加拿大家园论坛

加拿大教育情况有用素材收集帖子--梁溪香榭整理

原文链接:https://forum.iask.ca/threads/212079/

家园移民 : 2008-05-08#1
子女教育信息素材收集帖

嘎嘎

这个帖子也算历经了劫难了,丢了又重新找回来了。

主要整理的是关于教育的一些素材

从147楼开始,主要就是梁溪香榭搜集的一些的,推荐推荐!

推荐本书--从普通女孩到银行家,http://forum.iask.ca/showthread.php?t=233637

目录

tony 3213, PEI地区教育情况 Page 1, 5楼
tiantianxingchen 高中课程简介 Page 1, 6-9楼
守法公民 初三学生多伦多求学记 Page1, 18-22楼
守法公民 安省新移民高中指南 Page 2, 24-30楼
守法公民 安省新移民小学指南 Page 2, 34-36楼
守法公民 高中家长如是说 Page 3, 44-49楼
雨中冷百合 大多伦多地区高中排名 Page 3, 57楼, 59楼
守法公民 温哥华十佳公立中学 Page 4, 65楼
一缕阳光 菲沙关于BC省中学排名及相关简介 Page 4, 69楼
守法公民 部分大学对是否需要雅思或托福成绩的年限规定 Page 6, 101楼
守法公民 教育篇 Page 6, 113-121楼
梁溪香榭 高中留学须谨慎 Page 8, 149-152楼
梁溪香榭 加拿大高中教育感悟 Page 8, 153-155楼
梁溪香榭 根据成绩选择加拿大大学 Page 8, 159楼
梁溪香榭 如何选择最适合的加拿大大学 Page 9, 161楼
梁溪香榭 读高二的孩子去加拿大需要准备什么 Page 9, 164-165楼
梁溪香榭 高中英语课程介绍 Page 9, 166-169楼
梁溪香榭 高中英语课程介绍 Page 10, 188-191楼
梁溪香榭 多伦多著名私立学校收费一览 Page 10, 192楼
梁溪香榭 孩子的英语过关了吗? Page10, 194楼
梁溪香榭 关于A-LEVEL课程 Page 11, 217-228楼
shieley6811 关于A-LEVEL课程及学制 Page 13, 256-257楼
梁溪香榭 2007年安省高中拍名 Page 17, 322楼
朱三雀 安省, BC省, AB省高中拍名, Page 17, 329楼
响铃当当 加拿大教育优势 Page18, 347楼
梁溪香榭 A-LEVEL与IB课程的比较 Page 18, 358楼
朱三雀 中西学生的差异 Page 19, 364楼
梁溪香榭 大多伦多地区IB高中简介 Page 19, 372-375楼
梁溪香榭 IB课程与其他特殊教育课程的比较 Page 19, 376楼
梁溪香榭 IB与AP的比较, Page 19, 377楼
梁溪香榭 美国相关研究机构关于IB与AP的选择比较 Page 20, 387-391楼
梁溪香榭 群星灿烂不如一轮皓月当空-大学录取篇 Page 21, 409-413楼
梁溪香榭 在教育子女的过程中教育自己 Page 22, 422-424楼
梁溪香榭 关于美国大学 Page 22, 429-432楼
梁溪香榭 多伦多高中阶段特殊教育程序 Page 23, 445-446楼
梁溪香榭 加拿大高中生学科竞赛 Page 23, 447-448楼
梁溪香榭 美国大学 Page 24, 466楼
梁溪香榭 如何选择适合自己孩子的中小学 Page 25, 487-489楼
梁溪香榭 论孩子成长的内环境 Page 25, 490楼
梁溪香榭 犹太人的教育风格:朴实无华 讲求实际 Page 25, 497楼
梁溪香榭 培养子女的参政意识 Page 25, 498楼
梁溪香榭 美国大学点评 Page 25, 499-529楼
梁溪香榭 加拿大大学点评(商科) Page 27, 533-535楼
梁溪香榭 加拿大大学点评(理工科) Page 27, 547-549楼
梁溪香榭 安省高中教育咨询问答 Page 30, 584楼
梁溪香榭 毕业之际谈我的大学:滑铁卢大学 Page 30, 586-592楼
梁溪香榭 我在麦克马斯特 Page 30, 597楼
梁溪香榭 2005年度全球十大商学院 Page 30, 598楼
梁溪香榭 世界大学金融专业的排名 Page 30, 599楼
梁溪香榭 在加拿大IVEY商学院读MBA的日子 Page 31, 608楼
梁溪香榭 加拿大几个名牌大学比较之我见 Page 31, 616楼
梁溪香榭 加拿大的商学院MBA排名分析 Page 31, 618楼
梁溪香榭 加拿大本科申请指南 page 31-32, 620-624楼
yhyxll申请UBC请问需要什么条件,难度大否 Page 32, 633-634楼
willam131 关于A-LEVEL选课 page 34, 680楼
梁溪香榭 公立学校vs.私立学校
page 36, 713-716楼
梁溪香榭 路在自己脚下 --- 一位小留学生的自述 page 36, 717楼
惴惴不安 移民学生多“断层” 上公校还是私校难倒华人家长 page 36, 722楼
梁溪香榭 加拿大中学教育 page 38, 745楼
梁溪香榭 安大略省中学教育制度 page 38, 747楼
梁溪香榭 加拿大高中选课制 page 38, 748楼
梁溪香榭 高中毕业生该如何选择加拿大大学 page 38, 749楼
梁溪香榭 高中生如何入读加拿大大学?page 38, 750楼
梁溪香榭 加拿大大学怎样录取新生page 38, 751楼
梁溪香榭 加拿大大学对A-LEVEL的录取要求 page 38, 753-755楼

附件


菲碧 : 2008-05-08#2
回复: 子女教育信息素材收集帖子-冲置顶了

嘎嘎

教育其实很重要。这个话?狻E家恢泵挥锌??闹饕??蚴桥疾⒉皇呛茏ㄒ怠5?腔叵耄?蛭?蛔ㄒ担??愿??煤醚?鞍??


为了这个帖子的质量,建议大家开始可以把自己找到的有用的素材先帖上来,或者给个联结

偶尽量把这些素材整理好,然后整理成个大贴子。


这个帖子集中讨论,国内教育如何和加拿大的教育衔接的话题。

哈哈:wdb6:,第一次沙发!
守法开这个帖真是大好人啊!:wdb17:

Chechechen : 2008-05-08#3
回复: 子女教育信息素材收集帖子-冲置顶了

好帖,大家快来啊.

天南海北 : 2008-05-08#4
回复: 子女教育信息素材收集帖子-冲置顶了

好贴! 坐地板也乐意!

大家一起献计献策,共同进步!

tony3213 : 2008-05-08#5
回复: 子女教育信息素材收集帖子-冲置顶了

Last spring, students were selected randomly from a variety of schools across the country to test their abilities in reading, math and science. The country's education ministers sponsored the tests to see how their education systems compared.
P.E.I. did not compare well. Its students ranked last in reading and math, and next to last behind the Yukon in science. Education Minister Gerard Greenan is disappointed, but not surprised by the results.
"If you look at provinces like Ontario, Quebec, and Alberta, where the per capita expenditure for students is much higher than what we currently give to education, I guess we're going to continue to be in the lower end," said Greenan.
The government allocated new money for education in last week's budget, Greenan pointed out, particularly for very young Islanders, with an expansion of the Island's best-start program.
Opposition leader Olive Crane said she's worried about what these results mean for the Island's future workforce.
"My big area of concern has to do with the fact that the province is interested in going only into four sectors in the future for economic development: bioscience, information technology, aerospace and alternative energies. So math, science and reading competencies are extremely important," said Crane.
This is the second time in just a few months Islanders received bad news about the performance of young students. In December, results of an international testing regime also showed P.E.I. at or near the bottom of every category.
Crane wants public consultations on the education system to seek new ideas for a better way to teach Island students.

:wdb18::wdb24:

tiantianxingchen : 2008-05-08#6
回复: 子女教育信息素材收集帖子-冲置顶了

典型高一数学课
MATHEMATICS 421 (ALGEBRA)
- First Year - Academic - 1 credit PROVINCIAL CODE - MAT421A

DESCRIPTION - This course in algebra is a prerequisite for all other academic math courses. It is designed for students who have demonstrated, at intermediate school, an ability to cope with academic mathematics. It includes a review of number systems and their operations, solution of linear equations, inequalities and polynomial equations with related word problems, factoring polynomials, simplification and operations with rational and exponential expressions, and graphing linear relations.

科学Science 421
- SCIENCE 421 - First Year - Academic - 1 credit PROVINCIAL CODE - SCI421A

DESCRIPTION Science 421- This is an updated course using the new text, Coordinated Science. The focus of the course is to present topics at an introductory level in Chemistry, Physics and Biology leading to the 521 level. Topics include atomic structure, elements, compounds, cell structure, working with microscope and forces.

tiantianxingchen : 2008-05-08#7
回复: 子女教育信息素材收集帖子-冲置顶了

打字
KEYBOARDING SKILLS 701 - First or Second Year - 1 credit PROVINCIAL CODE - KEY701X

DESCRIPTION - This is a beginning course in keyboarding with emphasis on personal typing skills. A few of the areas covered are: display techniques, personal and business letter setups, tabulation and report typing. Emphasis will be placed on technique, posture and accuracy. Word processing concepts and applications are taught. The material covered will serve as a prerequisite for Typing 801. Students are expected to attain a minimum of 25 words per minute on a three-minute timing, within 97% accuracy.

tiantianxingchen : 2008-05-08#8
回复: 子女教育信息素材收集帖子-冲置顶了

高一英语!!
ENGLISH 421 - First year - Academic - 1 credit
PROVINCIAL CODE - ENG421A
LITERATURE ...This is a required course in Literature for students who have demonstrated success with the English program at the Intermediate School level. The literature segment which includes short stories, novels and a Shakespearean play will emphasize an active approach which teaches students to learn by asking critical questions, making predictions and forming hypotheses. They will respond to literature in various forms ranging from response journals to critical analyses.

WRITING ... The composition segment of this course will focus on the writing process and will emphasize such topics as brainstorming, researching, organizing, writing, revising, and will include small group peer editing. The overall goal is to enable students to write a unified, coherent composition on a variety of subjects for selected audiences. They will write in a variety of forms for various purposes.

tiantianxingchen : 2008-05-08#9
回复: 子女教育信息素材收集帖子-冲置顶了

PEI-China Public High School Program Schools
In 2004, PEI and CAN-ACHIEVE Educational Consultants Limited established the PEI-CHINA Public High School Program. Under the auspices of the program, CAN-ACHIEVE will establish and deliver the PEI public high school curriculum in selected Chinese schools.
<LI class=A_Item>Beijing Xicheng Foreign Language School北京西城区外国语学校
<LI class=A_Item>Foreign Languages School attached to Sichuan International Studied University <LI class=A_Item>He Nan Xuchang High School

梁溪香榭 : 2008-05-08#10
回复: 子女教育信息素材收集帖子-冲置顶了

真是我们想什么, 守法都知道啊.
大大地快人心啊.

家园移民 : 2008-05-08#11
回复: 子女教育信息素材收集帖子-冲置顶了

周末有时间了,偶就去好好的搜集整理点有用的素材去,最好是中文的

tony和tiantianxingcheng都厉害阿,嘎嘎

菲碧 : 2008-05-08#12
回复: 子女教育信息素材收集帖子-冲置顶了

周末有时间了,偶就去好好的搜集整理点有用的素材去,最好是中文的

tony和tiantianxingcheng都厉害阿,嘎嘎
:wdb10::wdb10::wdb10:守法!
偶正为儿子高中读哪一类的学校对今后去加更有帮助犯愁呢,北京的中学13号开始毕业考,成绩出来后马上就要填志愿了.:wdb24:
急等着守法的素材给偶参考噢!

山东大汉 : 2008-05-09#13
回复: 子女教育信息素材收集帖子-冲置顶了

这版主当的没治了,想什么来什么.
山东大汗 十七 男 高一 魁投 等面试

菲碧 : 2008-05-09#14
回复: 子女教育信息素材收集帖子-冲置顶了

收录一下:男,十六,高三,08年3月魁投,等面试.

菲碧 : 2008-05-09#15
回复: 子女教育信息素材收集帖子-冲置顶了

错了错了,打错了.是初三.

山东大汉 : 2008-05-09#16
回复: 子女教育信息素材收集帖子-冲置顶了

守法错了,是偶的仔.

家园移民 : 2008-05-09#17
回复: 子女教育信息素材收集帖子-冲置顶了

守法错了,是偶的仔.

修改

家园移民 : 2008-05-09#18
回复: 子女教育信息素材收集帖子-冲置顶了

摘录:

大家好。^ ^我是一个无锡外国语的初三学生,前天来到了多伦多。下面是我这三天求学,办手续等的经历。


前天:

没想到乘飞机的噪音那么大。虽然似乎应该用一些美好的词语形容我的旅程,但我还是要说:当我们到达温哥华,天空明亮的让人怀疑这里光污染泛滥。纯白色的阳光让我一阵炫目。

到了温哥华是耀眼的白天,而到了多伦多则是漆黑的夜晚。快降落前向窗外看去,整个多伦多被无数的路灯划分成一个一个金色的的正方形,整个城市都是金灿灿的一片,好像镶嵌了无数的宝石一般绚烂夺目。 当地晚上10点多,我终于到达“家”中。


昨天:

我获知要上学14岁以上的孩子先要去什么测试站测试一下英语和数学。我选择先考数学。拿出来一看,第一张简单得让人吐血。我相信小学的学弟学妹们也能非常正确的做完。可越往后他给我的卷子让我倍受刺激。居然出现了几的零次方。他们也学到那里了?!不是说外国的教育很松的?! 呜呜,最后一张更是英文的应用题,还好不难,我连猜带看算是弄懂了意思。

当然这些并不妨碍我最后得了满分,现在想想除了那个0次方,我好像六年级的时候来考也没问题。^ ^ 那边给数学卷子的老师用英文告诉我我的数学很棒! (他太可爱了)他问我是否在中国也是数学很好的学生。我真想非常自豪的告诉他这些题目太简单了!我中国的任何一个同学都会做呢!

英语考试我是带着坎坷的心情走了进去,没想到那白胡子白头发的老师非常和蔼和幽默的和我交谈一番。也对那张卷子进行了解释,那是一张看图画写文章的卷子,我非常轻松的写了出来。暗想也不难嘛!随后是两篇阅读理解,第一篇我全对了,第二篇虽然错了几个单词意思的选择,但是我也看得懂。

最惨的就是最后的一篇作文――television。他告诉我第一段写“什么是电视。”和我对电视的看法(看电视是/不是浪费时间)。后三段是三个理由,最后一段总结。我想虽然麻烦不过也没什么拉,学校里都一直写的。接着他拿给我一张很大的纸,告诉我每段写8-10行。 我傻傻得看着他,脑子一阵短路。天那!这么多简直是一篇作文!于是可怜的我死活用了很久很久才算凑够了那么多字。我一看,天,整整a4纸一大面还多。望着那细细的一行一行,我忽然觉得人的潜力果然是无穷的……

最后数学成绩不用说了,英语嘛,ABCDE4个等级(A最差,E就到了和这里学生一样的水平了。)我写作是D,听说全是C。没办法,在国内哪有时间练口语呢?(补充:虽然我最后没有用,不过这里英语测试好像能用电子词典呢! 只是老师看到我在那边,告诉我那会浪费很多时间,不要用。)

测试站还有个会普通话的阿婆。她对我谈了很久的话(那天她刚好不忙。)她还会问你一些关于爱好的问题填到你的档案上,到学校求学的时候好像要用的 。她自己也会问一些其他的问题。最后婆婆还很好心的介绍了许多事项给我,甚至给我我一张自己写的总纲。

今天

今天去学校报名,校长和老师都很和蔼,可我却听不大懂他们的话。 填了一大堆表签了无数个名以后我终于得知明天可以去上学啦!但他们居然告诉我除了英语专门上简单的以外其他都和正式班一起上!而且他们也有外语,就是法语!吓!我不由想到:明天找不到选修课教室怎么办?听不懂老师要求怎么办?要是……总之我心里乱糟糟的,紧张极了。 不过听说那里的同学都很热情,中国学生也很多,我总算放下心来。

家园移民 : 2008-05-09#19
回复: 子女教育信息素材收集帖子-冲置顶了

谢谢你们啦。以前看我妈妈青青草在这里发帖,我也来发发看。在今天(这里的周五)这个伟大的日子,我终于又一次能摸到我亲爱的电脑了。 下面是学校的外国学生。

去上学了,同学们都十分的热情 ,都会主动跟你打招呼,这个时候不能害羞 ,一定要尽力和他们讲话,仔细的听,这不,没几天我爸爸妈妈都说我说话语调越来越向外国人啦! 我在国内上的是无锡外国语学校,现在在学校里根本听不懂老师说什么 ,但却可以理解同学们的话(至少一半吧。 )

值得一提的是,作为一个中国学生,在加拿大学校你绝对不是孤立的 ,就说我们学校吧。我们学校号称最好的学校(St. Andrew's Junior high school,差不多叫这个吧。)我有三个中国同学,还有许多韩国同学。因为学校好,学区地价也是非常的贵(也有作业…… ),中国人算少了。在其他学校还会有更多中国人呢!

家园移民 : 2008-05-09#20
回复: 子女教育信息素材收集帖子-冲置顶了

找教室篇:

我们学校很奇怪,每上一节课都要换教室,好像每个老师专门给个教室,然后让学生去找教室!(和中国反一反啊 不公平!)一开始我很担心找不到教室,甚至因此作了恶梦 。

不过第二天我发现其实不要紧。 我们学校课表上每节课下面都会标上房间号,第一个数字是几楼,到了再找就简单了。实在找不到的话你只要指着那个房间号,随便走到一个学生边上问:“Excuse me,where is this classroom please.”那里的学生一定会很热情的告诉你,一般还会带你去。正如STAR 2004大大帖子里的,外国人真的很NICE。




学校作息时间篇:

也是很古怪的。

每人有一个柜子,要自己买把锁,在学校也可以买。6块钱,看上去米什么,可以想到要兑换成人民币,我的心那个痛啊…… 所以奉劝要去加拿大的同志们可以在国内先买上,不是有钥匙的,是那种一圈数字,然后转的。

早上8:55你需要先到主教室,然后会放国歌(?!),反正要站起来的,我第一天庄严的好像在天安门广场上升国旗似的 ,结果我却发现那些正宗的加拿大学生到在一边聊天嬉闹 ……然后开始上课。

上午有四节课,一二两节中间就一点点时间给你换教室(课程表上第一节下课和第二节上课只差两分钟,三四亦然。)中间是不能开柜子的。二三两节课中间倒是可以开柜子和休息15分钟。

下面就是午饭时间,你又可以开一次柜子,可以自带也可以买,但必须在餐厅吃。 50分钟时间,吃完了饭盒放到边上的架子上,放学的时候来拿(天那,什么破规矩,我忘了2个饭盒在那边,第三天抱着三个回去! )。吃完了就必须出去玩或者早上登记了可以去图书馆(?!这个还没弄明白。)反正你出去了打铃前就进不去了,除非有人正好出来。真不知道他那个锁怎么搞的(有人开?!)。最遭的是他不许你中午呆在学校楼里乱逛!

你问我怎么发现门是单方面开的?哦!那真不是什么美好的回忆!今天我把书放在门里的地上,然后出去感受了下冷空气,然后……然后…… (后记:我只好一个人孤独的在门边徘徊着……徘徊着……徘徊着…… )

中饭后又要上两节课。3点多放学

锁总结:我们学校一天里可以开这么几次锁

1,到学校
2,二三两节课间
3,中午吃饭前
4,下午上课打铃后
5,放学

家园移民 : 2008-05-09#21
回复: 子女教育信息素材收集帖子-冲置顶了

上面是学校介绍,这里就是我这三天上课的经历啦!

本着中国 迟到=被骂=扣分=可能站在外边 的数学公式 ,每上一节课我都以最快的速度,在两分钟内绝对抵达教室,似乎我每次不是第一个就是第二个。 其实可以定定心心的走,晚一会儿问题不大。每上一节新课(其实这里的课程表上一天内除了下课并没有重复的课…… )都要问老师要坐在什么座位上,哎,要是都像在主教室一样随便坐该多好!

虽然这里的同学和老师都非常非常友好 ,但是我在课上却依旧无法理解老师在说什么 ,尤其是SCIENCE!如果我不知道这里是加拿大,也许我连他讲的是哪国文都不知道。

相信过几个月就可以听懂大部分了吧。说到这里我又不得不赞一下我们老师,所有课的老师几乎都非常了解我的难处,SCIENCE老师更是叫个中国同学翻译告诉我不用担心,她会专门给我留些很简单的作业来锻炼英语。

下面就是体育课啦!是在室内上的,我依照规定买了一件统一的T恤和到膝盖的那种运动裤。课前都要求你到更衣室换好,然后就可以在偌大的场地里去啦。我和几个韩国同学一起打排球,虽然手上一片红红的,但是心里别提多开心啦!

在数学课上,虽然我还是不懂老师的话,但她在黑板上讲的4道计算题和讲完后说的“So noone is perfect in Math.”我可是理解了。 我问经常帮助我的那个亲爱的韩国同学是否那4道题目对他们来说很难?结果得到了她肯定的答复。 天啊,我承认那几道题有点复杂,但仅仅是数字比较大罢了,我们中国学生4年级做那个都未必有太多难度……

今天下午地理课我跟着同学们来到餐厅(吓!开始我以为来聚餐呢! )老师拿着话筒叽里咕噜讲了些什么,我倒是听懂了点关于交车费,午饭自己带,穿衣服有要求的话。难道是去秋游?我美美的想着 ,不过中国同学却告诉我是去做义工 。哭,我话都不会说呢!

那个经常帮我的韩国女同学,告诉他们有张关于义工的什么纸我来晚了所以没有,必须去二楼办公室拿。 我怕怕的告诉她我不懂英语,能不能请她帮帮我,她非常爽快的同意了。 我们之间的对话掺杂了很多的手势,也有几次电子词典牌上了用场,但她一点也没有不耐烦,而且多次帮助我,我实在是太感激她了!

家园移民 : 2008-05-09#22
回复: 子女教育信息素材收集帖子-冲置顶了

先说数学,考试的时候分好多张纸头。第一张简直是口算题! 做完后他根据你的水平决定你下一张。往后会难一点,但我相信你做最难的是没有问题的。 题目……俄,比如求正方形面积,周长(吓 ),还有给圆的半径叫你求周长,我暗想这还稍微复杂点 ,再往下一看,天那~他居然把公式都给你了! 最后那几张卷子上最难的也就是负几次方,除了这个毫无难度。

英语:阅读理解你肯定做过嘛,不难的。写作文就挑你会写的句子写。先前可以问问他:“Can I use my e-dictionery please?”


备注:那里的阿婆向我强调这不是考试,只是测试一下你的能力来决定你的就学。所以完全不用担心, 不管考的怎么样都可以上学的。那次有个一点英语也不懂的男孩也只是退了一级,还可以学学语言呢!好羡慕他! (数学?不考满分你就太过分了! )

家园移民 : 2008-05-09#23
回复: 子女教育信息素材收集帖子-冲置顶了

以下的是个宝贝,pdf的,太大,怎么都传不上去,估计过阵子才可以。先转化了word看看,呵呵

家园移民 : 2008-05-09#24
回复: 子女教育信息素材收集帖子-冲置顶了

家长指南第一章
(PARENT GUIDE ONE) 入学须知(Starting School)
目录
1 1. 中学的种类
2 2. 报名入学
3 3. 评估中心(接待中心)
4 4. 第一年选课
5 5. 承认以前的学分
6 6. 视力和听力健康
7 7. 使用计算机
8 8. 拿签证的国际学生
9 9. 各中学之间的区别
10 10.报读学区之外的学校
11 11.在加拿大之外的高中毕业的学生
12 12.为报道第一天做好准备
13 13.已满十八岁的学生

新移民学生抵达加拿大之后,大多就要立即入学。这份指南描述了孩子入学之初会碰到哪些事情、如何做好准备。
新移民到达加拿大的头几年可能非常艰难。家长忙于找工作、安顿新家。孩子则要结交新朋友,每个人都在适应新环境。如果子女就读高中(9-12 年级,secondary schools),家长还另有一项责任:每年都要协助孩子选择课程。课程种类繁多,选课并不容易,而高中怎么选课,会直接影响学生毕业后的出路。
指南的其它几章会告诉家长,如何协助子女对高中课程作出明智的选择。本指南建议家长和子女,在选科之前一起订出长期的学业计划(an education plan)。有了计划才可以目标明确(读大学、大专、学徒课程还是直接就业)。家长也可以请学校的老师或辅导员(the guidance counsellor)提供协助。
本指南专为新移民介绍安省高中制度,一共有六章,本章是其中之一。每一章都为家长如何辅导子女完成高中提供了资料和建议。这份免费资料有十八种语言,登在网上:www.settlement.org/edguide 。详情请参阅本章末尾。

Starting School Simplified Chinese
1. 中学的种类(Kinds of Secondary (High) Schools) 安省高中有四个体系:英语公立、英语天主教、法语公立、法语天主教。每个体系都由地区教育局(district school boards)管理。所有未满二十一岁的人都可以进高中读书。许多教育局还为成人开办高中课程。法语学校是为讲法
语的学生设立的。有关法语学校的详情,请向法语教育局查询。安省教育局的详情,请见:
http://www.edu.gov.ca/eng/general/elemsec/es_overview.html
2. 报名入学(Registering For School)
家长为子女报名入学的时候,请携带下列证件:
a) 学生年龄证明,二者有一即可:
?Ρ出生证明(a birth certificate)或者
?Ρ护照 (a passport)

b) 住址证明,文件上必须有姓名和地址,下列文件有一即可:银行月结单(a bank statement)、电话单(a telephone bill)、电费单(an electrical bill)、租约(apartment lease)。

c) 监护证明:只有未满十八岁、而且不和父母同住的学生才需要。
d) 学生移民身份证明,下列文件有一即可:
?Ρ加拿大护照 (Canadian Passport)
?Ρ移民入境纸(IMM1000)
?Ρ永久居民证明(IMM5292)
?Ρ枫叶卡(Permanent Resident Card)

e) 防疫注射记录(Immunization Record):证明学生已注射过防疫针
Starting School Simplified Chinese
3. 评估中心(接待中心)(Assessment Centres , or Reception)
难民身份的学生
有些教育局要求新移民学生到评估中心,让老师测试数学和英
(Students with Refugee Status)
语水平。测试的结果会送到学生就读的学校,让老师了解该生已经学过哪些内容。如果你有孩子以前的成绩单、教科书、已
不同教育局对难民学生、或及任何有助于评估的教学资料,都请拿给评估中心的老师看。
者没有《永久居民证明表》(Confirmation of Permanent Residence)的学生,有不同的报名方式。请向当地的教育局查询。
4. 第一年选课(Placing Students in Courses in the First Year)
在新移民学生入学的时候,辅导员会先与他们讨论以前的教育、兴趣、毕业后的目标,然后再协助他们选择适当的科目。
学校会考虑:
?Ρ 把已经学过的东西与安省高中课程的要求进行比较
?Ρ学生的能力和兴趣
?Ρ体现以前学业的文件,例如各科成绩表
?Ρ如果做过数学和英语评估的话,评估的成绩
?Ρ 学生在学校里上了多少年
?Ρ 学生的年龄

学校会暂时将学生分到某一年级,选读各种课目。然后,根据学生体现出来的知识水平,学校会进行必要的调整,将他们分配到更适当的班级。

Starting School Simplified Chinese
通常,在学年末、一般在二月,学生要选择下学年的课目。阅读成绩单(report cards)、与老师面谈 (parent teacher interview)、和子女讨论,都有助于你和子女选择课目。
与子女一起定下一个学业计划(an education plan)是非常重要的。由于课程种类繁多,如果事先没有计划好,选择起来会非常困难,并且现在的选择会影响到子女毕业后的出路.
有了一个计划,就可以一步一步来、保证家长和学生的目标得以实现(不论是读大学、大专院校、学徒课程还是直接就业)。学校的老师和辅导员也可以提供协助.
寻求帮助(Getting Help)
如果家长有疑问或担心,最好向有关老师查询。如果是一般性的问题,可以向“英语作为第二语言的特殊教师”(the ESL teacher)或辅导员查询。
英语为第二语言老师(The ESL Teacher) ESL 是English as Second Language 的简称,就是那些给英语为第二语言的学生开设的课程,教这种课程的老师就叫ESL 老师。ESL 老师,除了专门给移民学生教授英语,还可以帮助学生和家长理解他们的选择、毕业后的出路。这些老师在协助新移民学生方面有丰富的知识和经验,懂得怎样帮助他们顺利完成高中学业。
辅导员(The Guidance Counsellor) 辅导员可以提供信息和建议,帮助学生和家长理解如何选课、如何择业、毕业之后有哪些机会。
辅导员也可以帮助学生解决个人的疑难、学校里的各种问题。家长可以向辅导员咨询子女在校的表现。辅导员的丰富经验对学生和家长相当有帮助。
Starting School Simplified Chinese
5. 承认以前的学分(Credits for Previous School Learning)
安省学校采用学分制。学生完成每个课目之后会得到一个学分(a credit),学生必须拿够三十个学分才可以毕业。其中有一些学分是必修的,有些则可以选择。学校可能会承认新移民学生在原居地受过的教育,给一些学分。
学校通常会先让学生在学习中展示以前学到了多少,然后才决定原居住地的学业可以算多少个学分。请向辅导员查询详情。
?Ρ 所有可以体现学生以前学业的资料,都带到学校去,例如成绩表、教科书等等。

建 议
?Ρ帮助子女先做好准备,再与老师讨论以前的学业、毕业后的计划。
?Ρ 如有任何疑问,请致电辅导员查询.

6. 视力和听力健康(Vision and Hearing Health)
视觉或听觉有问题,学习起来会很困难。很多学生甚至不知道自己视觉或听觉有问题。他们可能认为自己的视听和别人是一样的。视觉和听觉会随着年龄增长而变化,所以定期检查是很重要的。
?Ρ 每二至三年让验光师(an Optometrists)或眼科医生(an Ophthalmologists)检查一次眼睛。安省医疗保险计划(OHIP)为十九岁以下的人支付每年一次的检查费用。
建 议

?Ρ 必要的时候,鼓励子女配戴眼镜、隐形眼镜、助听器。如果学生不想戴,与辅导员或老师见面,一起劝说鼓励。
?Ρ 跟医生谈一谈,看是否应检查学生的听觉

Starting School Simplified Chinese
7. 使用计算机(Access to Computers) 使用计算机是中学课程的重要部分。每所学校都有计算机供学生使用。
公立图书馆、一些社区中心也设有计算机,可免费使用。社区中心的计算机称为CAP 计算机(CAP 就是Community Access Program)。哪些社区中心设有计算机,名单在网上:http://cap.ic.gc.ca/english/4000newonurban.asp.
8. 签证学生(Visa Students)
需要付学费的外国学生被称为”签证学生”、或外国出生的付费学生。如果这些学生的父母不在加拿大居住,他们就必须有一名正式的监护人(an guardian)住在学区之内。监护人要负起当家长的责任。有关签证学生或国际学生的课程、学费,请浏览各个教育局的网址。

家园移民 : 2008-05-09#25
回复: 子女教育信息素材收集帖子-冲置顶了

9. 各中学之间的区别(How Secondary Schools Differ)
所有学校提供的核心课程都是一样的,但有些学校会提供一些特殊加强课程、或者学生的选择范围更广,例如英语为第二语言课程(English as Second Language,简称ESL)、英语识字课程(English Literacy Development,简称ELD)、半工半读实习教育(Co-operative Education)、各国语言(International Languages)等,例如中文,但是并非每所学校都有英语为第二语言(ESL)或英语读写课程(ELD)。
有些学校设有特殊加强课程,例如计算机、美术(arts)、自然科学(science)、体育(sports)。学生要先在相应领域表现出较强的基础,才能选择上述课程。
教育局有一本册子,里面列出每所中学所提供的课程。家长可以向学校免费索取这本册子,也可以到学校或教育局的网址查阅。

Starting School Simplified Chinese
10. 报读学区之外的学校(Schools Outside of Your Community)
通常学生读哪一所学校,是由他们的住址决定。每一所学校都有一个以街道和房屋划分的地区,称为学区。在有些教育局,如果某学校仍然有空缺的话,学区外的学生就可以报读。这些学校首先录取本区的学生,如果仍有空缺,才会录取区外的学生。
详情请向校务处查询,或浏览教育局的网址。
11. 在加拿大之外的高中毕业的学生(Secondary School Graduates from Other Countries)
有些在外国已经高中毕业的新移民学生,仍然想进中学,提高英语、修其它的课程,但是由于教育制度不同,学校很难对他们以前学过的每一个科目给予准确的学分。
建 议

?Ρ 向辅导员了解学校对学生以前的学业能给多少个学分,学生还需要多少个学分才可以取得安省高中文凭。
?Ρ调查一下,学生在原居地受过的教育是否够资格申请大学、大专院校,大学大专里有没有英语为第二语言的辅导(ESL)。
?Ρ如果学生的英语和数学做过评估(English and mathematics skills

assessment)的话,与辅导员一起分析讨论评估结果,另外看一看学校可以如
何协助。

Starting School Simplified Chinese
12. 为上学第一天做好准备(Planning for the First Day) 学生第一天报到,大多都感到紧张,加上英语有限、在学校里又举目无亲。
?Ρ 把子女介绍给一位同学,同学可以协助他们适应新学校
?Ρ购买第一天上学需要带的文具:横格纸(lined paper)、活页夹(binders)、圆珠笔或钢笔(pens)、铅笔(pencils)和字典(a dictionary)。
?Ρ 第一天放学后,询问子女当天是否顺利、可有意外。
2 13. 已满十八岁的学生(Students Who are 18 years of Age or Older) 在法律上,年满十八岁的学生都是成年人。学校会把所有资料交给他们,而非家长。如果家长想收到成绩单或老师的电话,必须由学生到校务处(school office)签署授权书。

建 议

?Ρ教育局(School Board)-管理一组学校的地区机关。
?Ρ学分(Credits)-学生每读完一门课(共110 小时) 会得到一个学分。大部分课程都是一个学分,学生要有至少三十个学分才可以毕业。有些学分是必修的,有些则可以选择。
?Ρ成绩单(Report Card)-学生在校成绩的正式报告。报告由每科老师填写,给学生和家长阅读。
?Ρ安省高中文凭(Ontario Secondary School Diploma ? 简称OSSD)-学生的高中毕业证书。
?Ρ评估(Assessment)-决定学生能力水平的过程。许多教育局要求学生在进高中之前先做数学和英语能力的评估。

本指南内的?家长?(parents)一词包括监护人、保姆、以及其他家人。高中(Secondary schools)也叫 high schools 或 collegiates。
Starting School Simplified Chinese
高中课程的家长指南包括以下六章:
1 1.入学须知 (Starting School)
2 2.学习英语和其它科目(Learning English and other Subjects)
3 3.学校政策和校务规则(School Policies and Procedures)
4 4.选课、拟定学业计划(Choosing Courses and Developing an Education Plan)
5 5.成绩单、家长老师面谈会――学校与家长沟通的两个渠道(Report Cards and Parent Teacher Interviews ? How the School Reports to You)
6 6.家长如何协助子女(Ways that Parents Can Help)

还需要更多信息吗?(Need More Information?)
家长可以采用下列方法得到更多信息:
?Ρ向老师、ESL 老师、辅导员(guidance counsellor)、校长(the principal ),副校长(the vice principal)或校务处职员(the school office staff)查询。
?Ρ联系地区教育局(the school board)或安省教育厅(the Ministry of Education),查询他们的政策和校务规则。
?Ρ安省教育厅的网址是http://www.edu.gov.on.ca/eng/welcome.html
?Ρ查询自己学区所属的教育局:http://sbinfo.edu.gov.on.ca
?Ρ安省家长议会(The Ontario Parent Council) 的网址:www.ontarioparentcouncil.org ,安省学校辅导员协会(The Ontario School Counsellor’s Association) 的网址:www.osca.ca

有关定居安省的资料,包括学习英语、工作就业、寻找住所、医疗服务等等一切,请浏览应有尽有的:www.settlement.org


家长指南第二章(PARENT GUIDE TWO)
学习英语和其它科目(Learning English and Other Subjects) 开学的头一两个月,大部分新
目录
1 1. 学习英语
2 2. 英语为第二语言课程(ESL)或英语识字课程(ELD)
3 3. 毕业条件与ESL 课程、ELD 课程
4 4. 升读大学、大专院校对英语水平的要求
5 5. 安省教学大纲
6 6. 教学方法
7 7. 学会自己对自己的学业负责
8 8. 家庭作业
9 9. 校外参观学习
10 10.法语
11 11.各国语言课程
12 12.课余和午餐时间的活动
13 13.特殊教育
14 14.已满十八岁的学生


移民学生都要面对两大困难:适应全新的教育制度、学习英语。二者加起来,可能是个很大的考验。
本章指南将会解释老师如何教授英语和其它科目,教学中与原来国家可能有哪些不同之处,而家长可以如何协助子女。
新移民到达加拿大的头几年可能非常艰难。家长忙于找工作、安顿新家。孩子则要结交新朋友,每个人都在适应新环境。如果子女就读高中(9-12 年级,secondary schools),家长还另有一项责任:每年都要协助孩子选择课程。课程种类繁多,选课并不容易,而高中怎么选课,会直接影响学生毕业后的出路。
指南的其它几章会告诉家长,如何协助子女对高中课程作出明智的选择。本指南建议家长和子女,在选科之前一起订出长期的学业计划(an education plan)。有了计划才可以目标明确(读大学、大专、学徒课程还是直接就业)。家长也可以请学校的老师或辅导员(the guidance counsellor)提供协助。
本指南专为新移民介绍安省高中制度,一共有六章,本章是其中之一。每一章都为家长如何辅导子女完成高中提供了资料和建议。这份免费资料有十八种语言,登在网上:www.settlement.org/edguide 。详情请参阅本章末尾。
1. 学习英语(Learning English)
每个人学习英语(或一门新语言)进度各异,即使一家人的兄弟姐妹也绝不一样。通常听说比读写要快一些。大部分学生过一两年就能够用英语进行日常交流,不过,对复杂的科目,读写和理解能力,要达到以英语为母语的同学那样,可能需要五至七年乃至更长时间。
来到加拿大的时候,大部分新移民学生都能够用母语读写,但英语能力可能有限。“英语为第二语言”(ESL)课程可以帮助这些学生赶上以英语为母语的同学。
不过,有些学生可能以前失学,没有读写的基础。英语识字课程(ELD)可以帮助他们用英语学习读写能力。
有关学习英语的详情,请浏览下列网址:
http://www.edu.gov.ca/eng/document/curricul/secondary/esl/eslful.html
家长可以用各种方法帮助子女学英语。下面是一些建议:
?Ρ 鼓励子女参加用英语进行的集体活动。午餐时间和放学后举行的活动,例如体育、音乐、兴趣小组等,是练习英语、结交朋友的好机会。

建 议
?Ρ向ESL 老师请教如何在家帮助子女学习英语。
?Ρ鼓励子女继续学习母语。如果学生有母语作为基础,学习英语会比较快。与子女讨论母语报刊上的新闻故事。参加以母语进行的社区活动。可能的话,为子女报读各国语言课程(International Language ,其中包括汉语)。有些公立图书馆还藏有多种语言的书籍.
?Ρ 安排子女参加讲英语的暑假活动。
?Ρ 鼓励子女参加课堂讨论。刚开始可能比较困难,但课堂讨论会加快学习英语的进度。

寻求帮助(Getting Help)
如果家长有疑问或担心,最好向有关老师查询。如果是一般性的问题,可以向“英语作为第二语言的特殊教师”(the ESL teacher)或辅导员查询。
英语为第二语言老师(The ESL Teacher) ESL 是English as Second Language 的简称,就是那些给英语为第二语言的学生开设的课程,教这种课程的老师就叫ESL 老师。ESL 老师,除了专门给移民学生教授英语,还可以帮助学生和家长理解他们的选择、毕业后的出路。这些老师在协助新移民学生方面有丰富的知识和经验,懂得怎样帮助他们顺利完成高中学业。
辅导员(The Guidance Counsellor) 辅导员可以提供信息和建议,帮助学生和家长理解如何选课、如何择业、毕业之后有哪些机会。
辅导员也可以帮助学生解决个人的疑难、学校里的各种问题。家长可以向辅导员咨询子女在校的表现。辅导员的丰富经验对学生和家长相当有帮助。
2. 英语为第二语言课程或英语识字课程(ESL or ELD Courses)
除了正常科目之外,学习英语的学生会修读英语为第二语言(ESL)或英语识字课程(ELD)。学校提供的ESL 课程,可以分为不同水平,最多分为五级。另有英语识字课程,分为四级。学校把学生分到哪一级,是根据学生的英语水平,而不是年龄或班级。
有些学校为移民学生设立了特殊科目,例如英语为第二语言历史,英语为第二语言地理,英语为第二语言自然科学,及英语为第二语言就业指导课(the career class)。

英语为第二语言课程和(ESL)英语识字课程(ELD)的简介
课程编码(Course 课程(Course) 教授内容(What it Teaches) code)
英语为第二语言(ESL)
ESL AO 第一级 英语交流初级班 英语入门,协助学生适应新生活
ESL BO 第二级 日常英语 增加学生的英语常用词汇,学习课堂用语
ESL CO 第三级 学校里、功课上的常用英语 提高学生在所有科目上的英语运用能力
ESL DO 第四级 英语环境里的学习技能 为学生修读学术类英语(Academic English Course)及应用类英语(Applied English Course)做好准备
ESL EO 第五级 英语之桥 学生要升大学、大专,需要修读较高水平的英语,这个级别的ESL 英语课程就是为升大学和大专做好准备。

英语识字课程(ELD)
ELD AO 第一级 读写初级班 介绍基本的读写技能,协助学生适应新生活
ELD BO 第二级 基本读写技能 协助学生增强基本的读写技能
ELD CO 第三级 日常读写应用 协助学生增强读、写技能,锻炼学生的学习和职业规划能力
ELD DO 第四级 学校和作业的读写应用 提高学生的读写技能,以便能够继续升学、寻找就业机会,并全面融入加拿大社会。

3. 毕业条件与ESL 课程、ELD 课程(Graduation Requirements and ESL and ELD Courses)
学生完成一门ESL 课程或一门ELD 课程,就会得到一个学分。学生必须拿够三十个学分才可以获得安省的高中毕业证。英语必修学分共有四个,其中三个可以是ELS 英语或ELD 英语,但剩下那一个必须是十一或十二年级的英语(grade 11 or 12 English)。

家园移民 : 2008-05-09#26
回复: 子女教育信息素材收集帖子-冲置顶了

4. 升读大学、大专院校对英语水平的要求(English Requirements for University and Community College Admission)
在加拿大未住满四(或五)年的的学生,大学和部分大专院校在录取之前会要求考托福(TOEFL,全称为Test of English as a Foreign Language)或类似的英语考试。大部分大学专业也规定学生必须有某些特定的中学英语学分。详情请参阅各大学大专的招生条件,以及本指南第四章14-16 页(总第49-51 页)。
有些大学和大专专门为英语为第二语言的学生开设课程,帮助他们。详情请参阅每所大学、大专的网页.
TOEFL 考试的各类信息,还有模拟题,都可在网上查到: http://www.toefl.org
5. 安省教学大纲(The Ontario Curriculum)
安省所有公立学校都采用安省的教学大纲。教学大纲描述了 安省教学大纲九年级
在读完一门课程之后学生应该学到了什么、会做什么. 的例子――数学原则
(学术类):
老师们都理解,正在读ESL 和ELD 课程的学生正在学语言、 ”在完成这个科目之
适应环境,所以会根据他们的英语水平来调整课程。 后,学生可以用不同
的公式算出一段直线
的斜度。”

安省教育厅为每个科目编定了教学大纲。老师根据这些教学大纲去安排教学。这些教学大纲包括:
英语(English) 科技教育(Technological Education) 英语为第二语言(ESL)或者英语读写课社会科学(Social Sciences) 程(ELD)艺术(The Arts) 法语为第二语言(French as a Second 法语(The French) Language) 数学(Mathematics) 辅导和职业教育(Guidance and Career 自然科学(Science) Education) 商科(Business Studies) 健康和体育(Health and Physical 加拿大和全球研究(Canadian and World Education) Studies) 跨科目研究(Interdisciplinary Studies) 古典语言和各国语言(Classical and 原住民研究(Native Studies) International Languages)
教学大纲发表在下列网址:
http://www.edu.gov.ca/eng/document/curricul/seccurric.html
专题研究(Projects)
6. 教学方法(How Students are Taught) 很多科目都要学生做专题研究。学生自己选择一个题目
老师运用许多方法去帮助学生学习,例如讲解、讨论、去研究,然后写一份报告或问答、调查研究、校外参观学习(field trips)、专题研者在全班讲解(a 究(projects)以及小组作业(team assignments)等。新presentation)。有时候几个学
移民学生可能对某些学习方法感到陌生. 生会小组合作、一起做一个专题。老师会提出一些问题让学生讨论和解答,而不是强迫学老师可以帮助学生安排他们生死记硬背。无论哪一科,老师都会鼓励学生培养批判的专题研究。性的思维,就是说学生要自己判断是非对错。学生学习如何对手头的信息进行解读、分析和评价。他们也要学习有条理地思考,提出问题,形成观点并向全班表达自己的看法。
老师希望学生参与课堂讨论,在全班或小组面前发言。例如在数学课上,老师会要求某位同学解释结果是怎样算出来的。学生应该有能力在图书馆、互联网上对一个题目进行调查研究,把找到的资料写出来,并提出自己的观点。然后把摘要写在研究报告里。
老师安排的教学会帮助学生体会到,不同背景、不同文化的人,他们的经验和贡献都同样可贵。
?Ρ 与子女讨论,安省和原来国家的教学风格和方法有什么差异。跟他们解释:适应新环境是需要时间的。
?Ρ 鼓励子女找老师谈谈自己的功课。这样老师可以看到学生的学习动力。
?Ρ 告诉子女,需要的时候就该请老师帮助。有疑问、需要帮助的时候就应该说出来。
?Ρ 鼓励子女对父母讲出心里的想法和原因。这对他们参与学校讨论、写作业,也是一种有效的练习。
?Ρ查询学校的课外活动,例如体育课、音乐课和兴趣小组等。通常所有学生都可以参加这些由老师指导的活动。参加课外活动是学生练习英语的好机会。

7. 学会对自己的学业负责(Learning Responsibility for School Work)
中学生已经是半个成人,只要再过几年他们就该独立了,要自己做决定、就业,负起一个成年人应有的责任。
老师为了学生的将来着想,会提供各种机会培养他们的自律精神,自己对作业负责。例如学生应该主动完成家庭作业,但老师并非每次都会检查是否完成。年级越高,就越该自律。
学会了自觉自律的学生,将来升入大学、大专院校,或者做其它行业,上手都将比较快,因为那时候就一切靠自觉自律了。

建 议

8. 家庭作业(Homework)
家长可以通过家庭作业了解学生在校的情况。学生几乎每晚都应该有作业。作业的多少视年级高低而定。
九年级的作业至少需要45 分钟至一小时。学生每晚会有作业,到十二年级的时候,最多达到两小时。学校要求学生每晚都读书。如果家长想知道要做些什么作业,可以向老师查询。
如果子女说“没有作业”?
?Ρ 要子女给你看他们在学校做的功课。
?Ρ 如果发觉子女经常没有作业,向老师查询.
?Ρ 鼓励子女复习前几天学过的内容,为以后的作业和测验做准备。
?Ρ 督促子女每晚都阅读英语或母语书籍。

9. 校外参观学习(Field Trips)
老师会经常安排学生去参观博物馆、社区活动、剧院。这些”校外参观学习”是课程的一个重要内容。学生在外出参观的前后,都会学习与参观有关的内容。
参加校外活动的学生,如果未满十八岁,必须事先得到家长的同意。学生会将同意书带回家给家长签字。家长在收到同意书之后请尽快签字,然后交回学校。有些校外参观活动需要付费。
10. 法语(French)
法语是加拿大两种官方语言之一。学生在中学毕业之前必须选修至少一门以法语为第二语言的科目。在特殊的情况之下,学校可以用其它科目代替法语学分。许多学生发现

对母语的知识和理解能够帮助他们学习法语。
11. 各国语言课程(International Language Classes)
有些学校提供“各国语言课程”,教授新移民的母语。这些各国语课程帮助学生保持、提高他们的母语水平,保持母语也有助于他们学习英语。有许多教育局在周末或晚上为学生提供各国语言课程,可以算学分。
如果报名的学生足够,教育局可以为现有的各国语言课程增加班级,或者开设新语种。详情可向ESL 老师、辅导员、继续教育部(the Continuing Education deparment)查询。
12. 课外活动、午餐时间的活动(After School and Lunch-time Programs)
每一所中学在课外或者午餐时间都有活动,例如体育、学生会、兴趣小组和音乐课程。人人都可以参加,这些活动由一名老师指导。
这些活动是新移民学生练习英语、结识朋友的大好机会。通常这些活动是每星期一次,每次一小时,在午餐时间或放学后举行。详情请向ESL 老师或辅导员查询。

13. 特殊教育(Special Education)
学生英语欠缺,不等于说就需要接受特殊教育。不过,如果你的子女对功课感到吃力,表示他们可能学习有困难。
特殊教育课程为学习有显著困难或有特殊需要的学生提供额外协助。其中有些学生只需要短期的协助,而有些学生有复杂的学习和健康问题,则时间较长。称为特殊教育,就是因为是专门为有特殊需求的学生设计的。
学生需要协助的原因可能多种多样。他们的问题可能是身体、智力、情绪、行为、对话、语言,可能是视觉或听觉上有问题,又可能是与人交往有严重的问题。特殊教育课程里也包括天才班。
要适应新语言、又要适应新学校,学业上出现困难是很正常的。告诉老师,学生的母语水平如何,这些信息会帮助老师找出问题的根源。
但是如果你担心子女在学习上确实有困难,对老师说出你的疑惑。老师会有许多非正式的方法去评估学生。如果需要的话,家长或者老师可以提出给学生进行正式的评估(a formal assessment)。
学校需要按照规定的手续,评估学生是否需要接受特殊教育。事先家长必须书面同意,让学校进行评估,自己也将参加一些会议,例如有一个被称为“辨认、排班和复核委员会议”(Identification, Placement and Review Committee ? 简称IPRC)。
每个教育局都有家长指南,解释接受特殊教育手续。
大部分大学和大专都设有课程,协助有特殊需要的学生。

14. 已满十八岁的学生(Students Who are 18 years of Age or Older)
在法律上,年满十八岁的学生都是成年人。学校会把所有资料交给他们,而非家长。如果家长想收到成绩单或老师的电话,必须由学生到校务处(school office)签署授权书。

?Ρ教学大纲(Curriculum)-一份正式文件,描述每门课程和学生必须掌握的内容。
?Ρ特殊教育(Special Education )――为有特殊需要学生而设的独特课程。
?Ρ学分(Credits)-学生每读完一门课程(110 小时) 会得到一个学分。大部分课程都是一个学分,学生至少需要三十个学分才可以毕业。有些学分是必修的,有些则可以选择。
?Ρ安省高中文凭(Ontario Secondary School Diploma ? 简称OSSD)-学生的高中毕业证书.
?Ρ毕业条件(Graduation Requirements )取得安省高中文凭必须具备哪些条件。
?Ρ录取条件(Admission Requirements )申请大学、大专、职业培训必须具备哪些条件。

此指南内的?家长?(parents)一词包括监护人、保姆、以及其他家人。高中(Secondary schools)也叫 high schools 或 collegiates 。
高中课程的家长指南包括以下六章:
1 1.入学须知 (Starting School)
2 2.学习英语和其它科目(Learning English and other Subjects)
3 3.学校政策和校务规则(School Policies and Procedures)
4 4.选课、拟定学业计划(Choosing Courses and Developing an Education Plan)
5 5.成绩单、家长老师面谈会――学校与家长沟通的两个渠道(Report Cards and Parent Teacher Interviews ? How the School Reports to You)
6 6.家长如何协助子女(Ways that Parents Can Help)

还需要更多信息吗?(Need More Information?)
家长可以采用下列方法得到更多信息:
?Ρ向老师、ESL 老师、辅导员(guidance counsellor)、校长(the principal ),副校长(the vice principal)或校务处职员(the school office staff)查询。
?Ρ联系地区教育局(the school board)或安省教育厅(the Ministry of Education),查询他们的政策和校务规则。
?Ρ安省教育厅的网址是http://www.edu.gov.on.ca/eng/welcome.html
?Ρ查询自己学区所属的教育局:http://sbinfo.edu.gov.on.ca
?Ρ安省家长议会(The Ontario Parent Council) 的网址:www.ontarioparentcouncil.org,安省学校辅导员协会(The Ontario School Counsellor’s Association) 的网址:

www.osca.ca
有关定居安省的资料,包括学习英语、工作就业、寻找住所、医疗服务等等一切,请浏览应有尽有的:www.settlement.org

家长指南第三章(PARENT GUIDE THREE)
学校政策和校务规则(School Policies and Procedures)
目录
1 1. 平等对待所有学生
2 2. 家长担心的问题
3 3. 如果学生可能受到伤害,如何举报
4 4. 包容其它宗教节日和信仰
5 5. 家长委员会
6 6. 行为守则
7 7. 强制性处罚
8 8. 子女的同学有让你担心的举动
9 9.纠正学生不良行为,家长如何参与
10 10. 适龄者必须上学
11 11. 午餐
12 12. 全天缺席或者缺课
13 13. 学期制与非学期制学校
14 14. 学校假期

1 15. 更新地址和电话
2 16. 如果学生在学校生病
3 17. 过敏症和健康问题
4 18. 恶劣天气
5 19. 防疫注射
6 20. 午餐活动和课后活动的通知
7 21. 上学着装和校服
8 22. 夜校和暑假班
9 23. 学生手册/学校记事簿
10 24. 校外参观学习
11 25. 课程表和课时表 (Student Timetables and Periods)
12 26. 教科书
13 27. 储物柜(Lockers)
14 28. 文具
15 29. 安省学生记录(Ontario Student Record)
16 30. 安省学生总成绩表(Ontario Student Transcript)
17 31. 已满十八岁的学生

这份新移民指南的目的,是让各位家长和子女对学校的生活有所了解。
安省法律规定,所有学校都要制定政策和措施,保护学生、帮助他们完成学业。政策有些是由单个学校自行决定,有些则由地区教育局和省教育厅规定。
指南的其它几章会告诉家长,如何协助子女对高中课程作出明智的选择。本指南建议家长和子女,在选科之前一起订出长期的学业计划(an education plan)。有了计划才可以目标明确(读大学、大专、学徒课程还是直接就业)。家长也可以请学校的老师或辅导员(the guidance counsellor)提供协助。

1. 平等对待所有学生(Equity for All Students)
对所有学生,学校都应该尊重其人格、理解包容。学校的”平等”政策确保对所有学生都应该一视同仁,无论种族、出生地、族裔、国籍,宗教、性别、性取向、残疾或家庭状况,都不该影响他们的学业成功。
2. 家长担心的问题 (Dealing with Concerns)
假如家长认为子女受到不公平的对待,和学校教职工沟通是非常重要的。通常第一步是向有关的老师反映,然后再找校长(the principal)或副校长(the vice-principal)。如果问题仍然未能解决,你可以向校监(the School Superintendent) 反映。
有关家长权利的详细资料,可以到安省人权委员会(Ontario Human Rights Commission)的网址查询:http://www.ohrc.on.ca
3. 如果学生可能受到伤害,如何举报(Reporting Possible Harm to Students)
当老师发觉十六岁以下的学生有遭到虐待、凌辱、遗弃的迹象,或者学生告诉老师他们家里人对他们不好,法律规定老师必须要通知儿童保护机构(这个机构称为儿童援助会-Children’s Aid Society ,或者是家庭与儿童服务中心-Family and Children’s Services )。
如果上述事情发生在满十六岁的学生的身上,学校也可能会报警。如果家长或学生怀疑有人遭到虐待、遗弃,也应该报告老师或通知儿童保护机构。有关未满十六岁的学生的信息,请浏览下列网址:
http://www.oacas.org
本指南专为新移民介绍安省高中制度,一共有六章,本章是其中之一。每一章都为家长如何辅导子女完成高中提供了资料和建议。这份免费资料有十八种语言,登在网上:www.settlement.org/edguide 。详情请参阅本章末尾。
4. 包容其它宗教节日和信仰 (Holy Days and Religious Accommodation)
如果学生和家长要求,学校会尽量提供方便、让学生奉行他们的宗教习俗,包括宗教节日、服饰和祈祷仪式。
家长最好事先与老师、副校长或校长讨论学生的需要。这样可以帮助学校了解每个家庭的独特需要,学校和家长也可以讨论如何因地制宜。
如果学生因宗教节日而不能上学,家长应该提前通知学校。
有关宗教和人权的资料,请浏览下列网址:
http://www.ohrc.on.ca/english/guides/religious-rights.shtml

家园移民 : 2008-05-09#27
回复: 子女教育信息素材收集帖子-冲置顶了

5. 家长委员会(School Council)
每一所学校都有一个由家长、本社区居民和学校教职员组成的顾问委员会, 简称为家长委员会。委员会经常和校长一起讨论学校的政策和计划。学校鼓励所有家长都出席家长委员会的会议、参与讨论。任何家长都有资格成为委员.
?Ρ 向校务处索取家长委员会主席的
姓名、电话号码,主席由一名家
长担任。与主席联系,讨论你可
以如何参加委员会。

?Ρ 出席委员会的会议、了解如何推
行工作,对你感兴趣的校内事务
提出问题、建议和看法。

?Ρ 与家长委员会的主席或成员讨
论,如何帮助学校教职员和其它
家长增进对本族裔和新移民学生
的了解。

?Ρ建立一个家长联络网, 以通知其他家长学校有什么活动,也可以接待欢迎新移民家庭。
建 议

6. 行为守则(Code of Conduct)
学校绝不会容忍虐待、欺压、歧视、恐吓、仇恨性的语言和行为,以及任何形式的暴力。学校有一份行为守则,提倡防患于未然、和平解决纷争。
学校有校规,帮助学生遵守行为守则。老师会将规则解释给学生听。学校通常会把规则张贴在校园里、刊登在学生手册上。
所有参加学校活动的人(包括学生、家长或监护人、义工、老师和其他的学校职员) 都必须遵守这份行为守则。
?Ρ 鼓励子女了解熟悉学校的规则。
?Ρ 提醒子女打架或其它不良行为都会有严重后果。
?Ρ 告诉老师和辅导员,如果对子女的操行有顾虑,立刻通知你。
?Ρ 向学校索取一份行为守则.
?Ρ 如果对行为守则或校规有任何疑问,可以向老师或辅导员查询。


7. 强制性处罚 (Mandatory Consequences)
一旦学生触犯安省政府制订的行为守则条例,学校就必须执行规定的处罚,所以称为强制性处罚。处罚的形式包括口头或书面警告(verbal or written warnings)、放学后留堂(detentions)、短期停学(suspension)、彻底开除出校(outright expulsion from school)。例如一名学生殴打另外一名学生,规定的处罚是停学五天,而且校长一定要报告警察。
在某些情况下,校长有权决定停学的时间长短。如果学生被停学或驱逐出校,学校会通知家长。

些行为会导致停学、开除出校、警察干预?
?Ρ 打架
?Ρ 恐吓
?Ρ 辱骂老师
?Ρ 贩卖毒品、武器
?Ρ 抢劫
?Ρ使用武器伤害他人身体,或扬言要伤害对方
?Ρ 伤害他人身体,导致需要医疗
?Ρ 蓄意破坏
?Ρ 性侵犯
?Ρ 提供含酒精的饮料给未成年者

8. 子女的同学有让你担心的举动 (Concerns About Another Student’s Behaviour)
如果有学生感到,同学对自己的态度和行为与行为守则相违背,应该向老师报告。不应该在没有老师的时候私下解决。例如当学生被另外一名学生打,应该设法离开现场、不要还手,应该立刻报告老师.
课间时,娜地亚在走廊里被数名男生不停地推来撞去.虽然她没有受伤,但感到不胜其扰,想要他们住手。虽然她对他们讲请住手,但是没用,于是就向ESL 老师求助。ESL 老师警告那些男生立刻停止这种行为,否则会受到惩罚。男生们就没有再犯了,虽然后来她听说那些男生对她很气愤,但她用自己的行为清清楚楚告诉周围的人:她懂得如何保护自己。
9. 纠正学生不良行为,家长如何参与 (The Role of Parents in Dealing with Behaviour Problems)
在适应新学校、新语言、新国家的过程当中,学生的举止有些改变是正常的。如果老师认为学生的行为不良、或者与同学的关系有问题,他们会通知家长。
老师会向家长解释学校会怎样帮助学生改善、家长在家可以做些什么。在双方努力合作之下,学生就会明确地知道上学时应该有什么样的态度。
如果家长对上述问题有任何疑问,请向老师或辅导员查询。
10. 适龄者必须上学(School Attendance is Obligatory)
法律规定所有六至十六岁的孩子都必须上学。如果学生长期旷课,教育局会调查旷课原因、通知有关当局.长期缺课的孩子学业容易出问题。
如果学生在较长一段时间内不能上学,家长必须事先通知学校。有时老师会建议学生在缺课期间如何学习。
11. 午餐(Lunch)
学生在中午有午餐时间。这段时间按照学生的课时表而定,所以每天可能不同。大部分学生都自带午餐上学。大部分的学校都没有设施让学生加热午餐。有些学校虽然设有小食部,但只有品种有限的便宜食品。
学生可以在午餐时间离开学校。
寻求帮助(Getting Help)
如果家长有疑问或担心,最好向有关的老师查询。如果是一般性的问题,可以向“英语作为第二语言的特殊教师”(the ESL teacher)或辅导员(the Guidance Counsellor)查询。
英语为第二语言老师(The ESL Teacher) ESL 就是English as Second Language 的简称,就是给英语为第二语言的学生开设的课程,教这种课程的老师就叫ESL 老师。英语为第二语言的老师,除了给移民学生专门教授英语,还可以帮助学生和家长理解他们的选择、毕业后的出路。这些老师在协助新移民学生方面有丰富的知识和经验,懂得怎样帮助他们顺利完成中学教育。
辅导员(The Guidance Counsellor) 辅导员可以提供信息和建议,帮助学生和家长理解如何选科、如何择业、毕业之后有哪些机会。
辅导员也可以帮助学生解决个人的疑难、各种学校里的问题。家长可以向辅导员咨询子女在校的表现。辅导员的丰富经验对学生和家长相当有帮助。
12. 全天缺席或者缺课(Absence from School or a Class)
学生必须每天准时上学,否则缺席情况会被老师记录下来,通知教务处。
电话留言例句:
如果你的子女不能上学,请在当天上学前通知学校。This is ( 父母姓名 ).大部分学校都有留言机,让家长在子女不能上学的时My son, ( 儿子姓名 )候留言通知学校。如果学生连续数天不能上学,请每who is in grade 11, is天都致电通知。留言的时候请说慢一点。sick today 。
有些学校利用录音通知家长。当你不在家,又没有留言装备就可能收不到通知。
除了致电通知之外,许多学校要求学生复课的时候交上有家长签字的便条.


13. 学期制与非学期制学校(Semestered and Non-Semestered Schools)
高中分为学期制及非学期制两种。
在学期制学校,学生通常于九月至一月(第一个学期,学期称为semester)读四科,然后二月至六月(第二个学期)再读四科。老师会在期中、期末发成绩单(The Report Card)。每个期末都有考试。
在非学期制学校,学生由九月至六月全年读八门科目.每年分为三个阶段(阶段称为term)。每个阶段末会发成绩单。在学年结束之前才有考试。有些学校则每个阶段末都要考试。



14. 学校假期(School Holidays)


高中在九月劳动节后第一个星期二开学,直到六月底。十二月最后有两个星期的假期,三月中旬有一个星期的假期。学校在开学的时候会告诉学生确切的日期。
15. 更新地址和电话(Keeping Contact Information Up-to-Date)
家长在为子女报名的时候要留下住宅电话和工作电话,还有紧急情况下可以联络的亲朋好友的姓名和电话。



16. 如果学生在学校生病(If the Student Gets Sick at School)
如果学生在学校生病或发生意外,学校会根据记录上的电话通知家长。如果联系不上,则会通知报名时家长指定的紧急情况联系人(the emergency contact person)。
出现下列情况时,请致电通知学校:
17. 过敏症和健康问题(Allergies or Health Problems) ?Ρ 你的子女因故
不能上学如果你的子女有过敏症(allergies)或其它健康问题,?Ρ 你的电话号码请通知学校。如果学生必须在上学时间服药,家长可能需或地址有所更要家庭医生填一张表格。改
?Ρ 你有任何疑问、担心或建
18. 恶劣天气(Bad Weather) 议
冬天暴风雪严重的时候,学校有时可能要关闭。学校会在当地电台发出通告。如果电台没有通告,则学校照常上课。


19. 防疫注射 (Immunization)
所有十八岁以下学生都必须接受防疫针注射,才可以上学。
公共卫生局(the Public Health Department)会在每一个社区强制执行。有时候需要的所有疫苗学生没有注射完全,或者学校的防疫记录上欠缺某种疫苗,家长就会收到公共卫生局发出的警告信。如果家长置之不理,学校可能不能让学生上学,直至防疫针注射问题得到解决为止。
有关详情,请浏览公共卫生局的网页:
http://www.health.gov.on.ca/english/public/pub/early/immunization.html
20. 午餐活动和课后活动的通知(Announcements About Lunch-Time and After-School Activities)
通常在每天上课之前,学校都会在广播里宣布午餐时间的活动、课后活动、兴趣小组、体育运动,还有如何参加这些活动。如果学生有任何疑问,可以向同学、ESL 老师、辅导员查询。


21. 上学着装和校服(Dressing for School/Uniforms)
每个学校都有“衣着规定”(a dress code),告诉学生哪些衣服上学可以穿。有些衣服是不能够穿着上学的。有些学校要求学生穿校服。体育课上,学校可能要求学生穿运动短裤。每个学校都有男女分开的更衣室。
如果家长或学生因宗教、信仰、风俗习惯等原因,对上学着装有任何疑问,可以向老师或辅导员查询。
22. 夜校和暑假班 (Night School and Summer School) 大部分教育局都举办夜校和暑假班。暑假班通常在七月份开办。主要面向两类学生:一是该学年本课程成绩不好、二是
要转换到另一类课程(例如由“就业类课程”转换到比较难的“大学-大专预备类课
程”)。夜校于晚间上课,学生通常是非全职学生(part-time student)。许多“各国语言课程”(International Languages)也在晚上或周末上课。
暑假和夜校课程通常都开设在同一间学校。
23. 学生手册/学校记事簿(Student Handbook/School Agendas)
有些学校印制了《学生手册》,解释校规、校务和其它重要的资料。学生亦可以利用这份手册记录自己的家庭作业和学习计划。这份手册会在开学的时候发给学生,但有些学校要学生付钱购买。

24. 校外参观学习(Field Trips)
老师会经常安排学生去参观博物馆、剧院或参加社区活动。这些”校外参观学习” 是学校课程中的一个重要环节。学生在出外参观的前后都会学习与该项活动有关的内容。
十八岁以下学生必须事先得到家长的同意才可以参加校外活动。学生会把同意书带回家给家长签字。请家长在收到后尽快签字、交回学校.有些校外参观活动是需要付费的。
25. 课程表和课时表(Student Timetables and Periods)
学生在学年或学期开始的时候,会收到一份个人的课程时间表。各门课的同学可能都不同。学校把每天上课的时间分为不同的节(每一节称为period),每节课从四十至九十分钟不等。
26. 教科书(Textbooks)
学校会在学年或学期开始的时候发给学生教科书。这些教科书是学校的财产,学生必须妥善保存,在学期结束时还给学校。如有遗失或损坏,学生必须赔偿。
27. 储物柜(Lockers)
学校会为学生提供存放书籍、文具和衣服的储物柜。储物柜是学校的公物,通常有专人在每个学期末进行清理。学生不应该将贵重财物放在储物柜里,如果柜内物品被偷窃或损坏,学校不会负责。
学校通常会卖给学生一把规定的锁。有些学校要多个学生合用一个储物柜。

28. 文具(School Supplies) 学校资料
家长可以通过下列途
老师会解释每个科目需用什么文具。大部分科目需要径获取有关课程、政笔记本(a notebook)和活页夹(a binder)。其它文具策和手续的信息:包括横格纸、铅笔、原珠笔等。老师会提供具体的要?Ρ 学校手册或者求,例如购买哪一种计算器。学校议程(the
school
29. 安省学生记录(Ontario Student Record) handbook, or
school agenda) 学生每年级的成绩单、每科的记录等各种教育文件,?Ρ 学校网页都存放在安省学生记录内(Ontario Student Record ? 简?Ρ 教育局网页和称OSR)。老师会参考记录内的文件,去了解学生、印制的刊物安排教学。虽然这份记录是保密的,但家长可以要求查阅。
如果学生转校,学校会把这份记录转到新学校。
30. 安省学生总成绩表(Ontario Student Transcript)
安省学生总成绩表是一份正式记录高中成绩的文件。大学或大专院校会根据成绩表来决定学生是否达到录取资格。如果学生转校,学校会把成绩表送到新的学校,新学校的老师就知道学生原来读过什么科目,将来有什么进步也可以记录进去。
31. 已满十八岁的学生(Students Who are 18 years of Age or Older)
在法律上,年满十八岁的学生都是成年人。学校会把所有资料交给他们,而非家长。如果家长想收到成绩单或老师的电话,必须由学生到校务处(school office)签署授权书。

学校常用词(School Words)
?Ρ留校(Detention)-放学后被留在学校,这是对行为不当的学生的轻微处罚.
?Ρ平等(Equity)-公平对待所有学生的政策,确保每个学生都能完成学业。
?Ρ开除(Expulsion)-把学生永久赶出学校,不准回学校上课。这项惩罚有省政府制定的法律严格监督,学生必须答应遵守某些条件才能返校上学。
?Ρ强制性处罚(Mandatory Consequences)-根据法律必须采取的行动――通常指学校对行为不良的学生所采取的措施。
?Ρ非学期制(Non-Semestered)-学生从每学年开始到结束,一次读八个科目。
?Ρ安省行为守则(Provincial Code of Conduct)-学生举止应当如何,守则里有解释
?Ρ宗教宽容(Religious Accommodation)-学校采取适当措施,允许学生奉行自己家庭的宗教习俗。
?Ρ学期制(Semestered)-学生在上半年读四科,下半年再读另外四科的学校。
?Ρ特殊教育(Special Education)-为有特殊需要学生而设的独特课程。
?Ρ停学(Suspension)-勒令学生不准到学校上课数天或数星期(最多二十一天)。有些行为,学校必须让该生停学。这是对行为不当的学生的严重处罚.

本指南内的?家长?(parents)一词包括监护人、保姆、以及其他家人。高中(Secondary schools)也叫 high schools 或 collegiates。
高中课程的家长指南包括以下六章:
1 1.入学须知 (Starting School)
2 2.学习英语和其它科目(Learning English and other Subjects)
3 3.学校政策和校务规则(School Policies and Procedures)
4 4.选课、拟定学业计划(Choosing Courses and Developing an Education Plan)
5 5.成绩单、家长老师面谈会――学校与家长沟通的两个渠道(Report Cards and Parent Teacher Interviews ? How the School Reports to You)
6 6.家长如何协助子女(Ways that Parents Can Help)

还需要更多信息吗?(Need More Information?)
家长可以采用下列方法得到更多信息:
?Ρ向老师、ESL 老师、辅导员(guidance counsellor)、校长(the principal),副校长(the vice principal)或校务处职员(the school office staff)查询。
?Ρ 联系地区教育局(the school board)或安省教育厅(the Ministry of Education) ,查询他们的政策和校务规则。
?Ρ 安省教育厅的网址是http://www.edu.gov.on.ca/eng/welcome.html
?Ρ 查询自己学区所属的教育局:http://sbinfo.edu.gov.on.ca
?Ρ 安省家长议会(The Ontario Parent Council) 的网址:www.ontarioparentcouncil.org,安省学校辅导员协会(The Ontario School Counsellor’s Association)的网址:www.osca.ca

有关定居安省的资料,包括学习英语、工作就业、寻找住所、医疗服务等等一切,请浏览应有尽有的:www.settlement.org

家园移民 : 2008-05-09#28
回复: 子女教育信息素材收集帖子-冲置顶了

家长指南第四章(PARENT GUIDE FOUR) 选课、拟定学业计划
(Choosing Courses & Developing an Education Plan)
目录
1 1. 学业计划为何如此重要
2 2. 老师如何帮助学生拟定计划
3 3. 家长如何帮助学生拟定计划
4 4. 抵达加拿大的第一年
5 5. 已满十八岁的学生
6 6.选课表格(the Course Selection Form)
7 7. 安省高中文凭
8 8. 修满三十个学分
9 9. 四十小时的社区服务
10 10. 对读写能力的要求
11 11. 毕业条件
12 12. 毕业前退学
13 13. 高中同一课程分为不同类型
14 14. 学术类课程和应用类课程:九年级和十年级的重要区别

1 15. 学术类和应用类课程相互转换
2 16. 十一和十二年级――四种水平的预科、毕业后四种出路
3 17. 过渡类课程
4 18. 预备课程
5 19. 英语为第二语言课程(ESL) 及英语读写课程(ELD)
6 20. 任选类课程(Open Courses)
7 21. 课程编号
8 22. 工作经验课程: 半工半读(Co-op )和安省青年学徒课程(OYAP)
9 23. 高中后的出路:学徒课程、升读大专、大学、私立职业学院
10 24. 各大院校如何录取
11 25. 大专院校
12 26. 学徒课程
13 27. 大学
14 28. 私立职业学院
15 29. 奖学金、助学金和学生贷款

新移民到达加拿大的头几年可能非常艰难。家长忙于找工作、安顿新家。孩子则要结交新朋友,每个人都在适应新环境。如果子女就读高中(9-12 年级,secondary schools),家长还另有一项责任:每年都要协助孩子选择课程。课程种类繁多,选课并不容易,而高中怎么选课,会直接影响学生毕业后的出路。
本章指南会告诉家长,如何协助子女对高中课程作出明智的选择。本指南建议家长和子女,在选科之前一起订出长期的学业计划(an education plan)。
有了计划才可以目标明确(读大学、大专、学徒课程还是直接就业)。家长也可以请学校的老师或辅导员(the guidance counsellor)提供协助。

1. 学业计划为何如此重要(Why an Education Plan is Important)
因为高中的课程选起来种类繁多,所以家长需要帮助子女拟定一份学业计划(an education plan)。有些类型的课程的学分是让学生毕业后直接就业,有些类型的课程则是申请大学、大专所用,有些则让学生成为学徒(apprenticeship)。要选读有些大学或大专类课程,学生必须先修读某些特定的课程,作为准备。学业计划就是在于帮助家长和子女选择课程,可以把兴趣、能力和志向三者结合起来。
学业计划就是一份协议书,列出学生为达到毕业后的目标、选读哪些课程。学生毕业之后可能开始工作,也可能先工作几年再回校进修。学生也可能直接上大学、大专、培训课程。学业计划可以帮助学生选择适当的课程、达成自己的目标,从而确保他们达到大学、大专、职业学院、学徒课程以及其它培训课程的录取要求。
学生在校期间,学业计划可能常常改变,因为人的兴趣、技能和能力都会变化。老师和辅导员可以告诉学生,有哪些就业、学习、和培训的机会。
对仍然要过英语关的学生,学业计划尤其重要,因为他们除了需要读一些目标之内的学科之外,学业计划里还需要包括如何提高自己的英语听、说、读、写。

2. 老师如何帮助学生拟定计划(How Teachers Help Students Plan)
全体学生通常到了十年级都会修读《职业研究》(the Career Studies course)这门课。课上,老师会引导学生学习如何规划自己的未来。老师不会为学生选定任何课程,也不会告诉他们毕业后应该干什么,但是老师会帮助每个学生达到自己的目标。
这种方式的好处,在于让学生逐渐学会自主自立、自负其责。学生培养起自立能力之后,也就为将来做好了准备――无论毕业之后读的是学徒、大学还是大专,到那个时候都应该自负其责了。但是,如果没有家长的参与,学生往往会选择不当,导致毕业后的出路受到限制。
3. 家长如何帮助学生拟定计划(How Parents Can Help Students)
家长也可以在子女制定计划的过程中给予指导。制定计划期间,您可以跟孩子一起做以下几件事:
?Ρ 和孩子探讨他们的兴趣、能力、毕业后的打算。问他们对什么行业感兴趣。如果子女反复修改计划,也不必担心,因为这是选择的过程中的必经阶段。
?Ρ 根据他们的职业计划,对他们目前需要修读的课程进行讨论、核对,看毕业之后需要的是什么类型的教育和培训。辅导员可以在这方面提供咨询和帮助。
?Ρ 可以参加放学后或午餐时间的校内活动。这些活动有助您的孩子发掘自己真正的兴趣、了解各行各业。同时也是结识朋友、练习英语的好地方。
?Ρ 参加学校有关职业信息的活动,了解各行各业。
?Ρ考虑让孩子参加有助于学习英语的课程和课外活动 (读、写、听、说),以便尽快适应学校生活。选一些注重会话能力的课程。例如在戏剧课上,学生就可以学会如何镇定自若地当众用英语演讲。
?Ρ征求学校辅导员(the guidance counselor)的意见。家长应该经常和辅导员讨论子女的职业和学业计划。
?Ρ 在定期的家长老师面谈会(parent-teacher interviews)上面,找老师讨论子女的成绩单和老师的评语。
?Ρ 与其他家长和学生沟通,听一听他们的计划和毕业后的择业情况。

建 议很多网页提供了择业信息,例如下列三个:
www.edu.gov.ca/prospects/index.html, www.edu.gov.ca/eng/career/, www.careercrusing.com

阮芝的教育计划阮芝十五岁,来到加拿大两年了。她在原居地读了两年英语,现在读高中ESL 英语的C 级(从易到难依次为A-E 级)。虽然她的口语逐渐进步,可是觉得写作仍然比较吃力。阮芝喜欢数学和自然科学,在父母的帮助下,她选了数学和自然科学的学术类型、历史的应用类型。她计划上大学、念工程系。她打算多读一年高中,让自己可以专攻几科、提高英语写作能力。她希望在十一年级多读几门大学预备课程。为了加强会话能力,她参加了课
余戏剧班,周末还到社区中心做义工。
4. 抵达加拿大的第一年(The First Year in Canada)
第一年初来乍到,很难计划将来。许多学生需要时间先逐渐适应,再考虑自己的将来。
学生在第一年会得到许多重要信息:
a) 学生能渐渐了解整个学校和课程,发现自己最喜欢科目。
b) 家长也可以从老师那里得到对子女的评价,英语听说读写各方面如
何,以及其它科目的成绩,还有子女对新学校适应得如何。
c) 家长还可以和其他家长讨论如何选课、子女毕业后有哪些出路。
d) 学校会决定学生移民前的教育可算多少个学分。

5. 已满十八岁的学生(Students Who are 18 years of Age or Older)
在法律上,满十八岁的学生都是成年人。学校会把所有资料交给他们而不是家长。如果家长想收到成绩单或老师的电话,必须由学生到校务处签署同意书。
6. 选课表格(The Course Selection Form)
通常在每个学年的中间,一般在二月,学生和家长要选择下学年或下学期的课程。学生要把所选的课程填在”选课表格”上面。学生和家长都要在表格上签名(如果学生未满十八岁的话)。家长的签字是表示同意子女所选的课程。
寻求帮助(Getting Help)
如果家长有疑问或担心,最好向有关老师查询。如果是一般性的问题,可以向“英语作为第二语言的特殊教师”(the ESL teacher)或辅导员查询。
英语为第二语言老师(The ESL Teacher) ESL 是English as Second Language 的简称,就是那些给英语为第二语言的学生开设的课程,教这种课程的老师就叫ESL 老师。ESL 老师,除了专门给移民学生教授英语,还可以帮助学生和家长理解他们的选择、毕业后的出路。这些老师在协助新移民学生方面有丰富的知识和经验,懂得怎样帮助他们顺利完成高中学业。
辅导员(The Guidance Counsellor) 辅导员可以提供信息和建议,帮助学生和家长理解如何选课、如何择业、毕业之后有哪些机会。
辅导员也可以帮助学生解决个人的疑难、学校里的各种问题。家长可以向辅导员咨询子女在校的表现。辅导员的丰富经验对学生和家长相当有帮助。
7. 安省高中文凭 ? 简称OSSD (The Ontario Secondary School Diploma - OSSD)
所有高中毕业的学生都会获得安省高中文凭。
无论学生在高中修读什么课程,毕业时都会得到相同的文凭,例如高中毕业后要升读大学也好,要直接就业也好,获得的高中文凭都是一样的。
高中之后要继续进修高等教育、或者参加培训课程,都必需有高中文凭。参加学徒课程、读大专、念大学,具体要求各不相同。详情可向辅导员查询。
8. 修满三十个学分(Earning Thirty Credits) 加拿大高中是学分制。学生需要修满三十个学分才可以毕业,学生每读完一门课程(110 小时)就会得到一个学分。学生必须得到50%以上的分数才算及格。三十个学分当中的十八个是必修课(compulsory),其余十二个学分是选修科(optional),学生可以自己选择。学校有一份课程表,列出每个年级的可选的课程。
什么情况下给学校打电话?
?? 子女因任何原因缺席
?? 你的电话号码、地址改了
?? 你有问题、顾虑和建议


虽然理论上允许学生每年修八个学分、四年读完高中课程,但是许多学生都会多读一年或一个学期。一来可以让他们每年少读几个学分,二来可以广泛探索对其它课程的兴趣。有些学生则会多读额外的学分。在一般的高中,学生可以一直上到满二十一岁。
十八个必修学分中的四个必须是英语课程,但这四个里有三个可以是ESL 英语(ESL)或ELD 英语(ELD)。不过至少要有一个是十一或十二年级的常规班英语
(regular English)。
在某些情况下,学校会容许学生用另外一门必修课去代替另一门必修课,例如学生可以多读一门英语学分代替法语学分。这种替换最多可以有三个学分。详情可向辅导员查询。
如果有一门不及格,学生要在下一学年重读此门课程,而其它正常课程照常进行。
9. 四十小时的社区服务(Forty Hours Community Service)
除了三十个学分之外,所有学生在高中毕业前都必须做满四十小时的社区服务工作。社区服务可以培养学生社区服务的例子:对社区的责任感。这四十个小时可以高中期间任何时?Ρ 在社区中心、图书馆候完成,但必须是在课外时间。或宗教组织做义工
?Ρ 为儿童做体育教练
社区服务需要学生和家长主动寻找、自行安排。辅导?Ρ 参加慈善筹款活动员会解释如何寻找适当的社区服务、如何向学校申请批准。
10. 对读写能力的要求(The Literacy Requirement)
学生要毕业,在九年级结束之前,就必须具备一定的读写能力。
衡量的标准就是通过《安省高中读写测验》(the Ontario Secondary School Literacy Test, 简称OSSLT)。不过,特殊情况下,测验不及格的学生可以修读《安省高中读写课程》(Ontario Secondary School Literacy Course, 简称OSSLC)。
安省高中读写测验(OSSLT) 学生可以在十年级参加第一次读写测验。不及格的学生可以补考,在十二年级结束之前有至少三次考试机会。测验分为两部分(阅读、写作),如果二者有一及格的话,只需要补考不及格的那一部分。
全省所有高中都在十月同时举行这个测验.
每名学生都会收到一份个人成绩单(Individual Student Report ? 简称ISR)。成绩单注明了学生的安省高中读写测验是否及格。不及格的学生会收到一份详细资料,帮助学生了解还需要学什么、如何才可以及格。学校有时候会为刚抵达加拿大或初学英语的学生另作安排。详情请向辅导员查询。
读写测验的结果对学生那年的成绩没有影响。
《安省高中读写课程》(OSSLC) 有些情况下,测验不及格的学生可以修读《安省高中读写课程》(OSSLC)。这门课为学生提供更多的练习,帮助他们达到九年级学生应有的读写水平。完成这门课会得到一个学分。
学生的成绩单只列明学生是否达到了读写的要求,不会标出学生考了几次读写测验, 或者学生是通过了读写测验还是修读了《安省高中读写课程》。
11. 毕业条件(Graduation Requirements)
要毕业、取得安省高中文凭,需要读的必修和选修课程如下。
所有学生都必须修读的课程:四门英语(其中三门可以是ESL 英语或ELD 英语) 三门数学(至少一门是11 或12 年级) 两门自然科学(science) 一门法语一门加拿大历史 一门加拿大地理一门美术(Arts) 一门健康和体育半门公民(Civics) 半门职业研究(Career Studies)
从下列任选一门:□ 一门额外的英语□ 一门英法以外的语言□ 一门社会科学/人文学(Social Science/Humanities) □ 一门额外的《加拿大与世界研究》 从下列任选一门:□ 商业研究□ 一门额外的健康和体育□ 一门额外的美术 从下列任选一门:□ 一门额外的自然科学(11 或12 年级) □ 科技教育(Technological Education)
十二个自选学分 -每所学校的自选课程都稍有不同
通过高中读写测验
完成四十小时的社区服务

12. 毕业前退学(Leaving School Before Graduation)
如果学生没有拿到安省高中文凭(a diploma)所规定的三十个学分,可以得到一张证书(a certificate):
1 1.取得了十四个学分的学生会得到一张《安省高中证书》(Ontario Secondary School Certificate).
2 2.不够十四个学分就退学的学生可以得到一张《成绩证明》(Certificate of Accomplishment).

13. 高中同一课程分为不同种类(Different Types of Secondary School Courses)
刚刚到达加拿大的学生,选起课来可能很难。他们正在学英语,还需要时间去考虑自己的兴趣和职业计划。选课的时候既要给自己多留出路、不要选死了,又要脚踏实地、量力而行。ESL 老师和辅导员可以帮助学生作出恰如其分的决定。
家长和学生在拟定学业计划的时候,应该放宽眼界,考虑各类科目。每种科目和课程里,学生都会学到不同的东西,读哪一科都可以得到文凭,但所选的课程对学生在毕业之后的出路会大有影响。
课程的种类包括:
?⒐学术类(Academic)?⒐应用类(Applied)?⒐大学预科类(University
Preparation)
?⒐大学-大专预科类(University/College Preparation)
?⒐大专预科类(Colloge Preparation) ?⒐就业预科类(Workplace
Preparation)?⒐过渡类(Transfer)?⒐任选类(Open)?⒐基本类(Essential)14. 学术类课程和应用类课程:九年级和十年级的重要区别(Academic or Applied Courses: Important Differences in Grade 9 and 10)
学生在九、十年级可以选读英语、数学、自然科学、历史、地理和法语,这些课程可以是学术类(academic courses),也可以是应用类(applied courses)。九、十年级的学生可以只选学术类或应用类,或者学术和应用两类同时选。
学术类课程:打算毕业后读大学、大专的学生,在11、12 年级就要修“大学预科
类”的课(University Preparation Courses),那么十年级的课就需要先选学术类的,作为基础。11、12 年纪的部分“大学-大专预科类”的课程(University/College Preparation courses)也需要同样的基础。
应用类课程:选了应用类课程,到11、12 年级就能选读的课程,就只有一部分“大学-大专预科类”的课程(University/College Preparation courses)、外加所有的“大专预科类”(College Preparation Courses)、所有的“就业预科类”(Workplace Preparation)。选应用课程的学生是打算进入大专、读学徒课程或直接就业。

基本类/地方自编类:学校可能为有困难的9、10 年级学生提供特殊的英语、数学和自然科学课。学生每读完一科会得到一个学分。
9、10 年级学术类课程和应用类课程的比较学生学习些什么:
学术类 应用类
z 基本概念和附加的内容 z 基本概念
z 更注重从理论上理解概念 z 更注重用概念的实际运用
z 需要更强的阅读能力和自学能力 z 较少注重自学能力
z 为11、12 年级选读所有“大学预科?⒐ 为11、12 年级选读部分“大学-大
学类”和一部分“大学-大专预科 专预科类”、所有“大专预科
类”做准备 类”、所有“职业预科类”做准备

15. 学术类和应用类课程之间的转换(Changing Between Academic and Applied)
如果学生想把某一门课程在学术类和应用类之间相互转换,可能需要做些额外的功课。例如,某生读完九年级应用类,想在十年级把同一科目转为学术类,学校就会建议该生自己多做功课,时间多达三十个小时。(这三十小时的额外功课称为搭桥课,就是 crossover course。)
15. 十一和十二年级---四种水平的预科、毕业后四种出路(Grades 11 and 12 ? Four Preparation Levels)
到了十一和十二年级,绝大多数学生的计划都已经相当明朗清晰了,于是所开课程也就是针对毕业后的不同出路。
十一和十二年级的预科课程
大学预科(University Preparation, 用U表示):这些课程符合大学录取条件。这些课程也可以用来申请大专院校、学徒课程、就业课程.
大学/大专预科(University / College Preparation ,用M表示):这些课程符合某些大学和大部分大专的录取条件。这类课程大多也可以用来申请学徒课程。
大专预科(College Preparation,用C表示):这些课程符合大部分大专的录取条件。大部分也可以用来申请学徒课程。
职业预科(Workplace Preparation,用E表示):这些课程是用来直接就业,也可以申请某些学徒课程、其它培训课程。
17. 过渡类课程(Transfer Courses)
10-12 年级的学生,如果想在下一年改换课程的类型,就必须先读过渡课程(a Transfer course)、补上没有读过的内容。例如大部分十一年级的“大学-大专预科类”,都要求首先修完十年级的学术类(请参阅下文)。如果一个学生只读了十年级“应用类数学”,现在要读十一年级的“大学-大专预科类数学”,就必须先修一个过渡课程、或者先读完十年级的“学术类数学”。
修读过渡课程有各种限制,例如只能在晚上、暑假或网上。详情可向辅导员查询。
18. 预备课程(Prerequisite Courses)
某些课程,必须先读完同一科目的低一年级、或者是类似科目的低一年级,才有资格选。这个低一级的科目就称为预备课程(Prerequisite Courses) 。例如,学生为了将来能读十二年级的“大专科技数学”(Mathematics for Community College Technology),在十一年级就必须先读“实用机能”(Functions,这是一个“大学-大专类”的课程),而要读“实用机能“,学生又必须先有十年级的“学术类数学
“。各科各级环环相扣,须有长期打算。

约翰的计划约翰打算在高中毕业后到大专读音乐系。他知道要申请必须要有十二年级的“大学-大专类音乐“。而要读十二年级音乐,又必须先读十一年级的音乐。
19. 英语为第二语言课程和英语识字课程(ESL and ELD Courses)
除了一般的课程之外,正在学英语的学生还可以读英语为第二语言课程(ESL)、或者英语识字课程(ELD)。
有些学校为ESL 学生对一些必修课进行特殊处理,提供ESL 历史、ESL 自然科学、ESL 职业研究、ESL 公民课等等。在学生的成绩单上这些课程通常和加拿大本地学生的常规课程一样,只列出课程名称,而不会显示出是ESL 课程。
20. 任选类课程(Open Courses)
任选课上教的是普通常用的技能和知识,例如美术、体育和戏剧等。任选课不分年级。所有ESL 和ELD 课程都是任选类。
21. 课程编号(Course Codes)
在学校档案里,每一门课程都有一个五位编号,显示课程的名称、年级和类型。有些教育局的课程编号是六位。
9、10 年级的课程编号例子:11、12 年级的例子:
MFM1P ? Foundations of Mathematics,
MDM4U ? Mathematics of Data Grade 9 (九年级的基础数学课程)
Management( 十二年级的数学数据管理课)
“MFM” 表示课程的名称.1 代表年级(1、2、3、4 分别是九、十、
“MDM”表示课程的名称。十一、十二年级)
4 代表年级(4 是十二年级) P 表示应用类。
U 表示大学预科类。22. 工作经验课程:半工半读(Co-op)和安省青年学徒课程(简称OYAP)
(Work Experience Programs: Co-op and Ontario Youth Apprenticeship Program
(OYAP))

参加这个课程的学生,利用全日或半日到实际的工作场合、在自己选择的行业实习。学生在拿学分的同时,又能积累实际的工作经验。
半工半读教育课程(Co-op):学校安排学生利用部分上学时间到校外工作。半工半读课程让学生有机会在实际的环境中、运用学到的技能和知识。例如有意当律师的学生,就每星期数次到律师行去实习半天。
安省青年学徒课程(OYAP):十一、十二年级的学生,如果满了十六岁、拿够了十六个学分,就可以参加学徒课程或职业培训。这些参加学徒训练的学生,实习可以算学分,所以是一边完成高中教育,一边就已经开始了学徒培训。有关详情,请向辅导员、半工半读课程老师查询.
美霖的计划美霖从小就喜欢动物。在上七、八年级自然科学的时候,她特别喜欢生物学和生态学。在九、十年级,她选的课程是应用类和学术类混合的。她与家长、辅导员讨论之后,决定在十一年级参加半工半读课程。她实习的地方是动物园(在学校附近)。她和动物园的职工一起照顾动物。她现在修的半工半读课程可以得到两个学分,她希望最后能够到大专去读动物科学专业。23. 高中后的出路:学徒课程、升读大专、大学、私立职业学院
(After High School: Getting into Apprenticeship, Community College, University or Private Career College)
辅导员会提供资料,列出修读这些课程的所需条件。自己也可以去浏览“职业入门”(the Career Gateway)的网页:
http://www.edu.gov.on.ca/eng/career

家园移民 : 2008-05-09#29
回复: 子女教育信息素材收集帖子-冲置顶了

24. 各大院校如何录取(How Student are Chosen)
大学、大专和各类培训课程,都需要学生读过某些指定的高中课程才能申请。如果名额有限,高校会优先录取成绩较好、有其它优势的学生。某些大学专业只录取平均分高于85-90%的高中毕业生,另一些大学专业可能只需要65-70%的平均分。大学录取主要是按照十二年级的成绩。
许多大学和大专会为正在学英语的学生提供协助。具体资料可以在他们的网页上找到。

25. 大专院校(Community College)
申请大专的“警察入门”
大专(Community College)又称为“应用文科 (Police Foundations)这门课程
和科技学院”(Colleges of Applied Arts 的条件:
and Technology),提供的课程有一年取得证 z 安省高中文凭
书的(证书称为certificate),也有二至三 z 十二年级的英语
年取得文凭(文凭称为diploma)的。现在有 ENMG4C(末尾的C 代
些大专院校开始提供四年的本科学位课程,称 表College,即大专),或
为“应用学位”(Applied Degree)。这些本科 者或者 ENG4U(末尾的
学位,部分大学的研究生院(university U 代表university,即大
graduate schools)是承认的。 学)
z 最好修过:数学:任何
大部分大专的院系录取的时候,要求必须有安 十二年级大专预科类的
省高中毕业文凭。 数学(MAP4C 或
MCT4C),或者大学预科
大专院校的有些专业,竞争很激烈。如果名额 类的数学。
有限,就只录取成绩更高的学生。申请安省大 z 最好能使用键盘,?得
专院校,需要哪些课程、分数,录取政策如 基本的计算机应用和计
何,都请到辅导员办事处索取《安省大专指 算机文字处理。
南》(the Ontario College Guide),或到网
上:www.ontariocolleges.ca

26. 学徒课程Apprenticeship 有些学生喜欢动手、善于实干,学徒课程就可以为他们提供全职工作训练。
这些训练课程所培训的行业,对技术水平、判断力和创造力的要求都相当高,而且薪金不菲。学生在积累经验的同时可以领取工资,技术越提高,工资就越多。
学徒课程中大约90%的时间都是由雇主在工作地点直接提供训练。其余10%在课堂学理论,上课通常是在大专院校里,或者其它核准的培训机构。
有关学徒课程的资料,请浏览下列网址:

http://edu.giv.on.ca/eng/training/apprenticeship/appren.html
27. 大学(University)
所有大学都提供四年的本科学位(undergraduate degree),个别大学也提供三年的课程。许多大学还有硕士、博士学位课程。虽然申请大学的最低要求是十二年级的“大学预科类”和“大学-大专预科类”加起来达到6 个学分(就是六门),然而,其实大部分大学课程都有自己附加的录取条件。如果申请的学生太多,会被大学优先录取的学生,选修过“大学预科类”课程,并且成绩优异、有其它优势。
有关大学录取要求的课程和分数,请浏览下列网址:www.ouac.on.ca。进入之后点击 INFO。
有些大学和大专院校联合提供一些课程(例如护理,就是nursing),学生念两年大专之后,再转到大学念两年。
28. 私立职业学院(Private Career College)
获得注册的私立职业学院,提供的培训注重实践。通常培训的时间比大专院校的课程短。由于学院是私人所有,没有政府拨款,所以学费通常比大专院校昂贵。他们一般只要求学生有安省高中毕业文凭。
29. 奖学金、助学金和学生贷款(Scholarships, Bursaries and Loans)
学生可以利用特设的奖学金(scholarships)、助学金(bursaries)和贷款(loans),减少大学、大专或培训的学费。
尽管可以向辅导员查询申请资格如何、怎样申请,但是这件事情还是要学生和家长自己负责。最好在申请大学、大专的时候一起申请这些资助。
有关奖学金的详情,请浏览下列网址:
http://www.osca.ca/finaid.html
省政府(the provincial goverment)和联邦政府(the federal government)都提供学生贷款。详情请浏览下列网址:
http://osap.gov.on.ca/eng/not_secure/general.html
(School Words)
?Ρ学术类课程(Academic Courses)-九、十年级修读的课程,修读之后给学生的选择面最广,可以升读大学、大专、学徒课程、职业培训、直接就业。
?Ρ录取条件(Admission Requirements)-申请大学、大专、培训课程必须具备条件。
?Ρ应用类课程(Applied Courses)-九、十年级修读的课程,这类课程给学生的选择面包括升读大专、学徒课程、职业培训、直接就业。
?Ρ学徒课程(Apprenticeship)-想成为技术工人(a skilled trade) 的学生,可以在学徒课程里培养动手技能、实地学习。学徒课程为学生在上学时、甚至毕业后都提供实践经验和训练。
?Ρ职业途径(Career Path)-要进入某种职业、某一职位之前,需要通过的一系列步骤和程序。
?Ρ半工半读课程(Co-op)-由学校安排、学生利用部分上课时间到一些雇主那里实习学到的技能和知识。
?Ρ学分(Credits)-学生每读完一门课程(110 小时) 会得到一个学分。大部分课程都是一个学分,学生至少要读满三十个学分才可以毕业。有些学分是必修的,有些则可以选择。
?Ρ基本类课程(Essential Courses)-为了帮助学习有困难的学生,九、十年级的数学、自然科学、英语经过调整修改,就称为基本类课程。
?Ρ毕业条件(Graduation Requirements )学生想取得安省高中毕业文凭必须具备哪些条件。
?Ρ非学期制(Non-Semestered)-有些学校里,学生从学年开始到结束一次读完八个课程。
?Ρ任选类课程(Open Courses)-这些课程教授基本、普遍的技能和知识,有学分,例如美术、音乐、体育。
?Ρ预备课程(Prerequisite)-学生必须先修一定的课程,下一年纪才能升读某一课程。例如要读十一年级的“物理大学预科“,就必须先读十年级的学术类自然科学。
?Ρ学期制(Semestered)-学生在上下半年、各读四科的学校。
?Ρ学生的总成绩档案(Transcript)-一份正式记录,包括学生的学习进度、所读课程、所在年级。学校会在学生申请大学、大专、培训课程的时候把它寄到申请的学校。

本指南内的?家长?(parents)一词包括监护人、保姆、以及其他家人。高中(Secondary schools)也叫 high schools 或 collegiates。
高中课程的家长指南包括以下六章:
1 1.入学须知 (Starting School)
2 2.学习英语和其它科目(Learning English and other Subjects)
3 3.学校政策和校务规则(School Policies and Procedures)
4 4.选课、拟定学业计划(Choosing Courses and Developing an Education Plan)
5 5.成绩单、家长老师面谈会――学校与家长沟通的两个渠道(Report Cards and Parent Teacher Interviews ? How the School Reports to You)
6 6.家长如何协助子女(Ways that Parents Can Help)

还需要更多信息吗?(Need More Information?)
家长可以采用下列方法得到更多信息:
?Ρ向老师、ESL 老师、辅导员(guidance counsellor)、校长(the principal),副校长(the vice principal)或校务处职员(the school office staff)查询。
?Ρ 联系地区教育局(the school board)或安省教育厅(the Ministry of Education) ,查询他们的政策和校务规则。
?Ρ 安省教育厅的网址是http://www.edu.gov.on.ca/eng/welcome.html
?Ρ 查询自己学区所属的教育局:http://sbinfo.edu.gov.on.ca
?Ρ 安省家长议会(The Ontario Parent Council) 的网址:www.ontarioparentcouncil.org,安省学校辅导员协会(The Ontario School Counsellor’s Association)的网址:www.osca.ca

有关定居安省的资料,包括学习英语、工作就业、寻找住所、医疗服务等等一切,请浏览应有尽有的:www.settlement.org


家长指南第五章(PARENT GUIDE FIVE)
成绩单、家长老师面谈会??学校与家长沟通的两个渠道
(Report Cards and Parent-Teacher Interviews ? How Teachers Report to You)
目录
1 1. 成绩单(The Report Card)
2 2. 成绩单上ESL 和ELD 课程一栏
3 3. 老师如何评定学生成绩
4 4. 在发成绩单之间如何了解子女的学习进度
5 5. 学习能力
6 6. 家长和学生的意见回条
7 7. 已满十八岁的学生

8. 家长老师面谈会(Parent-Teacher Interview)
9. 向老师提问家长忙于找工作、安顿新家。孩子则要结交新朋友,每个人都在适应新环境。如果子女就读高中(9-12 年级,secondary schools),家长还另有一项责任:每年都要协助孩子选择课程。课程种类繁多,选课并不容易,而高中怎么选课,会直接影响学生毕业后的出路。
指南的其它几章会告诉家长,如何协助子女对高中课程作出明智的选择。本指南建议家长和子女,在选科之前一起订出长期的学业计划(an education plan)。
有了计划才可以目标明确(读大学、大专、学徒课程还是直接就业)。家长也可以请学校的老师或辅导员(the guidance counsellor)提供协助。
本指南专为新移民介绍安省高中制度,一共有六章,本章是其中之一。每一章都为家长如何辅导子女完成高中提供了资料和建议。这份免费资料有十八种语言,登在网上:www.settlement.org/edguide 。详情请参阅本章末尾。
成绩单(The Report Card) 按照安省教学大纲(the Ontario Curriculum)的规定,安省成绩单用于报告学生每科的成
绩。(如果学生已满十八岁,就是独立的成年人,一定要经过学生签字同意,学校才会将成绩单直接寄给家长。请参看总第58 页)。每个学年里,学校会给家长邮寄几份成绩单。邮寄时间要看该校是学期制还是非学期制。
(欲知详情,请参看指南最后的”学校常用词”一栏)。在非学期制的学校,家长每年会收到三次成绩单。还会在第一学期收到期中成绩单。在学期制的学校,家长会在每个学期期中和期末收到成绩单,一年四次。分数是百分制,体现出:
a) 学生对每科所要求的能力和知识掌握了多少b) 学生的成绩和安省标准(the provincial standard)相比如何,安省标准是用来比较全省学校和学生的。
成绩级别 说明 安省标准 分数:百分制
第4 级 优 超过安省标准:有学分 80-100%
第3 级 良 达到安省标准:有学分 70-79%
第2 级 中 接近安省标准:有学分 60-69%
第1 级 及格 低于安省标准:有学分 50-59%
不及格 没有学分 50%以下

成绩单刊登在下列网址:
http://www.edu.gov.on.ca/eng/document/forms/report/1998/report98.html
第3 级(70-79%)是安省的标准。达到第3、4 级的学生就已准备充分、可以升到高一年级、或选修下一门课程。大多数学生都应该达到第三级。
50%以下意味着该学生不及格,得不到学分。学生需要重读、转科或接受额外帮助。详情请向老师或辅导员查询。
即使学生的第一次成绩单中有课程不及格,如果在学期或学年结束之前有明显进步,最终成绩仍然有可能及格。
成绩单上的ESL 和ELD 课程一栏(The ESL and ELD Boxes on the Report Card) 如果老师认为学生的英语水平太低,完全不能学习所需知识,就会将科目稍作调整以适应学生的英语水平。老师会在成绩单上的ESL 或ELD 一栏里做记号,老师将会根据调整后的课程情况来给学生评分。

寻求帮助(Getting Help)
如果家长有疑问或担心,最好向有关老师查询。如果是一般性的问题,可以向“英语作为第二语言的特殊教师”(the ESL teacher)或辅导员查询。
英语为第二语言老师(The ESL Teacher) ESL 是English as Second Language 的简称,就是那些给英语为第二语言的学生开设的课程,教这种课程的老师就叫ESL 老师。ESL 老师,除了专门给移民学生教授英语,还可以帮助学生和家长理解他们的选择、毕业后的出路。这些老师在协助新移民学生方面有丰富的知识和经验,懂得怎样帮助他们顺利完成高中学业。
辅导员(The Guidance Counsellor) 辅导员可以提供信息和建议,帮助学生和家长理解如何选课、如何择业、毕业之后有哪些机会。
辅导员也可以帮助学生解决个人的疑难、学校里的各种问题。家长可以向辅导员咨询子女在校的表现。辅导员的丰富经验对学生和家长相当有帮助。
3 老师如何评定学生分数(How Teachers Determine The Student’s Marks) 每科总成绩中的70%是平时作文、平时测验、以及其它平时的评估。其余的30%是期末考试、期末论文、及其它期末评估。
4 在发成绩单之间如何了解子女的学习进度(How to Follow The Student’s Progress Between Report Cards)
家长在收到成绩单之间,如何了解子女的学业?建议如下:
经常与子女讨论学校的事情。家长可以感觉到子女在哪些方面一帆风顺;哪些方面遇到困难。抽查子女的功课和测验成绩。态度要积极正面。表扬比批评更能够鼓励子女奋发向上。如有任何担心和顾虑,应向老师或辅导员提出。

学习能力(Learning Skills)
从成绩单上家长可以了解到子女是否具备六种重要的学习能力:
1 1. 自觉自律、独立学习
2 2. 团队精神(与其他学生合作)
3 3. 组织能力、有条不紊
4 4. 自发性、主动性
5 5. 学习习惯
6 6. 完成作业、善始善终


家长和学生的意见回条(The Parent and Student Response Forms)
成绩单还附有一份意见回条。家长与子女讨论过成绩单上的 家长在意见回条上的
分数和评语之后,必须在这份回条上签名,然后交回学校。 意见样本:
有些家长还会在回条上写下意见。这份回条会与学生的成绩 我们对他在数学上的
单一起,保存在安省学生记录(Ontario Student Record ? 简称 进步深感欣慰。我们
OSR)里面。 的儿子几乎每晚都在
阅读英语或西班牙语
学校鼓励学生在回条上写上自己的评注。这可以激发他们对 的书籍。
自我学业的责任感。

7 已满十八岁的学生(Students Who are 18 years of Age or Older)
在法律上,年满十八岁的学生都是成年人。学校会把所有资料交给他们,而非家长。如果家长想收到成绩单或老师的电话,必须由学生到校务处(school office)签署授权书。
8 家长老师面谈会(Parent-Teacher Interviews)
面谈的时候,家长可以与每一位老师交谈子女的在校表现。面谈时间为五至十分钟,通常是在教室、体育馆、或食堂。如果家长提前要求,许多学校都可以提供翻译(an interpreter)。
两次面谈各在秋季和春季举行。
老师在面谈时会与家长讨论学生的成绩单、如何评分。有时也会谈到老师如何帮助该生, 家长可以如何在家帮助子女。
面谈时,老师和家长可能会约定再次见面、彼此通电话。即使子女的成绩不错,也应与老师保持联络。告诉老师无论他们有任何担心、或者子女表现优异,都希望能够通知你。
z 在与老师面谈之前,和子女谈谈他们在学校喜欢什么、有什么困难。讨论一
下,成绩单上的成绩对选课和毕业后的计划有何影响。
z 想一想要向老师提出什么问题。最好把问题写下来,面谈时带上,以免忘记。
z 如果需要翻译(an interpreter),应事先向学校查询。如果学校不能提供翻
译,请带上懂英语的家人或朋友帮忙翻译(最好不要叫子女翻译)。
z 回家后与子女讨论面谈的情况。多说老师称赞他们的长处,对需要改进的地
方,要说得清楚具体。


10. 向老师提问(Question to ask the Teacher)
以下是家长向老师提出的常见问题。
z 学生的英语有哪些长处和短处?z 学生的家庭作业应该用多少时间?z 学生是否对某科特别感兴趣?z 学生与其他同学关系如何?z 我在家里应该怎样帮助他?z 学生在下一年应该选修什么课程?

?Ρ教学大纲(Curriculum )正式文件,描述每门课程及其学生必须掌握的内容。
?Ρ成绩单(Report Card)-反映学生学业概况的正式报告。报告是由每科老师写给学生和家长参阅的。
?Ρ成绩单的意见回条(Report Card Response Form)-附在成绩单上、寄给家长的一份回条。家长应在回条上签名并交回学校。
?Ρ学期制(Semestered)-学生在上下半年、各读四科的学校。
?Ρ非学期制(Non-Semestered)-有些学校里,学生从学年开始到结束一次读完八个科目。
?Ρ安省标准(Provincial Standard)-用百分制表示的分数,达到70-79%的学生就已准备充分,可以升读下一年或选读下一科。
?Ρ安省学生记录(Ontario Student Record ? 简称OSR)-学生在校期间的成绩单、所学科目、其它教育文件都存在这个记录里。

本指南内的?家长?(parents)一词包括监护人、保姆、以及其他家人。高中(Secondary schools)也叫 high schools 或 collegiates 。
高中课程的家长指南包括以下六章:
1 1.入学须知 (Starting School)
2 2.学习英语和其它科目(Learning English and other Subjects)
3 3.学校政策和校务规则(School Policies and Procedures)
4 4.选课、拟定学业计划(Choosing Courses and Developing an Education Plan)
5 5.成绩单、家长老师面谈会――学校与家长沟通的两个渠道(Report Cards and Parent Teacher Interviews ? How the School Reports to You)
6 6.家长如何协助子女(Ways that Parents Can Help)

家园移民 : 2008-05-09#30
回复: 子女教育信息素材收集帖子-冲置顶了

还需要更多信息吗?(Need More Information?)
家长可以采用下列方法得到更多信息:
?Ρ向老师、ESL 老师、辅导员(guidance counsellor)、校长(the principal ),副校长(the vice principal)或校务处职员(the school office staff)查询。
?Ρ 联系地区教育局(the school board)或安省教育厅(the Ministry of Education),查询他们的政策和校务规则。
?Ρ 安省教育厅的网址是http://www.edu.gov.on.ca/eng/welcome.html
?Ρ 查询自己学区所属的教育局:http://sbinfo.edu.gov.on.ca
?Ρ 安省家长议会(The Ontario Parent Council) 的网址:www.ontarioparentcouncil.org, 安省学校辅导员协会(The Ontario School Counsellor’s Association) 的网址:www.osca.ca

有关定居安省的资料,包括学习英语、工作就业、寻找住所、医疗服务等等一切,请浏览应有尽有的:www.settlement.org


家长指南第六章(PARENT GUIDE SIX) 家长如何协助子女(Ways That Parents Can Help)
目录
1 1. 多与子女交谈
2 2. 帮助子女的七种方法
3 3. 与老师讨论子女的学业
4 4. 如何与老师和辅导员联系
5 5. 已满十八岁的学生
6 6. 保密
7 7. 学校发来的信件
8 8. 不懂英语也可以帮助子女
9 9. 协助子女做作业
10 10. 解决问题
11 11. 恐吓和骚扰

本指南专为新移民介绍安省高中制度,一共有六章,本章是其中之一。每一章都为家长如何辅导子女完成高中提供了资料和建议。这份免费资料有十八种语言,登在网上:www.settlement.org/edguide 。详情请参阅本章末尾。新移民到达加拿大的头几年可能非常艰难。家长忙于找工作、安顿新家。孩子则要结交新朋友,每个人都在适应新环境。
本指南讲述家长可以如何帮助孩子顺利完成学业。提供的建议包括如何在家里讨论学业、如何和老师联系、留意学生的家庭作业、协助学生解决问题。
如果子女就读高中(9-12 年级,secondary schools),家长还另有一项责任:每年都要协助孩子选择课程。课程种类繁多,选课并不容易,而高中怎么选课,会直接影响学生毕业后的出路。
指南的其它几章会告诉家长,如何协助子女对高中课程作出明智的选择。这几章建议,在选科之前一起订出长期的学业计划(an education plan)。有了计划才可以目标明确(读大学、大专、学徒课程还是直接就业)。家长也可以请学校的老师或辅导员(the guidance counsellor)提供协助1. 多与子女交谈(Talking with Your Son or Daughter)
经常与子女讨论在学校的事情,谈天一来可以帮助家长了解子女的学校生活,二来家长也可以趁机提供建议和支持。交谈之中,家长可以了解子女遇到的困难,从而决定是否需要联系老师或辅导员。
年轻人通常不愿意远离亲友、移民到陌生的国家。在适应新生活的过程中,出现种种喜怒哀乐、兴奋失落、五味杂陈,是很正常的。
适应新的学校也可能给他们增添不少压力。他们可能觉得,自己要和其他同学一样的穿衣打扮、言行举止,既想融入其中,但又格格不入。家长可以借机和子女讨论,对如何适应新生活,提出建议。
z 经常与子女讨论在学校的事情。防患于未然,不要等到出了问题才去处理。
z 询问子女在学校发生了什么事情。鼓励他们讲述学校的生活。聆听他们的顺境
和逆境。
z 家人相聚时,抽时间一起讨论如何适应新生活。向子女解释,重大转变期间,
焦虑不安是很正常的。
z 谈谈哪些以前是困境、而现在好转了。就算是小小的成就也该应庆祝一下,例
如来到加拿大满了两个月。
z 认识子女的朋友。欢迎他们来做客。
z 鼓励子女把心里话讲出来、解释给你听。这实际上对课堂讨论和写作也是一种
很好的练习。



2. 帮助子女的七种方法(Seven Ways to Help Your Son or Daughter)
家长要帮助子女取得好成绩,有如下七种方法。
参加家长老师面谈会(parent-teacher interviews)。与子女讨论如何选课,拟定一个长期的学业计划。经常与子女讨论学业。积极参加学校举行的活动。为子女做功课提供一个舒服合适的地方。协助子女安排做功课的时间。有疑问或者担心的时候,和学校联系。
3. 与老师讨论子女的学业(Talking with the Teacher)
家长和老师交流的时候,家长会了解到老师对学生的期望和要求,家长可以如何在家协助。良好的沟通也帮助老师加深对学生的了解,从而帮助学生更快地地适应新的学校。安省高中鼓励家长积极参与子女的教育,如果有疑问或者担心,向老师查询。

4. 如何与老师或辅导员联系(How to Contact Teacher or the Guidance Counsellor)
老师和辅导员都随时欢迎家长致电查询。
z 致电校务处,留言给老师或辅导员。留下你的姓名、电话、子女姓名、最佳的回电时间。
?⒐如果需要,询问学校是否可以提供翻译(an interpreter),协助你与老师或辅导员沟通。
?⒐请一位懂英语的亲友(最好不要叫子女)陪同你到学校做翻译,或代你打电话到学校。会面几方都一定要把会面的时间弄清楚。
?⒐如果可以的话,使用电话上的三人会谈功能(*71,称为Three Way Calling),这样你、老师或辅导员、翻译(或懂英语的亲友)可以同时在电话上交谈。电话公司会收取小额的服务费。详情可查阅电话簿上三人电话会谈(*71)部份。
寻求帮助(Getting Help)
如果家长有疑问或担心,最好向有关老师查询。如果是一般性的问题,可以向“英语作为第二语言的特殊教师”(the ESL teacher)或辅导员查询。
英语为第二语言老师(The ESL Teacher) ESL 是English as Second Language 的简称,就是那些给英语为第二语言的学生开设的课程,教这种课程的老师就叫ESL 老师。ESL 老师,除了专门给移民学生教授英语,还可以帮助学生和家长理解他们的选择、毕业后的出路。这些老师在协助新移民学生方面有丰富的知识和经验,懂得怎样帮助他们顺利完成高中学业。
辅导员(The Guidance Counsellor) 辅导员可以提供信息和建议,帮助学生和家长理解如何选课、如何择业、毕业之后有哪些机会。
辅导员也可以帮助学生解决个人的疑难、学校里的各种问题。家长可以向辅导员咨询子女在校的表现。辅导员的丰富经验对学生和家长相当有帮助。
5. 已满十八岁的学生(Student Who Are 18 Years of Age or Older)
在法律上,年满十八岁的学生都是成年人。学校会把所有资料交给他们,而非家长。如果家长想收到学生的成绩表或老师的电话,学生必须到校务处(school office)签署授权书。
6. 保密(Confidentiality)
家长和学校之间所有的谈话都会得到保密。除非家长同意,否则学校不会将学生的资料或家庭状况泄露给非学校职员、其他家长或其他人。
7. 学校发来的信件(Letters from the School) 学校经常让学生把信息带回家,包括学校动态、校外旅行的许可书和其他通知。家长一定要留心、每份必看。
提醒子女,有关学校的各种信息你都需要了解。在家里专门找一个的地方,存放所有学校的信息。把重要的学校通知贴在家里。需要交回学校的表格尽快交回去。如有疑问,向校务处职员、老师或辅导员查询。8.不懂英语也可以帮助子女(If You Don’t Speak or Read English, You Can Still Help Your Son or Daghter) 就算家长正在学英语,也一样可以帮助子女。以下是一些建议: 提供一个安静的地方给子女做作业。
帮助子女安排时间做作业。与子女一起定出计划,如何学英语、如何选课、毕业后有何打算。与子女讨论正在学习的内容。要真心实意听他们说些什么。鼓励子女多看英语书籍和母语书籍。通过翻译与学校沟通。9. 协助子女做作业(Helping with Homework)
通过家庭作业,家长可以了解子女的在校情况。学生差不多每晚都该有作业,但是各个年级的作业数量不同。
九年级的家庭作业大约有45 分钟到一小时。到了十二年级,每晚的作业可能最多要两小时。每个学生都应该每晚看书。老师可以告诉你,他们应该有哪些作业。
z 在家为子女安排一个适当的地方用来做作业,光线充足,没有电视或其它干
扰。为他们提供钢笔、圆珠笔、铅笔、橡皮、纸和字典。
z 规定出一个固定的时间,用来做作业,帮助他们学会安排时间。
z 考虑一下,子女与同学一起做作业是否更好。有些学生与同学一起做功课,成
建 绩会比较好。学校有时要学生一起做小组作业。
议 z 询问子女他们做的是什么作业,做完之后拿来一起讨论。
z 如果功课很难,多鼓励他们, 要时常称赞他们的付出的努力。家长的表扬是很
大的鼓励。
z 如果发觉子女的家庭作业太难,时间太长或太短,应该找老师查询。
z 如果学校或社区里有家庭作业小组,鼓励子女参加。

没有作业?
?Ρ 要子女给你看在学校做的功课。
?Ρ如果经常没有家庭作业,联系老师查询。
?Ρ鼓励子女复习最近几天学的东西,为后面的测验和考试预习。
?Ρ 督促子女,每晚都要阅读英语或母语书籍。

10. 解决问题(Solving Problems)
如果子女觉得某科太难,或者家长有疑虑和建议,立刻联系老师。如果家长想知道子女的多门成绩,可以要求辅导员代你向每位老师查询。
如果子女与同学之间出现问题,请告诉辅导员或副校长(the vice-principal)。
学校一般都鼓励学生自己解决问题,但有些情况下需要家长和老师干涉。下面是一些解决问题的方法。
?⒐先提问,问清楚,了解发生了什么事情。?⒐向老师、辅导员解释,子女是如何告诉你的。
?⒐听听老师、辅导员的看法。例如,问老师在学校里会怎样处理,看看是否可以在家里用同样的办法处理。
z 与老师一起制定方案,统一口径,这
样学校和家长给学生的意见就是一致
的。

z 提出将再次会面,看看解决方法是否
有效。

?⒐如果问题仍然不能解决,与副校长或校长讨论。如果仍然不满意,应向校监(the school superintendent)反映。校务处有校监的联系电话。

11. 欺负和骚扰(Bullying and Harassment)
如果家长怀疑子女被同学欺负、骚扰,先问清楚他们在学校究竟发生了什么事情。
有些学生害怕报告老师之后,会遭到对方报复。与学校讨论如何保障子女的安全。所有学校的政策都严禁恐吓和骚扰行为,如果有学生被欺负或骚扰,学校想及时了解到,哪怕这些事情是发生在校外。
帮助学生学习自己解决问题

家园移民 : 2008-05-09#31
回复: 子女教育信息素材收集帖子-冲置顶了

安大略省新移民小学指南!!!

星星的新家 : 2008-05-09#32
回复: 子女教育信息素材收集帖子-冲置顶了

守法,辛苦了!这个帖子很好呢,建议继续补充一些样本什么的,比如说去加拿大时需要携带的一些证明的样本等等。

wxdwyq : 2008-05-10#33
回复: 子女教育信息素材收集帖子-冲置顶了

谢谢守法的资料,学习中。

家园移民 : 2008-05-10#34
回复: 子女教育信息素材收集帖子-冲置顶了

前言 1

入学前的准备安省公立小学制度2 入学手续2 接待中心3 视听健康3

帮助孩子适应新的学校帮助孩子结交朋友4 处理不安的情绪5

学习英语英语为第二语言 (ESL) 和英语读写学习(ELD)课程6

学校生活程序
午餐8 禁果仁的学校8 缺课8 如果孩子在学校身体不适8 过敏症或健康问题9 恶劣天气9 校车服务9 预防疫苗注射9 学生最新联络资料9 校园内学生的看管10 上学衣着要求10 校外参观学习10 学校分区范围10 法语浸习课程11 国际语言课程11 包容其它宗教节日和信仰11 筹款活动11 升级分班的安排11 安省学生记录(OSR) 11

安省教学和学习 教学方式 12
两级合班 13
教科书和学习材料 13
教育政策 平等对待所有学生 14
按年龄编级 14
怀疑学生受到伤害的举报 14
学生必须上学 14


学生行为 行为守则 15
家长的角色 15
不适当的行为和处罚 16


学习内容 安省教学大纲初学英语的学生法语特殊教育 17 17 17 18
成绩单 了解成绩单安省的成绩单老师如何打分学习技巧幼儿园的成绩单家长意见回馈 19 20 20 20 20 20
如何评估ESL 和ELD ESL 和ELD 学生成绩的评估 21
学生的成绩 英语为第二语言( ESL)和英语能力学习(ESD)的项目英语学习的阶段 21 22


家长参与和与老师的良好沟通
你的参与有助于孩子的成功 23
和老师沟通 23
联络老师 24
保密守则 24
学校和家长的沟通 24
家庭作业 25
会见老师之夜 26
解决问题 27


家长老师面谈 家长老师面谈是什么? 28
参与孩子学校的事务 义务工作 30
学校的活动和会议 30
家长会 30
安省测验-EQAO 教育质量和问责办事处EQAO 测验 31


6 , 7, 8 年级上高中的准备

需要更多资料需要更多资料
Newcomers’ Guide to Elementary School in Ontario Draft Six

Introduction
前言
这份指南为您和孩子的老师之间建立良好沟通关系提供一些资料和建议。良好的沟通可以帮助老师了解你的孩子, 同时也能帮助你了解学校的情况, 使你们能更顺利地适应一个不同的学校体制。你的参与对孩子学业成功将有所帮助。
这份指南是根据许多新移民和老师提供的意见编辑而成。没有他们的协助, 这份指南无法完成。
孩子在上小学的整个期间,你可以用这份指南作为参

考。请使用目录寻找你需要的项目。 这份指南共有18 种语言的译本: 阿拉伯文、孟加拉文、中文(繁体和简体)、
克罗西亚文、波斯文、法文、古祖拉蒂文、印地文、韩文、巴斯土文、菲律宾文、旁遮普文、俄文、塞尔维亚文、索马里文、西班牙文、塔米尔文、乌尔都文。 指南内的"家长"一词, 泛指监护人、照顾孩子者以及其它家庭成员。 这份指南的网址是: www.settlement.org/edguide


Getting Ready for School
入学前的准备
安省公立小学体制安省共有四种由政府资助的学校体制: 英语公立学校、英语天主教学校、法语公立学校和法语天主教学校。
在安省, 由政府资助的学校是由教育局负责管理。英语公立学校向所有的学生公开。天主教学校只招收已经洗礼信奉罗马天主教的学生及罗马天主教徒的子女。一般来说, 法语学校招收所有寻求法语教育的学生。有关法语教育的资料, 请向教育局查询。

入学手续在孩子报名入学时, 家长需要提供以下所有的资料:
1 1. 孩子年龄证明? 出生证明或护照。
2 2.住址证明?银行月结单、电话或电费帐单, 或有你的姓名和地址的房屋租约。
3 3.监护权证明?只适用于18 岁或以下,且没有跟父母亲同住的孩子。
4 4.孩子移民身份的证明?出生证明、护照、移民纸 (IMM1000)或永久居民的证明(IMM5292)。大部份教育局都要求孩子必须是加拿大公民或移民, 才可以直接在自己地区的学校报名。
5 5.疫苗接种记录?证明孩子已经按照安省规定的时间表, 接受疫苗接种。

2

Getting Ready for School Chinese Simplified
接待中心
有些教育局要求新移民学生必须在接待中心办理入学手续。在接待中心, 将会由一位老师负责评估孩子的数学和英语能力。如果可能, 请把孩子以前的成绩单带到接待中心。有关资料将会送交孩子就读的学校, 以帮助老师为孩子制定学习计划。
视力听力健康
视力和听力上有问题会令孩子在学习方面倍=================== 感困难。每六名孩子中, 就有一名会有视力问父母正在申请难民身份的题。特别是在冬季, 很多年幼的孩子会有持续学生性中耳炎, 使他们在课堂听课时有困难。有关这类学生的报名手续, 各区
很多孩子并不知道自己的视力或听力有问题, 的教育局都有不同的政策, 请向而以为自己所看到的和听到的和别人一样。你住区的教育局入学办有些孩子想办法付出额外努力去克服这些问事处查询。题; =================== 但是, 如果他们的视力或听力问题得到改善的话, 他们可能会有更好的成绩。
?Ρ孩子三岁时, 带他/她去见验光师或眼科医生检查眼睛。一般来说, 孩子应该每两或三年检查一次眼睛。安省的医疗保险计划(OHIP)支付每年一次的检查费用。
建议 > > ?Ρ如果孩子需要配戴眼镜, 请鼓励他/她经常戴眼镜。如果孩子不愿意戴的话, 家长可以告诉老师, 一起帮助孩子消除顾虑。
?Ρ 请家庭医生为孩子检查听觉。

3

Helping Your Child Adjust to a New School
帮助孩子适应新的学校
帮助孩子结交朋友
能够结交朋友和感到被新环境接纳, 可以令孩子在学校感到安心,把精力集中在学业上。家长可以用多种方法来帮助孩子结交朋友和认识周围的环境。

?Ρ 帮助孩子认识周围的环境和可以安全玩耍的地点。
?Ρ参观和使用社区设施, 例如图书馆。年龄较小的孩子可以去"毋须预先登记的活动中心"、"家长中心"和"亲子中心"。年龄较大的孩子可以去康乐活动中心。校务处的职员可以告诉你这些社区活动的地点。

建议 > > ?Ρ陪孩子在课余时间去学校的操场, 让他/她和朋友一起玩耍。
?Ρ结识你的邻居, 让孩子看到你在结交朋友。
?Ρ带孩子参加社区举办的周末和课余的活动; 这类活动大都是免费的。
?Ρ与孩子谈谈他ㄍ她的新同学并记住他们的名字。当孩子希望请同学回家时, 和对方的家长联络作出安排。


4

Helping Your Child Adjust to a New School Chinese Simplified
处理不安的情绪
孩子可能因为在一个新的国家,上新的学校和学新的语言而感到焦虑不安。焦虑的情绪可以影响孩子的想法和感受。如果你的孩子感到焦虑,可能会影响到他/她的学习能力。焦虑不安的一些征状是: 腹痛、头痛、容易疲倦、粗暴、忧伤、沉默和拒绝改变。
?Ρ和孩子谈谈他/她的感受, 分享你自己在一个新的国家适应生活的感想。
?Ρ向孩子解释, 如果生活中有重大变化的时候, 感到焦虑不安是正常的。
?Ρ 鼓励孩子谈谈在学校发生的事情。聆听
孩子的心声可以帮助你了解他们在哪些


建议 > > 方面很顺利, 哪些方面有困难。采用提问的方式与孩子交谈。
?Ρ提醒孩子回想一些以前感到困难, 而现在已经变得容易的情况。在面对困难时, 正视每一个小小的成功。例如: 庆祝完成两个月的学业。
?Ρ告诉老师你的孩子感到有些焦虑, 使老师可以帮助孩子感到安心。

5

Learning English
学习英语
英语班和英语基础班(ELD)课程
每个学生都有不同的学习进度,即使是同一个家庭中的两孩子也会不同。一两年内, 大部份新移民学生都可以在日常生活中用英语沟通。但是, 有些学生可能需要五到七年的时间去学习读写能力和理解课堂上复杂的内容, 以达到其他同学说英语的水平。
初来加拿大的时候, 大部份新移民学生都可以流利地使用母语, 但英语能力可能有限。有些学生虽然在加拿================== 大出生, 但是只在上学时才开始学习英语。英语为第二ESL 和ELD 课程的种类语言 (ESL)的课程可以帮助这些学生赶上英语是母语同学校可能会提供以下任何一
学的水平。种或多种ESL/ ELD 帮助。
你的孩子可能: 有些学生可能有相当长一段时间失去上学机会, 或从来没有学习用母语去读和写。英语基础班 (ELD)课程可?Ρ由班主任老师提供额外以帮助这些学生学习英语阅读和写作能力。的帮助
所有的孩子在学习一种新语言的时候, 都会经历不同的?Ρ在自己课堂上跟ESL/
ELD 老师学习阶段。孩子的老师可以帮助你了解孩子的学习程度和?Ρ离开自己的教室跟ESL/
告诉你怎样去帮助孩子。ELD 老师单独学习
有关学习英语不同阶段的资料, 请浏览以下的网址: ==================
http://www.edu.gov.on.ca/eng/document/curricul/esl18.pdf

Learning English Chinese Simplified
?Ρ经常用母语和孩子交谈。让孩子学习英语的同时继续学习母语。如果孩子能用母语充份表达自己, 在学习英语时会感到比较容易。
?Ρ鼓励孩子继续用母语阅读和写作。老师发现, 如果学生能够很好地用母语去读和写, 学习英语的进步会比较快。老师还发现

建议 > > 这些学生在科学和数学方面和其他同学保持同样的进度。很多公立图书馆都有多种语言的书籍、录音带和录像带。
?Ρ鼓励孩子在放学后和其他学生一起参加小组活动, 让他们有机会用英语交谈。

7


School Procedures


学校生活
午餐如果孩子不能回家吃午饭, 他/她可以留在学校吃。请给孩子带些家里吃惯的食物, 但学校也许不能替学生加热午餐。午餐应包括果汁或水, 而不是含糖量高的汽水。有些家长会让孩子带些零食。请在午餐袋子或盒子写上孩子的名字。
午饭后, 学生都要到户外去玩耍, 直到上课。学校职员会负责在校园照管学生。
“无果仁”政策有些学生对果仁有严重的过敏反应, 如果他们进食或甚至接触到含有极小量果仁的食物, 都会感到严重不适。为确保这些学生的安全, 许多学校都采取 ”无果仁” 的政策。如果孩子的学校是”禁果仁”, 请不要让孩子携带任何含有果仁的食物到学校。
缺课 ====================
为确保学生安全到达学校,在学生不能上学时学校要求家长打电话到通知学校。如果孩子在上午9 时仍然没有到学校, 而家长也没有打电话通知, 学校会与家长联络。如果你的 电话留言的例子: This is (your name), my child (child’s name) is sick today. My child’s teacher is (teacher’s
孩子不能上学, 请在上午9 时前打电话到学校。 name).
我是(你的名字), 我的孩子
(孩子的名字)今天生病。孩子
有些学校设有电话留言机, 以便家长留言请假。如果孩子 的老师是(老师的名字)。
连续不能上学, 请你每一天都打电话通知学校。 ====================
孩子在学校身体不适

如果孩子在学校身体不适或发生意外, 学校会根据学校记录的电话号码与家长联络。你或紧急联络人必须尽快来学校接回你的孩子。大部份的学校都有一位受过急救训练的教职员。如果遇到严重的意外或病情, 学校会叫救护车, 由一位教职员陪同孩子前往医院。


School Procedures Chinese Simplified
过敏症或健康问题如果孩子有任何过敏症或健康问题, 请通知老师。如果孩子需要在上学时间服药, 你和孩子的医生都要填写一份表格。
恶劣天气在冬季,有时候因为暴风雪学校可能要关闭和取消校车服务。遇到恶劣天气的时候, 请留意广播电台有关学校关闭或取消校车服务的报导。在恶劣的冬季天气或下雨的时候, 学生将会在午饭和休息的时间留在室内。
校车服务如果孩子就读特殊课程或你的家和自己区内的学校距离很远 ,可能会有校车接送。详情请向校务处职员查询。
预防疫苗注射======================= 公共卫生局规定为了预防严重儿童疾病,遇到以下的情况, 请打电话通知安省所有的孩子必须注射疫苗, 才可以上学学校: 读书。如果孩子没有注射所有的预防疫苗?Ρ 你的孩子因为任何缘故不能或学校的记录不全, 你将会收到公共卫生局上学发出的警告信。如果你不遵守规定, 你的孩?Ρ你更改电话号码或地址, 或
子将不能回到学校上课。?Ρ 你有任何问题、担忧或建议。
=======================
你的医生会给你一张黄色的记录卡, 列明孩子曾经注射的预防疫苗。请把黄卡交给学校登记, 以确保学校有最新的记录。有关详情, 请向你自己区内的公共卫生局查询或浏览:
http://www.health.gov.on.ca/english/public/pub/early/immunization.html
联络资料的更新当你为孩子报名入学的时候, 你曾将家庭与工作地方的电话号码, 以及一位亲友的姓名和电话写给学校。在需要讨论孩子的学业或发生紧急事故的时候, 老师会根据这些电话号码跟你联络。如果这些电话有所改变, 请通知学校。

家园移民 : 2008-05-10#35
回复: 子女教育信息素材收集帖子-冲置顶了

校园内学生的看管在上课前至少15 分钟以及放学后15 分钟, 将会有一位或多位老师在校园值勤。在休息的时候, 也会有人在校园内看着孩子。在其它时间, 学校不会有人看管学生, 请不要在早上太早或放学后太晚的时间把年幼的孩子留在校园。
如果你的孩子留在学校吃午饭, 学校将会负责看管你的孩子。
着装要求有些学校可能对着装有要求, 说明学生在上学时可以穿些什么。在冬天或雨天, 请所有的学生多带一双鞋子留在学校。在整个学年中, 年幼的学生应该多带一些可以替换的衣服留在学校。
学校可能要求学生在上体育课时穿上运动鞋、短裤和T 恤。如果你对学校衣着的要求有任何问题, 请和孩子的老师商谈。
除非是下雨, 所有学生在休息和上课前后都要在户外活动。请让孩子穿上适合天气的衣服。
学校要求学前班的学生自已穿衣服, 老师会帮助那些仍然在学穿衣的学生。
校外参观学习有的时候, 老师会安排学生去博物馆、公园或剧院看戏剧。校外参观学习是班上课程的重要部份。在活动前后, 学生会学习与参观有关的资料。
学校需要你的书面同意, 才可以让你的孩子参加校外参观学习活动。你的孩子会把一份同意书带回家让你签名。请把表格签好, 然后尽快交回学校。你可能需要支付部份校外参观的入场费。
一般来说, 老师欢迎家长陪同学生前往校外参观学习。
校区范围孩子就读的学校是按你的住址而定的。每一所学校根据附近的街道和住家去划分招收学生的范围或区域。
有些教育局允许入学人数不足的学校收取居住在学校范围以外的学生。详情请向校务处职员查询。
10

School Procedures Chinese Simplified
法语浸习课程为帮助孩子掌握加拿大的两种官方语言----英法双语, 有些教育局提供法语浸习课程。法语浸习课程是一项自选课程, 适合母语并非法语的孩子选读。
在法语浸习课程中, 学生将会用法语学习大部份的课程; 但在较高年级的时候, 学生会用英语修一些课程。开始法文浸习课程的年级是由各区的教育局决定, 课程会持续到中学。详情请向校务处职员查询。
国际语言课程你自己地区的教育局可能会为学生提供多种周末或放学后的国际语言课程。详情请向校务处职员查询。
宗教节日和信仰如果你的孩子因为宗教节日而不能到学校上课, 请提前打电话通知学校, 让学校知道你的孩子不能上学。
学校会尽最大的努力让你的家庭奉行自己的宗教和信仰仪式。详情请向学校老师或校长查询。
筹款活动有时候, 你可能会收到学校家长会的来信, 要求捐款, 以资助一些没有得到经费的学校活动。捐款纯属自愿。
升级分班的安排在安排下学年班级的时候, 学校经常会调配来自不同教室的学生。因此, 你的孩子有可能在下学年不能和原来的同班同学在一起。
安省学生记录(OSR) 当你的孩子在学校就读的期间, 他/她的成绩报告单、教育挡案以及老师评语, 都会存放在安省学生记录中(OSR) 。老师会参阅学生记录内的文件, 以了解你的孩子和制定他/她的学习计划。安省学生记录属于保密文件, 但家长可以要求参阅。
11

Teaching and Learning in Ontario Classrooms
安省教学和学习
教学方式
所有学生都有自己的独特性, 在同一班里每个学生对同一科目的理解都会有点不同。例如: 有些学生的数学较好, 有些写作较好。

为帮助每一位学生都能进一步去学习, 老师会分别为全班、小组和个别的学生编排一些课堂上和带回家的不同的学习活动。在学年结束前, 老师会确保每一位学生都能够学到所有规定的课程内容。
老师也会综合不同的教学方法。例如: 老师可能指导全班学生学习故事的结构, 然后将学生分组学习不同的故事。
老师发现, 如果学生可以 ”看见和感受到” 所学习的概念, 他们会更容易理解。例如: 年龄小的学生用沙======================== 子和大小不同的杯子去学习体积。当学生做完后, 老小组学习
有的时候, 老师会重新编排学生
师会向全班说明他们学到了什么。的座位, 以便学生分组学习。
语言能力是每一个科目的重要部份。例如: 在数学课小组学习可以帮助你的孩子和上, 算出正确的答案固然重要, 但也要能够解释如何其它同学合作、分担责任、聆算出答案。听和学习其它学生的观点。你
老师会准备一些问题让学生去讨论和解答, 而不是强的孩子可以练习英语和学到多
种技能, 例如: 调查研究、
调死记硬背。在所有的科目中, 学生都要学习如何作组织和验证观点、采取主动、分析判断性的思考。学生学习分辨事实和假设, 用逻提出问题和勇于尝试。辑去思考,提出问题, 根据所学的内容建立自己的观======================== 点, 然后向全班同学讲解自己的观点。
我们生活在一个全球一体化的社会。老师编排的学习活动可以帮助学生重视所有人类的经验和贡献。

Teaching and Learning in Ontario Classrooms Chinese Simplified
?Ρ创造家人交谈的机会, 以便每一个人都可以参与, 例如: 在吃饭时间。
?Ρ鼓励孩子谈谈自己的观点 (他/她的想法)。这样对孩子的学习会有帮助。

建议 > > ?Ρ当孩子要做小组专题作业的时候, 帮助孩子和其它同学一起去做。
?Ρ如果你希望进一步了解课堂上的情况, 请和老师谈谈。
?Ρ请到下列网址索取以下的资料 “家长指南: 如何帮助你的孩子学习数学”, 和“家长指南: 如何帮助孩子学习阅读” :

http://www.edu.gov.on.ca/eng/document/brochure/brochure.html
混班教学
有些学校把两个不同年级的学生安排在同一间课堂里。在这些课堂内, 老师会确保学生学到自己所属的班级课程。老师会为全班学生和两级的学生分别编排学习活动。
有的时候, 学生连续两年都有同一位老师。这样的好处是老师可以了解孩子的学习需要, 并且已经与孩子的家庭建立了联系。
教科书和学习材料
在安省的学校, 学生不一定每个科目都使用教科书和习题书, 然后每天逐页去学。在学习活动中, 老师会采用不同的材料: 教科书、图书馆的书藉、互联网、报纸、录像带以及其它的资料。
13

Education Policies


教育政策
安省法律要求所有的学校都要遵守一些政策和程序, 以保障和帮助孩子在学校顺利地学习。
学生人人平等学校致力于提供一个所有学生都可以得到尊重、尊严和理解的学校环境。平等政策确保不会因为种族、血统、原居地、肤色、族裔、国藉、宗教、性别、性取向、残疾和家庭状况的不同而受到歧视。此政策同时也确保不会因为上述情况而妨碍学生的学业。如果你有任何关注, 请和老师或校长商谈。

有关学校所有人士的权利, 请参考安省人权委员会及其刊物
http://www.ohrc.on.ca/english/publications/index.shtml
按年龄编级当学生报名入学的时候, 他们所读的班级是根据年龄而分配的, 并非照原居住国家所完成的年级。但是, 在编排学习活动的时候, 老师会考虑到学生以前的教育水平。
潜在危害的举报当学校教职员观察到可能有人虐待、苛待或疏忽照顾儿童, 或者是有学生报告受到不良对待, 法律规定老师必需通知负责保护儿童的机构。 (名为”Children’s Aid Society 儿童保护中心”或”Family and Children’s Services 家庭及儿童服务中心”)。有关详情, 请浏览: http://www.oacas.org/resources/whentoreport.htm
出勤规定法律规定所有六至十八岁的学生都必须上学读书。如果学生长时间没有到学校上学, 教育局将会调查缺课的原因, 并且在必要时通知有关当局。
如果孩子将会在一段较长的时间内不能上学, 请预先告诉老师。有时候, 老师会建议一些给孩子在缺课期间学习的内容。

14


Student Behaviour
学生行为
行为守则学校的教职员致力于消除校内的不良情况, 例如: 虐待、欺辱、歧视、威吓、仇恨性的言语和行为, 以及任何形式的暴力。学校制订行为守则, 是为了防患于未然, 鼓励和平解决问题的技巧, 以及创造一个安全和友善的环境。

学校和孩子的老师会制订一些规则, 帮助学生遵守行为守则。老师会向学生解释规则的内容, 通常也会把规则张贴在学校内。
任何参与学校活动的人, 包括学生、家长或监护人、义工、老师或其它职员, 都必须遵守行为守则。行为守则也适用于在学校以外进行的学校活动。
?Ρ 鼓励孩子熟悉学校和课堂的规则。
?Ρ 提醒孩子打架斗殴和其它不当的行为所引起的严重后果。

?Ρ请老师通知家长如果老师对孩子的行为有所关注。建议 > > ?Ρ索取一份学校的行为守则。
?Ρ如果你感到行为守则或校规对你的孩子不公平, 你可以和老师商谈, 必要时可以和校长商谈。
?Ρ如果你对另一位学生的行为感到关注, 请与老师商讨。

家长的角色孩子行为的改变可能是由于适应新学校、新语言和新国家而产生的正常表现。如果老师对你孩子的行为或跟其它同学的关系感到关注时, 他/ 她可能会跟你联络。
老师会向你解释他/ 她在学校如何帮助你的孩子, 也可能会建议你在家里如何帮助孩子。家长和老师的配合可以带给孩子一致的信息,知道在学校应有的行为。
如果你有任何疑虑, 请和老师联络。
15
Student Behaviour Chinese Simplified
不适当的行为和处罚( Inappropriate Behaviour and Discipline)
学校会处罚不遵守学生守则的学生。根据情节的轻重,处罚包括口头和书面警告,课后留校,停课(一到二十天)或者开除。
校长根据具体的情况确定停课的长短。校长对学生是否明白了他或她的行为后果以及其他因素会予以考虑。类似盗窃和造成身体伤害的侵犯行为,将导致自动停课。如果学生停课或开除,家长将会得到通知。
=========================
可能导致停课、开除学籍和警方介入的行为
?Ρ 打架
?Ρ 威胁他人
?Ρ 对老师使用粗言秽语
?Ρ 贩卖违禁药物或武器
?Ρ 抢劫
?Ρ使用武器伤害他人身体, 或恐吓要对他人造成严重伤害
?Ρ伤害他人身体, 因而需要医护人员护理
?Ρ 恶意破坏
?Ρ 性侵犯
?Ρ 提供酒精给未成年者

=============================

16


What Students Learn
学习内容
安省教学大纲
在安省, 所有公立学校都要求遵照安省的教学大纲。教学大纲列出由学前班到各年级的每一个科目中, 学生应该达到的要求。
每一个科目都有教学大纲: 英语、数学、科学技术、社会学科、法语、健康及体育和美术。学前班也有教学大纲。老师要按照教学大纲去编排各种学习活动。
校务处职员可以告诉你如何取得这份课程指南。该文件登载在以下网址: http://www.edu.gov.on.ca/eng/document/curricul/curricul.html.
初学英语的学生
如果有必要, 老师会编排适合你的孩子的
========================= 学习活动。根据孩子的英语能力, 老师
摘录自1 至8 年级的语言教学大纲: 可能修改对孩子的学习要求; 或编排某些
“在3 年级结束时, 学生将学会如何 学习活动, 以帮助你的孩子与其它孩子保持
正确地使用名词、动词、形容词和 同样的学习进度。
副词, 把信息组织成一些简短的段落。”
========================= 法语
为帮助孩子学习加拿大两种官方语言之一----
法语的基本沟通技能, , 所有学生都要学习法
语及其文化。教育局必须在4 年级开始提供
法语课程, 也有些教育局在4 年级以前已经
开设法语课程。

17

What Students Learn Chinese Simplified
特殊教育
特殊教育课程为有明显学习困难或特殊需要的孩子提供额外的帮助。有些孩子只需要短期的帮助, 而另外一些学生有较复杂的学习和健康需要。
孩子需要帮助的原因可能是: 身体健康、智力、情绪、行为、表达、语言、视力听力困难, 或与他人交往沟通有严重困难。特殊教育课程也为智商较高的学生提供。
缺乏英语能力并不表示需要特殊教育。但是, 如果孩子在学业或与其它同学相处方面有困难, 就可能是学习有困难的表现。有时候, 问题的出现是孩子适应新语言和新学校的正常过程。有关孩子以母语学习的学习技能的信息, 例如: 以前的成绩单, 往往可以帮助老师找到学生感到困难的原因。
如果你担心孩子可能有明显的学习困难, 请和老师商谈。老师有多种非正式的方法可以评估你的孩子。如果有必要, 你或老师都可以提出要求给孩子进行一次正式的评估。
学校需要根据特殊教育的程序, 评估和决定你的孩子是否符合接受特殊教育的条件。为决定孩子的需要, 你必须提供书面的同意书并出席会议, 包括鉴定安置审查委员会(IPRC) 会议。
18


The Report Card
成绩单
理解成绩单从1 至8 年级,每年你都会收到三次成绩单。安省的成绩单是根据安省的教学大纲, 告诉你有关孩子在每个科目的学习表现。通过字母等级或百分比, 你会知道:
?Ρ 你的孩子是否达到所读年级应有的技能和知识。
?Ρ 你的孩子在该科目是否超过、达到、接近或低于安省的标准。

定义

成绩超过安省的标准
成绩达到安省的标准
成绩接近安省的标准
成绩远低于安省的标准
需要多方面的额外辅导学生能够表现应有的知识和技能

学生能够表现大部份应有的知识和技能
学生能够表现部份应有的知识和技能
学生能够有限度地表现一些应有的知识和技能
学生不能够表现应有的知识
和技能成绩单登载于下列的网址:

字母等级 百分比
(1 至 6 年级) (7 和8 年级)
A + 90-100
A 85-89
A- 80-84
B+ 77-79
B 73-76
B- 70-72
C+ 67-69
C 63-66
C- 60-62
D+ 57-59
D 53-56
D- 50-52
R 低于 50

http://www.edu.gov.on.ca/eng/document/forms/report/1998/report98.html#elem

19

The Report Card Chinese Simplified
安省的成绩单
达到安省标准的第 3级 (B+ 至 B-), 是表示你的孩子已经做好升级的准备。我们期望大部份的学生都可以达到这个水平。
“R “或”低于50”表示学生需要多方面的额外辅导, 你和老师需要准备一项帮助孩子成功的计划。
评分规则
老师会评估孩子在整个学期中多方面的表现: 测验、个人和小组作业、家庭作业和每天在班上的参与。老师会根据教学大纲中的水平要求去选出最接近学生成绩的等级, 然后给予适当的字母等级或百分比。
有关成绩等级的详情, 请参阅安省的教学大纲。校务处职员可以告诉你如何取得一份教学大纲。
教学大纲有登载于: http://www.edu.gov.on.ca/eng/document/curricul/elemcurric.html.
学习技巧
在成绩单中, 有一个部份是关于孩子的学习技能。这部份可以让你了解孩子在各方面的能力, 包括: 解决问题、课堂参与、制定目标、主动性、完成作业、使用资料、与别人合作和独立完成工作的能力。
学前班的成绩单
===========================
每一个教育局都有自己的学前班成绩单。在收到成绩单之间的期间, 家长如何跟老师根据对学生在学前班的学习要求, 评估进孩子的学习 :
学生的进度并填写成绩单。?Ρ 定期和孩子谈谈学校的生活。这样你
将会察觉哪些方面很顺利, 哪些方面有家长意见反馈困难。成绩单附有一份意见反馈表格。请你在签?Ρ请孩子给你看看他/她的作业和测验
名后, 由孩子交回学校, 你也可以在出席家成绩。
?Ρ和孩子一起阅读, 观察他/她的阅读
长教师面谈时带来学校。有些家长会在反技能。
馈表格写上意见。回执将会连同孩子的成?Ρ采取积极的态度, 赞赏孩子的努力。绩单一起存放在安省学生记录的档案内。这样可以鼓励孩子多谈学校生活。
?Ρ如果你有特别的关注, 请和老师商谈。=============================
20

家园移民 : 2008-05-10#36
回复: 子女教育信息素材收集帖子-冲置顶了

How the Work of ESL and ELD Students is Evaluated on the Report Card
如何评估ESL 和ELD 学生的成绩
正在学习英语的学生在每个科目上都需要时间和帮助以增强他们的英语知识和技能。

一些科目,学生可能与同年级的其他同学做同样的功课。在另一些科目,老师可能根据孩子的英语水平调整学生应该达到的学习要求。随着孩子英语的提高,老师会开始给他或她愈来愈多的与其它同学一样的功课。
如果孩子刚刚到加拿大,老师可能不给具体成绩。
英语班(ESL) 和英语基础班(ESD) 的项目
如果老师在成绩单上的ESL 或ESD 栏目作了记号,那表明:
?Ρ 学生与同年级其他学生所学的功课不同
?Ρ 老师就学生的英语水平对应该达到的学习要求做了调整
?Ρ 成绩单上的学生成绩是根据已调整后的要求,而不是与同年级其他同学相同的学习要求而评定的。
?Ρ老师在成绩单上写下如下评语:‘某某科目的成绩是根据学生在ESL 或ELD 项目中的要求而评定的,这与同年级的一般要求不同。



21

How the Work of ESL & ELD Students is Evaluated on Report Card Chinese Simplified
如果老师没有在成绩单上的ESL 或ESD 栏目作记号,那么成绩单上的成绩是根据与同年级其他同学相同的要求而评定的。
如果需要更多学习英语的信息,请参考本册有关学习英语章节。
英语学习的阶段老师可能会谈到有关学习英语的各个阶段。简单地说, 是指:
1 1. 用英语进行基本的沟通
2 2. 在熟悉的环境和有人帮助的时候学用英语
3 3. 在新的环境和有人帮助的时候学用英语 ? 孩子有较强的独立能力
4 4. 用和其它说英语的同学相同水平的英语。这阶段一般需要最长的时间

有关英语学习的阶段的详情, 请浏览: http://www.edu.gov.on.ca/eng/document/curricul/esl18.pdf
22

Parent Involvement and Good Communication with the Teacher
家长参与和与老师的良好沟通
你的参与有助于孩子的成功
在安省,学校鼓励家长积极参与孩子的教育, 以及和老师建立良好的沟通。研究发现, 家长的参与可以使孩子更加成功。
良好的沟通可以帮助老师认识孩子也可以帮助家长了解学校的情况。同时可以帮助孩子更顺利地适应一个不同的学校体制。
与老师沟通老师知道有些正在学英语的家长在与老师沟通时可能会有困难。老师仍然欢迎你和他们谈谈有关孩子的事情。
?Ρ 向学校查询是否有翻译可以帮助你和老师交谈。=======================
?Ρ请一位亲友(不能是小孩) 陪你到如果我不会说或看英语学校, 或跟老师在电话交谈。请当家长仍在学习英语时, 也可以有多种方
预约时间, 以确保老师有空和你法去帮助孩子:
建议 > 交谈。?Ρ 帮孩子安排做作业的时间
> ?Ρ和孩子谈谈他/她学习的内容
?Ρ在电话交谈的时候, 你可以用电?Ρ用母语为孩子朗读, 或听孩子朗读
话的三方通话功能(*71)。该功?Ρ安排孩子参加图书馆或其社区的活动能可以让你、老师和翻译(或懂来充实自己英语的亲友) 同时对话。该功能======================= 需要一点费用。详情请看电话簿内的特别服务项目。
?Ρ把你的问题或忧虑写下并放进信封内, 由孩子交给老师。

23

Parent Involvement & Good Communication with the Teacher Chinese Simplified
与老师联络
打电话到校务处给老师留言给。把你的姓名和电话、孩子的姓名和教室号码, 以及最方便与你联络的时间告诉校务处的职员。老师将会在白天或在黄昏时回电给你。
保密规定
你和老师谈话的所有内容都会保密。老师不会把孩子的学业或家庭情况告诉其它家长或社区人士。
学校和家长的沟通
学校会定期让孩子带一些资料给你, 例如: 课堂和学校活动的通知、家长通讯、校外参观学习的同意书、以及其它重要事情的通知。
?Ρ提醒你的孩子, 你需要看看学校派发的资料。
?Ρ请孩子检查自己的书包, 看看有没有学校的通知或信件。孩子经常会忘记交给你。

建议 > > ?Ρ在家里安排一个固定的地方, 存放学校的通知。
?Ρ 把重要的通知贴在家里。
?Ρ 尽快把回执让孩子带回学校。
?Ρ如果你有任何疑问, 请和学校联络。

24

Parent Involvement & Good Communication with the Teacher Chinese Simplified
家庭作业
检查家庭作业是家长了解孩子在学校学习的一个途径。作业的多少是根据孩子的班级而定的。每天晚上, 学生都要看书。老师会解释对作业的要求。
?Ρ和孩子一起安排一个舒适的学习地方,避免电视机或其它声音的干扰。提供学习需要的原珠笔、铅笔、橡皮擦、纸和字典。
?Ρ固定做作业的时间, 同时帮助孩子制定学习计划完成他/她的作业。
?Ρ问孩子作业的内容, 在孩子做完后, ====================

与他们讨论作业的内容。 如果你的孩子没有作业:
?Ρ当孩子遇到困难时, 要鼓励他/她, ?Ρ让孩子告诉你他在白天并且经常赞扬他/她的努力。这样做了些什么
建议 > > 会令孩子感到有信心。?Ρ读书给孩子听, 或让孩子读给你听
?Ρ 如果作业太难、需要太长时间、?Ρ鼓励孩子玩拼图, 或一些或者太容易, 请告诉老师。训练思考和对话的游戏
?Ρ如果孩子经常没有作业,
?Ρ 定期与孩子谈谈学业。就算是你和老师谈谈不熟悉这个题目, 你仍然可以做一==================== 位良好的听众。

25

==========================

Parent Involvement & Good Communication with the Teacher Chinese Simplified
会见老师会见老师的内容:
?Ρ 学生学习的内容
大部份学校都会给家长提供和老师会面以及?Ρ 课室中的特别专题作业或活动了解学校的课程的机会。这可能称为”会见?Ρ 对家庭作业的要求老师之夜”、”课程之夜”或”开放日”。这些?Ρ 学生行为守则的要点活动一般都是在学年初举行。?Ρ 你如何在家里帮助孩子
==========================
会见老师并不是和老师讨论孩子表现的最适当时间, 如果你对孩子有特别的关注, 你可以预约时间和老师面谈或通电话。
?Ρ告诉老师如果他/她对孩的表现子有关注, 或孩子取得优异成绩时打电话给你。
建议 > > ?Ρ 在学校内到处走走。参观学校图书馆和体育馆。这样可以帮助你和孩子谈论他们的体育和图书馆活动。
26

Parent Involvement & Good Communication with the Teacher Chinese Simplified
解决问题如果你有任何建议或对孩子有任何关注, 请和老师商谈。如果你的孩子和其它的孩子有冲突, 请告诉老师。
在可能的情况下, 学校会鼓励并协助学生自己解决问题, 但有时候也需要家长的介入。
建议 > >
?Ρ 向孩子提问可以帮助你了解情况。
?Ρ 向老师解释孩子告诉你的情况和孩子的感受。告诉老师你是如何帮助孩子解决问题。
?Ρ 听听老师的观点并听听老师在学校的处理方法。考虑在家里是否可行。
?Ρ制定一个你和老师能一起实行的的计划, 以便孩子可以从你和老师得到同样的信息。
?Ρ和老师约定再次商谈, 以便看看这办法是否有效。
?Ρ如果问题仍没法解决, 请找校长商谈。如果校长也不能够解决问题, 请找学校校监。校务处有校监的电话号码。

====================
帮助孩子尝试自已去解决问题
?Ρ 鼓励孩子和老师谈谈他们面临的问题
?Ρ 帮助孩子练习怎样去解释有关的问题
?Ρ 提醒孩子学校的规则是不准用打架和用粗俗语言去解决问题
?Ρ告诉孩子, 如果她/他自己不能解决时, 你会介入

====================
27


Parent Teacher Interviews
家长老师面谈
什么是家长老师面谈?
家长老师面谈一般是每年举行两次, 时间是在11 月或12 月和3 月份。家长必须出席秋季的家长老师面谈, 而3 月份的面谈是根据家长或老师的要求而定。你的孩子会把有关家长老师面谈的通知带回家。

家长老师面谈是家长和老师讨论什么是帮助孩子进步的最好途径。这是一个好的机会让家长和老师商谈彼此可以做些什么使孩子的学业成功。面谈通常是10 至15 分钟。
在面谈中, 老师可能会向你解释成绩单的内容, 并谈谈他/她目前是如何教导你的孩子的。老师会把孩子从学期开始到最近的作业进行比较,让你看到孩子学到些什么。老师可能会向你建议如何在家里去帮助孩子。
有些老师会鼓励学生出席面谈,谈谈自己的学业。如果你希望单独和老师讨论特别的事情, 请和老师另约时间。
?Ρ面谈前预先和孩子谈谈, 了解他/她喜欢学校生活的那些方面以及在那些方面有困难。
?Ρ想想有什么问题要问老师。用笔写下来, 可以帮你记住问题。

建议 > > ?Ρ如果需要的话, 请向学校查问是否可以提供翻译。如果学校没有翻译, 请带一位亲友 (不能是小孩) 出席面谈。
?Ρ面谈一般是10 至15 分钟。请至少在面谈前5 分钟到达, 以便尽量使用全部安排的时间。
28

Parent Teacher Interviews Chinese Simplified
建议 > >
?Ρ有时候, 家长和老师需要再次见面或用电话联络, 以跟进一些事项。你或老师都可以建议再次交谈。
?Ρ面谈后, 跟孩子讨论一下面谈的内容。侧重好的方面, 在谈到你担心的问题时要具体明确向孩子解释任何你和老师制定的计划。
?Ρ既使是孩子的学业顺利, 也要和老师保持联络。这样对孩子有鼓励的作用。

====================
家长经常向老师提出的问题:
?Ρ我的孩子学习英语的进度如何?
?Ρ我的孩子在那些科目可以学到所属班级的正常课程?
?Ρ我的孩子在那些科目需要额外的辅导? 要什么形式的辅导?
?Ρ我的孩子需要花多少时间做作业?
?Ρ我的孩子和其它学生的关系是怎样?
?Ρ我在家里应该如何帮助孩子?

====================
29

Getting Involved in Your Child’s School
参与学校的事务
义务工作学校经常鼓励家长义务为学校的活动服务。家长义工可以陪同学生参加校外参观学习, 在图书馆工作, 听学生读书, 或在其它必要的方面提供服务。向孩子的老师建议一些你自己母语或有关你自己文化的书籍、报纸、杂志或录像带。和老师谈谈有哪些适合课堂和图书馆的材料。和孩子的老师谈谈有关义务工作的机会。
学校的活动和会议学校可能会为家长和学生组织一些特别的活动, 例如: 学生表演和学校游艺会。有时, 学校会组织有关学业的讲座, 例如: 阅读写作、数学、如何帮助孩子学业成功。这些活动都是家长和孩子共渡时光的好方法, 并且可以增加家长对学校的了解。
校务委员会每一间学校都有由家长、社区人士和学校教职员组成的顾问委员会, 简称为校务委员会。校务委员会和校长讨论学校的政策和计划。所有家长都可以参加校务委员会的会议和参与讨论。同时, 所有家长都有资格成为校务委员会的干事。有些学校的校务委员会干事是通过正式选举产生; 另一些学校的校务委员会干事是由家长自愿担任的。
?Ρ 向校务处索取校务委员会主席的姓名和电话。
?Ρ和校务委员会主席联络, 讨论你想怎样参与校务委员会。

?Ρ出席校务委员会会议、了解校务委员会如何进行工作、对校内你感兴建议 > > 趣的事情提出问题, 建议, 并分享你的观点。
?Ρ 和校务委员会主席或干事谈谈如何帮助学校教职员以及其它家长了解你的族裔和新移民学生的需要。
?Ρ建立一个家长联络网, 以通知其它家长有关学校的活动及欢迎新移民家庭。


30


Provincial Testing ? EQAO
安省统考 - EQAO
3 年级和6 年级的学生将会参加由教育质量和问责办事处( EQAO)负责的统一测验。学校会把有关测验日期的通知信寄到学生家里, 测验一般是在 5 月份举行。
测验是根据安省的教学大纲, 目的是要了解全省各地学生在阅读、写作和数学方面的程度。
教育质量和问责办事处(EQAO) 测验的成绩不会记录在成绩单或学院及大学的申请表上。也不会用于===================== 决定孩子将来任何方面的发展。EQAO 的例题学生可能需要: 教育质量和问责办事处将会准备一份报告, 详细说明
?Ρ 写一篇文章以说明一个你的孩子在阅读、写作和数学方面的成绩。报告观念将会于下一个学年由学生带回家。
?Ρ找出一篇文章的中心思想在有些情况, 如果你的孩子是刚到加拿大并处于初学
?Ρ计算完成一个旅程所需要英语的阶段, 他/她可能获免参加测验, 或可以在参加的时间并用文字解释计算测验时得到更多的时间。方法

===================== 有关详情, 请浏览教育质量和问责办事处的网址:
www.eqao.com/

31

The Middle Grades (6, 7 & 8)
6, 7 , 8 年级6、7 和8 年级的课程是为学生进入高中准备的。学生有时候需要转到另一间学校升读6、 7 和
8 年级, 但有时候也会留在原校。
如果你的孩子需要转校, 你的学校会为家长和学生安排活动, 帮助你们做好转学的准备。ESL 学生家长常提出的问题:
在6、 7 和 8 年级, 学生会有专科老师教授不同的?Ρ在附近的中学是否有ESL 科目。学生将会有一份时间表, 说明每一个科目课程?
的上课日期和时间。?Ρ有没有配合我的孩子兴趣其中一位和你接触最多的老师将负责和你联络, 的特别课程? 讨论你对孩子的问题、忧虑和建议。这位老师会?Ρ 中学学校如何配合新移民在家长老师面谈的时候和你见面, 你也可以要求学生的需要? 会见其它的老师。=====================
上高中的准备
在初中6、 7 和 8 年级, 学生开始计划他们的高中教育。老师会帮助学生考虑自己的专长、兴趣和毕业后的计划。这些计划可以帮助学生准备在高中的选课。
在8 年级, 学校会为学生和家长组织一些活动, 帮助他们做好升读高中和9 年级的准备。这些活动都是家长提问和讨论什么对孩子最适合的好机会。
?Ρ和孩子谈谈在高中毕业后的计划。讨论他/她的优点和兴趣。大部份的学生都不能够确定将来的计划, 这是很正常的。
建议 > > ?Ρ 和老师讨论什么对孩子是最适合的。有些学校有辅导老师。
?Ρ参加学校或高中为8 年级学生家长举行的高中信息会议。
32

Need More Information?
需要更多资料
这份指南提供了很多方面的基本资料。以下是取得进一步资料的方法:
?Ρ 和孩子的老师、校长或校务处职员商谈
?Ρ 向教育局和教育部索取更多有关政策和程序的资料

taylor2755 : 2008-05-10#37
回复: 子女教育信息素材收集帖子-冲置顶了

好贴啊,不顶简直是犯罪了.

Luthy : 2008-05-10#38
回复: 子女教育信息素材收集帖子-冲置顶了

守法真是好人,额想啥恁来啥:wdb6::wdb17::wdb20:

tiantianxingchen : 2008-05-10#39
回复: 子女教育信息素材收集帖子-冲置顶了

很及时!很有用!很实用!

jsnjlxy : 2008-05-10#40
回复: 子女教育信息素材收集帖子-冲置顶了

非常感谢,加SW了。

菲碧 : 2008-05-10#41
回复: 子女教育信息素材收集帖子-冲置顶了

守法辛苦了!收集了那么全面的资料给我们!
努力发贴冲50就可以给你加声望!现在只能动嘴来"感谢感谢了"

zzblyllzg : 2008-05-10#42
回复: 子女教育信息素材收集帖子-冲置顶了

守法:受累啦.谢谢,太全面啦,
有用,太有用啦,加分,加分

poohdl2005 : 2008-05-10#43
回复: 子女教育信息素材收集帖子-冲置顶了

也来参与一下!

家园移民 : 2008-05-10#44
Lucia的说法

我儿子是90年的,去年5月份到加,插班就读10年级(高一)。就说一下我的情况,希望能对后来的有所帮助。
请孩子所在学校开一个学习成绩证明,中英文的各一份,自己交公证处公证。因为我儿子在国内高中一年级读了一个学期,所以就出示了两份证明(初中、高中各一,证明样本附后)。到教育局报到时,出示了出生公证、学习成绩公证和移民纸,并做一些测试就按照你的所住地分配到学校上课了。到现在他就读11年级,前几天又把学习成绩公证交到学校,可以得到16个学分。这边上大学至少要有30个学分才可以,其中要有6个12年级与大学专业相关的学分。高中每个学期开四门课,一学年可以得到8个学分(这当然要在每门课都PASS前提下),如果学分不够,也可以通过上夜校和summer school来赚学分,每学期最多可以上2个夜校,所以能转到学分是很重要的。另外,那个预防针的证件我们在国内也没有找到,对于我们的大孩子(好像是16岁以上)是没有什么用的,所以没有的也不必担心或费神去弄了。

学习成绩证明

XXX同学生于XX年XX月XX日,男(女),系我校2006级初(XX)班学生,2003年9月至2006年7月在我校就读初中,经过三年的学习,成绩优秀,于2006年7月30日初中毕业,初中学年的成绩(百分制)如下:

因为表格在这里发不了,有需要的给我悄悄话。 高中的也比照这样的就行。

家园移民 : 2008-05-10#45
Lucia的说法2

“作者: wangxiaoyu-62
LUCIA,你写的很详细,很有帮助.我也是90年的,我是四月的.我现在正读高三,马上准备登录,但我有些问题想请教一下:我原本打算参加完国内的高考,拿到成绩后赴加.我是想到加后能进入象多大这样的大学学习.但听说这些名牌大学对象我这样的新移民申请时需提供TOFEL或者IELTS成绩,而我没有.象我这种情况,朋友给了我几种方案1)不要参加国内的高考,马上赴加来读这里的高三.------这样的话语言上会有很大进步,但我觉得还是不能申请大学,因为我还是没有TOFEL成绩,我上高三的课程那有时间考啊,还有就象你说的,学分也不够啊.其实我已留了一级,很有可能留两级呢有点浪费时间(2)参加完高考再赴加,然后读一年的预科,这期间学校有TOFEL的课程并可以参加考试.然后可以申请大学-----但听说这类预科学校都是私立的,不可靠,会随时倒闭,这又让我很迷惑.所以我很拿不定主意该怎么办? ”



就你的情况,我和儿子讨论了下,他认为,还是尽快过来,因为大学要求很高,如果成绩不好,或者fail课了,就会被踢出来,所以,到了大学首先语言要过关,不然课听不懂肯定fail了。高中过来读的好处呢,就是有时间适应英语,而且学分不是问题, 国内的学分可以转,他一个同学今年圣诞节过来的,到多伦多,她在国内正读高2,上个月学分转了,得到了20个学分,一年半轻轻松松就可以把学分拿够,学分到不是问题;还有你说的TOFEL,不是所有大学都要TOFEL,有的大学有它专门的英语考试,过了就可以上。在高中还有许多大学的竞赛,waerloo的macmaster的都经常在儿子的学校搞数学竞赛、物理竞赛什么的,他还在数学竞赛中拿了个全校第一,题目很简单;这个竞赛中成绩好的,大学录取的时候就会优先考虑。
在预科呢,他也有认识的同学在读,但是就是像你说的不可靠,主要不是怕倒闭,主要是学习环境差,教学质量也差,而且很贵。
我这里有个方案,你现在已经高三,没几个月就高考了,考不考无所谓,因为你考了过来还可以上高中,只要你是21岁以下的都可以上高中,然后就像我前面说的,大学要6门专业课学分,你就想好要学哪个专业,然后到http://www.electronicinfo.ca/en/上面看...难事。然后还有就是弄好国内成绩的翻译和公正,拿过来转学分,你的情况应该可以转24个学分吧。
就看你怎么安排,读一年,辛苦点,早点毕业,读2年,轻松点,可以有更多选择,比如你可以多?藜该鸥咧械目危?ù笱У氖焙蚓筒恢劣谥荒鼙ㄒ桓鲎ㄒ怠?
我儿子是90年10月的,现在才在这里高中读11年级,算正常年龄。你年龄不算大,不用着急的,主要是要能上个好大学,而且要能坚持住,不被T出来。

家园移民 : 2008-05-10#46
Lucia的成绩单

[/ATTACH]

附件


家园移民 : 2008-05-10#47
henryj60的说法1

引用:
作者: cyzhangch
希望已经在加国或准备让孩子在加国念初中或高中的妈妈们在这里多交流.这方面的信息太少了,已经在加国的妈妈能否将你的孩子的学习\交友\饮食等方面的经验\经历呈现给大家,让大家一起分享快乐!分担痛苦!准备去加国的父母能否将你们已经掌握或搜集的信息呈上,大家一起学习\补充和完善?谢谢!

这是可怜天下父母心呐。其实你们的任务就是把孩子带到加拿大来。关键是,如果决定来,就早来比晚来好。早一天到,早一天适应。
下一步就不用太多费心了。特别是移民集中的大城市,这里有很完善的教育体制来迎接新移民的到来。孩子会慢慢的适应学习环境,确立自己的位置。
学习\课程基本上是孩子自己选。学校有辅导员可以帮助孩子。可以说他们比家长更了解孩子在学习上的表现。比家长参和要好。你们的职责就是听听汇报,把把关。我参加过两次家长会,由于说了一句谦虚的话,害得我女儿在ESL班多呆了2个月。以后就再也没去过。这里有很多文化上的差异,与其我适应,还不如让孩子适应。
交友\孩子到一个新地方,或早或晚都会有自己的朋友。我女儿的第一个小朋友是刚来时住公寓里的一个小女孩。我们是6月末登陆的,学校正好放假。这个假期我女儿也就有了上学前的一个适应期。在公寓的游泳池认识了这个小女孩,比她还小一年级。这个小女孩是4岁随父母出来,转暂美国、法国,最后定居加拿大,基本说英语。这个假期我女儿就有了对新环境的了解和认识。上学以后,有通过第一个朋友认识了同年级的新朋友。就这样,身边就有越来越多的朋友了。家长不但有关心鼓励孩子学习,还有鼓励他们参与各种活动。不要不放手孩子。她出去和朋友玩、看电影什么的,我都鼓励。只有告诉我和谁在一起,都去哪里就行。
饮食\刚开始上学时,午餐基本上是热狗、三明治之类的快餐。一来简单,不用热(有的学校没有微波炉),二来造价也低。后来逐渐带保温饭盒,带中餐,偶而买点pizza。

家园移民 : 2008-05-10#48
henryj60的说法2

引用:
作者: cyzhangch
谢谢henryj60的回复!!可以看出你在教育方面是一个非常明智的家长.我们在等PL,估计年后应该到了.不知道您的孩子多大了?我的女儿现在正上初二,打算让她念完初三再过去,原因有二,1:我女儿的班主任很?训茫?谂嘌?⒆拥拢苤牵芴寮父龇矫婧苡蟹椒ǎ?玻核?挠镂睦鲜σ膊淮恚?蚁肴门??谡夥矫娴幕?「?我恍???币怖?谜庖荒臧爰忧克?耐庥铮?醵坛龉?蟮牟皇视ζ冢?恢?闳衔?绾危坑捎诙院芏嗍虑椴涣私猓??杂惺币不嵊行┙粽藕兔H唬?M?芰私飧?啵?热缂庸?醒У募倨谑奔洌渴切扪Х致穑?
是不是有必修课和选修课之分?我的女儿不爱学数学,是不是在国内的时候就应该让她把一些基本的数学名词的英语背一背?.....

我女儿6月在国内读完初一来的。9月份开学在这里上的初二(8年级)。住在北约克。先是在公寓辖区的初中读8年级(就是 Georges Vanier SS 对门的初中)。后来为了去 AY Jackson 搬到了 AY Jackson 辖区的 Highland 初中,读的9年级。之后升入 AY Jackson。高中在 AY Jackson 读了2年半。最后半年在安省科学学校 ( Ontatio Science Schoo)。

对于你的情况,如果年后应该到PL,我强烈建议初二毕业就过来。我的观点是:良师固然有用,但孩子最终是要自我发展的。老师会按他的方式培养孩子,但孩子应该按自己的兴趣成长。个性绝不能被束缚。中国的教育机构就如同一个庞大的机器,进去的是充满个性的孩子,出来的是眼镜+分数的统一产品。绝大多数孩子都炼成了学虫,缺少了创造性。这是为什么我们培养不出诺贝尔奖得主;不管哪里,教育都有些连贯性。你插入的越晚就越跟不上连贯的步伐。日后高中学习时,就会感到不适应; 还有就是语言环境。孩子13岁进入英语环境和14岁进入就有很大的不同。这不是倍数关系,是指数关系。与其在国内补习英语,不如来这里亲身实践;最后就是,这里上大学不仅要求高中学分和成绩,还要求英语。很多学校要求学生在英语教育有3、4或5年的经历。否则就要托福成绩。虽说考托福对她将来不会是大问题,但能减轻孩子压力为什么不呢?最后,入大学还要有一定的社会服务时间(就是做义工的小时数)。你越晚来,就有越少的时间考虑这些问题。最后把所有的东西都压到一起,岂能忙开呀。

以上是我个人观点,希望你能认真考虑。




引用:
作者: 龙井茶
非常感谢你的介绍!请问:
1.如果在国内初中毕业后过去,应当读9年还是10年?
2.如果不加试托福,加拿大的大学要求至少有几年的本地教育背景?

1, 和国内的学年一样的(取决你是几年的初中),你在国内该上几年,到了这里就上几年级。一般应该是上10年。但这里是按年龄上学,同一年出生的一般都在同一年级。6岁那年上学,18岁那年高中毕业。

2, 最多要求5年,最少要请3年。大专要通过英语考试,一般不要求托福。

家园移民 : 2008-05-10#49
henryj60的说法3

引用:
作者: 龙井茶
多谢了老乡,给你加分了。我是沈阳的。
我孩子今年初三毕业,准备今年8月去多伦多读高中。但高中我还没选好,您女儿读的AY JACKSON好像只接收10~12年的学生?从网上看不同的学校接收不同年龄段的学生,感到很乱。另外如果大学(例如多大)要求至少4年的本地高中背景的话,我儿子估计还要重读9年了,那样他的年龄就显得大一些。现在很着急,麻烦你多介绍一些选校的经验。多谢。


中小学教育归市教育局管,不同是教育局有不同的管理。多伦多是有5个市合并成的,还保留着以前个市的教育体制。北约克有小学(6年)、初中(3年)、高中(3年);世家堡有小学(8年)、中学(4年)。学制不同没有关系,都是12年教育。

你儿子来了是不是要重读9年,多半要有学校(和测试中心)决定,当然你们可以坚持自己的观点。他们主要考虑让同龄的孩子在一起。


引用:
作者: Ammie
我们也是92年底的,现在读初三,在加拿大想读10年纪,(因为我们在国内没读10年纪),很担心孩子适应过程太短,跟不上. 请问我们可以读10年纪吗?
谢谢您了.

你当然可以和测试中心的人讲。你国内的成绩单有明显的学年(9年级)。这里应该会安排到适应的年级去。

你想的一定要和他们沟通,这里还是挺有人情味的。




引用:
作者: 花谢枫红
我也遇到孩子到那儿怎么入学问题,求朋友们帮我参谋一下并给些建议,先谢了:

我的孩子是91年8月的,在国内北京的学校读到初三时(2005年11月)感觉跟班吃力,就蹲班转到大连枫叶学校读初二(2006年3月去的)并于2007年在枫叶初中毕业。07年9月回北京上了一个私立高中,读了四个月,因为这个学校实在不能给孩子好的教育和影响,加上考虑很快就要移民,就在07年年底退了学,没有期终考试的成绩。现在我想让她四月登陆后先找个高中学一下语言和适应一下环境,然后9月读10年级,但是,我的埃德蒙顿朋友给我咨询的结果是这样的:
如果在本地高中注册,那么到9月份她只能跟着升入11年级;在本高中学区找了一个对应的初中去问,该校认为孩子已经17了,年纪偏大,不好再读9年级。朋友建议我做个在国内读初三的证明,说她因为蹲班和入学晚等原因就是在读初三,可以和他们沟通考虑读9年级。

孩子目前只有在枫叶学校初中毕业证书和在北京澳华学校(高中)的一个在校证明,我是否需要给她把大连的学籍成绩一类的东西拿回来?是否需要在北京找个学校做一个初中在校的学习成绩及证明?
孩子如果9月上11年级,她没有10年级的成绩,是否影响她将来对大学的申请?是否必须要考托福?

我这些日子头都大了,这个漫长的移民进程把孩子的学习进程都打乱了,本来移民的目的就是想让孩子受更好的教育,让她快乐的学习,有兴趣的学习,主动的学习,没想到时间拖了那么久,孩子年龄都大了,唉!
求求各位给我一个答复,谢了谢了!!

看来你事先作了不少工作。我不是指打听加拿大入学情况。是指办移民以后对孩子的“特殊”安排。就是这个安排,使孩子放松了继续学习的劲头,过分憧憬移民以后的日子,造成了现在的局面。我这里不是责备你,只是想让其他刚办移民的家长重视。学习要靠自己,环境变了,只会带来不力的因素,只有加倍努力才能赶上。

你的情况确实很难说。你想,17岁的孩子和15岁的在一起。对他的心理影响会有多坏。如果和17岁的一起,学习有赶不上。对他也不好。还有关键的问题:孩子现在还有学习的尽头吗?孩子的心还能收回来吗?这应该是目前你们最担心的才对。

不管孩子来后上几年级,我建议不选名校或好学校。参与竞争恐怕不是他的强项,如果在学校里总是打狼,势必挫伤孩子的信心,从而一蹶不振。在一般的学校,同学平均水平不高,加上你们督促,孩子就有领先的优势,树立信心。

即使不能上9年级,也可以争取进10年级。就是进11年级学不好,还可以再留级一年,来年再读11年。

最后是不是要考托福进大学,也不是在加拿大学4年(9、10、11、12)就一定不需要托福。还是需要高中的英语成绩达到大学要求才免。我朋友的孩子就被多大要求提供托福成绩,因为他的高中英语成绩不太好。

tiantianxingchen : 2008-05-11#50
回复: 子女教育信息素材收集帖子-冲置顶了

谢谢守法搜出来的信息!很有用!

家园移民 : 2008-05-11#51
回复: 子女教育信息素材收集帖子-冲置顶了

谢谢守法搜出来的信息!很有用!
有用就好,嘎嘎

大家有没有什么压箱的宝贝阿?嘎嘎

螺母 : 2008-05-11#52
回复: 子女教育信息素材收集帖子-冲置顶了

先顶一顶,谢谢完了再慢慢看!。。。

nakijin : 2008-05-11#53
回复: 子女教育信息素材收集帖子-冲置顶了

好帖,守法辛苦了。

shine : 2008-05-12#54
回复: 子女教育信息素材收集帖子-冲置顶了

thanks!

雨中冷百合 : 2008-05-13#55
回复: 子女教育信息素材收集帖子-冲置顶了

有用就好,嘎嘎

大家有没有什么压箱的宝贝阿?嘎嘎

我有呀,嘎嘎:wdb20::wdb20:

家园移民 : 2008-05-13#56
回复: 子女教育信息素材收集帖子-冲置顶了

我有呀,嘎嘎:wdb20::wdb20:
啧啧

哪儿啊
哪儿啊

雨中冷百合 : 2008-05-13#57
先来点排名吧。这是05年统计的,也不知道是不是管用,

大多伦多地区高中排名High School Ranking in Greater Toronto Area
以下高中学校排名(High School Ranking)是由Fraser Institute研究发表,National Post(国家邮报)发布的。目前最新的排名报告发表于2001年。是根据1998、99年度的学校资料进行的排名。表格中显示“n/a”的地方,表示资料不足,难以进行名次排定。此外,表格中也并没有包括本地区所有高中学校。 ・ 高中12年级人数不足15人的学校没有包括,因为其学生资料不够很难得出公正可靠的分析结果。 ・ 没有包括成人教育和继续教育中心、主要面向外国学生的学校、以及只是侧重某些方面课程教育的学校。当然,没有包括这些学校,并不意味否定这些学校的教育。 值得注意的是,多伦多教育局(Toronto DSB)的一些在华人社区中名气很大的高中,在Fraser排名中不一定理想。如A Y Jackson Secondary School,被认为资料不足,难以确定名次;而在华人社区较少听说的 R H King Academy ,确排名第20,是多伦多教育局在大多伦多地区排名最前的高中。安省排名 District Name School Name 本地区排名
Branksome Hall
Havergal College
St Andrew's College
St Michael's College School
St Clement's School
University Of Toronto Schools
The Crescent School
The Academy for Gifted Children
Holy Trinity School
Ner Israel Yeshiva College
St Mildred's-Lightbourn School
Richmond Hill High School
R H King Academy(Toronto DSB)
The Woodlands Secondary School
Pickering College
&amp;Eacute;cole secondaire Cardinal Carter de langue fran&amp;ccedil;aise
Bayview Glen
Kingsway College
Ursula Franklin Academy(Toronto DSB)
Beth Jacob Private School
Langstaff Secondary School
Northview Heights Secondary School(Toronto DSB)
Holy Name of Mary Secondary School
Georges Vanier Secondary School(Toronto DSB)
Unionville High School
Woburn Collegiate Institute(Toronto DSB)
Heart Lake Secondary School
Albert Campbell Collegiate Institute!(Toronto DSB)************
Forest Hill Collegiate Institute(Toronto DSB)
Martingrove Collegiate Institute(Toronto DSB)
Richview Collegiate Institute(Toronto DSB)
Parkdale Collegiate Institute(Toronto DSB)
Peoples Christian Academy
Leaside High School
The Country Day School(Toronto DSB)
Lorne Park Secondary School
Agincourt Collegiate Institute(Toronto DSB)
Thornlea Secondary School
Malvern Collegiate Institute(Toronto DSB)
North Toronto Collegiate Institute
Mayfield Secondary School
Streetsville Secondary School
Midland Avenue Collegiate Institute
David &amp; Mary Thomson Collegiate Institute
Glenforest Secondary School
Woodbridge College
St Joseph Secondary School
Markville Secondary School
Lester B Pearson Collegiate Institute
Loyola Catholic Secondary School
Dr Norman Bethune Collegiate Institute
Marc Garneau Collegiate Institute
O'Neill Collegiate and Vocational Institute
Father Leo J Austin Catholic Secondary School
St Ignatius of Loyola Secondary School
Erindale Secondary School
Stephen Leacock Collegiate Institute
Bayview Secondary School
The Humberview School
Brampton Centennial Secondary School
Pine Ridge Secondary School
Sir John A Macdonald Collegiate Institute
Bramalea Secondary School
Madison Academy
Milliken Mills High School
Westmount Collegiate Institute
&amp;Eacute;cole secondaire E J Lajeunesse

雨中冷百合 : 2008-05-13#58
回复: 子女教育信息素材收集帖子-冲置顶了

我收集点学校排名的资料,都是转贴来的,但可能时间有点久了,大家先参考着看看吧

雨中冷百合 : 2008-05-13#59
回复: 子女教育信息素材收集帖子-冲置顶了

安省的教育网站http://www.edu.gov.on.ca/eng/document/brochure/osslt/

cyzhangch : 2008-05-13#60
回复: 子女教育信息素材收集帖子-冲置顶了

感谢,加SW

zhaozq : 2008-05-13#61
回复: 子女教育信息素材收集帖子-冲置顶了

感谢!加SW.

我也想移民20061122 : 2008-05-13#62
回复: 子女教育信息素材收集帖子-冲置顶了

女,初二,08.3魁投,等面试

菲碧 : 2008-05-13#63
回复: 子女教育信息素材收集帖子-冲置顶了

女,初二,08.3魁投,等面试

我们俩是同月的FN,我儿子初三,共同度过这漫长的移民路吧!:wdb9::wdb9::wdb9::wdb32::wdb32::wdb32:

吉人天相 : 2008-05-14#64
回复: 子女教育信息素材收集帖子-冲置顶了

请守法给些温哥华的中学资料好吗?

家园移民 : 2008-05-14#65
回复: 子女教育信息素材收集帖子-冲置顶了

请守法给些温哥华的中学资料好吗?
温哥华十佳公立中学
排名 校区 学校名称
1 温哥华 University Hill
2 西温 Sentinel
3 本那比 Burnaby North
4 列治文 Steveston
5 温哥华 Prince of Wales
6 温哥华 Point Grey
7 温哥华 Lord Byng
8 温哥华 Magee
9 列治文 Hugh Mcroberts
10 素里 Elgin Park

家园移民 : 2008-05-14#66
回复: 子女教育信息素材收集帖子-冲置顶了

女,初二,08.3魁投,等面试
更新了

黑色光明 : 2008-05-15#67
回复: 子女教育信息素材收集帖子-冲置顶了

绝对有用的贴子!辛苦守法!

Chechechen : 2008-05-19#68
回复: 子女教育信息素材收集帖

守法,有没有多伦多中学的信息,我儿子94年的,现在初二,顺利的话10月份过去.

一缕阳光 : 2008-05-22#69
回复: 子女教育信息素材收集帖

 温哥华的一个亲戚发给我的,希望跟大家共享。

菲沙研究所(The Fraser Institute)周六公布卑诗省及育空地区的中学排行榜,名列榜首5校全属私校,其中四所位於温哥华,另一家私校位於素里。报告中列治文各校排名变动最剧烈,本拿比则全面下滑,无一公校进入前百名。

  继上周公布本省小学排名榜之后,菲沙研究所再次公布本省的中学排名榜,这次排名针对全省298所公立及私立高中,以十至十二年级学生的省考成绩作为主要依据,排名中私立中学依旧包办前13名。得分满分、排名第一位的5所学校是:温哥华的小花中学(Little Flower Secondary)、圣乔治中学(St. George's)、约克豪斯中学(York House Secondary)、温哥华学院中学(Vancouver College Secondary),以及位於素里的索思里奇中学(Southridge Secondary)。

  在公校当中,排名最靠前的高中为温哥华大学山中学(University Hill),位於排名榜的14位,比前5年排名第10的成绩有所下降。大温地区排名最后的学校为不列顛尼亚中学(Britannia Secondary),名列排行榜283位,比前5年排名232的成绩,下降了51位。

去年首度拉下西区Eric Hamber的东区明星学校Killarney,今年表现欠佳,甚至被东区的Templeton、David Thompson和Windermere超越。Windermere和Templeton是大温地区排名进度最多的前四所学校之二,Templeton首度挤入前百名,表现抢眼。

  近年来在列治文公校中称王的Hugh McRoerts,今年排名大幅滑落,在列市公校中名列第五,被Steveston、R. A. McMath、J. N. Burnett和Richmond赶过。但这五所学校排名均在百名之内,使列市公校整体成绩在大温地区名列前茅,仅次于西温哥华、温哥华西区和新西敏。 R. A. McMath和Richmond均名列大温排名进步最多的10校之列。

  本拿比各公校今年排名普遍下滑,无一校挤进百名之内。Moscrop取代Burnaby North,居该区公校榜首,Burnaby Mountain紧随两校之后,居该区第三名。新成立、位于本拿比东南近新西敏的Byrne Creek中学,排名吊车尾,在大温地区居倒数第四。

  Semiahmoo和Fraser Heights并列素里公校第一名,Elgin Park退居第三。紧随其后的North Surrey今年进步显著,为大温排名进步最多的10校之一。

评论:
菲沙学会(Fraser Institute)公布了卑诗省中学最新年度排名榜。该学会研究亦揭示,学校表现与所在社区的贫富程度及学生父母的受教育程度并没有必然联系。

菲沙学会的中学和小学评鉴,往往被批评者诟病,其排名结果只反映所在地区的经济条件,也就是说,富裕社区的学生成绩一般较佳。菲沙学会从多方面对这种批评作出回应。

  一周前发表的小学评鉴,特别为此举例说明,同一社区内背景相同的学校,成绩可以很不一样。譬如温哥华东区近在咫尺的JW Sexsmith小学和John Henderson小学,景背近似,可是在菲沙学会五年的评比中,前者排名198,后者排名596。

  昨天发表的中学评鉴,则特别列举13所学校表明,即使在并不富裕、父母教育程度偏低的社区,同样可以找到好一些排名在平均以上的学校。

  这13间“特别的”中学是:金贝尔河基督教中学(Campbell River Christian),伽迈拿斯中学(Chemainus Secondary),麦健士中学(MacKenzie Secondary),米逊中学(Mission Secondary),位于克雷梅斯(Keremeos)的希密凯明中学(Similkameen Secondary),素里(Surrey)的玛嘉烈公主中学(Princess Margaret Secondary),Midway的邦德威中心中学(Boundary Central),Lumby的查理斯布伦中学(Charles Bloom),阿伯茨福(Abbotsford) 的茅特中学(W. J. Mouat)和理克汉森中学(Rick Hansen),以及温哥华的谭普顿中学(Templeton)、戴维汤逊中学(David Thompson)和温地密尔中学(Windermere)。

  中学评鉴报告主撰人柯里(Peter Cowley)表示,这13所学校的教师和管理者,都能找到对抗不利因素的办法,帮助学生获得比一般预期为佳的表现,这些都难以用学生的家庭特征加以预测。一方面,这些学校都挤身卑诗省中学排名前列,而另方面,其学生父母的教育程度则是该省最低的。

  柯里表示,有人习惯把学校成绩欠佳归诸其所处的社会经济环境因素。类似看法实质认为学生不可能摆脱家庭经济条件而取得学业成功。但是,“无论学生住在哪里或他们的父母收入如何,公立学校系统应能够以同一标准教育所有的孩子”。

家园移民 : 2008-05-22#70
回复: 子女教育信息素材收集帖

守法,有没有多伦多中学的信息,我儿子94年的,现在初二,顺利的话10月份过去.
多伦多偶还真不熟悉阿

梁溪香榭 : 2008-05-27#71
回复: 子女教育信息素材收集帖

多伦多偶还真不熟悉阿

守法, 拜托想辙, 提供点安省中学的情况好吗?
因为加拿大很多好大学都在安省啊.

有2个问题:
1. 高一在读(刚过16岁)过去可否重读高一. 可否跟学校坚持重读高一?
2. 希望提供安省私立全寄宿中学情况. 不是指国际学校. 可以是教会学校. 不一定要在多伦多. 入学需要什么条件?

辛苦守法, 估计有不少家长也对这个问题感兴趣. 我们就指望您了.

家园移民 : 2008-05-27#72
回复: 子女教育信息素材收集帖

守法, 拜托想辙, 提供点安省中学的情况好吗?
因为加拿大很多好大学都在安省啊.

有2个问题:
1. 高一在读(刚过16岁)过去可否重读高一. 可否跟学校坚持重读高一?
2. 希望提供安省私立全寄宿中学情况. 不是指国际学校. 可以是教会学校. 不一定要在多伦多. 入学需要什么条件?

辛苦守法, 估计有不少家长也对这个问题感兴趣. 我们就指望您了.
1、困难,不过也看学校
2、这个就真不知道了。多伦多这个就得在www.rolia.net这些网站查询了,或者自己在网站搜索

家园移民 : 2008-05-27#73
回复: 子女教育信息素材收集帖

明天下午偶公司有个培训,有个小孩,北京的,初二到了温哥华,现在在多大了,去年同时被5个最好的大学录取了。

等偶培训的时候,好好的询问询问,嘎嘎

梁溪香榭 : 2008-05-27#74
回复: 子女教育信息素材收集帖

1、困难,不过也看学校
2、这个就真不知道了。多伦多这个就得在www.rolia.net这些网站查询了,或者自己在网站搜索

谢谢守法, 可否提供BC省私立全寄宿中学情况. 不是指国际学校. 可以是教会学校. 入学需要什么条件?

主要问题是家长可能无法长期待在加拿大。 只能两边跑几年。

家园移民 : 2008-05-28#75
回复: 子女教育信息素材收集帖

谢谢守法, 可否提供BC省私立全寄宿中学情况. 不是指国际学校. 可以是教会学校. 入学需要什么条件?

主要问题是家长可能无法长期待在加拿大。 只能两边跑几年。
公司有本大温学校年监

有不少这个内容,因为是从加拿大邮寄过来的,所以不多版本

建议嫩给北京环球打个电话,一般顾问都有,就说嫩需要复印件,就说是网上公司的守法答应嫩的,嘎嘎


实在不行了,给嫩个邮寄地址,偶给嫩安排个

咖啡壶 : 2008-05-28#76
回复: 子女教育信息素材收集帖

明天下午偶公司有个培训,有个小孩,北京的,初二到了温哥华,现在在多大了,去年同时被5个最好的大学录取了。

等偶培训的时候,好好的询问询问,嘎嘎
是啊,辛苦守法仔细问问.

惴惴不安 : 2008-05-28#77
回复: 子女教育信息素材收集帖

我是投资移民,已申请一年了,如果09年7月能成行,我女92年9月生,现初三,打算在加重读高一,听说小城镇容易融入,去哪里好呢?当然还要考虑国内生意,时差问题,传说对吗?请指教,先谢谢!

梁溪香榭 : 2008-05-28#78
回复: 子女教育信息素材收集帖

公司有本大温学校年监

有不少这个内容,因为是从加拿大邮寄过来的,所以不多版本

建议嫩给北京环球打个电话,一般顾问都有,就说嫩需要复印件,就说是网上公司的守法答应嫩的,嘎嘎


实在不行了,给嫩个邮寄地址,偶给嫩安排个

守法,悄悄话了。

家园移民 : 2008-05-28#79
回复: 子女教育信息素材收集帖

我是投资移民,已申请一年了,如果09年7月能成行,我女92年9月生,现初三,打算在加重读高一,听说小城镇容易融入,去哪里好呢?当然还要考虑国内生意,时差问题,传说对吗?请指教,先谢谢!
教育方面,偶也不怎么专业,最近恶补恶补

还是温哥华好!自然环境时差教育质量等都很好

按照嫩女儿的年龄,可能要读11年级了,具体的还要和教育局商量的,加拿大这个地方,想留级还真不容易

菲碧 : 2008-05-28#80
回复: 子女教育信息素材收集帖

公司有本大温学校年监

有不少这个内容,因为是从加拿大邮寄过来的,所以不多版本

建议嫩给北京环球打个电话,一般顾问都有,就说嫩需要复印件,就说是网上公司的守法答应嫩的,嘎嘎


实在不行了,给嫩个邮寄地址,偶给嫩安排个

守法:偶是在北京的,儿子初三.可否您公司也提供一份复印件给偶?偶可以自己来拿喔! :wdb10:偶一下!:wdb6:

家园移民 : 2008-05-28#81
回复: 子女教育信息素材收集帖

守法:偶是在北京的,儿子初三.可否您公司也提供一份复印件给偶?偶可以自己来拿喔! :wdb10:偶一下!:wdb6:
悄悄话给偶地址,偶尽量准备阿

菲碧 : 2008-05-28#82
回复: 子女教育信息素材收集帖

悄悄话给偶地址,偶尽量准备阿

太感谢守法了!见悄悄话.
不过偶第一次用这个功能,研究了半天,:wdb7:
不知收不收的到?

家园移民 : 2008-05-29#83
回复: 子女教育信息素材收集帖

收到

梁溪香榭 : 2008-05-29#84
回复: 子女教育信息素材收集帖

菲碧, 我也发悄悄话给你了. 查收

jenniferlee : 2008-05-31#85
回复: 子女教育信息素材收集帖

先顶一个再说!守法真是好筒子,想偶们所想,急偶们所急啊。声望支持!

偶的仔仔:12岁,男,小六,九月就上初中了。魁投,FN等待ING。。。

jenniferlee : 2008-05-31#86
回复: 子女教育信息素材收集帖

这里的TZ们,你们登陆后准备去哪儿啊?迷茫中。。。

家园移民 : 2008-05-31#87
回复: 子女教育信息素材收集帖

先顶一个再说!守法真是好筒子,想偶们所想,急偶们所急啊。声望支持!

偶的仔仔:12岁,男,小六,九月就上初中了。魁投,FN等待ING。。。
收录了

attack122008 : 2008-06-02#88
回复: 子女教育信息素材收集帖

?

梁溪香榭 : 2008-06-02#89
回复: 子女教育信息素材收集帖

明天下午偶公司有个培训,有个小孩,北京的,初二到了温哥华,现在在多大了,去年同时被5个最好的大学录取了。

等偶培训的时候,好好的询问询问,嘎嘎

守法, 询问了末有?
我们等着泥。

家园移民 : 2008-06-03#90
回复: 子女教育信息素材收集帖

问了

不过人家是公立的,对私立的学校还真是不了解阿

不过小伙子刚上大一,1米9,看着还真阳光

梁溪香榭 : 2008-06-03#91
回复: 子女教育信息素材收集帖

问了

不过人家是公立的,对私立的学校还真是不了解阿

不过小伙子刚上大一,1米9,看着还真阳光

有没有问问这个小伙子几年级出去的?
高中选什么课?
现在哪所大学, 什么专业?
守法, 给多点资料以便培训培训我们嘛.

美滋滋810 : 2008-06-12#92
回复: 子女教育信息素材收集帖

先顶一个再说!守法真是好同志,谢谢了。
投资移民 香港——联邦 5月30号已体检,准备登温哥华。
儿子90年的,高二转到国际部,读出国预备班。今年没参加高考。到加拿大后一定要再读一年高三,才可以进大学吗?

wanglj96 : 2008-06-14#93
回复: 子女教育信息素材收集帖

守法,经常看你的帖子,辛苦了。我想要你手里学生成绩证明的中/英文摸板。能否传到我邮箱里,谢谢。wanglj92@163.com

changchang : 2008-06-14#94
回复: 子女教育信息素材收集帖

谢谢守法,辛苦了,先作记号,慢慢读,希望很快可以有能力加声望!

移民心愿 : 2008-06-14#95
回复: 子女教育信息素材收集帖

先谢了.偶也在学习中顶

MN719 : 2008-06-14#96
回复: 子女教育信息素材收集帖

守法,经常看你的帖子,辛苦了。我想要你手里学生成绩证明的中/英文摸板。能否传到我邮箱里,谢谢。mn719@163.com

家园移民 : 2008-06-14#97
回复: 子女教育信息素材收集帖

守法,经常看你的帖子,辛苦了。我想要你手里学生成绩证明的中/英文摸板。能否传到我邮箱里,谢谢。wanglj92@163.com
46楼有样本

家园移民 : 2008-06-14#98
回复: 子女教育信息素材收集帖

守法,经常看你的帖子,辛苦了。我想要你手里学生成绩证明的中/英文摸板。能否传到我邮箱里,谢谢。mn719@163.com
46楼有样本

家园移民 : 2008-06-14#99
回复: 子女教育信息素材收集帖

先顶一个再说!守法真是好同志,谢谢了。
投资移民 香港——联邦 5月30号已体检,准备登温哥华。
儿子90年的,高二转到国际部,读出国预备班。今年没参加高考。到加拿大后一定要再读一年高三,才可以进大学吗?
拿到高中毕业证书就可以的

sz艾美 : 2008-06-14#100
回复: 子女教育信息素材收集帖

守法:请问小孩在加中学至少要读4年还是5年,进入大学才不需要考雅思.

家园移民 : 2008-06-14#101
回复: 子女教育信息素材收集帖

各个学校不一样

ubc3年

[FONT='Arial','sans-serif']English language requirement[/FONT]
[FONT='Verdana','sans-serif']English is the primary language of instruction at UBC. All prospective students are required to demonstrate a minimum level of English before they’re admitted. [/FONT]
[FONT='Arial','sans-serif']There are eight ways to meet this requirement[/FONT]
[FONT='Arial','sans-serif']1. [/FONT][FONT='Arial','sans-serif']Complete three or more consecutive years of full-time education in English within Canada immediately prior to attending UBC. [/FONT]
[FONT='Arial','sans-serif']2. [/FONT][FONT='Arial','sans-serif']Complete four or more consecutive years of full-time education in English in a [FONT='Verdana','sans-serif']country other than Canada where English is the principal language[/FONT]. These four years must be immediately prior to attending UBC. [/FONT]
[FONT='Arial','sans-serif']3. [/FONT][FONT='Arial','sans-serif']Achieve a grade of 70% or better on the provincial examination portion of BC English 12 or English Literature 12 or the equivalent. [/FONT]
[FONT='Arial','sans-serif']4. [/FONT][FONT='Arial','sans-serif']Achieve a final grade of 4 or better on Advanced Placement (AP) English Language & Composition or AP Literature & Composition; or achieve a final grade of 5 or better on International Baccalaureate English A1 or A2 (higher-level or standard-level). [/FONT]
[FONT='Arial','sans-serif']5. [/FONT][FONT='Arial','sans-serif']Achieve the competence standard indicated by one of the [FONT='Verdana','sans-serif']tests of English language proficiency[/FONT] that evaluates skills in listening, reading, speaking, and writing. [/FONT]
[FONT='Arial','sans-serif']6. [/FONT][FONT='Arial','sans-serif']Successfully complete six credits of post-secondary first-year English studies that are eligible for transfer credit to UBC. To be eligible, the course must be taken at a recognized university in an English-speaking country. A transcript showing completion of these courses is required by June 30. For students at North American post-secondary institutions, this usually requires you to complete the courses by the end of the January to April term. [/FONT]
[FONT='Arial','sans-serif']7. [/FONT][FONT='Arial','sans-serif']Graduate from a recognized degree program at an accredited university at which English is the primary language of instruction in [FONT='Verdana','sans-serif']a country where English is the principal language[/FONT]. [/FONT]
[FONT='Arial','sans-serif']8. [/FONT][FONT='Arial','sans-serif']Attend four or more years at an [FONT='Verdana','sans-serif']eligible international secondary school[/FONT] that uses English as the language of instruction but operates in a country where the primary language is not English. These four years must be immediately prior to attending UBC. [/FONT]
[FONT='Verdana','sans-serif']If you are fully proficient in English but do not meet any of the eight options listed above, you may request a waiver of the English language requirement.[/FONT]
[FONT='Arial','sans-serif']Submitting documentation[/FONT]
[FONT='Verdana','sans-serif']You will receive an email and/or letter if you are required to submit evidence of English language proficiency. If you submit an application that does not have proof of English language proficiency, you will not be evaluated until we receive such proof or until a waiver is granted.[/FONT]

美滋滋810 : 2008-06-16#102
回复: 子女教育信息素材收集帖

拿到高中毕业证书就可以的
非常谢谢:wdb6:

NO way : 2008-06-17#103
回复: 子女教育信息素材收集帖

请教守法大人!!我90的现在读高二。可是没有会考 T T。如果我过去加可以从高一开始读吗?555....

家园移民 : 2008-06-17#104
回复: 子女教育信息素材收集帖

估计没戏,能上11年级就不错了

NO way : 2008-06-17#105
回复: 子女教育信息素材收集帖

哦~那上11年级不需要会考成绩吧?

家园移民 : 2008-06-17#106
回复: 子女教育信息素材收集帖

哦~那上11年级不需要会考成绩吧?
不需要

NO way : 2008-06-17#107
回复: 子女教育信息素材收集帖

不需要


哦。谢谢

海边小舞 : 2008-06-17#108
回复: 子女教育信息素材收集帖

严重感谢守法!感谢热心的好朋友们!:wdb9:

顺其自然HZJ : 2008-06-17#109
回复: 子女教育信息素材收集帖

女,94年,初二,07.6魁投,面试已过,等体检

家园移民 : 2008-06-17#110
回复: 子女教育信息素材收集帖

收录了

xxwe : 2008-06-18#111
回复: 子女教育信息素材收集帖

守法先生,
我的女儿18岁,高二,我们在等ME,预计09年7、8月才会PL。女儿高中会考结束,现在就可以拿到高中毕业证了。以后的高三学习全是为了明年的高考。
请问:
1、女儿还继续在校学习,迎接明年的高考,拿到高中毕业证和高考成绩,然后去加申请大学?
2、让女儿现在就去新东方学英语,放弃高三的在校学习,去过英语关,明年到加后继续读12年级(高三)?
以上那种方法更好点呢?
还有:
3、加国新移民入12年级,在年龄和国内的学习上有什么限制吗?比如,在国内高二已算是高中毕业了,加国还让继续上12年级吗?还有孩子明年就19岁了,年龄上还允许上高中吗?
有人说,孩子过了18岁,到啦那边只能上成人高中,成人高中收费额高吗?与普通高中相比,对孩子有什么不利吗?
谢谢
也欢迎有经验的其他同学帮忙出出主意。

家园移民 : 2008-06-21#112
回复: 子女教育信息素材收集帖

守法先生,
我的女儿18岁,高二,我们在等ME,预计09年7、8月才会PL。女儿高中会考结束,现在就可以拿到高中毕业证了。以后的高三学习全是为了明年的高考。
请问:
1、女儿还继续在校学习,迎接明年的高考,拿到高中毕业证和高考成绩,然后去加申请大学?
2、让女儿现在就去新东方学英语,放弃高三的在校学习,去过英语关,明年到加后继续读12年级(高三)?
以上那种方法更好点呢?
还有:
3、加国新移民入12年级,在年龄和国内的学习上有什么限制吗?比如,在国内高二已算是高中毕业了,加国还让继续上12年级吗?还有孩子明年就19岁了,年龄上还允许上高中吗?
有人说,孩子过了18岁,到啦那边只能上成人高中,成人高中收费额高吗?与普通高中相比,对孩子有什么不利吗?
谢谢
也欢迎有经验的其他同学帮忙出出主意。
教育的问题涉及太重要,不能随便发言

不过从技术角度来说,只要没有拿到pl,就不敢保证不出意外,建议还是继续读高三,直到pl了,再考虑进一步的。否则如果有耽误,耽误移民事小,孩子教育事大

家园移民 : 2008-06-21#113
jenniferlee的精彩发言

教育篇


为什么要移民,这个话题可谓老生常谈,在其他版中见过几次.


虽然每个人的移民目的不尽相同,虽然每个人在做出移民或不移民的抉择时,多少都会有些犹豫,彷徨和困惑,但是对于投资移民而言,下这个决心更加艰难.因为我们不同于那些青春年少的莘莘学子,他们风华正茂,手握大把如金的岁月可供挥洒;我们也不同于那些技术移民,他们中的大多数人都有一技傍身,无论走到世界的哪一个角落,依靠技术都能混口饭吃.最重要的是:无论是莘莘学子还是技术移民,他们中的绝大部分人,都因为种种原因,还没有能够在国内获得长足的发展,因此,他们有足够的理由出去奋斗,冀望在水的那方为自己和家人建设一个更加理想的家园,哪怕暂时要为此付出不少的心血和汗水.


对于我们这些改革开放的受益者,这些已经或接近不惑之年,这些通过十多二十年的辛勤努力,已然在各自的领域当仁不让地各领风骚,这些事业基础已经牢牢地根植在自己的祖国,这些在国内受人尊敬,生活舒适轻松,有钱有闲一族而言,有什么理由驱使我们弃易奔难;有什么理由让我们放弃多年的基础,却奔赴那未知的世界;又是什么理由,足以让我们宁愿甘冒夫妻父子长相分离的痛苦,常年承受万里长途跋涉的辛苦劳顿呢?


收到CSQ,打了投资款,当初面试成功的喜悦心情平复之后,就不可避免地想起这些问题.虽然俺的思想简单,天生一乐观派,但毕竟不是懵懂无知的少年人了,还是觉得如果能把这些问题考虑清楚,会更加有助于我们未来尽快地适应和享受加拿大的新生活,不至于届时心理落差太大.


为此,特开此帖,抛砖引玉,希望大家畅所欲言,互相启示.


就俺个人来说,早在十年前,就产生移民念头了,当时想办技术移民(孤陋寡闻,还不知道有投资移民).朋友中有人当时极力怂恿,要一块儿办.但因为听到一些负面消息,说某某人的妻子孩子在加拿大,自己在国内挣钱养活他们.因为耐不住寂寞,弄个老二养着云云:wdb1:...听上去很恐怖,遂不做移民之想.


一晃又过了几年,LG再次提起这个动议.借着公出之际,偶去加拿大蒙城,温哥华和多伦多看了看,与当地的朋友亲戚同学促膝竞夜长谈,到他们家中实地考察,得出的结论就是:他们的日子远不如俺原来想象的那样好.也因此,再一次地打消了移民的念头.


事实上,俺在加拿大,除了办理公事而外,其余的时间,就都是在为那里的男性朋友们答疑解惑,因为他们都听说国内发展机会多多,跃跃欲试地想要回国发展.不过,女同胞们则与之相反,她们都让俺劝阻她们的先生们,因为国内的狐狸精们实在有够厉害:wdb20:.


俺在加期间,当地地接为俺们安排的几个临时翻译,都是才过去不久(半年到一年多)的新移民,没有FULLTIME 工作,看上去很是凄惨可怜.其中有一位去国前任商务部某处的付处长(不像贪官),去了之后因为没有技术,只能靠临时为人做翻译谋生.俺当时真是感觉心酸,心有戚戚焉.仿佛又看见了小时侯那些戴着眼镜的臭老九的模样,难受ING...

家园移民 : 2008-06-21#114
回复: jenniferlee的精彩发言

俺觉得只要是循正途去往加拿大的同胞们,绝大部分都是优秀人才.但是由于天时地利人和等诸多原因,导致他们中的一部分人的生活,远不如俺们这些土鳖在国内来得舒服.


加拿大地广人稀,区区三千万人口,市场的局限性很大.加之多年来的高税收高福利政策,导致那里的许多人(尤其是当地人)懒惰成性,一点儿开拓精神,进取精神都没有.


举例说来:偶们在加参观了许多企业,并且进行了初步的商谈.但是,回国后,主动与我们联系,到我们的企业座谈考察的,只有一家大陆移民企业.虽然最终也没有合作成功,但是,华人的勤奋,努力可见一斑.而那十多二十家西人企业,就都音信渺然了.

很难想象这样的民族能有快速发展的可能.

当然,诚如上面一位朋友所言,加拿大的确具有令俺们向往的许多特质.用俺在加友人们当年为俺做的总结来说明一,二:

1.加拿大有着全世界最丰富的淡水资源.而世界上许多国家都已经,或即将面临淡水资源短缺的危机.MAYBE某一天,列强们将像现在为能源而开战这样,为了巧取豪夺淡水资源而骤起纷争.

实际上,现已探明加拿大的油砂储备量似乎也不少.


2.加拿大位于北美大陆,经济军事上长期仰赖世界第一强国,相对安全.


3.加拿大实际上是一个较高级的社会主义社会,相对贫富差距较小,因而社会的稳定度高,和谐感强,社会矛盾较为和缓,因此犯罪率也不至于太高.


4.整体的人口素质高.


5.语言优势.


6.民主政体,法制社会.相对而言,个人的权益更容易受到保障.

导致俺最终做出移民决定的最主要原因,同有得就有失一样,也是为了孩子.

一晃去年儿子上小学五年级了,即将面临小升初的人生第一次搏弈.

孩子从小比较聪明,但是非常不用功.他对数学比较敏感(可能与俺俩都是学工科的有关),从一年级开始,就在学校的数学兴趣班里学奥数,并于三年级和四年级时,两次入选北京奥数集训队.第一次入选时,俺们还得意洋洋地送他去参加集训.可是上了两次课后,孩子说,他只能听懂很少的一部分.俺们马上与教练员深入地聊了聊,这才发现,只有真正的数学天才才可能拿到国际国内比赛的大奖.而一个8岁孩子的教材,却是俺们花了很大的劲,还难以全部搞懂的.

当机立断,退出培训,因为一是俺们的孩子并非什么天才,二是也不希望他成为什么天才.记得孩子7岁时去进行"超常儿童测试",他用那稚嫩的嗓音说:"妈妈,我是正常儿童,不是超常儿童."童言无忌,说得多么在理啊.

儿子应付学校的教学大纲绰绰有余,但因为其他的大部分同学课余和假日里都忙着上各种各样的补习班,而偶们却不愿意他小小年纪就负担过重,因此,除了不断地上各类运动型的班之外,课外的其他兴趣班都没有上.也因此,他在学校数学兴趣班的位子,就只能是每况愈下.每年参加市里的奥数比赛,就只能拿个三等奖了(有N次了).

去年五月果断地做出移民决定后,马上随机找了家中介公司(孤陋寡闻,如果那时就知道家园,一定找受法),当时他们说,如果俺们马上办,那么可能可以赶上今年的秋季入学.现在看来,这实在太不现实了.

万幸的是,去年当机立断的决定,却使得俺们这一年来少着了许多的急(傻人有傻福:wdb6:),孩子更是少受了N多的罪.

今年寒假,俺俩去HK见考官,孩子自己回了外公外婆家,和表哥一起,天天玩个不亦乐乎.俺们却鞭长莫及,难以管束.与此同时,他的同班同学都在上各个顶尖中学的网校,以期获得参加考那所学校的资格.不少孩子报了两所,甚至三所中学的网校,把孩子们累得东倒西歪的:wdb14:.离俺家最近的*大附中,只有十多个自主招生的名额,网校却报了2700多人.竞争的那个残酷,令人震惊.

当时俺们笃笃定定地等待CSQ,天真地以为拿到CSQ就可以开拔了.殊不知,原来取得CSQ还只是刚刚过了雪山草地,万里长征的移民之路还遥远着哪.这才开始着急为他找学校.


好在CSQ在手,心里毕竟塌实了很多,所以,只要是所还不错的学校就行,不再苛求顶尖学校了.曾经打听过,孩子三年初中,要上顶尖学校,除非你是仁华学校(一所奥数学校)一,二班的学生,或者你有强硬的关系,否则一个孩子需要花十多,二十W:wdb5:,而且还得托人找关系,否则有钱都不知朝哪送.:wdb26:

家园移民 : 2008-06-21#115
回复: jenniferlee的精彩发言

到目前为止,孩子同学的初中基本都已经有了着落.


今年,北京市小升初改革,规定各所学校除了文艺和科技特长生而外,不可以考试为测试标准.还有一部分推优生,要求是获得过全国以及市区级三好生的孩子可以参加推优学校(俺们海淀区就有十多所,其中大部分都是很一般的学校)的电脑随机大派位,其他孩子则是参加指定学校的电脑派位.


俺家儿子,从小玩乐高玩具,从三年级开始就参加学校机器人小组,所以,大大小小拿过不少机器人比赛的奖项,其中包括去年7月美国亚特兰大机器人世界杯大赛的一等奖.


凭着这些奖状,以及林林总总的奥数获奖证书,春蕾杯全国中小学生作文竞赛的获奖证书,俺带着儿子考了两所中学,结果全部铩羽而归.其中的**中学,招两个特长生班,而参加考试的竟达到3300多人!


班主任让俺们去找人,俺也没找.


孩子开始时很郁闷,俺们借机教育他:天才都得努力尚且能够成功,更遽论咱们凡人了.


但随着同学们入学之事渐渐地沉埃落定,孩子自己就得出了结论:原来要上好学校,主要还是得爸妈有关系肯花钱.因为,他们班上许多远不如他的孩子,都能够进顶级的学校.


今年的改革结果是:大量的学生都必须去通关系,走后门花钱.否则,好学校一定是对你说NO的.


教育不公由此可见一斑.但这实际上只是我们的社会众多不公的表现之一.也因此,成为俺们移民的理由之一.


事实上,对于俺们来说,花点钱让孩子受好一点的教育并非什么难事.适者生存,咱一介平头百姓也无力扭转乾坤.但问题是:费了这么大的劲,花了这么多的钱,而孩子们又能得到什么呢?


一直对中国的应试教育制度不敢恭维,因为自己多少也是这种教育制度下的牺牲品.而现在的孩子们,则一生都有可能为应试教育所累.自小过重的学赋,使得大部分的孩子不同程度地厌学,他们中的许多人,在年幼时分被拔苗助长,被剥夺了无忧无虑玩耍的机会,而当他们心志业已成熟,理当读书进取时,却终于脱离了父母的掌控,开始无忧无虑,或者有忧不虑地玩耍.长此以往,这种违背自然规律的本末倒置,会给俺们这个民族带来很大的危害.



此外,大人和孩子这么辛苦,学到的又是些什么呢?参加过几次公司招聘,发现真的很难从应届生中寻觅到合适的人选.自小开始过于繁重的学业造就的创造力缺失,,脱离社会实践的教学内容,闭门读书导致的对社会现状,人情世故的一无了解,以及多年来家庭对孩子教育的高投入所对应的毕业生对于薪资待遇的高期望值,在在的都成为大学毕业生成功就业的障碍.



我们的社会没有为孩子们创造让他们成为具有独立的思想,健全的心志,健康的体魄,丰富宽泛的学识的社会中流砥柱的条件.

我们的孩子中的许多人从小做过几万道数学题,因此而没有时间读闲书,杂书,从而令人遗憾地没有培养出可以受用一生的良好的读书习惯.

我们的学校鼓励孩子听老师的话,压抑了个性的发展,扼杀了创造力的萌芽,致使我们的孩子,只能在世界之林中,领跑短暂的十数年(应试能力那叫一绝),而却落后于其后漫长的几十年.

我们的孩子每天披星戴月地上学,周末假日也少有闲暇.我们的孩子大都是独生子女,因而习惯了惟我独尊,习惯了从小到大被人伺候,很难想象未来他们时代的离婚率会是多么地高(早就给儿子灌输了不可以找独生子女做太太的理念).


真正的人才所需要具备的情商,财商,胆商,智商和体商,学校能够赋予予我们的孩子多少呢?他们中的许多人,只怕连自己独立生活的能力都欠奉.


我们的教育体制只认同很有限的几种人的天分,而却视另外的人类与生俱来的几十种天赋于不见,为他们打上"不够聪明,笨"的标签,从而挫伤了大部分孩子的学习积极性,导致其中不少人破罐子破摔,不仅不能发挥他们的长才,反而会给未来社会制造麻烦.

这一切,是俺移民的主要原因之一.

家园移民 : 2008-06-21#116
回复: jenniferlee的精彩发言

去年俺一姐妹淘从美国来京参加国庆观礼,她与俺一起待了几乎一天的时间,跟着俺去买菜购物,接送孩子上学,回家安排小保姆干活做饭.最后她得出结论:你的日子过得很舒服.

俺问她:你去了美国十多年,觉得美国最好的地方是什么?

她想了半天,说道:我觉得美国最好的地方是自由.只要你不触犯法律,没有人管你这样那样的.

这话对俺有振聋发聩的作用.

前人云:不自由,毋宁死.生命诚可贵,爱情价更高.若为自由故,两者皆可抛.

自由于人的重要性实不必言.

因此,追求更加自由的空间,成为俺移民的重要理由.

大学所学的化学知识中,唯一被俺记住的是:分子的密度(单位空间里的分子数)越大,分子间相互碰撞的机会越多.

把这个理论应用到社会生物学中,俺们可以得出这样的结论:人口密度越大,人与人之间产生矛盾,摩擦,甚至斗争的可能性就越大.

没有生存竞争问题的分子间尚且具有这样的特点,由此可以推论:在资源有限的社会中,人口密度越大,必须仰赖资源生存的人类之间的生存竞争就越激烈,乃至残酷.


这也是俺希望移民的理由之一:不希望在残酷的争斗中不慎被挤扁.不希望成为被别人暗算的对象.当然,俺希望活在一个和谐的社会中,哪怕不再是鸡群中的立鹤.须知:老祖宗们早有遗训,木秀于林风摧之啊

家园移民 : 2008-06-21#117
回复: jenniferlee的精彩发言

当初选择移民目的地时,俺LG给出了两个选择:加拿大和澳大利亚。

这两个国家俺都去过,因此断然地否决了后者,尽管LG说澳大利亚的气候比加拿大好。


之所以选择加拿大,主要还是从孩子教育的角度考虑的,但也有其他因素:

1。加拿大位于北美,文化背景与美国几乎相同。如果以后孩子或俺们想去美国发展,要容易很多。

2。加拿大英语的口音较澳大利亚的英语口音为轻。

3。比较喜欢四季分明的地方。从生理学的角度考虑,平均温度高的地区,人口平均寿命为低(在其他条件相同的基础上比较),因为人体基础代谢率较高的缘故。

4。加拿大地大物博,发展空间较大。

俺的一朋友3年前技术移民加拿大,俺问她感觉加拿大的中小学教育如何,她给俺举了个实例:

她的表妹就读于八年级(还是七年级?),相当于咱们的初一初二吧。那孩子有一次的地理课作业是这样要求的:

给你1,000刀,要求你用这笔钱在暑期中安排一次欧洲游,除了必须游览几个指定城市而外,你可以自己安排其他行程,但费用不得超支。

那个孩子为了做这份作业,上网查机票价格,查旅馆费用,查各大航空公司的联航信息,然后根据各个国家和城市的区域位置,合理安排行程。为了节约费用,为了能够多去几个自己想去的国家或城市,她就必须要像旅行社那样,尽可能地安排坐红眼航班,住青年旅馆。。。


这样的作业没有绝对的对与错,但是却给孩子提供了一个非常好的机会,使他们不仅能够对相关的地理知识印象深刻,同时还是结合生活实际的学习统筹安排事物,合理分配支出的实战型训练。

依俺看来,这样的学习方法比枯燥无味地死背硬记强得太多了。常年接受这种训练的孩子,他们的综合能力一定不会很差。

家园移民 : 2008-06-21#118
回复: jenniferlee的精彩发言

俺LG常说:俺们一辈子最大的投资,无论是金钱,还是精力和时间,其投资方向都非儿子莫属。


从教育投资的角度考虑,如果孩子迟早要出国念书,那么作为外国人所需支付的四年大学的费用,以及之前有可能要花费的语言学校的费用,加一起,估计与俺们现在支付的投资款相去也不会太远。


这半年来,给俺儿子找了外教一对一学习英语口语(JUST FREE TALK)。说实话,没有语境的外语学习,对于本身就还不怎么懂得要学的孩子来说,钱没有少花,而效果实在是差强人意。


去年一儿时玩伴从美国回来,她的一席话对俺也有所启发。

她说自己观察过中国孩子的英语学习,发现如果是在20岁之前去的孩子,那么基本上可以做到没有口音。


此外,她也提到,因为她的孩子出生在美国,所以汉语只能听懂基本的生活用语,表达能力较差。而俺朋友两口子都是大学毕业后去的美国,英语口语表达能力有限,从而导致他们目前已经明显地不能就人生,社会方面深层次些的许多问题,与孩子进行交流了。他们很担心孩子进入青春期后,父母无法对他进行有效的指导,而这些对于孩子的健康成长,实际上是非常重要的。


今年初,俺一从事IT业的表妹从加拿大回来。据她说:她发现在国外,多数老印比老中们混得好的原因,在于老印们从小接受的是英语教学,他们的思维方式比较老中西化得多。尽管他们也许技术上不见得比老中们强,但语言优势和思维方式的接近,却使得他们更容易为西人所接受,从而更易获得高薪和职位晋升。


这让俺联想到上海人与北京人在子女教育选择方向上的差异。北京的学校特别注重奥数成绩,而上海人则看重英语水平。上海人重视英语的原因,是与上海的外企公司众多,并且外企员工外语水平的高低,能够在很大程度上,决定他/她的收入水平相关。俺家有几个亲戚自大学毕业就在摩托罗拉,英特儿这样的大外企混的,其收入水平(算上期权)比在加拿大工作还高得多。


大家都知道上海人的注重实用。就这点而言,上海人是比北京人精明些。试想,学好奥数充其量能够保证考上好大学(还不能完全保证),即便进了好大学,出来也未必见得就能找着好工作,更别提确保一生的无虞了。而生活在平的地球上的人类,语言作为最宽泛使用的沟通工具,其重要性无论怎样强调,也是不为过的。


每个人一生的时间和精力都有限,如何帮助孩子合理分配,善用宝贵的时间和精力,是每个为人父母的人面前的重大课题。套用梁溪香榭的话说,鱼和熊掌无法兼得。用俺的土语表达,叫针无两头利。道理一样,就看自己如何取舍啦。


经常教育儿子:一生只做一件事,那就是推销自己。推销自己的能力,推销自己的公司,推销自己的产品。只要你自己的个人价值能够为世人所接受,那么,你这辈子就注定能与成功结缘。然而,如果没有流畅的语言交流能力,就很难想象如何能够事半功倍,高效率地推销自己.如果没有对于不同文化背景下产生的对许多事物看法的认知和理解,也难以保障推销自己工作的有效进行.


为了这个原因,俺也想选择合适的时机,带孩子移民过去,让他在自然的环境中自然地学习外语,同时也接触和接受与俺们的文化迥然不同的另一种文化,为他将来的人生道路,多做一些铺垫。

家园移民 : 2008-06-21#119
回复: jenniferlee的精彩发言

说来惭愧,俺从小到大,是个没有信仰的人:wdb4:。无忧无虑无任何思想负担无任何党团费要交地幸福成长(傻人有傻福):wdb6:


俺的外公外婆都是虔诚的基督教徒,一辈子乐善好施,终得善果。俺外婆80岁时某个周日,在做完礼拜回家的途中倒地不起,驾鹤西归时心境一派澄澈。俺外公80岁开始学德语,也是在心灵异常清静的状态下,撒手人寰的。两位老人家都没有受什么罪。


俺的不少朋友近年来皈依教门,这多少使俺有所触动。虽然俺过去从不少文艺和历史作品中,部分地了解了天主教,基督教的起源,以及她的发展史(其中不乏太多令人发指的残暴罪行)。但不可否认的是:基督教的教义还是劝人向善的。而且,大部分教徒的心地相对纯洁,戒律规范着他们的言行举止。所以,俺也在考虑,是否要选择一个基督教的社会,让俺的小儿也受些正面影响?


LG总是提点俺:加拿大很是寂寞.为了到那里也能够找到组织,俺也在考虑加入姐妹们的行列.至少对于俺来说,信奉基督是有百利而无一害的选择.也许这样,俺的灵魂能够得到拯救,俺的精神世界会更加充实吧.


这也成为俺移民的理由之一,虽然相对弱了些。

家园移民 : 2008-06-21#120
回复: jenniferlee的精彩发言

在陈述下面的理由之前,俺得先把钢盔戴上,以防遭拍时被砸破脑袋:wdb13:


昔日孟母三迁,是为了给孩子创造一个“谈笑有鸿儒,往来无白丁”的生活和学习环境。今天的张王李赵母,为了类似的,或亚类似的理由,也在不断地迁徙,甚至不远万里地择地而居。


不是矫情,但俺的确觉得,现在的孩子由于物质生活条件太过优越,因此而失去了太多奋斗的动力。这使得俺非常担心儿子的未来。


所谓富不过三代,是非常有道理的。第一代的创业者胼手胝足地打下了江山,自己不舍得吃不舍得花的。到了第二代,能够将祖业发扬光大者又有几人?多的是纨绔子弟,坐吃山空的败家子。


虽然俺们并非什么富人,但相对而言,孩子从小没有吃过一点苦。不穷则不思变,多年充裕丰富的物质享受,已然使得他的进取精神廖剩无几,前途堪忧。


因此,俺想改变一下这种状况,带着孩子去加拿大受点苦。这也是俺移民的理由之一。

工作多年,多少有些见识.


俺发现无论哪个领域的成功者,一般而言都是具有高度责任心的人.他们对社会,对事业,对工作,对家庭,对自己,对他人,都秉持着高度负责任的态度.正因为如此,他们才能踏踏实实,认认真真地做好每一件事,即便那是件在世人眼中,无足轻重的琐事一桩.也正因为如此,他们的成长伴随着一次又一次的成功,才得以不断地获得领导的首肯,群众的认同,才能够一路青云,扶摇直上.


无论是在哪一个范畴,无论是在社会,单位,还是在家庭,很难想象一个欠缺责任心的人,能够长期地在该范畴内获得稳定的成就.


俺们的孩子多是独生子女,生活条件优越,学业负担沉重.因此,他们无论在心理上,还是在实际生活中,很少为自己的家庭考虑或做过些什么,更遽论为社会,为他人做些什么了.在国内的这种惟有分数高的氛围中,成绩决定了一切,成绩主宰了一切,成绩绑架了一切.


因为心疼他们的辛苦,俺们自觉不自觉地忽视了他们承担家庭责任的意愿;因为欠缺社会环境的支持,他们也很少有机会得以学习承担社会责任.因为长期施以援手之故,俺们的孩子对自己都不是很负责任,对家长的依赖心理严重.


鉴于此,俺希望能够带着孩子去到学业负担轻一些的国度,让他有限的时间和精力,多放在培养自己对社会,家庭,工作,个人和他人的责任感之上.这也构成俺选择移民的理由.

家园移民 : 2008-06-21#121
回复: jenniferlee的精彩发言

五.一期间,儿子受同学邀请,参加了一个在俺看来非常有意义的活动.一位极有头脑的家长,组织了年龄在6--12岁的十一个孩子,去京郊雁栖湖游玩.她买了不少玫瑰,让孩子们分成两组,采访游客,每人出售一枝鲜花,最后按照利润总额和采访到的信息量的多寡和采访质量,进行总结评比.


这种活动在国内鲜少见到,因此,孩子们卖花的举动频频地遭到防范心重的部分游人的质疑.这在很大程度上挫伤了部分孩子的积极性.俺大致统计了一下,俺儿子所在的这组(他最大,是组长)六个孩子,平均每卖出一枝鲜花,需要向5,6个成人售卖,尽管孩子们事先已经定下销售策略,专找那些衣着光鲜的情侣去进行推销.奈何许多国人欠缺这种扶助幼小的爱心,因此售卖活动进展地不太顺利.与俺儿同班的一女孩率领着另一个小组,身为大队委的这个可爱的小姑娘,在碰了几十次钉子后,居然一朵花都没有卖出去.最后孩子痛哭失声,让人不忍.


俺家小儿在是次活动中表现极为优异.尽管屡屡遭拒,但他们愈挫愈勇,令俺对其刮目相看.通过这次活动,大幅提高了孩子的自信心,语言表达水平和组织管理能力.咱们的孩子其实一个比一个聪明,如果社会和学校,家庭能够多为他们创造出体现他们自身价值和能力的条件,他们的前途会是多么地光明啊.


几年前,曾经看过薛岳的一篇博客文章,讲述了他刚去美国的9岁女儿第一次参加童子军活动的经历.孩子们准备了一些饼干,向路过的大人们售卖.由于成年人们也是曾经的童子军,因此,他们大都微笑着掏钱买下了高价的饼干.成功的喜悦令开始时畏缩不前的小姑娘信心倍增.事后,孩子们一起协商,将成本归还大人后,赢利部分又分为几份:一部分捐给社会福利机构(虽然数量很少,但培养爱心,意义重大),一部分送给曾经帮助他们组织该次活动的大人(学会投桃报李),最后剩下的,才是他们自己的所得(通过自己努力所得到的,与家长们给予的相比,其意义不可同日而语).


一次,孩子对他的老外老师说:他很想去卖报纸挣钱.老外对俺说:他听了这话非常诧异,因为跟着他学英语的中国孩子,家里生活条件都不差.他在中国教了三年英语,还从来没有听说哪个孩子有过这种意愿的.然而在美国,那是再正常不过的事了.他本人从12岁起,每当向妈妈要求某样非学习和生活必需品时,他的妈妈就会说:好啊,买去吧,不过你要自己去挣这钱.年已26岁的他,至今非常感谢妈妈当年的不予纵容.他说,他的妈妈让他从小就学会了自立,自强,和自信.他在课余时,就去帮邻居剪草坪,送报纸,等到可以合法开车的年龄时,他就用自己多年积攒下的钱,为自己买了一辆车,然后开着车去大学报到.


相形之下,北大清华校园旁的民宅里,却租住着许多外地前来陪读的家长.俺一亲戚家的孩子,目前在号称北大清华摇篮的四中就读,北京市奥林匹克数学和物理竞赛一等奖的获得者,却不会自己剪指甲.反差之大,令人咂舌.


儿子8岁时,俺就曾有带他去卖报纸的想法,目的是想通过体验普通劳动者的劳动,让孩子体会挣钱养家的不易,从而激励他为了自己的前程而用功学习,同时帮助他杜绝大手大脚浪费金钱的坏习惯的养成.奈何国情如此,真要将这种想法付诸实施,大概警察叔叔就该出面制止了.


为了孩子能够尽早地学会自立,俺选择了移民加拿大,因为那是能够实现俺多年愿望的合适国家之一.

天海茫茫 : 2008-06-30#122
回复: Lucia的说法

我儿子是90年的,去年5月份到加,插班就读10年级(高一)。就说一下我的情况,希望能对后来的有所帮助。
请孩子所在学校开一个学习成绩证明,中英文的各一份,自己交公证处公证。因为我儿子在国内高中一年级读了一个学期,所以就出示了两份证明(初中、高中各一,证明样本附后)。到教育局报到时,出示了出生公证、学习成绩公证和移民纸,并做一些测试就按照你的所住地分配到学校上课了。到现在他就读11年级,前几天又把学习成绩公证交到学校,可以得到16个学分。这边上大学至少要有30个学分才可以,其中要有6个12年级与大学专业相关的学分。高中每个学期开四门课,一学年可以得到8个学分(这当然要在每门课都PASS前提下),如果学分不够,也可以通过上夜校和summer school来赚学分,每学期最多可以上2个夜校,所以能转到学分是很重要的。另外,那个预防针的证件我们在国内也没有找到,对于我们的大孩子(好像是16岁以上)是没有什么用的,所以没有的也不必担心或费神去弄了。

学习成绩证明

XXX同学生于XX年XX月XX日,男(女),系我校2006级初(XX)班学生,2003年9月至2006年7月在我校就读初中,经过三年的学习,成绩优秀,于2006年7月30日初中毕业,初中学年的成绩(百分制)如下:

因为表格在这里发不了,有需要的给我悄悄话。 高中的也比照这样的就行。
有事向你请教,如何和你联系。谢谢!

家园移民 : 2008-06-30#123
回复: Lucia的说法

有事向你请教,如何和你联系。谢谢!
这个是偶摘录的,偶也找不到阿

天海茫茫 : 2008-06-30#124
回复: Lucia的说法

这个是偶摘录的,偶也找不到阿
噢,谢谢守法先生了

googlebig : 2008-07-12#125
回复: 子女教育信息素材收集帖

I don't know why my New PC can not write Chinese.

But anyway, Thanks for your hard work.

Now, My child's School holidays is comming, I cn't find the teaher to write my son's record or certification. But we have a student manual where can show the record, is it ok to register middle school 9 grade in Canada?:wdb21:

家园移民 : 2008-07-12#126
回复: 子女教育信息素材收集帖

I don't know why my New PC can not write Chinese.

But anyway, Thanks for your hard work.

Now, My child's School holidays is comming, I cn't find the teaher to write my son's record or certification. But we have a student manual where can show the record, is it ok to register middle school 9 grade in Canada?:wdb21:
细节还真不了解阿,呵呵

xj830 : 2008-07-16#127
回复: 子女教育信息素材收集帖

请问守法:在伯城公立小学较好的有吗?住在伯城可以上ubc大学中的公立小学吗?

老牛牛 : 2008-08-10#128
回复: 子女教育信息素材收集帖

太好了,谢谢!

sunnytan : 2008-08-12#129
回复: jenniferlee的精彩发言

说来惭愧,俺从小到大,是个没有信仰的人:wdb4:。无忧无虑无任何思想负担无任何党团费要交地幸福成长(傻人有傻福):wdb6:


俺的外公外婆都是虔诚的基督教徒,一辈子乐善好施,终得善果。俺外婆80岁时某个周日,在做完礼拜回家的途中倒地不起,驾鹤西归时心境一派澄澈。俺外公80岁开始学德语,也是在心灵异常清静的状态下,撒手人寰的。两位老人家都没有受什么罪。

从不少文艺和历史作品中,部分地了解了天主教,基督教的起源,以及她的发展史(其中不乏太多令人发指的残暴罪行)。但不可否认的是:基督教的教义还是劝人向善的。而且,大部分教徒的心地相对纯洁,戒律规范着他们的言行举止。所以,俺也在考虑,是否要选择一个基督教的社会,让俺的小儿也受些正面影响?


LG总是提点俺:加拿大很是寂寞.为了到那里也能够找到组织,俺也在考虑加入姐妹们的行列.至少对于俺来说,信奉基督是有百利而无一害的选择.也许这样,俺的灵魂能够得到拯救,俺的精神世界会更加充实吧.


这也成为俺移民的理由之一,虽然相对弱了些。
GOD BLESS YOU!
我移民的一个理由也是如此!作为一个基督徒去一个基督教文明的国度是幸福的!
愿耶和华神保佑我们在移民路上一路走好!
报告:女儿 89年生 刚参加完高考
   儿子 92年生 升高二
谢谢守法和大侠以上信息!真正的家园感觉:wdb6:

惴惴不安 : 2008-08-16#130
回复: 子女教育信息素材收集帖

楼主好,我朋友的儿子91年生,现在高三读书,成绩不错大约能考上国内一本大学,她办理移民加拿大明年能登陆,孩子随行上大学,她的准备什末材料?国内高考成绩在加有用吗?加拿大工科大学那所有名?请帮忙解答,先谢了。

咖啡壶 : 2008-08-16#131
回复: 子女教育信息素材收集帖

请问一个问题:温哥华初中的春、秋季学期的开学时间分别是几月份?谢谢!

家园移民 : 2008-08-17#132
回复: 子女教育信息素材收集帖

请问一个问题:温哥华初中的春、秋季学期的开学时间分别是几月份?谢谢!
1月和9月

家园移民 : 2008-08-17#133
回复: 子女教育信息素材收集帖

楼主好,我朋友的儿子91年生,现在高三读书,成绩不错大约能考上国内一本大学,她办理移民加拿大明年能登陆,孩子随行上大学,她的准备什末材料?国内高考成绩在加有用吗?加拿大工科大学那所有名?请帮忙解答,先谢了。
高考成绩有用
需要准备的资料:高考成绩单、会考成绩单、高中三年的成绩单、老师的推荐信,以及其他的专业特长的证明
工科大学:迈基尔,多大,滑铁卢等的都不错的

惴惴不安 : 2008-08-17#134
回复: 子女教育信息素材收集帖

谢谢首法:这些材料都要公正吗?

咖啡壶 : 2008-08-17#135
回复: 子女教育信息素材收集帖

1月和9月
谢谢守法!那么请问想过去读1月份,那么什么时候要报名的呢?

家园移民 : 2008-08-17#136
回复: 子女教育信息素材收集帖

谢谢首法:这些材料都要公正吗?
需要公证阿

家园移民 : 2008-08-17#137
回复: 子女教育信息素材收集帖

谢谢守法!那么请问想过去读1月份,那么什么时候要报名的呢?
偶给嫩确认下去

塞北的雪 : 2008-08-17#138
回复: 子女教育信息素材收集帖

非常有用,收藏了。俺儿子15岁了,开学就读初三了。

惴惴不安 : 2008-08-17#139
回复: 子女教育信息素材收集帖

他还需要托弗或雅思成绩吧?是在国内考?还是在国外考?需多少分?他家办的是联邦投资移民hk07年4月份fn,06年8月递材料,请守法帮分析一下,他该如何办理才能不耽误孩子的学业.

Hellen luo : 2008-08-17#140
回复: 子女教育信息素材收集帖

谢谢了!

家园移民 : 2008-08-18#141
回复: 子女教育信息素材收集帖

他还需要托弗或雅思成绩吧?是在国内考?还是在国外考?需多少分?他家办的是联邦投资移民hk07年4月份fn,06年8月递材料,请守法帮分析一下,他该如何办理才能不耽误孩子的学业.
目前要求亚斯成绩的比较多,国内考比较容易出成绩的

各个学校的要求不一样的,具体可以看看各个学校的网站了

惴惴不安 : 2008-08-19#142
回复: 子女教育信息素材收集帖

如果孩子在大一读书时才能登陆,能转学吗?如何转?需准备什末材料?能转学分吗?

小小资 : 2008-08-21#143
回复: 子女教育信息素材收集帖

男,93年7月,现在新高一,已体检,估计过完年登陆,请问:这半年是在高中读还是去专读语言?谢啦

家园移民 : 2008-08-21#144
回复: 子女教育信息素材收集帖

如果孩子在大一读书时才能登陆,能转学吗?如何转?需准备什末材料?能转学分吗?
可以转阿

具体的规定其实各个学校的网站写的挺清楚的。最好找个英文利索的,然后针对相应的大学看下,不是很复杂的

家园移民 : 2008-08-21#145
回复: 子女教育信息素材收集帖

男,93年7月,现在新高一,已体检,估计过完年登陆,请问:这半年是在高中读还是去专读语言?谢啦
建议:

一定等pl了,再做决定。me后并不能完全保证。如果耽误了孩子的教育就是大事了。

小小资 : 2008-08-22#146
回复: 子女教育信息素材收集帖

建议:

一定等pl了,再做决定。me后并不能完全保证。如果耽误了孩子的教育就是大事了。

万分感谢。

mmintwd : 2008-09-24#147
回复: 子女教育信息素材收集帖

非常有用。谢谢

陌上守望 : 2008-09-25#148
回复: 子女教育信息素材收集帖

非常有用。谢谢
这个帖子太有用了,强烈要求置顶!:wdb23::wdb23::wdb23:

梁溪香榭 : 2008-11-16#149
偶搜集到的一些关于加拿大高中教育的资料 与大家分享

与很多这里的同学一样, 预计俺的儿子也是要到高中阶段才能出去.
搜集了一些高中教育资料, 希望能帮到大家.

ZT
加拿大高中留学需谨慎

篇首语:
1、如果你是在国内高二毕业,想在一年之内顺利的考上加拿大大学,请放弃(除非你是天才)
2、如果在你选择的是私立学校,不用浪费时间看这篇文章了

或许我开篇的两句话有点偏激,但也是不争的事实,我是在一名在加拿大留学的高中生,因此我不会像中介一样去美化这个留学有多好、前途有多光明,我会实事求是的写下我经历的一切。我在国内完成了高二的学业,按照计划,应该是在加拿大完成Grade12也就是高三的学业就申请大学,但听完多伦多教育局举办的orientation(类似介绍会)后,我明白了,1年是不可能顺利的考上大学。

教育体制的差异。加拿大的教育体系多是以学分制为主体,也就是说,你每一门课都有一个学分,当你把学分修满后才可以高中毕业,所谓的年级:Grade10、Grade11、Grade12(国内叫高一高二高三)在加拿大实际上是没区别的,只是作为一个作为区别年龄的标准(我是指对像我这种国际交流生),就算你是在Grade12你也可能同时修Grade11和Grade12的学科,因为加拿大要求如果你要读更高一级的学科,必须有低一级的学科作为基础,不能随便越级,例如说Grade12的微积分就需要Grade11的函数作为基础,当你拿到了Grade11函数的学分后才可以修Grade12的微积分,而当你拿齐了所有的必修的学分(30分)才可以申请大学,因此,作为一个国际生,我认为一年之内很难或者说不可能解决语言的问题并且修齐所有的必修学分。​

梁溪香榭 : 2008-11-16#150
回复: 偶搜集到的一些关于加拿大高中教育的资料 与大家分享

语言是最大的障碍。首先我先说说International Student最先接触的英语分级制度,这个等级制度是对immigration的英语水平进行分级的制度,英文简称:ESL,全称:English As Second Language。以我自己本身做例子:当我抵达多伦多的第2天就去Toronto District School Board(多伦多教育局)注册,并且给了我一个档案,档案的封面上写着我Assessment和Orientation,接着按照日期去考试,考的内容在我前一篇日志已经写了,就是ESL和数学,我的ESL级别是C。整个ESL级别有ABCDE,A是代表你水平不行,B代表一般般,C代表中等(和加拿大高一学生的水平一样),D和E就是代表已经不错了,当你ESL得到E了之后,还有ENG这一个等级制度,这个ENG就考的全是各科的专业词汇了,很难很难,ENG里包含了3U和4U,本地的学生考4U也有一定难度,但是具体考什么我还真不清楚。如果你ESL级别越低,你的高中就会给你安排越多的课程以提升你的英语水平,如:艺术、加拿大历史、加拿大地理等等,以使你可以接触更多的词汇从而提升英语水平。

梁溪香榭 : 2008-11-16#151
回复: 偶搜集到的一些关于加拿大高中教育的资料 与大家分享

还是学分问题,我前面提到了,30个学分就可以高中毕业,但这并不准,并且我至今也不知道教育局是按照什么标准算出这30个学分的,接下来我好好算算学分你就清楚了。例如我,是在国内的高二毕业,把我所有的成绩拿到加拿大可以换成25个学分,30-25=5,那是不是说我再修5个学分就可以毕业了?不可能,2-1不等于1,因为你英语水平不够,就算你国内成绩再好,数学再厉害,那也是中文的,加拿大所有教材都是英语,虽然你是满腹经纶,但都是Chinese不是English,也正因为鬼佬知道你是满腹经纶、你是有学过这些知识但是语言不通为了让你学习语言才认可你在国内的成绩,正是因为这个原因把我国内的成绩转化为25个学分,但是你在加拿大学习的时候,课程不仅仅是英语,还有数学什么的,主要是看你选哪课,反正一共是8课,最终的结果是你在加拿大又获得了十几个学分,加起来可能有40个学分才申请大学。接着还是用我自己做例子,我测试的数学的水平是Grade10,也就是高一,那我就要从Grade10学起,虽然可能都是我学过的,但是我仍然要再次学习,目的还是为了语言!因为那些题目如果换成Chinese,唰唰唰一会就可以搞定,问题在这里,题目全是English,有太多太多的专业词汇了,在我学习Grade10的数学的同时也为我累计了学分,有助于大学录取。

梁溪香榭 : 2008-11-16#152
回复: 偶搜集到的一些关于加拿大高中教育的资料 与大家分享

琐碎的小问题很多很多。国内的课改不是要把平时的成绩、课堂表现、作业、团队合作等等都纳入最终衡量的标准吗?当然有高考的存在,这一切都是徒劳,但起码这样的做法在加拿大就是有效的,加拿大没有高考,大学的录取是依据你平时成绩、课堂表现、作业、团队合作的情况、有无义工的工作记录等等综合起来再加上你各学科的分数以及Toefl的成绩来决定的,就我个人而言,我国内的数学水平以及是高二了,但是在加拿大我仍然必须重新学习高一的数学,并且我不能因为说太简单就逃课,因为逃课会记录入档案,严重影响大学的录取,说白了,j就是要老老实实的和加拿大的学生一起学习。我也向加拿大教育局咨询过有没有什么方法可以加速这整个进程,例如去私校或是越级什么的,人家的回答绝了:“有这个必要吗?如果你的孩子的水平并没到达这个水平,但是你硬是推他去面对这一次,只会令孩子一次次的失败,你希望你的孩子有无比沉重的挫折感吗?”就从这一句话中可以明白为什么加拿大的孩子都这么轻松,不知道什么时候国内的家长也可以领悟到这一点,或许等到中国也和加拿大一样的发展水平的时候,这一切才会有改变。

国内有很多中介说保证可以把你送进大学,但是聪明人都知道,国外的大学都是送进严出,水平不行是不可能毕业的,所以我选择不相信中介,在加拿大的公立高中好好的打一年基础再决定

梁溪香榭 : 2008-11-16#153
回复: 偶搜集到的一些关于加拿大高中教育的资料 与大家分享

ZT 加拿大高中教育感悟(转)
移民加拿大五年多来,陪女儿读书是我的主要生活内容。通过陪读,使我对加拿大高中教育体系有了一定的了解。我感到,加拿大的高中教育,无论是课程设置,还是教学方法,都体现了实用性和趣味性的特点,能够充分发挥孩子的创造力和想象力,让孩子对学习和学校的生活充满渴望,感到充满乐趣。

  加拿大的高中采取学分制,每门课一个学分,修满30分即可毕业。30分中,有16分是必修课,其中包括英文,数学,自然科学,历史,地理,体育,艺术,家政,法语,基础电脑应用等;其余的14个学分,由学生根据自己的爱好和未来的发展设想进行选课。在选修课中,有五个系列的课程,学生以一个系列为侧重点,由低向高选修上去。

  五种选修课程系列为:商科系列:包括统计、会计、各种商业法律法规、消费学、生产力布局学等;自然科学系列:包括物理、化学、生物、高等数学各个分支、电脑高级语言等;艺术系列:包括戏剧、绘画、摄影、各种美术设计等;技术系列:包括电工、木工、管工、汽车修理、房屋修缮等;社会科学系列:包括各历史学科分支、社会学、人类学、人文学等。另外还专门开设驾驶课程,以便让16岁以上的学生尽快取得驾照。这样的课程设置,使学生从9-10年级 (14-15岁) 起就开始考虑自己未来的发展方向,开始侧重选修与这一方向有关的课程,并根据自己的兴趣在这一领域里进行一些深入的研究。所以,加拿大的高中毕业生尽管在数理化总的方面不如中国学生学的深,但在某一领域的知识和参与能力却非常强。

  这种课程设置使加拿大的高中毕业生适应社会的多种需求,一部分学生进入大学,将来可能成为政治家、科学家、工程师、工商管理人员、医生、律师、教师等,而那些不能进入大学甚至不想上大学的学生也能通过自己的努力,凭着自己的一技之长在社会上自立。

梁溪香榭 : 2008-11-16#154
回复: 偶搜集到的一些关于加拿大高中教育的资料 与大家分享

加拿大的高中教育不仅在课程设置上有上述特点,而且教学方法上也有其独到之处。老师注重培养学生的实践和动手能力,从学生的日常生活能力教起。例如,家政课是一门必修课,其内容包括烹调、缝纫、家用电器的使用和简单维护,房屋的简单修缮,如何做父母及喂养小孩等,而且每项内容都配之以实践:学校有专用的厨房让学生们实践做饭,有缝纫机供学生们做自己设计的衣服,有车间共学生们做木工活,还有专门设计好带电脑程序的假娃娃供学生们带回家里”喂养” ,老师根据电脑记录给分,老师还专门带学生去附近的幼儿园实践如何带弟弟妹妹。再如,学校专门为学商科课程的学生开了一个小卖部,只在学校内部营业,经营管理和日常运作全部由这些学生负责。

  注重发挥学生的创造能力和参与意识是其教学方法的另一个特点。老师讲课并不面面俱到,而是提倡学生自学和思考。有时老师专门安排一两个自学单元,让学生自学后,选一个重点给全班讲出来或提交读书报告。这几年来,我女儿不知写了多少读书报告,每一次都要上网或去图书馆查阅大量资料,有些读书报告还要求图文并茂,所有报告后面都必须附上参考书目录和引文出处,而且要打印得干干净净,装订得整整齐齐,才能交给老师,俨然像中国大学生或研究生提交的论文。考试也不只要求死记硬背,很大部分内容要求学生的独立思考。

例如,我女儿上10年级的地理课考试,老师出的题目为:在某个社区拟建一个超级市场,老师给出了这个社区的基本状况,如地理位置、人口及结构、收入水平、交通环境、超市的规模等,让学生们通过自己的分析得出建超市是否可行的结论。而且,答案也不要求与老师的看法一致,只要自圆其说即可。当时我感到,这种题目在中国的学校,只有大学学城市地理的学生才会遇到。尽管对于10年级的学生来说,这个题目比较难,他们不可能分析得很透彻全面,但毕竟是一次很好的实践。又如,一次我女儿美术课的期末考试是,老师给出几个主题,学生们任选一个主题,设计一张加拿大邮票并附设计说明。电脑课的期末考试是用所学的语言设计一个自己喜欢的游戏软件,我女儿为设计这个软件,熬了好几个通宵,不仅一句怨言也没有,而且还其乐无穷。她经常跟我说,”上学太有意思了,每天都很 EXCITING(振奋) ” 。孩子有这样的心境面对学校和学习,她能学不好吗?

梁溪香榭 : 2008-11-16#155
回复: 偶搜集到的一些关于加拿大高中教育的资料 与大家分享

加拿大的学校还注重培养学生与他人的合作协调能力,鼓励学生在公众面前表现自己。从小在加拿大上学的孩子对PROJECT(课题) 和PRESENTATION(演示) 这两个词都不陌生。PROJECT大都需要和几个同伴一起完成,根据老师的题目,大家商量对策,分头准备,然后一起动手写,或画,最后在班上宣讲或表演出来,也就是PRESENTATION 出来。几乎每一门课都有这样的教学环节。学生们通过这样的教学环节,不仅锻炼了与他人的合作协调能力和组织能力,提高了自信心,而且还享受着思考与创作的乐趣。例如,我女儿选的戏剧课,高年级期末考试是和几个同学一起写一个剧本并把它表演出来,从写作到表演,从服装、道具到音乐、灯光,全由他们自己解决,为了得到好成绩和与其他小组竞争,每个人都认认真真的工作,积极地出谋划策,这无形中培养了他们的团队意识和集体荣誉感。当时我就想,这对他们将来走上社会与他人一起配合工作是一种多么好的体验和锻炼埃

  另外值得一提的还有他们的考试制度。加拿大高中的期末考试并不可怕,因为期末考试仅占总成绩的25%,其余的75%由平时的每一次作业,每一个小测验,每一项PROJECT和PRESENTATION以及课堂上的参与意识、出勤等因素综合组成。总之,一门课要想得到好成绩,就要付出持之以恒的努力,就要积极地参与。上大学也没有类似中国的”高考” ,而是以高中最后一年6门课的平均分去申请。但如果一个学生历年来经常参加学校和社区的活动,或做志愿者,或有一些工作经验,即使平均分低一些,大学也愿意优先录取这样的学生,因为加拿大的大学也不愿意培养那些只会读书,高分低能的“书虫” 。这里的人们认为,人的能力可以从各个方面表现出来,而只会读书的人并不一定能够适应社会。

  同时,学校也为学生早日适应社会,提高各种能力提供了许多条件。在加拿大,如果孩子有兴趣,可以在学校参加各种社团组织和活动,或去社区做志愿者,以锻炼他们的各种能力。例如,学校有数学俱乐部,科学俱乐部,戏剧俱乐部,足球队,冰球队,小乐队等等,社区图书馆、敬老院、活动中心等机构都欢迎高中生去那里做志愿者,有些机构还为长期在那里做志愿者的学生提供上大学的奖学金。我女儿从13岁到了这里,一直参加学校的话剧俱乐部,一连五年,年年在学校的戏剧节演出。从排练到演出,所有的演员,工作人员,舞台监督,布景,化妆,灯光、音乐,道具等工作全部由学生承担,老师只负责指导,这是一个涉及几十人的大 PROJECT。通过参加这项社会活动,她不仅很快提高了英语水平,弥补了其英语的先天不足,增强了自信心,锻炼了她的组织能力和与别人的协调配合能力,而且通过在话剧俱乐部里和当地孩子的接触,更多地了解了这个社会的风俗、文化,为其将来走入这个社会做了准备。

梁溪香榭 : 2008-11-16#156
回复: 偶搜集到的一些关于加拿大高中教育的资料 与大家分享

英语, 重中之重

北美高中的词汇量是8万左右,大学毕业要达到20多万了。

而我国有些所谓的专家说,英国农民2000词汇量足以应付日常生活了,并以此推而广之,把大学的4级词汇量定为4000左右。这完全是一种误导

惴惴不安 : 2008-11-16#157
回复: 偶搜集到的一些关于加拿大高中教育的资料 与大家分享

溪溪,这个帖子太好了,谢谢!!

lucky_gg : 2008-11-16#158
回复: 偶搜集到的一些关于加拿大高中教育的资料 与大家分享

好贴

梁溪香榭 : 2008-11-16#159
回复: 偶搜集到的一些关于加拿大高中教育的资料 与大家分享

纯粹是搜集来的资料, 其真伪请大家自行辨别.

ZT 看成绩选加拿大学校
1、高考分数一本以上:有这样的好成绩你可考虑直接进入加拿大最好的大学。如女王大学、多伦多大学、不列颠哥伦比亚大学、麦吉尔大学、西安大略大学、麦克马斯特大学、约克大学、西门・菲沙大学和其他大学。对于大多数四年课程的大学,还要求托福成绩在580/600分左右。

  2、高考分数在二本:要在著名的加拿大大学中完成4年课程的学习,挑战性是非常强的,如果你的高考分数低于二本,或没有高考成绩,想进入加拿大著名的大学是非常困难的,只能进入大学的英语强化训练班,然后转到预科课程,预科合格后才能获得直接进入大学的资格。

  根据特殊的安排,你可以直接进入圭尔夫大学、布鲁克大学、全特大学、马尼托巴大学、圣马丽大学和其他大学。你还可以直接进入马拉斯比娜大学学院、乔治亚学院、不列颠哥伦比亚理工大学、乔治・布朗学院和其他大学。一般来说,对于直接入学,要求托福成绩在550/560分左右。

  3、高中学习成绩在C级或以上,但未参加高考:你可以直接进入马拉斯比娜大学学院、乔治亚学院、卡里布大学学院、不列颠哥伦比亚理工大学、乔治・布朗学院和其他大学。一般来说,对于直接进入大学,要求托福成绩在550/560分左右。如果你是高三的在读生,现在可以开始申请学校,你可以选择进入圭尔夫大学、布鲁克大学、马拉斯比娜大学、乔治亚学院、卡里布大学学院和其他大学的转学或预科课程的学习。

家园移民 : 2008-11-16#160
回复: 偶搜集到的一些关于加拿大高中教育的资料 与大家分享

坐个板凳啊

梁溪香榭 : 2008-11-16#161
回复: 偶搜集到的一些关于加拿大高中教育的资料 与大家分享

如何选择最适合的加拿大大学
加拿大有70几所大学,大多数大学教学质量优良,其中不乏国际知名的一流学府,如英属哥伦比亚大学(University of British Columbia)、多伦多大学(University of Toronto)、西安大略大学(West Ontario University)和皇后大学(Queens University)等。

首先,在众多的学院中,如何选择一个或几个适合自己的学校呢?以下的几个方面是你必须注意的:
一、大学的历史
学术成就是累积而得的结果,新近成立的大学虽然设备先进、校园美观和朝气蓬勃,但在短期内很难有较大的学术成就;反观历史悠久的大学,虽然校舍较为古旧,作风传统,但长久以来,即使不是人才辈出,也会有一些杰出精英,这样,积聚数十或百多年的成就和经验,在学术和社会上,早已建立一定的基础与荣誉。
有时,两所大学的实际水平可能差不多,但来自历史较久大学的学生,在就业时会略占优势,因为一般雇主对大学的认识,多在“听过”和“从未听过”的层次。此外,早十几年毕业的师兄师姐,在行内已奠定所谓“江湖地位”,照顾同校的师弟、师妹只是举手之劳,也是人之常情。
笔者有位同学专程由加拿大西岸到东岸的戴候斯大学(Dalhousie University)读法律,就是为了它的法学院有悠久历史,现在加拿大不少大法官大律师都是出自这间大学的。所以,历史悠久的大学自然建立一定的名声,慕名而来的学生自然不少,大学便可从中挑选资质较佳者,学生的素质高,校誉自然日见昌隆。
二、教师与学生的人数
这两个数字直接反映大学的规模。很多规模较小的大学只有一幢建筑物,资源缺乏,别说没有大片草坪供人坐卧,就连图书馆的座位也少得可怜,其它设施,如学生中心、运动场等多在大学范围以外,或者根本就没有。此外,小型大学开办的课程也有限,能提供给学生的选科甚少;有时,名义上是同一个学位,如Bachelor of Commerce,大型大学的学生会多读几科较特别的科目。
虽然小型大学的条件不足,发展大受制肘,但也有其优点:1)学生人数少,教师对学生,尤其是英语较差的留学生,照顾较周到,所以师生关系甚为亲切;2)由于声望稍逊一筹,有进取心的小型大学会努力发展,在师生齐心之下,往往出现令人意外的成就。
另外,从这两个数字可以计算出师生的比例,理论上师生比例越高,教师对个别学生照顾越少。
三、图书馆藏量
图书馆提供资料的质与量往往决定大学的成就,图书馆的藏量高,学生寻找资料自然得心应手;否则,明明知道某几本书或期刊合用,图书馆却偏偏没有,学生便要到别的地方找,费时费事;若改用其它次要的资料,又影响成绩。所以,在选择大学时,图书馆的藏书量是其中一个重要的考虑因素。
四、重点院系
有些中、小型大学会集中全力发展个别学系,以求该系水平在短期内大幅提升,于芸芸学府中建立良好声望,再由学系的成名促成整所大学的成名。于是,这类学系在大学里成为骄子,师资、设备和课程设计等都凌驾于其它学系之上,读这类学系的学生必定获益良多,当然值得考虑。如滑铁卢大学的计算机系和约克大学的商学院等。

梁溪香榭 : 2008-11-16#162
回复: 偶搜集到的一些关于加拿大高中教育的资料 与大家分享

坐个板凳啊

领导讲坛上请 :wdb6::wdb6::wdb6:

梁溪香榭 : 2008-11-16#163
回复: 偶搜集到的一些关于加拿大高中教育的资料 与大家分享

溪溪,这个帖子太好了,谢谢!!


谢谢鼓励
这些都是俺特别关注的, 相信大家也都一样. 哈哈.

梁溪香榭 : 2008-11-16#164
回复: 偶搜集到的一些关于加拿大高中教育的资料 与大家分享

读高二的孩子到加拿大需准备什么

孩子尤为重要的准备:英文,英文,还是英文!
从去年10月开始,在加拿大境内没有受满4年义务教育的移民孩子上大学都要考托,而且是改了形式的考托(以前重读写,去年10月开始重听说)这次改进的托福形式考试对中国大陆孩子很不利。250分为合格。
如果是来加拿大读高三,对孩子来说还是很具挑战性的。因为在加拿大孩子如果快到18岁了(或19岁?)还没有拿到所有高中毕业的学分的话,教育局会让孩子到成人高中继续修课,直到所有学分拿到,完成高中毕业的条件才可以申请大学,另外还要加试托福(或雅丝)。这样就有可能会比国内的同学晚几年上大学。
安省的高中可以读到21岁,直到修完相应学分和必修课为止,很多高中过来的孩子都是在高中多读一两年,为的是适应英文写作,为将来上大学打好基础,也免得上大学太辛苦。所以孩子过来进入高中后,可以根据孩子的情况决定何时申请大学。

梁溪香榭 : 2008-11-16#165
回复: 偶搜集到的一些关于加拿大高中教育的资料 与大家分享

温哥华读高中?

1 在温哥华必须读满4年才能不考托福申请大学.也就是说不能让孩子来读高一(国内的说法),而要从初三读起才不用考托福,否则要考托福.为什么讲得这么罗嗦,因为身边很多朋友一说移民都说等孩子初中毕业去加拿大读高中,基础知识也牢固了又不用高考那么辛苦,这是一大误区.

2 高中必须修满一定学分才能申请.通常孩子来了要上ESL班(那些英语超好的孩子不在此列啊),对第一年的选课造成限制,导致第一年的学分不满.这个好像可以上暑期班来弥补,但是孩子还是蛮辛苦的.

3 高中毕业的年龄不超过19岁,比如你在12年上半学期年龄达到19岁,下半学期就要离开学校去上成人高中. 成人高中学习环境没有普通高中好(主要是指气氛吧).当然要避免这点你可以选择到其他省读高中.

4 在成人高中要修三门成人高中自己的课才能毕业.成人高中要交选课费.

5 或者去读COLLEGE,修满学分再转大学.这样好耽误时间也花钱.

将孩子送来读高中不是很好的选择,能早就尽早吧,否则在国内读个国际高中再申请国外大学也蛮好的.

梁溪香榭 : 2008-11-16#166
回复: 偶搜集到的一些关于加拿大高中教育的资料 与大家分享

高中英语第一门课程:English Skill 之内容篇
它是一个相对比较综合的课程,但重点还是写作和语法。我很喜欢这里老师的授课方式,灵活多样,互动性强!这门课的老师经常因为我们不跟他互动,很生气~~, 没办法,他想一天两天改变我们几十年的“优良传统”,那怎么可能呀!

课程内容主要包括以下几个方面:
(1)针对说:
a.老师给一个主题和相关的问题,然后分小组讨论。老师会到每一个小组的讨论看看讨论的情况,然后给一些意见!
b.老师会给一些词,两个人一组,一个人描述,另一个猜!很惭愧,第一次有机会玩猜词游戏!

(2)针对听:
a.老师会介绍一些连读,爆破,重音等基本的知识。这部分很少,只有一节课时介绍这方面的。但是感觉这部分自己很强呀!没办法,在国内把语法记得太死啦!有利有弊
b.老师读一篇文章,让大家抓主要的信息,然后做一些练习。有点像我们中国的听力练习。关键是老师太高估我们啦,就给听一边,鬼才知道说的是什么!也许是因为这部分不是本门课程的重点吧!

(3)针对读:老师会给一篇文章,并且列出十个生词和一系列针对这是个生词的练习。
值得注意的是,老师给的文章很多都是真实故事,除了学英语外,还能领会一些精神!例如:一个癌病患者,完成了Triathlon比赛,成为了一个真正的Ironman!老师说这是一个真人真事,而且就发生在vancouver。
另外,还有两个印象比较深的练习方式:a.用生词造句:以前学单词,很少想到“造句“。不过想想,我们开始学汉语的时候,不就是从造句开始吗?看来语言虽有它的不同特性,但有些学的方法是相通的!b.Crossword Puzzle:虽然已经有了所填词的范围,但是依旧不容呀!

(4)针对写:感觉这是这门课程的重点。
a.每周四都会写一篇作文,应该算考试!老师会给你批改,然后给你一个成绩,这个成绩决定着你最后的成绩。作文题目都是比较贴近生活,例如:海啸,彩票,职业计划... ...
b.几乎每次课,老师都会给homework,一般是写一个paragraph。要注意:paragraph 和passage有区别呀!开始我就没注意到,?里?唆,费劲扒力,写了一大篇!虽然老师不是每次都查,但是他会抽查。很郁闷呀,只有一次没写,被他逮个正着!所以,人 不能存在侥幸心理!

(5)针对语法:老师会给一些不好的句子,或有错误的句子,然后找出问题,给出改正的方法。关于这部分,有同学感觉有一个缺点:这里的老师不会像中国的老师告诉你具体的句法规则。我也不知道原因,也许他认为我们应该知道这些规则,或者他希望你自己体会。

(6)针对词汇:老师采用连线的方式教我们掌握部分词汇。一栏是生词,一栏是解释,要求学生不能查字典,试图去连线!试着去猜!
但是有的同学有不同的看法:他说我连解释中的单词我都不认识,我怎么去猜亚?也许他有他的理解。但是我感觉除了词义,我们还可以利用词性去猜。我并不太反对老师的观点,也许因为我一直就是老师喜欢的那种“乖乖”的学生,唉!
值得注意的是:词汇一定要复习!虽然当时猜对啦,感觉认识了!但是如果不及时复习,过些天,我又毫无保留地把它“还”给老师啦!可惜呀!

主要是这六部分,但是老师会利用一些琐碎的时间告诉一些小知识!
例如:加拿大最好的coffee, chocolate, cheesecake and ice cream!
还有,老师会要求学生借本小说读,三周以后,给出题目,写篇针对小说的作文。
对于懒惰的学生,是件好事!比如我,如果老师不要求我看,我还不知道猴年马月会去读一本小说,因为中文的小说我都不太喜欢读,字太多,累得慌!
唉!说实话,既是为了英语老师,我也没读完呀!惭愧... ...

除了课程内容,我相信很多人会关心pass和fail的问题!

梁溪香榭 : 2008-11-16#167
回复: 偶搜集到的一些关于加拿大高中教育的资料 与大家分享

高中英语第一门课程:English Skill 之结果篇
我相信,每个人都很在乎这个结果,因为自己毕竟付出啦!

其实,我开始真得不在乎是否pass,因为我没想过一定要拿到grade12,主要目的是提高英语。但是到了课程快结束的时候,我真的有些在乎这个“成绩”, 因为它是对我的一个肯定!

整个课程学生能拿到两个成绩:一是student progress report,感觉就是我们的期中成绩;二是最后的report card,这个直接决定你是否可以升到上一个级别!我还没有拿到最后的成绩,希望可以pass!

个人感觉,关于高中课程是否pass的一个结论:想pass,并非轻而易举!
原因有很多:
1。这里并不是60分pass,而是65分pass! 老师对写作要求跟高的,很少能拿到4分(我只有一篇),一般都是3或3.5,我看很多同学得了2.5!我想说的是:即使每次都能拿到3,最后也是Fail.
2。课程过程中的每一个环节,都与最后的结果息息相关!即使一个环节出错,它就可能导致Fail。发生在我身上的一个例子:老师要求我们借一本小说。因为逃了一节课程,所以当他登记的时候我就没带,在我的期中成绩中,novel哪里就是0分!所以我的期中成绩是fail拉,好在,我这个还可以弥补,因为下次课程来了就好!
3。出勤情况。据说:如果缺勤三次或四次,肯定不能pass啦!

一个很有意思的现象:开始我们班有20左右的学生,可是到最后两节课程,仅仅剩下了10个!我猜可能有的人看自己没希望pass啦,所以索性不来啦!

可见,最高通过率也只有50%。

梁溪香榭 : 2008-11-16#168
回复: 偶搜集到的一些关于加拿大高中教育的资料 与大家分享

高中英语第二门课程:Classroom Communication 之内容篇
本门课程不是必修课程,但是它是高中英语中唯一一门侧重听说的课程,我岂能放过!哈哈

基本上可以分成三大块:
1。说:基本还是以讨论为主。跟English Skill中分组讨论有些相似,但又不完全相同!
a.都是分小组讨论, 但是CC(Classroom Communication )的partner经常是你的“邻居”,而且大家基本都固定座位,所以感觉不太好;而ES(English Skill)比较随机!
b.CC会给篇文章和一系列问题;ES大多数就没有问题,需要大家自由发挥(哈哈,经常跑题,而且跑得很远)!
c.CC的老师就不会参与到每个组中,只是最后在前面一起讨论讨论!

2。听:虽然知道是自己的弱项,但是很不喜欢上听力!唉!
a.开始老师会介绍一些“速记”的符号和方法。例如:增加就用向上箭头等。。。有的时候,老师会叫一些同学在黑板上边听边记,唉!始终没有勇气上去!
b.剩下的就是听力练习啦!跟中国的区别不大,老师放磁带,同学使劲听!但是老师也会在黑板上写一些关键的词和中心句子!

3。Presentation:原计划本门课程要有两个presentation的,可惜让罢工给搅啦!我们只完成了一个!很喜欢这部分,虽然做presentation的人几乎都是我们的同胞,都用着蹩脚的英文,但是我们能吃到很多美食,学到很多“知识“!由于老师说:实物是最好的“Visual Aid”!

a.题目自选:我们题目是:How to do something? something自己确定!所以大家各尽其能,题目是五花八门,有如何做蛋糕?如何发豆芽?如何抓螃蟹?如何米酒?如何理发?如何做土豆沙拉?如何识别中国的象形文字?如何成为教徒?如何清理地毯?等等。。。有些陌生的领域真的是不懂呀,例如:有个人介绍如何改金子的纯度!

b.明确要求:老师会提前给出几个方面的要求,例如:Preparation, Pronunciation, Organization, Grammar, Fluency, Energy, Information, Visual Aid。也就是,最后老师会从这几个方面给分,综合之后才是总分!根据老师的评分,你也知道自己那部分做得不好。同时,老师也会指出学生那些音发的不正确,注意呦,只是指出,但是不负责改正,需要学生自己去Learning Centre借书和磁带,需要个人的努力!惭愧,我还不知道Learning Centre在哪?

课程内容基本就这三方面!
这个老师“酷爱“绕口令,经常利用零碎的时间给我们写几个。
我想,当我们这些中国学生能把这些绕口令读顺流啦,我们的舌头就完全适应了英语的发音!

梁溪香榭 : 2008-11-16#169
回复: 偶搜集到的一些关于加拿大高中教育的资料 与大家分享

高中英语第三门课程:Grammer and Writing之内容篇一
请注意:不同的老师,授课的内容可能是不同的!

从时间安排上,课程内容可以分三部分:
1.Homework:每次老师都会分配课后作业,而且还不少!所以,上课的部分时间是讲课后作业,包括一些语法规则。偶尔也会课堂上做一些练习!

2.Writing:每次课堂都会写一篇,时间大概半个多小时到一个小时!老师会给修改,偶尔集中讲一些普遍的错误,但是不会一对一给讲。(这一点不如English Skill的老师,因为有些修改学生是不能理解的)

3.Test: 三次比较大型的考试和最后的考试。第一次关于形容词/副词的比较级和最高级;第二次是关于动词的时态;第三次是关于连接词;最后的考试比较综合,没有选择题555!

吉人天相 : 2008-11-16#170
回复: 偶搜集到的一些关于加拿大高中教育的资料 与大家分享

多谢分享。看得我心里直没底。我们都是为孩子读书做的移民选择,这种情况孩子能行吗?

梁溪香榭 : 2008-11-16#171
回复: 偶搜集到的一些关于加拿大高中教育的资料 与大家分享

多谢分享。看得我心里直没底。我们都是为孩子读书做的移民选择,这种情况孩子能行吗?

俺朋友的孩子初三结束后出去的, 现在出去了一年半, 英语已经很好了. 据俺朋友说孩子的英语问题基本可以在一年左右跟上趟, 当然前提是孩子自身的努力.

海怡宝贝 : 2008-11-16#172
回复: 偶搜集到的一些关于加拿大高中教育的资料 与大家分享

你的贴太好了,我正关心着呢,赶紧收藏!:wdb17::wdb17:

xxwe : 2008-11-16#173
回复: 偶搜集到的一些关于加拿大高中教育的资料 与大家分享

x香榭
我女儿明年参加高考,高考后明年这会儿就登陆了,女儿肯定会考到本一线以上的成绩,并且会有一个大学的录取通知书或已是某大学的在校生。想问:如果有了雅思或托福成绩,孩子登陆后可以直接申请大学吗?还需要补高中学分吗?雅思或托福有哪一个的成绩好更好呢?
谢谢!!!

梁溪香榭 : 2008-11-16#174
回复: 偶搜集到的一些关于加拿大高中教育的资料 与大家分享

x香榭
我女儿明年参加高考,高考后明年这会儿就登陆了,女儿肯定会考到本一线以上的成绩,并且会有一个大学的录取通知书或已是某大学的在校生。想问:如果有了雅思或托福成绩,孩子登陆后可以直接申请大学吗?还需要补高中学分吗?雅思或托福有哪一个的成绩好更好呢?
谢谢!!!

据我所知是这样的. 凭高考成绩和托福或雅思成绩可以直接申请加国高校. 具体嫩可以去看各个学校的网站. 每个学校都会对招生有明文规定, 很详细, 包括不同专业对托福及雅思的分数要求都有明文分列. 应该谈不上哪个更好. 二者取其一即可. 具体哪个更好考俺就不知道了.

惴惴不安 : 2008-11-17#175
回复: 偶搜集到的一些关于加拿大高中教育的资料 与大家分享

我女儿班里来了一个澳大利亚籍的男孩,大约初一或小学6年纪去的,英语很好,其他的科目都不行,差得很远,我女儿在学校成绩一般,都能拉他200分,据说这个同学在奥成绩还不错,并且读私立学校(不知是否属实)。我问这个同学是不是其他地方很突出,综合素质好些,我闺女说,除了篮球打得还可以,没看出有什莫突出的地方。刚来的时候他们班上女生(包括我闺女)喜欢他,觉得他长得好看,现在巨不喜欢,嫌他太傲气。

惴惴不安 : 2008-11-17#176
回复: 偶搜集到的一些关于加拿大高中教育的资料 与大家分享

看了你的帖子,我的考虑去多伦多了,哪里孩子可以在21岁前上高中,明年9月我闺女就满17周岁了,要是明年走不了,就的考虑多伦多了。

秦川八百里 : 2008-11-17#177
回复: 偶搜集到的一些关于加拿大高中教育的资料 与大家分享

好贴!楼主辛苦了,谢谢!
我觉得楼主的帖子说得非常对。我们刚到加拿大不久,我孩子刚去上课时一点都听不懂,这在我意料之中,可是,过了一两周问她,还是这样,我心里就不踏实了。正常的作业老师都说她可以不做她就不做,所以她每天很闲,就上网和国内同学聊天,我自己也发觉她一天天过去英语也看不见什么长进,我心里着急起来!我让她这个周末把课本带回来我看了看,觉得一点也不简单,以她那点词汇量和语法知识,她对着字典都有很多句子看不懂,更别说听老师讲课了。我一下子就感到了问题的严重性:看来靠学校EAL课学的那点英语远远不够,以为到了国外有英语环境了英语自然就好的观念也是错误的,因为不会有人把那些英语词汇知识一下子复制到你脑中。在国外想学好英语依旧是任重道远,得靠自己下功夫去记、去领悟才行。
记得以前有报导过说有的小留学生,在国外呆几年了英语还过不了关,我现在突然能理解他们了,不完全是孩子不争气 ,是大人们小看了孩子学语言所面对的困难。小孩子自制力差,没有人督促和鼓励,很容易选择逃避。

北美枫 : 2008-11-17#178
回复: 偶搜集到的一些关于加拿大高中教育的资料 与大家分享

好贴!楼主辛苦了,谢谢!
我觉得楼主的帖子说得非常对。我们刚到加拿大不久,我孩子刚去上课时一点都听不懂,这在我意料之中,可是,过了一两周问她,还是这样,我心里就不踏实了。正常的作业老师都说她可以不做她就不做,所以她每天很闲,就上网和国内同学聊天,我自己也发觉她一天天过去英语也看不见什么长进,我心里着急起来!我让她这个周末把课本带回来我看了看,觉得一点也不简单,以她那点词汇量和语法知识,她对着字典都有很多句子看不懂,更别说听老师讲课了。我一下子就感到了问题的严重性:看来靠学校EAL课学的那点英语远远不够,以为到了国外有英语环境了英语自然就好的观念也是错误的,因为不会有人把那些英语词汇知识一下子复制到你脑中。在国外想学好英语依旧是任重道远,得靠自己下功夫去记、去领悟才行。
记得以前有报导过说有的小留学生,在国外呆几年了英语还过不了关,我现在突然能理解他们了,不完全是孩子不争气 ,是大人们小看了孩子学语言所面对的困难。小孩子自制力差,没有人督促和鼓励,很容易选择逃避。
请问:您的孩子有多大?在国内上几年级的时候去的?我的孩子现在在国内上初2

梁溪香榭 : 2008-11-17#179
回复: 偶搜集到的一些关于加拿大高中教育的资料 与大家分享

看了你的帖子,我的考虑去多伦多了,哪里孩子可以在21岁前上高中,明年9月我闺女就满17周岁了,要是明年走不了,就的考虑多伦多了。

是啊. 所以俺也不考虑温革华. 去安省吧. 就是回国麻烦点.

梁溪香榭 : 2008-11-17#180
回复: 偶搜集到的一些关于加拿大高中教育的资料 与大家分享

好贴!楼主辛苦了,谢谢!
我觉得楼主的帖子说得非常对。我们刚到加拿大不久,我孩子刚去上课时一点都听不懂,这在我意料之中,可是,过了一两周问她,还是这样,我心里就不踏实了。正常的作业老师都说她可以不做她就不做,所以她每天很闲,就上网和国内同学聊天,我自己也发觉她一天天过去英语也看不见什么长进,我心里着急起来!我让她这个周末把课本带回来我看了看,觉得一点也不简单,以她那点词汇量和语法知识,她对着字典都有很多句子看不懂,更别说听老师讲课了。我一下子就感到了问题的严重性:看来靠学校EAL课学的那点英语远远不够,以为到了国外有英语环境了英语自然就好的观念也是错误的,因为不会有人把那些英语词汇知识一下子复制到你脑中。在国外想学好英语依旧是任重道远,得靠自己下功夫去记、去领悟才行。
记得以前有报导过说有的小留学生,在国外呆几年了英语还过不了关,我现在突然能理解他们了,不完全是孩子不争气 ,是大人们小看了孩子学语言所面对的困难。小孩子自制力差,没有人督促和鼓励,很容易选择逃避。

谢谢提点. 嫩这是第一手资料. 珍贵!
嫩孩子多大? 去了多久啦?

zhaozq : 2008-11-17#181
回复: 偶搜集到的一些关于加拿大高中教育的资料 与大家分享

我儿子现在国内读初二,有机会也让我儿子自己看一下这个帖子,好有个思想准备,顺利的话,估计明年他就面临这些问题了。

梁溪香榭 : 2008-11-17#182
回复: 偶搜集到的一些关于加拿大高中教育的资料 与大家分享

我儿子现在国内读初二,有机会也让我儿子自己看一下这个帖子,好有个思想准备,顺利的话,估计明年他就面临这些问题了。

是的. 俺发这个贴滴目的就是有个思想准备.

sz艾美 : 2008-11-17#183
回复: 偶搜集到的一些关于加拿大高中教育的资料 与大家分享

慢慢仔细看,先给加声望.

响铃当当 : 2008-11-17#184
回复: 偶搜集到的一些关于加拿大高中教育的资料 与大家分享

好贴呀,加分:wdb6:

jenniferlee : 2008-11-17#185
回复: 偶搜集到的一些关于加拿大高中教育的资料 与大家分享

好帖要顶!

梁溪香榭 : 2008-11-17#186
回复: 偶搜集到的一些关于加拿大高中教育的资料 与大家分享

谢谢各位鼓励. 俺回礼.

yuanman : 2008-11-17#187
回复: 偶搜集到的一些关于加拿大高中教育的资料 与大家分享

谢谢lz,好贴!

梁溪香榭 : 2008-11-17#188
回复: 偶搜集到的一些关于加拿大高中教育的资料 与大家分享

高中英语第三门课程:Grammer and Writing之内容篇二
这部分写了三天啦,还没完成!今天一定要一气呵成~~

从句法知识角度,主要包括:

1. 词性:主要是单词在句子中的词性,所以称:Part of speech!好像英语中没有,词性活用这个说法。

2. 同义词:找常用词的Synonym!这个对我们的写作非常有好处,可惜但是没有记录下所有的。应该当时要求老师写在黑板上(5555)。后悔,所以有想法一定要表达!

3. 句型:称Sentence Types,其实就是我们说的简单句,复合句和从句!但是俺们是不知道他们称:Simple Sentence, Compound Sentence and Complex Sentence.
强调的重点一:不是让我们简单区分句型,而是训练我们能自己写出后两种。
强调的重点二:详细介绍了构成Compound Sentence的连接词。

4. 时态:重点难点!
Present tense: Simple present, Present progressive, Present Perfect
Past tense: Simple Past, Past Progressive, Past Perfect
Future Time: Will and Be going to的区别!(汗,以前没注意过)
Time Shifts and Tense: 一篇文章中使用不同的时态!
Present Versus Present Progressive: 感觉不难,但是区别清楚不是很容易
Present Perfect Progressive for New Habits:

5. 主谓一致(Subject-Verb Agreement): 相对比较简单

6. 形容词的比较级和最高级:知识点是两个或两个以上音节的词(不以Y结尾的)用more/most

7. Word Forms: 俺不记得以前说过这项,所以没对上号!主要是:The form of a word changes according to its part of speech。

8. 并列结构:Parallel Structure

9. 不定式和现在分词:Infinitive and Gerund

10. 比较和对比:Comparing-相同点 and Contrasting-不同点

11. Fragment and Sentence: 直接翻译就是句子碎片和句子!

很遗憾,有学到这里,课程就结束啦!其实后面应该是比较难的句子结构。关于后面的部分,在English Skill里学了一点点,但是那个老师不给Profile,所以学的稀里糊涂的~

梁溪香榭 : 2008-11-17#189
回复: 偶搜集到的一些关于加拿大高中教育的资料 与大家分享

高中英语第三门课程:Grammer and Writing之收获篇
总体感觉:还不错!

优点:
1. 可以系统地巩固我们已有的语法知识,对继续学习比较有好处!
2. 对我们已有的语法,进行了Localization
3. 对写作有一点的好处

缺点:
1. 不理解的语法知识,仍然不能理解呀!原因一:有的时候,老师也说不出来原因;原因二:以前用中文讲解都没理解,理解用英文讲的谈何容易!

2. 时间有限,不能学完全部的语法知识!老学到Fragment or Sentence,郁闷!

梁溪香榭 : 2008-11-17#190
回复: 偶搜集到的一些关于加拿大高中教育的资料 与大家分享

高中英语第四门课程:Literature之内容篇

上课流程:
1.每次课程后,老师都会给一篇不同问题的文章,而且长短不一
2.课上,老师会给一份Discussion Question about this passage
3.然后就是同学直接进行无聊的讨论,总所周知,课堂上大部分都是我们的同胞
4.老师总体总结,然后让课后写一篇作文,当然是离不开this passage

考试:基本上三次考试。考试内容也是给一篇文章,然后又多选和写作

梁溪香榭 : 2008-11-17#191
回复: 偶搜集到的一些关于加拿大高中教育的资料 与大家分享

高中英语第四门课程:Literature之收获篇

先谈谈这个老师吧!

这个老师属于思维特别活跃,而且想象力及其丰富,给的文章基本都是Open Ending!

她最得意地问题就是:What is the source of the conflict about the story? When start? How to end? 每次考试,最后的写作都是这个问题,如果找不到这个正确的Source,想得分难呀!她评作文侧重内容,不侧重语法。也许这也是Literature课程的与众不同吧!

收获:
1. 督促自己读一些文章,也可以说是小说的一部分
2. 对英语文学有一点点地了解 (有的时候感觉像学语文)

缺点:
同学之间,尤其是同胞之间讨论,经常跑题,而且容易用大家的母语(爽)!

梁溪香榭 : 2008-11-17#192
回复: 偶搜集到的一些关于加拿大高中教育的资料 与大家分享

多伦多几所著名私立学校学费一览

Branksome Hall (girls only)

Sk - Grade 10 $20700
Grade 11 $22360
Grade 12 $20700
Boarding student fee $18300
Initial application fee $5000

Crescent School (boys only)

Tuition fee/year: $20750
Initial application fee(1st year only): $5000
Other fees/year: $2600

Havergal College (girls only)

Day school tuition fee: $19515
Boarding student tuition & residence : $39300
Initial application fee: $5000

Saint Andrews College (boys only)

Boarding student: $37335
Day school student: $23255
Initial application fee: $3850

Upper Canada College (boys only)

Day school tuition fee: $22700
Day school IB program tuition fee: $24700
Boarding student : $40500
Boarding student IB program: $42500
Initial application fee: $5000

pwjnd : 2008-11-17#193
回复: 偶搜集到的一些关于加拿大高中教育的资料 与大家分享

是啊. 所以俺也不考虑温革华. 去安省吧. 就是回国麻烦点.
好帖子!:wdb17::wdb17:我们家孩子现上高一,看样子我们多花点钱让他上A-LEVEL还对了,我也想去安省,但现在的移民进度太慢了:wdb12:但愿我们能尽快在加拿大相会,尽快安排好孩子的学习。

梁溪香榭 : 2008-11-17#194
回复: 偶搜集到的一些关于加拿大高中教育的资料 与大家分享

你孩子的英语真的过关吗?


  一个十三、四岁随父母移民加拿大的中国孩子,经过本地四年高中、三年名牌大学的学习,英文应该没有任何问题吧?像大多数同胞一样,老牛会毫不犹豫地回答:中文可能有问题,英文当然不会有问题。可事实并非如此乐观!

  这得从老牛目前所在的公司每年招实习大学生(Intern)说起。由于公司开发业务的需要,每年会从多伦多大学(University of Toronto)、女皇大学(Queens University)和阿尔伯塔大学(University of Alberta)等招收大量科学工程类实习生。由于要求实习的时间至少一年,使得一些只提供四个月实习期的大学如著名的滑铁卢大学(University of Waterloo)就不能给我们提供实习生源。好在公司给实习大学生的工资相当不菲;打个比方吧,如果移民同胞夫妻两人都打一般的Labour工,那么老牛所在公司实习大学生的工资绝不少于这对夫妻的工资总和。另外公司每年还会从实习生中挑选部分优秀学生,在他们结束实习后提前一年给他们Offer,这样这些学生返回大学完成最后一年学业后,就可直接回公司上班。正是由于如此优厚的实习待遇,每年大学生之间的竞争非常激烈。大家都知道,科学工程类专业的学生大多是来自亚洲及东欧国家移民的后代。这样一来,竞争一般会在华裔、印度裔和东欧裔学生之间展开;今年也不例外。不过例外的是,今年负责招聘实习大学生的任务是由老牛的顶头上司、一个至少祖宗三代生活在加拿大的本地人负责。虽然顶头上司负责招聘实习生,但分配到我们组的实习生最终干活还得老牛领着,所以无论如何老牛还是要稍稍关心一下的。

  谈起这次招聘的结果,顶头上司除了问老牛本文开头的这个问题外,还问道:为什么一个来自俄罗斯的学生,在加拿大只生活了三年,英文交流水平会高于在加拿大生活了七年的华裔学生呢?俄语和英语可也是不同的语系啊。老牛听完之后,非常震惊。心想,不会吧?老牛周围有不少朋友的子女可都是上了名牌大学,英语至少在老牛看来是不会有问题的。于是就追问,这是不是个别现象?顶头上司就说至少从这次的招聘面试来看,华裔学生虽然学习成绩不错甚至很好,但普遍有英语交流问题。顶头上司还认为,这可能是由于在加拿大,尤其是多伦多,华裔社区越来越大,华裔学生很容易找到同族裔的同学和朋友,互相之间很容易用母语交流;加上在家里和父母交流也是用母语;这自然会影响到英文交流水平的提高。

  无论老牛的顶头上司一家之言是否完全有道理,但移民的孩子,尤其是那些在中国接受了小学甚至中学教育的孩子,他们的英文水平真的过关了吗?真的能满足将来工作的需求吗?毕竟,移民他乡,没有一个父母愿意自己的孩子还像自己一样,因为憋脚的英语终身影响着找工和升迁,头顶上总有一块“玻璃天花板”,更枉论融入主流社会了。

  那么,在加拿大上了三年高中和四年大学的孩子,英语到底应该是在什么样的水平呢?按老牛顶头上司的说法是:英语交流应该完全没有问题。有没有口音呢?有,但只应该是一点点(原话是:a little bit)。这样的要求是不是有些苛刻?但这至少是一个本地人的真实期望。

  移民孩子的英语水平能否达到本地人的期望,不可否认有许多因素。孩子自身的素质是一个重要的因素,但作为父母的移民同胞是不是也有值得深思的部分呢?答案是不容置疑的。

  在过去的数年中,在我们的社区,在我们的免费中文报纸上,“名校区”这个词大概同胞们都不陌生吧,房产经纪对此可是作出了重要“贡献”的。结果呢,华人同胞一哄而上,涌进了“名校区”。 “名校区”的房价扶摇直上,这可让原来的房主大大地赚了一笔;可从此“名校”也就成了“华裔名校”,一个班上清一色黑头发已不是个别现象。本文前面所提到的华裔大学生应该是在1998或1999年左右移民加拿大的,老牛不敢肯定是不是也是从一些“华裔名校”走出来的;但这“华裔名校”的后果肯定会慢慢呈现出来。

  老牛曾以为“名校区”这种现象在多伦多发生过,同胞们已经引以为戒。不想,随着华人同胞“西进”,这“名校区”现象在密西沙加正在发生。在密西沙加中部一所所谓的“名高中”,一个ESL班上清一色来自中国大陆的孩子,校方这么安排可能是便于管理。但后果呢?这些孩子除了上课用英文,写作业用英文,听说讨论完全可以接着用中文。可想而知,经过四年本地“名高中”,他们的英语水平能有多少提高?老牛不知道这些孩子的父母了不了解这个事实;而知道后又是如何感想的?毕竟孩子们将来不可能永远生活在华人小圈圈的,是要在加拿大这个大社区工作生活的。

  其实,掌握语言技巧对孩子们来说仅仅是第一步,还要理解和掌握本地的文化风俗。这是必须的,但也是比较难的。老牛的小儿五岁移民加拿大,中文对他来说有问题,英语对他来说应该不成任何问题。可是,有一次和他的老师面谈时,他的老师诚恳地建议要让孩子多参加一些集体的课外活动或体育活动或社区活动。为什么呢?他的老师说,有时他不太理解她说的笑话。无疑,小儿遇到了一位很负责任的好老师,她点出了很多华裔孩子的一个通病。华裔孩子聪明好学,华裔家庭重视教育,但大多数华人父母是技术移民,由于在中国接受的教育以及过去大多从事技术工作等原因,往往看重知识的获取,而忽视其它人文方面的教育。由于华人家庭的社会活动大多局限于华人社区,孩子们除了学校外,也很难有机会和其它族裔的孩子真正交流,以致学习成绩很好,却听不懂一个笑话。

  很多同胞移民的一个理由是为了孩子,但如果孩子的英语水平都没有真的过关,我们对得起自己移民的初衷吗?是不是真的是时候问一问自己:孩子的英语是否真的过关?但愿老牛不是杞人忧天。

pwjnd : 2008-11-17#195
回复: 偶搜集到的一些关于加拿大高中教育的资料 与大家分享

多伦多几所著名私立学校学费一览

Branksome Hall (girls only)

Sk - Grade 10 $20700
Grade 11 $22360
Grade 12 $20700
Boarding student fee $18300
Initial application fee $5000

Crescent School (boys only)

Tuition fee/year: $20750
Initial application fee(1st year only): $5000
Other fees/year: $2600

Havergal College (girls only)

Day school tuition fee: $19515
Boarding student tuition & residence : $39300
Initial application fee: $5000

Saint Andrews College (boys only)

Boarding student: $37335
Day school student: $23255
Initial application fee: $3850

Upper Canada College (boys only)

Day school tuition fee: $22700
Day school IB program tuition fee: $24700
Boarding student : $40500
Boarding student IB program: $42500
Initial application fee: $5000
收藏了,感谢梁溪给坛子里作出的巨大贡献哦!

梁溪香榭 : 2008-11-17#196
回复: 偶搜集到的一些关于加拿大高中教育的资料 与大家分享

好帖子!:wdb17::wdb17:我们家孩子现上高一,看样子我们多花点钱让他上A-LEVEL还对了,我也想去安省,但现在的移民进度太慢了:wdb12:但愿我们能尽快在加拿大相会,尽快安排好孩子的学习。

握手握手 :wdb6::wdb6::wdb6:
俺也准备让儿子上A-LEVEL.
他现在初三, 高一就准备读A-LEVEL啦.

梁溪香榭 : 2008-11-17#197
回复: 偶搜集到的一些关于加拿大高中教育的资料 与大家分享

收藏了,感谢梁溪给坛子里作出的巨大贡献哦!

谢谢
今天分分加不上了. 明天加.

大家鼓励俺, 俺就有干劲. 努力多刨点资料来. :wdb6::wdb6::wdb6:

JACKYZHOU : 2008-11-17#198
回复: 偶搜集到的一些关于加拿大高中教育的资料 与大家分享

好贴!虽
然越看越担心,困难重重啊,但是仍然相信未来是美好的!

吉人天相 : 2008-11-17#199
回复: 偶搜集到的一些关于加拿大高中教育的资料 与大家分享

看了你的帖子,我的考虑去多伦多了,哪里孩子可以在21岁前上高中,明年9月我闺女就满17周岁了,要是明年走不了,就的考虑多伦多了。
这个信息很重要,以前一心想到温哥华,根本没想过其他地方。现在看为了孩子的学习,真的有必要全面衡量一下了。多谢了!

惴惴不安 : 2008-11-17#200
回复: 偶搜集到的一些关于加拿大高中教育的资料 与大家分享

是啊. 所以俺也不考虑温革华. 去安省吧. 就是回国麻烦点.
我要考虑去多伦多了,移民的动力就是子女求学,这是第一要务,必须办好没商量。估计你已经搜集不少多伦多的资料,还得向你取经啊。

惴惴不安 : 2008-11-17#201
回复: 偶搜集到的一些关于加拿大高中教育的资料 与大家分享

这个信息很重要,以前一心想到温哥华,根本没想过其他地方。现在看为了孩子的学习,真的有必要全面衡量一下了。多谢了!
9494

:wdb10::wdb10::wdb10:

秦川八百里 : 2008-11-17#202
回复: 偶搜集到的一些关于加拿大高中教育的资料 与大家分享

请问:您的孩子有多大?在国内上几年级的时候去的?我的孩子现在在国内上初2

我们今年十月份登陆,孩子跟你们一样,在国内上初二,到这里进八年级。
原来只看过大家都在讲孩子过来一半年后英语就没问题了,从没有想过是怎么没问题的。我在这里看到过有来了两年的高中生跟人交流还有问题的,也看到过有来了三年的大学生英语就已经非常好的,我觉得这关键是看你怎么学了。

xxwe : 2008-11-17#203
回复: 偶搜集到的一些关于加拿大高中教育的资料 与大家分享

香榭顶顶顶!!!!

惴惴不安 : 2008-11-17#204
回复: 偶搜集到的一些关于加拿大高中教育的资料 与大家分享

溪溪 300贴纪念

zhaozq : 2008-11-17#205
回复: 偶搜集到的一些关于加拿大高中教育的资料 与大家分享

good

梁溪香榭 : 2008-11-17#206
回复: 偶搜集到的一些关于加拿大高中教育的资料 与大家分享

溪溪 300贴纪念

谢谢嫩把嫩滴第300贴放在俺滴楼里. 珍贵啊.
:wdb6::wdb6::wdb6:

BTW, 嫩亲家母玩失踪? :wdb23::wdb23:

梁溪香榭 : 2008-11-17#207
回复: 偶搜集到的一些关于加拿大高中教育的资料 与大家分享

香榭顶顶顶!!!!


谢谢支持. 都加纷纷.

(俺咋还不解冻捏. :wdb12:)

梁溪香榭 : 2008-11-17#208
回复: 偶搜集到的一些关于加拿大高中教育的资料 与大家分享

我要考虑去多伦多了,移民的动力就是子女求学,这是第一要务,必须办好没商量。估计你已经搜集不少多伦多的资料,还得向你取经啊。

嘿嘿, 还没来得及呢.
俺还早着呢. 等拿到联邦FN再动手也不迟.
不过既然嫩说起, 俺就留心啦.

其实俺也很可能不去多伦多, 虽然那里有朋友.
俺比较倾向于安省南部的中等城市, 学校里华人孩子少些, 有利于孩子英语提到. 等孩子上了大学俺们再考虑定居城市.

梁溪香榭 : 2008-11-17#209
回复: 偶搜集到的一些关于加拿大高中教育的资料 与大家分享

好贴!虽
然越看越担心,困难重重啊,但是仍然相信未来是美好的!

:wdb10::wdb10::wdb10::wdb9::wdb9::wdb9::wdb20::wdb20::wdb20:

9494. 人生就是要过道道坎. 没有途中的艰险又怎会体味登顶的快乐.

shine : 2008-11-17#210
回复: 偶搜集到的一些关于加拿大高中教育的资料 与大家分享

这个帖子太好太及时有用了.

FredHe : 2008-11-17#211
回复: 偶搜集到的一些关于加拿大高中教育的资料 与大家分享

好帖
请问可以介绍一下成人读大学的情况吗

梁溪香榭 : 2008-11-17#212
回复: 偶搜集到的一些关于加拿大高中教育的资料 与大家分享

这个帖子太好太及时有用了.

谢谢鼓励. :wdb6:

好帖
请问可以介绍一下成人读大学的情况吗

不好意思, 这个俺不了解.

FredHe : 2008-11-17#213
回复: 偶搜集到的一些关于加拿大高中教育的资料 与大家分享

你是读高中?有成人高中吗?

梁溪香榭 : 2008-11-17#214
回复: 偶搜集到的一些关于加拿大高中教育的资料 与大家分享

你是读高中?有成人高中吗?

是俺儿子将(2年后)要去读高中.
帖子里都有说啊, 都是俺预先搜集的一些资料啊.

FredHe : 2008-11-17#215
回复: 偶搜集到的一些关于加拿大高中教育的资料 与大家分享

"以我自己本身做例子:当我抵达多伦多的第2天就去Toronto District School Board(多伦多教育局)注册,并且给了我一个档案,档案的封面上写着我Assessment和Orientation,接着按照日期去考试,考的内容在我前一篇日志已经写了,就是ESL和数学,我的ESL级别是C。"
令我误会了

尘烟一缕 : 2008-11-18#216
握手握手 :wdb6::wdb6::wdb6:
俺也准备让儿子上A-LEVEL.
他现在初三, 高一就准备读A-LEVEL啦.

粱姐姐,弱弱地问一声,A-LEVEL是啥呀?偶和你是同期的申请呢!

梁溪香榭 : 2008-11-18#217
回复: 偶搜集到的一些关于加拿大高中教育的资料 与大家分享

关于A-LEVEL

A-Level的全称是Advanced Level,它是英国的普通中等教育证书考试高级水平课程,通常被称为英国的金牌教育方式(British Golden Education System),英国学生一般在16岁或稍大一些开始学习这种课程,圆满修完这种课程即可进入大学就读。这种课程相当于我国中学最后两年的课程。(你可以将它理解为我国高中的高三以及高四两年的课程)

大部分英国学生都是用两年的时间修完这种课程,但能力很强的学生有时也可在更短的时间内修完。这种课程要求学生学习三门或四门主科课程并参加毕业考试,考试合格者即可进入大学就读。学生的考试成绩及其所选修的A-Level课程在很大程度上决定着能否进入理想的大学和学习所选择的学位课程。


除大学预科课程之外,A-Level课程是中国学生入读英国大学的最佳途径。这是因为A-Level课程要大大优于大学基础课程,其优点有以下三方面:

梁溪香榭 : 2008-11-18#218
回复: 偶搜集到的一些关于加拿大高中教育的资料 与大家分享

第一,中国学生在国内的高一或高二的在校学生可以赴英国学习A-Level课程。这样,他们再经过两年的学习就可以进入英国大学就读,而不是像在中国那样,高中毕业然后还要学习一年的大学预科课程,才能入读英国大学。


第二,这种课程是为中国学生进入英国大学做准备的理想课程。无论是在学业方面还是在语言方面都会高于大学基础课程可达到的程度。中国学生要用英语学习各门课程,亲自体验新的教学方法,其英语也会达到相当熟练的程度。


第三,牛津、剑桥、帝国理工和伦敦大学学院这样的名牌英国大学几乎是从不录取大学预科的学生,而只录取A Level毕业证书或同等学历的学生。同样,如果没有A-Level毕业证书或同等学历,要想就读像医科或医疗卫生领域的某些课程也是难上加难。但是,如果中国学生在A-Level课程考试中取得了好成绩,他们就会有很多机会进入英国非常好的大学攻读任何学科的课程。而就读一年制的大学预科课程则不具备进入这些好大学的条件。

梁溪香榭 : 2008-11-18#219
回复: 偶搜集到的一些关于加拿大高中教育的资料 与大家分享

A-LEVEL课程详解

英国高中课程(A-Level)有60多门课供学生选择,学生可以任意选修其中的3至4门。英国的大多数中学开设的A-Level课程科目相当广泛,有文科、商科、经济、语言、数学、理科、计算、法律、媒体、音乐等等。数学、进阶数学和物理是大多数大学和专业招生时要求学过的A-Level科目(只有极专业的学科除外,例如法学),所以我们建议选择这三门课。除了被广泛地接受外,学生还有其他的收益。相对于西方学生,中国学生在数理化方面的训练更为严格,基础扎实;而且学习数理化对英语能力的要求比其他科目较低。所以这样的选择能够体现中国学生的优势,当然,海外学生还有一门额外的必修课英语。

但是学生如果对将来所学专业有了清晰的选择,那么选课就必须谨慎,因为有的专业是具有特殊要求的,例如:将来学习医学,现在就需要学习化学和生物学。如果学生对将来留学的大学或专业有了选择,可以向中心的工作人员咨询具体大学和专业对A- Level课程的专门要求,以便让学生选课时具有针对性。在选择A-Level课程门类时要考虑两个基本因素:一是学生将来计划申请就读的学位课程;二是学生本身的能力与天赋。

学生第一年各门功课和第二年各门功课的统考成绩(满分600分)都将按比例被计入总成绩换算,得出的最终结果就是A-level成绩,并以ABCDE划分等级。

梁溪香榭 : 2008-11-18#220
回复: 偶搜集到的一些关于加拿大高中教育的资料 与大家分享

英国国家考试局对每一个开设A- Level的高级附属中心都进行严格的教学质量控制措施。每一位教授A- Level课程的老师都经过严格的筛选和测评。同时,老师们也可以通过多种渠道得到英国国家考试局的协助和培训。为了监控分布在世界上150多个国家教育中心的教学质量,这些考试局已经在全球各地建立了完善的组织网络;通过其批准的高级附属中心,不定期地对教学质量进行检查和评定。从而保证每一个就读A- Level课程的学生能接受到高质量的教育。

四、证书的国际认可度

持A-Level证书可以进入的大学有:

英 国:剑桥大学、牛津大学、帝国理工学院、伦敦政治经济学院、巴斯大学、曼彻斯特大学、拉夫堡大学、利兹大 学、布里斯托大学等所有大学

加拿大: 多伦多大学、不列颠哥伦比亚大学、麦吉尔大学、女皇大学、西安大略大学、劳伦西大学、蒙特埃里森大学、特伦特大学等大学

澳大利亚: 悉尼大学、澳大利亚国立大学、新南威尔士大学、西澳大学、蒙那什大学、卧龙岗大学、墨尔本大学等所有大学

新西兰: 奥克兰大学、梅西大学、林肯大学、怀卡托大学、马努卡理工学院等全部大学

爱尔兰: 都柏林大学三一学院、爱尔兰国家大学梅努斯学院、科克学院、戈尔威学院、都柏林学院、利默里克大学、都柏林城市大学等全部大学

新加坡: 新加坡国立大学、新加坡南洋理工大学等所有大学

香港: 所有大学

南非: 所有大学

美国: 部分大学

若需获取全球认可A-Level证书的大学名单,请查询各个考试局的官方网站 www.ocr.org.uk, www.aqa.org.uk, www.edexcel.org.uk,www.cie.org.uk 或英国大学招生委员会的官方网站 www.ucas.com.

梁溪香榭 : 2008-11-18#221
回复: 偶搜集到的一些关于加拿大高中教育的资料 与大家分享

五、课程目的

在中国开设A-Level课程旨在为中国学生提供进入国外大学的有效途径,具体目标为:

培养在国内初高中成绩优秀的学生进入世界顶尖大学

培养在国内初高中成绩中等的学生进入世界一流大学

培养在国内初高中成绩一般的学生考取适合自己的大学

六、课程适用对象

A-Level课程适用于年龄在1518之间,初中毕业以上的在校中学生或具有同等学历的职高生。
七、英国高中课程(A-Level)的学制

英国高中课程(A-Level)的学制为两年:

第一年称为AS水准,学生通常选择自己最擅长且最有兴趣的34门课,通过考试后获得AS证书。

第二年称为A2水准,学生可选择AS水准中优秀的3门课继续学习,通过考试后获得A-Level证书。

梁溪香榭 : 2008-11-18#222
回复: 偶搜集到的一些关于加拿大高中教育的资料 与大家分享

八、课程设置及课程结构

1.课程设置

英国高中课程(A-Level)有60多门课供学生选择,学生可以任意选修其中的3至4门

课程包括:数学、进阶数学、物理、化学、生物学、会计学、商业学、经济学、英国文学、心理学、计算机学等科目。

综合国外大学的录取要求及中国学生的优势和特点,A- Level课程将首先在中国开设数学、进阶数学、物理、计算机学、会计学、商业学、经济学等课程供学生选择。

应该选择哪几门课程是学生和家长都普遍关注的问题,但也是非常难回答的一个问题。英国、加拿大、爱尔兰、澳大利亚、新西兰、新加坡等英语国家没有统一的大学入学标准,虽然它们都认可A-Level证书,但是各所大学、各个专业对学生学过哪几门A-Level课程以及成绩都有不同的要求。所以怎样选择课程并没有一个唯一的答案。

学生选择课程时,一般要考虑现在自己的优势科目和将来的发展方向,即你想选择哪个大学、什么专业,从而根据他们的要求有的放矢地选课。然而,对于16-18岁的学生做这样的选择也是很难的,因为自己可能还没有一个清晰的决定。所以我们建议选择适合大部分大学和专业的课程,给自己今后发展留下比较大的选择空间。

数学、进阶数学和物理是大多数大学和专业招生时要求学过的A-Level科目(只有极专业的学科除外,例如法学),所以我们建议选择这三门课。除了被广泛地接受外,学生还有其他的收益。相对于西方学生,中国学生在数理化方面的训练更为严格,基础扎实;而且学习数理化对英语能力的要求比其他科目较低。所以这样的选择能够体现中国学生的优势。

但是学生如果对将来所学专业有了清晰的选择,那么选课就必须谨慎,因为有的专业是具有特殊要求的,例如:将来学习医学,现在就需要学习化学和生物学。如果学生对将来留学的大学或专业有了选择,可以向中心的工作人员咨询具体大学和专业对A- Level课程的专门要求,以便让学生选课时具有针对性。学生也可以自己上网查询相关信息,以下是一些大学的网站:

澳大利亚:www.dest.gov.au/highered ausiunis.htm

新 西 兰:www.educationnz.org.nz/institutions/universities.html

美 国:www.ed.gov , www.usjournal.com

加 拿 大:www.studycanada.ca

英 国:www.ucas.com/instit/index.html


2.课程结构 [学生可根据自身特点和兴趣,从中选择3至4门课]

基础数学 中国学生在数学学科上有很大的优势,一般学生都会选择基础数学。基础数学的内容涵盖:纯粹数学、概率统计、机械学。考试以笔试的形式,分为六个模块。

进阶数学 在基础数学的基础上,进阶数学在内容的深度和广度上略有提高。那些在理科方面有特长的同学,通常会选择进阶数学。

物理学 如果学生要进入大学的理工类专业,通常要选物理学。物理学的内容包括:普通物理、牛顿力学、物质、振动及波、电学与磁学、现代物理。

商业学 商业学内容包括:商务及环境、人与组织、市场营销、运作管理、商业会计学、决策与支持、信息学等。考试以笔试为主,题型包括:简答、小论文、案例分析等。

经济学 经济学内容包括:经济学基础、价格体系及公司理论、价格体系的政府干预行为、国际贸易、宏观经济学基础、宏观经济学问题、宏观经济学政策。考试以笔试为主,题型有多项选择、数据分析、结构化问题、小论文等。

计算机科学 计算机科学内容包括:计算机系统学、计算机通讯与软件学、结构化实践任务、系统软件技术、数据库理论、程序模块和集成信息系统、计算工程。考试以笔试为主,题型有问答及编程。

梁溪香榭 : 2008-11-18#223
回复: 偶搜集到的一些关于加拿大高中教育的资料 与大家分享

九、考试时间、地点及评审方法

1.考试时间:A-Level 的考试将由所属的考试委员会分别于每年5-6月和10-11月在全球统一组织,成绩分别在8月和次年 2月公布。

2.考试方法:英国高中课程的考试方法非常灵活,学生可以选择分阶段测试或者一次报考所学所有课程。而且,每门课程均有多次考试机会,最终成绩以最好的一次计算。

3.考试地点:学生在就读的各地高级附属中心参加考试。

4.考试试题:A-Level的考试试题由所属的考试委员会统一命题。试卷为全英文,要求学生用英文回答。

5.考卷评阅:学生的答卷将由学校采用严格的保密措施,在规定的时间内送达英国。试卷也将由所属的英国国家考试局派选专家在英国当地统一阅卷。

6.考试成绩:A-Level的成绩分为A、B、C、D、E、U六个等级,A为最优,E为通过,U为不及格。如果学生对某门课的成绩不满意,可以选择重考,最终成绩以最好的一次为准。

7.评分标准:与中国的考试不同,A-Levle考试的及格等级E相当于百分制40分。学生达到及格成绩,就可以申请国外的大学。具体的等级与分制之间的关系如下表所示:

A-LEVEL成绩

百分制

A 80-100

B 70-80

C 60-70

D 50-60

E 40-50

U 40以下

梁溪香榭 : 2008-11-18#224
回复: 偶搜集到的一些关于加拿大高中教育的资料 与大家分享

十、大学录取标准

全球不同大学,不同专业的入学标准对于A-Level的成绩要求不尽相同。概括地说,学生至少学习三门课程,只要在两门课的考试中取得E即可达到一些普通大学的入学标准。而对于较好的大学,则要求学生3门课的成绩均应达到C以上;而对于世界一流大学如牛津、剑桥、哈佛等名校,则要求申请学生3门课的成绩达到 AAA或AAB。

各大学对英语成绩的要求也不尽相同,剑桥、牛津等大学要求雅思 > 7.0 ,较好的大学要求雅思 >6.5,一般的大学要求雅思 5.5-6.0

梁溪香榭 : 2008-11-18#225
回复: 偶搜集到的一些关于加拿大高中教育的资料 与大家分享

A LEVEL课程适合中国学生

一: 在美国、加拿大、澳大利亚和英国这四个国家中,只有英国有全国性的课程体系(从小学到预科)和全国性的教育监督体系。

英国英格兰地区的主流预科课程体系是A LEVEL。A LEVEL是英国全国性的课程体系,苏格兰和威尔士的许多学校也提供A LEVEL教学。

A LEVEL本身,是英语世界最成熟的预科课程体系。在世界范围,只有IB课程体系,可以与A LEVEL媲美,甚至超越A LEVEL。

可以说,IB课程在原则上是攻读世界名牌大学最好的课程。西方各国的名牌大学都接受修读IB预科课程的学生。只可惜,一般说来,IB课程并不太适合中国留学生。

IB是一种国际课程体系,总部设在瑞士、研究基地主要在瑞士和英国。这个课程体系的重要理念之一,是不仅要学生掌握知识和技能,而且要通过培养和发展对知识的批判性思维,来发展学生的独立思考能力和创造性。

IB课程体系中,必修课中包括知识论(Theory of Knowledge)、创新、实践和社会服务,以及拓展论文等这些独特的部分。这些部分,要求学生有很强的独立思考能力,分析批判能力,和组织活动能力。这些都不是中国学生的强项,甚至是中国学生的死穴。而且,独立思考能力和分析批判能力,都需要通过语言和论文来体现,对语言(英语)运用的要求非常高。初到国外留学的中国学生,显然难以胜任。

而且开设IB课程的学校比较少。虽然全世界有很多国家的学校开设IB课程,但目前没有任何一个国家把它作为主流课程。

而A LEVEL课程,不仅为英国所有大学承认,也为美国、加拿大、澳大利亚等英语国家承认,实际上,它也为德国和法国的大学承认。因此,修读A LEVEL课程,只要成绩合格,可以报读美国、加拿大、英国、澳大利亚、德国和法国的任何一间大学。

梁溪香榭 : 2008-11-18#226
回复: 偶搜集到的一些关于加拿大高中教育的资料 与大家分享

A LEVEL比较容易?绝对不是。A LEVEL体系的教学内容,拿到中国任何一间重点中学,都足以令教师畏缩。

那么为什么读A LEVEL比较容易升入世界名牌大学呢?

这有几个原因。

第一,按A LEVEL课程体系规定,每个学生修读的课程比较少。对留学生,可以把有限的精力集中于这几门课,相对说来更有把握获得好成绩。

作为对比,可以考虑澳洲维多利亚省的11-12年级课程,一般要求学生修读6门课,加拿大安大略省的预科课程,一般也要求学生选读6门课。而英国的A LEVEL,第一年只要求学生选读4门课,第二年可以进一步减少到3门。这对初来乍到、语言文化都相当陌生的国际留学生非常有利。对中国学生更是如此。以中国学生的勤奋,要在三四门课程中取得好成绩,不是很艰难的事。

第二,A LEVEL没有必修课的规定,学生可以根据自己的知识基础、兴趣爱好和特长,选择自己最有希望、最有把握获得好成绩的课程。其他国家的课程体系,一般都有规定的必修课。

第三,A LEVEL的数学课,可以当作两门课、甚至三门课来修读,这对中国学生最有利。

中国学生从小学高年级起,在数学训练上,就比西方国家学生普遍更严格,数学往往是中国学生的优势。世界上所有的中学和预科课程体系,纯美术音乐类的除外,说到底,就是两大支柱,第一是语言,第二是数学。换句话说,在学校中要获得好成绩的关键条件,只有两条:或者语言好(英语好),或者数学好(当然还要加上学生愿意学习)。中国中学生到英语国家留学,英语再好,与本土优秀学生还差了老一大截。因此,中国中学生到英语国家留学,想要在那些极端依赖英语的课程上与本土学生竞争,是不现实的不明智的。中国中学生应该扬长避短,避开英语的劣势,发挥数学的优势和强势。而英国的A LEVEL课程,正好为中国学生发挥数学优势提供了舞台。

中国学生只要在国内时数学属于中等以上,那么读A LEVEL课程时,就可以选读2门数学。数学对英语语言的要求最低。原因很简单:数学使用的语言最规范、意义最精确,没有文学中那些比喻、含蓄、夸张、讽刺等等花里狐骚的东西,数学词汇相对比较少,数学自身的公式和符号世界统一,不仅不依赖特定语言,还可以反过来帮助理解数学教材/试卷中使用的自然语言。

如果中国学生选2门数学,就意味着在A LEVEL第一年,数学成绩占了总成绩的百分之五十,第二年占了三分之二。换句话说,如果数学好,那么去英国读A LEVEL课程,“高考”中三分之二的好成绩已经握在手中,三分之二个身子,已经踏进世界名牌大学的大门啦。

因此,对那些数学比较好的中国中学生,通过读A LEVEL升入世界名牌大学,简直如履平地。

人人都认为:世界上没有简单的“高考”,世界上没有敞开大门的名牌大学。他们错了,A LEVEL课程对中国优秀学生,是非常“简单”的“高考”,也是通往世界名牌大学校门的大道平川。

只是我们不知道,说不定什么时候,A LEVEL课程也会变化,变成不利于中国学生“钻空子”。

第四,A LEVEL课程的考试,是“边读边考”,两年内根据教学进度分四轮考完,如果一轮考试中,某一科某一模块没有考好,下一轮(隔半年)还可以再考。这样就大大减轻了每次考试的复习负担和心理压力,也更有利于学生表现出自己的正常水平。学生不会因为一次考试发挥失常,就丧失人生的重大机会。

第五,A LEVEL课程是两年中分四轮考完,如果一轮考试中,某一科某一模块没有考好,那么学生重新考试时,完全有把握提高成绩。这是因为英国学校的假期很多、很长,学生可以利用假期,针对性地复习和预习,解决掉薄弱环节。英国的中学/预科,一年有三个大假,三个小假。三个大假是:暑假2个月,圣诞节假近一个月,复活节假20多天。三个小假就是每个学期的期中假(half term holiday),每个期中假都是一个星期左右。英国学生自己没有假期补习的习惯。但对于那些渴望读世界名牌大学的中国少年,为了增加把握,不妨利用这些假期来扩大自己学术成绩上的把握和优势。

第六、英国的A LEVEL课程考试,既是一种应试成分很强的知识性课程,很适合应试教育下“成长”的中国学生,但又绝对不会有象中国考试中那种专门设陷阱让学生往里死的怪题偏题。考试中出现的题型,在教科书里都可以找到。只要掌握了教材中的基本概念、原理、定理、定律,和基本类型的题目,在考试中都有把握获得好成绩。在国内重点中学中,经历过“整人坑人算计人”试题训练的小留学生,看见英国“试卷”中考题的简单明了,不禁要仰天大笑:真是天助我哇。

梁溪香榭 : 2008-11-18#227
回复: 偶搜集到的一些关于加拿大高中教育的资料 与大家分享

不要走到另一个极端而误解。不要以为A LEVEL是很容易读的课程体系。

实际上,A LEVEL课程体系相当严谨,课程广度大,难度也很高。要读好同样要下功夫。只不过,A LEVEL课程体系的总体安排,由于上面分析的种种原因,特别适合中国中学生,特别是那些数学基础比较好的学生。不需要是数学天才,只要在国内数学属于中上就够了。

概括地说,A LEVEL之所以“简单”,不是因为它的内容不够深、不够广,而是因为,它特别适合这样的学生:第一,英语不够好而数学足够好,第二,会考试,第三,肯勤奋用功。而这三条,几乎条条是中国中学生的特点。

还要补充几句话。也有不少中国中学生,到英国读A LEVEL读不下来。发生这种状况,主要有以下几个原因。

第一,被中介误导掉进了英国烂校。烂校的教学素质一塌糊涂,在全国性的考试中,学生自然无法过关,更谈不上获得好成绩。

第二,A LEVEL选科不当。不少中国中学生认为经济类商学类课程比科学类(数学物理)课程好读,或不喜欢科学类课程,因而放弃数学物理,选择读A LEVEL的经济学、商学、地理等课程。这正是犯了留学生的大忌。经济学、商学对英语的要求远高于数学物理,如果中国中学生没有充分的准备,往往会读不下去。

第三,英语太弱。尽管数学物理等课程对语言要求相对比较低,但毕竟要完全用英语听课、阅读、做作业、讨论。英语太蹩脚,也是读不好的。而中国学校里的英语教学,令人无法恭维。中国大陆少年到英国后,他们的英语,普遍比不上香港学生、台湾学生,以及马来西亚的学生。

第四,准备不足。许多家长与学生,听信“中国的课程是世界最难”的误导,以为中国学生到国外都能获得好成绩。事实并非如此。国外的课程,由于体系不一样,有难有易。如果出国前充分了解,尽量准备,一般都能顺利适应,否则,读起来很挣扎的学生,也比比皆是。

第五,纨绔子弟。万贯家产来路不明的暴富家庭,家长自然也不会言传身教什么好素质,子女娇生惯养、好逸恶劳,根本不愿意读书学习。或者个性太弱太娇、自理能力太弱,整个无法适应国外的生活,殃及学业。

梁溪香榭 : 2008-11-18#228
回复: 偶搜集到的一些关于加拿大高中教育的资料 与大家分享

有时候,连英国人自己都怀疑,A LEVEL要求学习的科目是不是太少了。当中国人忙着用一纸又一纸行政命令“减负”的时候,英国人开始为他们的少年“加负”。

在2000年以前,A LEVEL两年课程中的第一年和第二年,学生都只需要选三门课。2000年开始,英国A LEVEL课程的改革,规定一般第一年要选四门课;此外,还增加了“关键技能”课等部分新内容。

好在,英国大学的录取标准,迄今还是只取三门A LEVEL科目的成绩。我们不知道哪一天,英国大学会提高录取的门槛。

全世界的教育和课程专家,长久以来一直在争吵一个问题:通才教育好,还是专才教育好。通俗地说,就是学得广博些好,还是学得专精些好。

其实,这个问题是一个伪问题。人的精力是有限的,不可能样样都学都懂。人也不可能做不食人间烟火的专才,除了他自己的专业,其他什么都不懂;人,应该享受人生,而要享受人生,应该具备广泛的常识,参与广泛的活动。而教育,在培养人至少能够谋生的基础上,要让少年、青年去快乐地享受他或她独特的人生。

但学校教育的最大问题,是把人当作无差别的可以批量塑造的泥巴,对谁,都用统一的考试来修理。

考试,是现代教育中的“必要之恶”。正如政府和政府的权威是现代社会中的必要之恶。人还没有发明出比考试更好的评估方法。但考试这个必要之恶越少越好。

所以,体现在课程体系中,对必要科目和内容的专与精,只能用考试来强制。但这必要的范围,越小越好。考试的范围小了,学生比较容易达到专精,学生同时也自然可以有时间有余力,参与各种活动,扩展知识面和实践经验,包括在有兴趣的领域中自学,自由发展,达到适当的广博。2000年以前的A LEVEL课程,规定学生在两年中只要修三门课,无意中达到了这个效果。尽管英国人这样做可能是不自觉的。

可惜,英国的A LEVEL课程改革,竟然走上了一条“加负”的道路!

不难理解英国人的立场。在高福利发达国家,青少年游手好闲、不思上进,是普遍现象。家长和企业界期望通过加大课程负担,促使或强迫青少年多学一点。

其实,何苦来着。不愿意学习、不愿意思考的人,用鞭子,是不会令他们学会学习、学会思考的。

而考试,往往使学生更不会思考。

不要考试的学习,是最快乐的学习。这是一个无法实现的理想。退而求其次,考试最少的学习,是比较快乐的学习。A LEVEL作为一种“高考”课程,只需要考三门课,是一种考试最少的课程,而且可以进入世界名牌大学,何乐而不为呢。

梁溪香榭 : 2008-11-18#229
回复: 偶搜集到的一些关于加拿大高中教育的资料 与大家分享

A-LEVEL课程现在在国内很多主流, 一流高中有开设.
作为准备将来去加拿大上高中或大学而目前还在等VISA的孩子是个不错的过渡.
个人认为. 呵呵

allown : 2008-11-18#230
回复: 偶搜集到的一些关于加拿大高中教育的资料 与大家分享

看了这么好的帖子。不加声望不回贴。就显得不道德了。真诚感谢!

梁溪香榭 : 2008-11-18#231
回复: 偶搜集到的一些关于加拿大高中教育的资料 与大家分享

谢谢 容俺慢慢给各位回礼.
一天只能加10个.
现加到62楼.

惴惴不安 : 2008-11-18#232
回复: 偶搜集到的一些关于加拿大高中教育的资料 与大家分享

谢谢嫩把嫩滴第300贴放在俺滴楼里. 珍贵啊.
:wdb6::wdb6::wdb6:

BTW, 嫩亲家母玩失踪? :wdb23::wdb23:
我亲家母每天在窥视我们灌水呢!!:wdb8::wdb6::wdb19:

菲碧 : 2008-11-18#233
回复: 偶搜集到的一些关于加拿大高中教育的资料 与大家分享

谢谢 容俺慢慢给各位回礼.
一天只能加10个.
现加到62楼.
今天上来又看到嫩的好贴,一口气看完并收藏,:wdb17::wdb17::wdb17:
加分给嫩!这会儿不会忘啦:wdb6:

菲碧 : 2008-11-18#234
回复: 偶搜集到的一些关于加拿大高中教育的资料 与大家分享

一口气给大伙都加分后才给嫩加上,不容易啊!O(∩_∩)O哈哈~

feixue08 : 2008-11-18#235
回复: 偶搜集到的一些关于加拿大高中教育的资料 与大家分享

好贴,加分分致谢哈:wdb19::wdb17::wdb6:

C调云野 : 2008-11-18#236
回复: 偶搜集到的一些关于加拿大高中教育的资料 与大家分享

好贴。加分分

梁溪香榭 : 2008-11-18#237
回复: 偶搜集到的一些关于加拿大高中教育的资料 与大家分享

谢谢各位鼓励哈.
都加分分致谢啦.

yuanman : 2008-11-18#238
回复: 偶搜集到的一些关于加拿大高中教育的资料 与大家分享

刚学会加分分,第一个加给lz了。谢谢lz的好贴!

王二麻子 : 2008-11-18#239
回复: 偶搜集到的一些关于加拿大高中教育的资料 与大家分享

目前最关注的问题,谢谢楼主了

朱三雀 : 2008-11-18#240
回复: 偶搜集到的一些关于加拿大高中教育的资料 与大家分享

你孩子的英语真的过关吗?


  一个十三、四岁随父母移民加拿大的中国孩子,经过本地四年高中、三年名牌大学的学习,英文应该没有任何问题吧?像大多数同胞一样,老牛会毫不犹豫地回答:中文可能有问题,英文当然不会有问题。可事实并非如此乐观!

  这得从老牛目前所在的公司每年招实习大学生(Intern)说起。由于公司开发业务的需要,每年会从多伦多大学(University of Toronto)、女皇大学(Queens University)和阿尔伯塔大学(University of Alberta)等招收大量科学工程类实习生。由于要求实习的时间至少一年,使得一些只提供四个月实习期的大学如著名的滑铁卢大学(University of Waterloo)就不能给我们提供实习生源。好在公司给实习大学生的工资相当不菲;打个比方吧,如果移民同胞夫妻两人都打一般的Labour工,那么老牛所在公司实习大学生的工资绝不少于这对夫妻的工资总和。另外公司每年还会从实习生中挑选部分优秀学生,在他们结束实习后提前一年给他们Offer,这样这些学生返回大学完成最后一年学业后,就可直接回公司上班。正是由于如此优厚的实习待遇,每年大学生之间的竞争非常激烈。大家都知道,科学工程类专业的学生大多是来自亚洲及东欧国家移民的后代。这样一来,竞争一般会在华裔、印度裔和东欧裔学生之间展开;今年也不例外。不过例外的是,今年负责招聘实习大学生的任务是由老牛的顶头上司、一个至少祖宗三代生活在加拿大的本地人负责。虽然顶头上司负责招聘实习生,但分配到我们组的实习生最终干活还得老牛领着,所以无论如何老牛还是要稍稍关心一下的。

  谈起这次招聘的结果,顶头上司除了问老牛本文开头的这个问题外,还问道:为什么一个来自俄罗斯的学生,在加拿大只生活了三年,英文交流水平会高于在加拿大生活了七年的华裔学生呢?俄语和英语可也是不同的语系啊。老牛听完之后,非常震惊。心想,不会吧?老牛周围有不少朋友的子女可都是上了名牌大学,英语至少在老牛看来是不会有问题的。于是就追问,这是不是个别现象?顶头上司就说至少从这次的招聘面试来看,华裔学生虽然学习成绩不错甚至很好,但普遍有英语交流问题。顶头上司还认为,这可能是由于在加拿大,尤其是多伦多,华裔社区越来越大,华裔学生很容易找到同族裔的同学和朋友,互相之间很容易用母语交流;加上在家里和父母交流也是用母语;这自然会影响到英文交流水平的提高。

  无论老牛的顶头上司一家之言是否完全有道理,但移民的孩子,尤其是那些在中国接受了小学甚至中学教育的孩子,他们的英文水平真的过关了吗?真的能满足将来工作的需求吗?毕竟,移民他乡,没有一个父母愿意自己的孩子还像自己一样,因为憋脚的英语终身影响着找工和升迁,头顶上总有一块“玻璃天花板”,更枉论融入主流社会了。

  那么,在加拿大上了三年高中和四年大学的孩子,英语到底应该是在什么样的水平呢?按老牛顶头上司的说法是:英语交流应该完全没有问题。有没有口音呢?有,但只应该是一点点(原话是:a little bit)。这样的要求是不是有些苛刻?但这至少是一个本地人的真实期望。

  移民孩子的英语水平能否达到本地人的期望,不可否认有许多因素。孩子自身的素质是一个重要的因素,但作为父母的移民同胞是不是也有值得深思的部分呢?答案是不容置疑的。

  在过去的数年中,在我们的社区,在我们的免费中文报纸上,“名校区”这个词大概同胞们都不陌生吧,房产经纪对此可是作出了重要“贡献”的。结果呢,华人同胞一哄而上,涌进了“名校区”。 “名校区”的房价扶摇直上,这可让原来的房主大大地赚了一笔;可从此“名校”也就成了“华裔名校”,一个班上清一色黑头发已不是个别现象。本文前面所提到的华裔大学生应该是在1998或1999年左右移民加拿大的,老牛不敢肯定是不是也是从一些“华裔名校”走出来的;但这“华裔名校”的后果肯定会慢慢呈现出来。

  老牛曾以为“名校区”这种现象在多伦多发生过,同胞们已经引以为戒。不想,随着华人同胞“西进”,这“名校区”现象在密西沙加正在发生。在密西沙加中部一所所谓的“名高中”,一个ESL班上清一色来自中国大陆的孩子,校方这么安排可能是便于管理。但后果呢?这些孩子除了上课用英文,写作业用英文,听说讨论完全可以接着用中文。可想而知,经过四年本地“名高中”,他们的英语水平能有多少提高?老牛不知道这些孩子的父母了不了解这个事实;而知道后又是如何感想的?毕竟孩子们将来不可能永远生活在华人小圈圈的,是要在加拿大这个大社区工作生活的。

  其实,掌握语言技巧对孩子们来说仅仅是第一步,还要理解和掌握本地的文化风俗。这是必须的,但也是比较难的。老牛的小儿五岁移民加拿大,中文对他来说有问题,英语对他来说应该不成任何问题。可是,有一次和他的老师面谈时,他的老师诚恳地建议要让孩子多参加一些集体的课外活动或体育活动或社区活动。为什么呢?他的老师说,有时他不太理解她说的笑话。无疑,小儿遇到了一位很负责任的好老师,她点出了很多华裔孩子的一个通病。华裔孩子聪明好学,华裔家庭重视教育,但大多数华人父母是技术移民,由于在中国接受的教育以及过去大多从事技术工作等原因,往往看重知识的获取,而忽视其它人文方面的教育。由于华人家庭的社会活动大多局限于华人社区,孩子们除了学校外,也很难有机会和其它族裔的孩子真正交流,以致学习成绩很好,却听不懂一个笑话。

  很多同胞移民的一个理由是为了孩子,但如果孩子的英语水平都没有真的过关,我们对得起自己移民的初衷吗?是不是真的是时候问一问自己:孩子的英语是否真的过关?但愿老牛不是杞人忧天。

很及时很好。刷卡感谢!

王二麻子 : 2008-11-18#241
回复: 偶搜集到的一些关于加拿大高中教育的资料 与大家分享

http://www.sinocanada.cn/viewpage.asp?id=113

这里有个中加学校,如果是去这读书的话,不知道读了Grade 11是不是可以去加拿大读Grade 12?

海怡宝贝 : 2008-11-19#242
回复: 偶搜集到的一些关于加拿大高中教育的资料 与大家分享

http://www.sinocanada.cn/viewpage.asp?id=113

这里有个中加学校,如果是去这读书的话,不知道读了Grade 11是不是可以去加拿大读Grade 12?
我女儿曾在这个学校读了一年Grade 10年级,现已转学。
就是不知在BC注册学籍还没求证过是否加拿大的学校就一定能接收正常升学,我也关注。明年我们如顺利到加国的话就应升Grade 12。听说,去了加国要参加考试,不知针对我们这样读书的会怎么样对待?但我已做好最坏的打算,最多去了多读一年,还好我们还小,早去早适应,多读对今后进入大学打好扎实基础。

王二麻子 : 2008-11-19#243
回复: 偶搜集到的一些关于加拿大高中教育的资料 与大家分享

俺家小麻子明年初中毕业,按照现在加拿大投资移民的进度,偶估计起码是后年,大后年才有希望,也就是说小孩子只能在高二和高三的时候才能去加拿大,看了这个贴,必须提前作准备了。
现在的选择有两个,一个是 A-Level,但这个好像和加拿大的高中不搭噶,并且如果在国内读的话,从IGCSE到A-Level要四年时间(当然也可以高二去申请A-Level 1年级,但时间又不对了,也许还没读完就办成了。)另外一个选择就是读中加学校的课程,貌似很加拿大的Grade10.11.12可以衔接上,但转学的问题不知道如何解决?并且就算转成了,因为在加拿大的读书没超过4年,还是得考e文,是不是这样理解?

梁溪香榭 : 2008-11-19#244
回复: 偶搜集到的一些关于加拿大高中教育的资料 与大家分享

俺家小麻子明年初中毕业,按照现在加拿大投资移民的进度,偶估计起码是后年,大后年才有希望,也就是说小孩子只能在高二和高三的时候才能去加拿大,看了这个贴,必须提前作准备了。
现在的选择有两个,一个是 A-Level,但这个好像和加拿大的高中不搭噶,并且如果在国内读的话,从IGCSE到A-Level要四年时间(当然也可以高二去申请A-Level 1年级,但时间又不对了,也许还没读完就办成了。)另外一个选择就是读中加学校的课程,貌似很加拿大的Grade10.11.12可以衔接上,但转学的问题不知道如何解决?并且就算转成了,因为在加拿大的读书没超过4年,还是得考e文,是不是这样理解?

俺家小伙子也是明年初中毕业. 俺们08年5月的FN, 瞧着加拿大的蜗牛速度, 2010年都不知道能不能登陆. :wdb23::wdb23::wdb23:
关于A-LEVEL课程, 国内一般都是在高一先上O-LEVEL, 作为过渡, 特别是语言方面. 然后高二高三才是A-LEVEL.
反正是全英文授课, 就当是提高语言吧. 啥时候VISA到啥时候走人.
相比中加学校, 俺感觉至少我们这边的A-LEVEL是由本市最好的高中开设的, 就收50个人, 2个班. 开展得还不错, 收的都是好孩子, 家长比较放心.
转学应该不是问题啊. 不可能不收吧. 呵呵.
考托福或雅思是避免不了的. 其实这也没那么难. 不用太担心这个.

王二麻子 : 2008-11-19#245
回复: 偶搜集到的一些关于加拿大高中教育的资料 与大家分享

08年5月的FN,就混到了这么高的威望,偶滴神啊!嫩天天不睡觉挂论坛吗?:wdb17:

偶是08年8月滴FN,紧跟上嫩的步伐!:wdb6:

shirley6811 : 2008-11-19#246
回复: 偶搜集到的一些关于加拿大高中教育的资料 与大家分享

:wdb10::wdb10::wdb10:太有用了

梁溪香榭 : 2008-11-19#247
回复: 偶搜集到的一些关于加拿大高中教育的资料 与大家分享

:wdb10::wdb10::wdb10:太有用了

谢谢支持.
回头嫩教教俺们炒股票哈 :wdb6:

梁溪香榭 : 2008-11-19#248
回复: 偶搜集到的一些关于加拿大高中教育的资料 与大家分享

纷纷加到98

shirley6811 : 2008-11-19#249
回复: 偶搜集到的一些关于加拿大高中教育的资料 与大家分享

外星人很有道理的,我还要向他学习呢。嫩是我的榜样,又会做菜,文笔又好!很会总结。:wdb10::wdb17::wdb17::wdb17:

尘烟一缕 : 2008-11-19#250
回复: 偶搜集到的一些关于加拿大高中教育的资料 与大家分享

一口起读完梁姐姐的帖子,真是佩服梁姐姐呀,是五体投地的那种!虽然我家丫头尚在小学四年级,暂时不用考虑A-LEVEL的事,但还是很有用,很值得收藏的,谢谢梁姐姐热情而耐心的解答!可是怎么给嫩加分分呢?偶是不是还没资格呀?

梁溪香榭 : 2008-11-19#251
回复: 偶搜集到的一些关于加拿大高中教育的资料 与大家分享

一口起读完梁姐姐的帖子,真是佩服梁姐姐呀,是五体投地的那种!虽然我家丫头尚在小学四年级,暂时不用考虑A-LEVEL的事,但还是很有用,很值得收藏的,谢谢梁姐姐热情而耐心的解答!可是怎么给嫩加分分呢?偶是不是还没资格呀?

心领啦. 嫩要发满50贴就可以加分啦.

能对大家有用俺就:wdb6:

吉人天相 : 2008-11-19#252
回复: 偶搜集到的一些关于加拿大高中教育的资料 与大家分享

梁溪,你太伟大了。哪找到的这么详细的资料,真是个称职的好妈妈。加声望了。

rwx1378 : 2008-11-19#253
回复: 偶搜集到的一些关于加拿大高中教育的资料 与大家分享

梁溪,你太伟大了。哪找到的这么详细的资料,真是个称职的好妈妈。加声望了。


俺也:wdb17:你,儿子太幸运了。
溪,想上美国前50的大学最好加上SAT的成绩。

梁溪香榭 : 2008-11-20#254
回复: 偶搜集到的一些关于加拿大高中教育的资料 与大家分享

梁溪,你太伟大了。哪找到的这么详细的资料,真是个称职的好妈妈。加声望了。

谢谢 回礼啦.
都是网上找的. 嫩也是称职的好妈妈.

梁溪香榭 : 2008-11-20#255
回复: 偶搜集到的一些关于加拿大高中教育的资料 与大家分享

俺也:wdb17:你,儿子太幸运了。
溪,想上美国前50的大学最好加上SAT的成绩。

俺崇拜嫩闺女. :wdb19:

上美国大学那移民监咋办捏?

shirley6811 : 2008-11-20#256
回复: 偶搜集到的一些关于加拿大高中教育的资料 与大家分享

学制设置

1. 中英学制对比

中 国
初 中
高 中
大学本科(4年)
硕士研究生

3 年
高一
高二
高三
大一
大二
大三
大四
3 年

英 国
普通初中
IGCSE课程
A Level课程
大学本科(3 年)
硕士研究生

3 年
G1
G2
AS
A2
大一
大二
大三
1-2年


2. 课程介绍

(1) A Level课程
A Level即General Certificate Of Education Advanced Level(剑桥高级水准课程),是现代文化、科技、教育、法治和工商的源头国家英国的国民课程,也是英国学生进入大学前的主要测试课程。与中国四年制本科教育相比较,英国大学本科是三年制。A Level课程是衔接高中教育与本科教育的桥梁,在其开设的70多门课程中,有相当多的科目是中国大学一年级的课程。如会计学、商业学、经济学、计算机学等等。

A Level 课程于1951年在英国正式启用,其证书由CIE即剑桥大学国际考试中心颁发,被称为英国的金牌课程和金牌证书。学生凭借3门以上A Level课程成绩即可申报世界上绝大多数名牌大学。美国大学给予拥有A Level证书的同学给予一定比例的学分转换。
如果你想具体了解哪些大学院校承认和接受IGCSE,请使用我们的认可搜索《认可A Level和IGCSE课程的大学》查询。

 

(2) IGCSE课程
IGCSE即INTERNATIONAL GENERAL CERTIFICATE OF SECONDARY EDUCATION(普通中等教育证书课程),它是专门中四和中五年级(相当于中国高一、高二)学生设计的一个国际课程体系。获得剑桥大学国际考试委员会授权的IGCSE资格证书的学生将为其以后的A 和AS Level学习和就业做很好的准备。IGCSE被在世界范围内的学术机构和用人单位广泛认可。

IGCSE所提供的学科范围非常广泛。在整个课程中,实际应用和理论知识达到一个很好的平衡互补。因其广泛和核心的学科设置,IGCSE适合不同能力程度的学生学习。IGCSE课程的教学内容允许和适合本土化,使之和不同地区的教学内容相结合。它也适合那些母语为非英语的学生,他们的整个考试过程都是被认可的。

IGCSE课程是全球最流行的针对15至17岁学生的国际课程。它成功培养了数以千计的学生,让为他们下一步的教育,包括继续学习A和AS Level学习做好准备;同时也使他们为将来的就业培养了必要的技能。IGCSE是被全球众多大学和就业单位认可的课程。

目前,IGCSE已在全球100多个国家推广,被视为和英国的GCSE相当的国际课程。

IGCSE的评分是从A*至G的八个等级,其中的A至E相当于O Level的A至E等级。在一些国家,获得IGCSE认可是满足大学的入学要求之一。在其他一些国家,IGCSE认可被更广泛地用于学习A Level,AS,IB和美国APC课程的准备。IGCSE英语学科的等级C(适用于英语为第一和第二语言的学生)适用于英国以及其他许多国家的大学英语水平要求。

如果你想具体了解哪些大学院校承认和接受IGCSE,请使用我们的认可搜索《认可A Level和IGCSE课程的大学》查询。

IGCSE、A LEVEL 课程体系是当今世界最广泛采用的通用教育课程体系,其课程体系及评估模式也是目前国内正在进行的基础教育课程改革所借鉴的主要课程体系之一。

IGCSE适用于15-17岁之间的学生;A LEVEL 课程适用于17-19岁之间的学生,这两项课程均适用于国内已经完成九年义务教育的中国中学生或具有同等学历的在校学生。

3.中心学制设置

中心参照中英学制对比和世界名牌大学对A Level课程学生中学修读课程的入学要求,学制设为4年。即IGCSE学部2年,参加剑桥大学国际考试委员会(CIE)的考试后将获得IGCSE证书;A Level学部2年,参加CIE的考试后将获得A Level课程证书,直接报读国外大学。

 

 学制设置

1. 中英学制对比

中 国
初 中
高 中
大学本科(4年)
硕士研究生

3 年
高一
高二
高三
大一
大二
大三
大四
3 年

英 国
普通初中
IGCSE课程
A Level课程
大学本科(3 年)
硕士研究生

3 年
G1
G2
AS
A2
大一
大二
大三
1-2年


2. 课程介绍

(1) A Level课程
A Level即General Certificate Of Education Advanced Level(剑桥高级水准课程),是现代文化、科技、教育、法治和工商的源头国家英国的国民课程,也是英国学生进入大学前的主要测试课程。与中国四年制本科教育相比较,英国大学本科是三年制。A Level课程是衔接高中教育与本科教育的桥梁,在其开设的70多门课程中,有相当多的科目是中国大学一年级的课程。如会计学、商业学、经济学、计算机学等等。

A Level 课程于1951年在英国正式启用,其证书由CIE即剑桥大学国际考试中心颁发,被称为英国的金牌课程和金牌证书。学生凭借3门以上A Level课程成绩即可申报世界上绝大多数名牌大学。美国大学给予拥有A Level证书的同学给予一定比例的学分转换。
如果你想具体了解哪些大学院校承认和接受IGCSE,请使用我们的认可搜索《认可A Level和IGCSE课程的大学》查询。

 

(2) IGCSE课程
IGCSE即INTERNATIONAL GENERAL CERTIFICATE OF SECONDARY EDUCATION(普通中等教育证书课程),它是专门中四和中五年级(相当于中国高一、高二)学生设计的一个国际课程体系。获得剑桥大学国际考试委员会授权的IGCSE资格证书的学生将为其以后的A 和AS Level学习和就业做很好的准备。IGCSE被在世界范围内的学术机构和用人单位广泛认可。

IGCSE所提供的学科范围非常广泛。在整个课程中,实际应用和理论知识达到一个很好的平衡互补。因其广泛和核心的学科设置,IGCSE适合不同能力程度的学生学习。IGCSE课程的教学内容允许和适合本土化,使之和不同地区的教学内容相结合。它也适合那些母语为非英语的学生,他们的整个考试过程都是被认可的。

IGCSE课程是全球最流行的针对15至17岁学生的国际课程。它成功培养了数以千计的学生,让为他们下一步的教育,包括继续学习A和AS Level学习做好准备;同时也使他们为将来的就业培养了必要的技能。IGCSE是被全球众多大学和就业单位认可的课程。

目前,IGCSE已在全球100多个国家推广,被视为和英国的GCSE相当的国际课程。

IGCSE的评分是从A*至G的八个等级,其中的A至E相当于O Level的A至E等级。在一些国家,获得IGCSE认可是满足大学的入学要求之一。在其他一些国家,IGCSE认可被更广泛地用于学习A Level,AS,IB和美国APC课程的准备。IGCSE英语学科的等级C(适用于英语为第一和第二语言的学生)适用于英国以及其他许多国家的大学英语水平要求。

如果你想具体了解哪些大学院校承认和接受IGCSE,请使用我们的认可搜索《认可A Level和IGCSE课程的大学》查询。

IGCSE、A LEVEL 课程体系是当今世界最广泛采用的通用教育课程体系,其课程体系及评估模式也是目前国内正在进行的基础教育课程改革所借鉴的主要课程体系之一。

IGCSE适用于15-17岁之间的学生;A LEVEL 课程适用于17-19岁之间的学生,这两项课程均适用于国内已经完成九年义务教育的中国中学生或具有同等学历的在校学生。

3.中心学制设置

中心参照中英学制对比和世界名牌大学对A Level课程学生中学修读课程的入学要求,学制设为4年。即IGCSE学部2年,参加剑桥大学国际考试委员会(CIE)的考试后将获得IGCSE证书;A Level学部2年,参加CIE的考试后将获得A Level课程证书,直接报读国外大学。

 

 

 



 

shirley6811 : 2008-11-20#257
回复: 偶搜集到的一些关于加拿大高中教育的资料 与大家分享

剑桥A Level是最适合中国高中生就读的国际课程:

  多样的学科选择:  
  剑桥A Level课程体系有70门课程可供选择,上海师大国际课程专家经过潜心研究,选择开设了中国学生最具竞争力的课程,既充分尊重兴趣,又充分考虑潜能。A Level第一年只需选修4门课程,第二年只需选修3门课程。包括牛津、剑桥在内的所有英国大学都只需要3到4门A Level课程的成绩。
  良好的数学基础:
   中国高中生的数学能力普遍比西方学生强,而A Level课程体系中数学可选修两门。对于绝大多数中国高中生而言,选择A Level数学就意味着一只脚已踏入了世界一流大学之门。
  稳定的考试模式:
   A Level课程的试卷全部由剑桥大学统一命题,统一阅卷,全球统一考试时间,统一评分标准,其权威性为世界公认。剑桥考试的标准化和稳定性,使从小受到严格考试训练的中国学生极易脱颖而出。
  等级式评分系统:
  剑桥A Level考试不是百分制,考试成绩分为ABCDE五个等级。这样对于细小的考试失误并不影响学生的等级评定。况且剑桥A Level考试不是一试定终身,每年有两次考试,如果学生对自己的成绩不满意,可以补考。申报大学以较好的成绩为准。
   可回避英文差距:
   中国学生与同龄的英语国家学生在英语基础有差距是正常的,因此文学、经济、法律、历史等文科类课程的学习成绩往往不理想。A Level课程不像IB课程那样要求必修文科课程,数理学科是中国学生的强项,而国外高校录取又不分文理科,甚至欢迎跨学科报考,因此报读上师大剑桥国际中心为学生特设的课程,既可以回避中国学生的英文差距,又不影响报读大学专业的选择

梁溪香榭 : 2008-11-20#258
回复: 偶搜集到的一些关于加拿大高中教育的资料 与大家分享

除了A-LEVEL, 还有一个选择是IB国际文凭
这个课程也是很好的, 难度可能比A-LEVEL更高些. 很注重能力培养.
可惜俺们这边没有学校开设.

rwx1378 : 2008-11-20#259
回复: 偶搜集到的一些关于加拿大高中教育的资料 与大家分享

俺崇拜嫩闺女. :wdb19:

上美国大学那移民监咋办捏?

说服闺女和老妈在一起呗,总不能一家三口呆三个地儿,三个国家吧。

shirley6811 : 2008-11-20#260
回复: 偶搜集到的一些关于加拿大高中教育的资料 与大家分享

梁姐姐,那么这种A-LEVEL 学校难进吗?

梁溪香榭 : 2008-11-20#261
回复: 偶搜集到的一些关于加拿大高中教育的资料 与大家分享

梁姐姐,那么这种A-LEVEL 学校难进吗?

好象不算太难

shirley6811 : 2008-11-20#262
回复: 偶搜集到的一些关于加拿大高中教育的资料 与大家分享

:wdb10::wdb10::wdb10:
好象不算太难

pwjnd : 2008-11-21#263
回复: 偶搜集到的一些关于加拿大高中教育的资料 与大家分享

:溪溪做啥事都这么周全认真!大大的赞!:wdb17::wdb17:

pwjnd : 2008-11-21#264
回复: 偶搜集到的一些关于加拿大高中教育的资料 与大家分享

剑桥A Level是最适合中国高中生就读的国际课程:

  多样的学科选择:  
  剑桥A Level课程体系有70门课程可供选择,上海师大国际课程专家经过潜心研究,选择开设了中国学生最具竞争力的课程,既充分尊重兴趣,又充分考虑潜能。A Level第一年只需选修4门课程,第二年只需选修3门课程。包括牛津、剑桥在内的所有英国大学都只需要3到4门A Level课程的成绩。
  良好的数学基础:
   中国高中生的数学能力普遍比西方学生强,而A Level课程体系中数学可选修两门。对于绝大多数中国高中生而言,选择A Level数学就意味着一只脚已踏入了世界一流大学之门。
  稳定的考试模式:
   A Level课程的试卷全部由剑桥大学统一命题,统一阅卷,全球统一考试时间,统一评分标准,其权威性为世界公认。剑桥考试的标准化和稳定性,使从小受到严格考试训练的中国学生极易脱颖而出。
  等级式评分系统:
  剑桥A Level考试不是百分制,考试成绩分为ABCDE五个等级。这样对于细小的考试失误并不影响学生的等级评定。况且剑桥A Level考试不是一试定终身,每年有两次考试,如果学生对自己的成绩不满意,可以补考。申报大学以较好的成绩为准。
   可回避英文差距:
   中国学生与同龄的英语国家学生在英语基础有差距是正常的,因此文学、经济、法律、历史等文科类课程的学习成绩往往不理想。A Level课程不像IB课程那样要求必修文科课程,数理学科是中国学生的强项,而国外高校录取又不分文理科,甚至欢迎跨学科报考,因此报读上师大剑桥国际中心为学生特设的课程,既可以回避中国学生的英文差距,又不影响报读大学专业的选择
shirley的信息好详尽,给您加纷纷。

kindchen : 2008-11-21#265
回复: 偶搜集到的一些关于加拿大高中教育的资料 与大家分享

10岁以上孩子来这里英语主要有以下不足:
1. 快速阅读能力差,严重影响到信息接受效率.
2. 专业词汇贫乏,影响学习的理解深度.
3. 写作能力低下,不能准确的表述思想和主题.
这几方面的提高不象口语交流短时间取得比较大的进步,要花很大的力气和很长的时间.我们请了家教来帮儿子,希望儿子能早点追上来.

FredHe : 2008-11-21#266
回复: 偶搜集到的一些关于加拿大高中教育的资料 与大家分享

10岁以上孩子来这里英语主要有以下不足:
1. 快速阅读能力差,严重影响到信息接受效率.
2. 专业词汇贫乏,影响学习的理解深度.
3. 写作能力低下,不能准确的表述思想和主题.
这几方面的提高不象口语交流短时间取得比较大的进步,要花很大的力气和很长的时间.我们请了家教来帮儿子,希望儿子能早点追上来.

请问家教是西人?费用多少?

shirley6811 : 2008-11-21#267
回复: 偶搜集到的一些关于加拿大高中教育的资料 与大家分享

请问家教是西人?费用多少?
同问?:wdb23::wdb23::wdb23:

0810准备 : 2008-11-21#268
回复: 偶搜集到的一些关于加拿大高中教育的资料 与大家分享

拜读了。lz你打算登陆哪个城市啊?我女儿跟你孩子差不多,但是我们现在才寄出申请资料,比你们晚大半年了。很担心登陆城市对小孩读书的影响。

agmyxl : 2008-11-21#269
回复: 偶搜集到的一些关于加拿大高中教育的资料 与大家分享

俺家小伙子也是明年初中毕业. 俺们08年5月的FN, 瞧着加拿大的蜗牛速度, 2010年都不知道能不能登陆. :wdb23::wdb23::wdb23:
.

2010年夏秋季就可毕业啦:wdb6::wdb6::wdb6:

kindchen : 2008-11-21#270
回复: 偶搜集到的一些关于加拿大高中教育的资料 与大家分享

请问家教是西人?费用多少?

是一个作过中学付校长的西人,非常负责人但是选学生也很严.费用每小时$50,每周课外给批改一篇作文.儿子跟这个老师以后进步挺大.

梁溪香榭 : 2008-11-21#271
回复: 偶搜集到的一些关于加拿大高中教育的资料 与大家分享

10岁以上孩子来这里英语主要有以下不足:
1. 快速阅读能力差,严重影响到信息接受效率.
2. 专业词汇贫乏,影响学习的理解深度.
3. 写作能力低下,不能准确的表述思想和主题.
这几方面的提高不象口语交流短时间取得比较大的进步,要花很大的力气和很长的时间.我们请了家教来帮儿子,希望儿子能早点追上来.

非常感谢嫩提供滴资料.
俺要给俺儿子看看.
加分啦.

梁溪香榭 : 2008-11-21#272
回复: 偶搜集到的一些关于加拿大高中教育的资料 与大家分享

拜读了。lz你打算登陆哪个城市啊?我女儿跟你孩子差不多,但是我们现在才寄出申请资料,比你们晚大半年了。很担心登陆城市对小孩读书的影响。

具体城市待定. 初步考虑安省.
咱们这个年纪的挺多的. 都面临类似的问题.

梁溪香榭 : 2008-11-21#273
回复: 偶搜集到的一些关于加拿大高中教育的资料 与大家分享

2010年夏秋季就可毕业啦:wdb6::wdb6::wdb6:

呵呵 托嫩吉言啦.
加分分

梁溪香榭 : 2008-11-21#274
回复: 偶搜集到的一些关于加拿大高中教育的资料 与大家分享

是一个作过中学付校长的西人,非常负责人但是选学生也很严.费用每小时$50,每周课外给批改一篇作文.儿子跟这个老师以后进步挺大.

很不错啊. :wdb19::wdb17:

这样的西人老师容易找吗? 还是可遇不可求呢?

朱三雀 : 2008-11-21#275
回复: 偶搜集到的一些关于加拿大高中教育的资料 与大家分享

发现了这个转帖一下想听听大家的意见。
ZT--
大学录取和奖学金的颁发不是凭学生毕业自哪所学校,而是学生在高二和高三的平均成绩及在学校的排名。而且,平均成绩再高,如果学生不是排名第一(既便是学校500-600 毕业生中的top 1%),他/她几乎没有希望拿到大学的最高奖学金。e.g. Dalhousie 的最高奖学金只给每所学校的第一名。如果学生的平均成绩达不到95或更高的话,第二,三档的奖学金怕也无缘了。总之,大学要的是成绩,不是学校。加拿大社会上为高中毕业生升大学提供的各种奖学金要的也是成绩,排名,外加学生的社会工作与贡献。

agmyxl : 2008-11-21#276
回复: 偶搜集到的一些关于加拿大高中教育的资料 与大家分享

梁溪,你可真是位用心的母亲,你孩子有你这般的母爱(关键是知道怎么去爱),真是他的幸福。

梁溪香榭 : 2008-11-22#277
回复: 偶搜集到的一些关于加拿大高中教育的资料 与大家分享

梁溪,你可真是位用心的母亲,你孩子有你这般的母爱(关键是知道怎么去爱),真是他的幸福。

谢谢哈
这个俺也要拿去给儿子看看.:wdb6:

梁溪香榭 : 2008-11-22#278
回复: 偶搜集到的一些关于加拿大高中教育的资料 与大家分享

发现了这个转帖一下想听听大家的意见。
ZT--
大学录取和奖学金的颁发不是凭学生毕业自哪所学校,而是学生在高二和高三的平均成绩及在学校的排名。而且,平均成绩再高,如果学生不是排名第一(既便是学校500-600 毕业生中的top 1%),他/她几乎没有希望拿到大学的最高奖学金。e.g. Dalhousie 的最高奖学金只给每所学校的第一名。如果学生的平均成绩达不到95或更高的话,第二,三档的奖学金怕也无缘了。总之,大学要的是成绩,不是学校。加拿大社会上为高中毕业生升大学提供的各种奖学金要的也是成绩,排名,外加学生的社会工作与贡献。

关于这方面我也问过, 的确是这样的. 包括大学录取看的也是在高二高三的平均成绩. 也就是说在那些所谓名校如果不能名列前茅的, 不一定能上心仪的专业. 而那些在烂校拔尖的反而还能拿奖学金.

FredHe : 2008-11-22#279
回复: 偶搜集到的一些关于加拿大高中教育的资料 与大家分享

是一个作过中学付校长的西人,非常负责人但是选学生也很严.费用每小时$50,每周课外给批改一篇作文.儿子跟这个老师以后进步挺大.

谢谢分享

朱三雀 : 2008-11-22#280
回复: 偶搜集到的一些关于加拿大高中教育的资料 与大家分享

关于这方面我也问过, 的确是这样的. 包括大学录取看的也是在高二高三的平均成绩. 也就是说在那些所谓名校如果不能名列前茅的, 不一定能上心仪的专业. 而那些在烂校拔尖的反而还能拿奖学金.
谢谢。和中国的规则不一样啊呵呵。刷卡感谢!

梁溪香榭 : 2008-11-22#281
回复: 偶搜集到的一些关于加拿大高中教育的资料 与大家分享

谢谢。和中国的规则不一样啊呵呵。刷卡感谢!

是啊. 好多地方和国内的理念是不同啊.
谢谢嫩滴纷纷哈.
冻着呢. 回头回礼哈.

0810准备 : 2008-11-24#282
回复: 偶搜集到的一些关于加拿大高中教育的资料 与大家分享

具体城市待定. 初步考虑安省.
咱们这个年纪的挺多的. 都面临类似的问题.
就是就是啊。如果登陆温哥华,要是满19岁了,学分还不够,那就只能上成人高中,还是一边读学院,一边读成人高中修学分啊?

agmyxl : 2008-11-24#283
回复: 偶搜集到的一些关于加拿大高中教育的资料 与大家分享

“一般进入加拿大大学要综合考虑中学平时成绩和省试成绩”
据我自己了解的对这句话做个解释,以温哥华为例,申请大学时,中学成绩占60%,省试成绩占40%,但很多时候申请大学时,省考还没有进行,所以申请大学的成绩往往就是中学的平时成绩了。而这个中学成绩是不分学校的好坏的,不管是在好的学校还是在差的学校的学生根据平时的成绩申请好大学的几率是平等的。这里就可以扭转一部分新移民的思维,部门新移民家长刚开始登陆的时候总是固执地要求把孩子送到排名靠前(在加拿大中小学的排名,除华人等少数族裔外,基本上是既不被鼓励也不被看重)的中学,而不考虑孩子的实际情况,往往原本成绩比较好的孩子在那里成绩反倒越来越不好,或者丝毫没有优势了,倒申请不到好的大学了;其实根据孩子的实际情况,让他在一般的学校里学习,让他能跟得上学习进度,平时成绩及校考得比较好的话,倒能申请到好的大学。所以根据孩子的实际情况入学是最好,不能盲目地用国内的观点去安排孩子的教育问题。


转载,供参考

kindchen : 2008-11-24#284
回复: 偶搜集到的一些关于加拿大高中教育的资料 与大家分享

非常感谢嫩提供滴资料.
俺要给俺儿子看看.
加分啦.

谢谢,都是为了孩子.

sun and moon : 2008-11-24#285
回复: 偶搜集到的一些关于加拿大高中教育的资料 与大家分享

谢谢分享!:wdb10::wdb10::wdb10:

梁溪香榭 : 2008-11-25#286
回复: 偶搜集到的一些关于加拿大高中教育的资料 与大家分享

就是就是啊。如果登陆温哥华,要是满19岁了,学分还不够,那就只能上成人高中,还是一边读学院,一边读成人高中修学分啊?

这方面俺不清楚啊. SORRY啊

梁溪香榭 : 2008-11-25#287
回复: 偶搜集到的一些关于加拿大高中教育的资料 与大家分享

“一般进入加拿大大学要综合考虑中学平时成绩和省试成绩”
据我自己了解的对这句话做个解释,以温哥华为例,申请大学时,中学成绩占60%,省试成绩占40%,但很多时候申请大学时,省考还没有进行,所以申请大学的成绩往往就是中学的平时成绩了。而这个中学成绩是不分学校的好坏的,不管是在好的学校还是在差的学校的学生根据平时的成绩申请好大学的几率是平等的。这里就可以扭转一部分新移民的思维,部门新移民家长刚开始登陆的时候总是固执地要求把孩子送到排名靠前(在加拿大中小学的排名,除华人等少数族裔外,基本上是既不被鼓励也不被看重)的中学,而不考虑孩子的实际情况,往往原本成绩比较好的孩子在那里成绩反倒越来越不好,或者丝毫没有优势了,倒申请不到好的大学了;其实根据孩子的实际情况,让他在一般的学校里学习,让他能跟得上学习进度,平时成绩及校考得比较好的话,倒能申请到好的大学。所以根据孩子的实际情况入学是最好,不能盲目地用国内的观点去安排孩子的教育问题。


转载,供参考

谢谢好资料.
等解冻就给嫩加分分

梁溪香榭 : 2008-11-25#288
回复: 偶搜集到的一些关于加拿大高中教育的资料 与大家分享

谢谢,都是为了孩子.

9494
谢谢理解 :wdb6:

jenniferlee : 2008-11-26#289
回复: 偶搜集到的一些关于加拿大高中教育的资料 与大家分享

好帖子要顶起来,让更多的人受益!

海怡宝贝 : 2008-11-26#290
回复: 偶搜集到的一些关于加拿大高中教育的资料 与大家分享

顶!

菲碧 : 2008-11-26#291
回复: 偶搜集到的一些关于加拿大高中教育的资料 与大家分享

溪溪的好贴也来再顶一下!

鼠小妹 : 2008-11-27#292
回复: 偶搜集到的一些关于加拿大高中教育的资料 与大家分享

推荐大家看一下youku.com上北京精华中学袁腾飞讲的历史,很好玩,遇到这样的老师孩子们肯定不会逃学

梁溪香榭 : 2008-11-27#293
回复: 偶搜集到的一些关于加拿大高中教育的资料 与大家分享

谢谢 
去看看

kindchen : 2008-11-27#294
回复: 偶搜集到的一些关于加拿大高中教育的资料 与大家分享

很不错啊. :wdb19::wdb17:

这样的西人老师容易找吗? 还是可遇不可求呢?

香榭,抱歉出差几天,今天才回复.这个老师很好但是挺难找,我们是一个偶然机会朋友介绍,等了两个月才开课的.这里找家教在7,8月份新学年开始以前会容易一点.

梁溪香榭 : 2008-11-27#295
回复: 偶搜集到的一些关于加拿大高中教育的资料 与大家分享

香榭,抱歉出差几天,今天才回复.这个老师很好但是挺难找,我们是一个偶然机会朋友介绍,等了两个月才开课的.这里找家教在7,8月份新学年开始以前会容易一点.

谢谢
有个好老师真是太好了. 嫩也放心许多.

梁溪香榭 : 2008-11-27#296
回复: 偶搜集到的一些关于加拿大高中教育的资料 与大家分享

贴个笑话, 也有点道理.

要对学生好

有一个新任老师,上课第一天校长就把他请到校长室,语重心长地对他说;
假如学生考试得a,那么你要对他好,因为他以后是科学家,会对社会有所贡献。
假如学生得b ,你也要对他好,因为他以后会返校当老师,可能是你得同事。
假如有学生得c,你也要对他好,因为他以后会赚大钱,会捐给学校好多钱;
假如有学生作弊抓到,你更要对他好,因为他以后要竞选议员或总统,所以要对他更好。

海怡宝贝 : 2008-11-27#297
回复: 偶搜集到的一些关于加拿大高中教育的资料 与大家分享

贴个笑话, 也有点道理.

要对学生好

有一个新任老师,上课第一天校长就把他请到校长室,语重心长地对他说;
假如学生考试得a,那么你要对他好,因为他以后是科学家,会对社会有所贡献。
假如学生得b ,你也要对他好,因为他以后会返校当老师,可能是你得同事。
假如有学生得c,你也要对他好,因为他以后会赚大钱,会捐给学校好多钱;
假如有学生作弊抓到,你更要对他好,因为他以后要竞选议员或总统,所以要对他更好。
不错,挺有道理的。:wdb6:

菲碧 : 2008-11-28#298
回复: 偶搜集到的一些关于加拿大高中教育的资料 与大家分享

贴个笑话, 也有点道理.

要对学生好

有一个新任老师,上课第一天校长就把他请到校长室,语重心长地对他说;
假如学生考试得a,那么你要对他好,因为他以后是科学家,会对社会有所贡献。
假如学生得b ,你也要对他好,因为他以后会返校当老师,可能是你得同事。
假如有学生得c,你也要对他好,因为他以后会赚大钱,会捐给学校好多钱;
假如有学生作弊抓到,你更要对他好,因为他以后要竞选议员或总统,所以要对他更好。

O(∩_∩)O哈哈~:wdb6:

lydh : 2008-11-28#299
回复: 偶搜集到的一些关于加拿大高中教育的资料 与大家分享

感谢LZ,您辛苦了!珍藏备用。

shirley6811 : 2008-11-28#300
回复: 偶搜集到的一些关于加拿大高中教育的资料 与大家分享

贴个笑话, 也有点道理.

要对学生好

有一个新任老师,上课第一天校长就把他请到校长室,语重心长地对他说;
假如学生考试得a,那么你要对他好,因为他以后是科学家,会对社会有所贡献。
假如学生得b ,你也要对他好,因为他以后会返校当老师,可能是你得同事。
假如有学生得c,你也要对他好,因为他以后会赚大钱,会捐给学校好多钱;
假如有学生作弊抓到,你更要对他好,因为他以后要竞选议员或总统,所以要对他更好。
:wdb20::wdb20::wdb20:,很贴切!

尘烟一缕 : 2008-11-28#301
回复: 偶搜集到的一些关于加拿大高中教育的资料 与大家分享

贴个笑话, 也有点道理.

要对学生好

有一个新任老师,上课第一天校长就把他请到校长室,语重心长地对他说;
假如学生考试得a,那么你要对他好,因为他以后是科学家,会对社会有所贡献。
假如学生得b ,你也要对他好,因为他以后会返校当老师,可能是你得同事。
假如有学生得c,你也要对他好,因为他以后会赚大钱,会捐给学校好多钱;
假如有学生作弊抓到,你更要对他好,因为他以后要竞选议员或总统,所以要对他更好。
虽然是个笑话,但不无道理!:wdb10:

梁溪香榭 : 2008-11-28#302
回复: 偶搜集到的一些关于加拿大高中教育的资料 与大家分享

给各位加纷纷
谢谢鼓励

gxy1968 : 2008-11-28#303
回复: 偶搜集到的一些关于加拿大高中教育的资料 与大家分享

俺家小伙子也是明年初中毕业. 俺们08年5月的FN, 瞧着加拿大的蜗牛速度, 2010年都不知道能不能登陆. :wdb23::wdb23::wdb23:
关于A-LEVEL课程, 国内一般都是在高一先上O-LEVEL, 作为过渡, 特别是语言方面. 然后高二高三才是A-LEVEL.
反正是全英文授课, 就当是提高语言吧. 啥时候VISA到啥时候走人.
相比中加学校, 俺感觉至少我们这边的A-LEVEL是由本市最好的高中开设的, 就收50个人, 2个班. 开展得还不错, 收的都是好孩子, 家长比较放心.
转学应该不是问题啊. 不可能不收吧. 呵呵.
考托福或雅思是避免不了的. 其实这也没那么难. 不用太担心这个.
我是08年4月的FN,孩子也是明年上高中,感谢楼主的信息:wdb17:加分感谢了

梁溪香榭 : 2008-11-28#304
回复: 偶搜集到的一些关于加拿大高中教育的资料 与大家分享

我是08年4月的FN,孩子也是明年上高中,感谢楼主的信息:wdb17:加分感谢了

:wdb6::wdb6::wdb6:

sz艾美 : 2008-11-28#305
回复: 偶搜集到的一些关于加拿大高中教育的资料 与大家分享

大家都是高中学生的家长,有没初中的同道人啊

梁溪香榭 : 2008-11-28#306
回复: 偶搜集到的一些关于加拿大高中教育的资料 与大家分享

大家都是高中学生的家长,有没初中的同道人啊

JENNIFER就是啊

lxn1463 : 2008-11-30#307
回复: 偶搜集到的一些关于加拿大高中教育的资料 与大家分享

我想了解一下,我的弟弟09年2月份就20岁了,我们打算1月份登陆多伦多,他应该可以入读高中吧?还有,会不会到了21岁就要转移去成人高中?我想了解清楚!我也希望楼主可以给D意见我们参考!
PS:我弟弟已经高中毕业,也读了1年多雅思!:wdb17:

lxn1463 : 2008-12-01#308
回复: 偶搜集到的一些关于加拿大高中教育的资料 与大家分享

期考快到了,没有多少时间可以上网,希望知情人士可以解答一下我的问题,谢谢

梁溪香榭 : 2008-12-02#309
回复: 偶搜集到的一些关于加拿大高中教育的资料 与大家分享

我想了解一下,我的弟弟09年2月份就20岁了,我们打算1月份登陆多伦多,他应该可以入读高中吧?还有,会不会到了21岁就要转移去成人高中?我想了解清楚!我也希望楼主可以给D意见我们参考!
PS:我弟弟已经高中毕业,也读了1年多雅思!:wdb17:

期考快到了,没有多少时间可以上网,希望知情人士可以解答一下我的问题,谢谢

俺没有权威答案, 所以没有及时回答.
20岁应该是可以在安省上高中的. 21岁之前都可以.
建议你去多伦多版块和教育版块问问.

bluecat : 2008-12-03#310
回复: 偶搜集到的一些关于加拿大高中教育的资料 与大家分享

谢谢分享:)

wucedric : 2008-12-04#311
回复: 偶搜集到的一些关于加拿大高中教育的资料 与大家分享

与很多这里的同学一样, 预计俺的儿子也是要到高中阶段才能出去.
搜集了一些高中教育资料, 希望能帮到大家.

ZT
加拿大高中留学需谨慎

篇首语:
1、如果你是在国内高二毕业,想在一年之内顺利的考上加拿大大学,请放弃(除非你是天才)
2、如果在你选择的是私立学校,不用浪费时间看这篇文章了

或许我开篇的两句话有点偏激,但也是不争的事实,我是在一名在加拿大留学的高中生,因此我不会像中介一样去美化这个留学有多好、前途有多光明,我会实事求是的写下我经历的一切。我在国内完成了高二的学业,按照计划,应该是在加拿大完成Grade12也就是高三的学业就申请大学,但听完多伦多教育局举办的orientation(类似介绍会)后,我明白了,1年是不可能顺利的考上大学。​


教育体制的差异。加拿大的教育体系多是以学分制为主体,也就是说,你每一门课都有一个学分,当你把学分修满后才可以高中毕业,所谓的年级:Grade10、Grade11、Grade12(国内叫高一高二高三)在加拿大实际上是没区别的,只是作为一个作为区别年龄的标准(我是指对像我这种国际交流生),就算你是在Grade12你也可能同时修Grade11和Grade12的学科,因为加拿大要求如果你要读更高一级的学科,必须有低一级的学科作为基础,不能随便越级,例如说Grade12的微积分就需要Grade11的函数作为基础,当你拿到了Grade11函数的学分后才可以修Grade12的微积分,而当你拿齐了所有的必修的学分(30分)才可以申请大学,因此,作为一个国际生,我认为一年之内很难或者说不可能解决语言的问题并且修齐所有的必修学分。​
首先太感谢你了,发了这么多这么有用的资料上来:wdb19::wdb17::wdb19::wdb10:
问你几个问题,希望帮帮我:我已经20岁了:wdb7:,大概明年登陆,有国内的高中毕业证。
:wdb2:1、像我这样的申请国外的大学要怎么办呀?有雅思和托福的成绩就可以了吗?我还需要什么东西呀?
:wdb2:2、想申请好一点的大学(想去西安大略)有可能吗?
:wdb2:3、还用不用念国外的高中了?
:wdb2:4、念国外的私立高中和公立的对申请大学有什么区别吗?

wucedric : 2008-12-04#312
回复: 偶搜集到的一些关于加拿大高中教育的资料 与大家分享

我想了解一下,我的弟弟09年2月份就20岁了,我们打算1月份登陆多伦多,他应该可以入读高中吧?还有,会不会到了21岁就要转移去成人高中?我想了解清楚!我也希望楼主可以给D意见我们参考!
PS:我弟弟已经高中毕业,也读了1年多雅思!:wdb17:

我觉得我跟你弟弟的情况很像啊!我一月份也20岁了,也有国内的高中毕业证,不过刚刚ME,可能要明年中旬才能登陆,现在对于申请国外的大学也一头雾水,:wdb7::wdb7::wdb7:移民中介说有了国内高中毕业证和托福雅思成绩就可以申请国外大学,是这样吗?:wdb2:

梁溪香榭 : 2008-12-05#313
回复: 偶搜集到的一些关于加拿大高中教育的资料 与大家分享

首先太感谢你了,发了这么多这么有用的资料上来:wdb19::wdb17::wdb19::wdb10:
问你几个问题,希望帮帮我:我已经20岁了:wdb7:,大概明年登陆,有国内的高中毕业证。
:wdb2:1、像我这样的申请国外的大学要怎么办呀?有雅思和托福的成绩就可以了吗?我还需要什么东西呀?
:wdb2:2、想申请好一点的大学(想去西安大略)有可能吗?
:wdb2:3、还用不用念国外的高中了?
:wdb2:4、念国外的私立高中和公立的对申请大学有什么区别吗?

对于大学申请, 我也曾经留意过. 那些名气大的大学基本上都要参考高考成绩的. 对雅思托福有最低分数规定. 很多热门大学专业要求雅思都在7.0以上.
关于大学申请, 建议你去心仪大学的网站, 那里才能找到最权威的答案.
每个学校都有针对来自不同高中教育体系的毕业生申请不同专业的非常具体的要求.

lxn1463 : 2008-12-06#314
回复: 偶搜集到的一些关于加拿大高中教育的资料 与大家分享

感谢楼主的热心解答,东西很受用..

盈婆婆 : 2008-12-06#315
回复: 偶搜集到的一些关于加拿大高中教育的资料 与大家分享

太谢谢楼主了。
想请教一下。在你看来,一个十二三岁的普通中国孩子。正处于青春期,比较内向。来安省之后,私校和公校哪个更加适合呢?

梁溪香榭 : 2008-12-06#316
回复: 偶搜集到的一些关于加拿大高中教育的资料 与大家分享

太谢谢楼主了。
想请教一下。在你看来,一个十二三岁的普通中国孩子。正处于青春期,比较内向。来安省之后,私校和公校哪个更加适合呢?

关于这个问题可谓见仁见智.
俺问过在那边的朋友.
首先要上好的私立学校是要排队,要考试的.
先不论排队(据说有些顶尖私立学校排几年的队都不希奇), 私立学校不设ESL, 直接要求英语达到本地学生水平. 这个对我们的孩子有难度.
俺认为, 只能先上公立, 同时排队. 英语过关再考虑私立.
私立的确有私立的优势, 但一定是要真正好的私立学校. 不然还不如在公立呢. 嫩说呢?

盈婆婆 : 2008-12-06#317
回复: 偶搜集到的一些关于加拿大高中教育的资料 与大家分享

关于这个问题可谓见仁见智.
俺问过在那边的朋友.
首先要上好的私立学校是要排队,要考试的.
先不论排队(据说有些顶尖私立学校排几年的队都不希奇), 私立学校不设ESL, 直接要求英语达到本地学生水平. 这个对我们的孩子有难度.
俺认为, 只能先上公立, 同时排队. 英语过关再考虑私立.
私立的确有私立的优势, 但一定是要真正好的私立学校. 不然还不如在公立呢. 嫩说呢?
先公立再私立,这个主意不错。就公立学校而言,你觉得是好学区,华人孩子多的学校好些呢?还是排名前一点的天主教学校更加好呢?

梁溪香榭 : 2008-12-07#318
回复: 偶搜集到的一些关于加拿大高中教育的资料 与大家分享

先公立再私立,这个主意不错。就公立学校而言,你觉得是好学区,华人孩子多的学校好些呢?还是排名前一点的天主教学校更加好呢?

俺信基督教. 但这个问题俺不了解. 不能瞎说.
有信仰绝对是好事. 但中国教育出来的孩子要接受西方宗教不容易啊.

其实西方文化是建立在宗教基础上的.
如果我们的孩子接受宗教教育就能更加好滴理解西方文化.

樱桃老丸子 : 2008-12-07#319
回复: 偶搜集到的一些关于加拿大高中教育的资料 与大家分享

:wdb17::wdb17::wdb17:谢谢分享,很有价值的资料.

盈婆婆 : 2008-12-07#320
回复: 偶搜集到的一些关于加拿大高中教育的资料 与大家分享

俺信基督教. 但这个问题俺不了解. 不能瞎说.
有信仰绝对是好事. 但中国教育出来的孩子要接受西方宗教不容易啊.

其实西方文化是建立在宗教基础上的.
如果我们的孩子接受宗教教育就能更加好滴理解西方文化.
太感谢了。

盈婆婆 : 2008-12-07#321
回复: 偶搜集到的一些关于加拿大高中教育的资料 与大家分享

再问一下楼主。在哪里可以查到安省初高中排名。最好是公私校在一起的。现在我找的都是被地产经纪打乱了的。

梁溪香榭 : 2008-12-07#322
回复: 偶搜集到的一些关于加拿大高中教育的资料 与大家分享

再问一下楼主。在哪里可以查到安省初高中排名。最好是公私校在一起的。现在我找的都是被地产经纪打乱了的。

这是我以前找到的.
其实个人认为也没什么意思.
心理安慰而已.
仅供参考

附件


盈婆婆 : 2008-12-07#323
回复: 偶搜集到的一些关于加拿大高中教育的资料 与大家分享

这是我以前找到的.
其实个人认为也没什么意思.
心理安慰而已.
仅供参考
除了加分,俺还能说啥?

梁溪香榭 : 2008-12-07#324
回复: 偶搜集到的一些关于加拿大高中教育的资料 与大家分享

谢谢嫩滴分分. 等俺解冻了就回礼

盈婆婆 : 2008-12-07#325
回复: 偶搜集到的一些关于加拿大高中教育的资料 与大家分享

刚才大概看了下你给我的排名,七百多学校,可是够壮观的。不知道是不是公私校都在内了。

梁溪香榭 : 2008-12-07#326
回复: 偶搜集到的一些关于加拿大高中教育的资料 与大家分享

刚才大概看了下你给我的排名,七百多学校,可是够壮观的。不知道是不是公私校都在内了。

貌似都在里面了.
看前面100个足够了吧

盈婆婆 : 2008-12-07#327
回复: 偶搜集到的一些关于加拿大高中教育的资料 与大家分享

够了够了够了。。。。。俺老实滴只知道点头聊。呵呵。
以后还要向楼上请教。谢谢先。

sz艾美 : 2008-12-08#328
回复: 偶搜集到的一些关于加拿大高中教育的资料 与大家分享

关于这个问题可谓见仁见智.
俺问过在那边的朋友.
首先要上好的私立学校是要排队,要考试的.
先不论排队(据说有些顶尖私立学校排几年的队都不希奇), 私立学校不设ESL, 直接要求英语达到本地学生水平. 这个对我们的孩子有难度.
俺认为, 只能先上公立, 同时排队. 英语过关再考虑私立.
私立的确有私立的优势, 但一定是要真正好的私立学校. 不然还不如在公立呢. 嫩说呢?
说的很正确啊

朱三雀 : 2008-12-08#329
回复: 偶搜集到的一些关于加拿大高中教育的资料 与大家分享

为不怎么上网的TZ提供一下高中排名:
安省高中排名:(page 11/20)

http://www.fraserinstitute.org/Commerce.Web/product_files/70ONESC08RANK.pdf

BC省:

http://www.fraserinstitute.org/Commerce.Web/product_files/70BCESC08RANK.pdf

AB省(page 36/41)

http://www.fraserinstitute.org/Commerce.Web/product_files/70ABHSC08COMP.pdf

agmyxl : 2008-12-08#330
回复: 偶搜集到的一些关于加拿大高中教育的资料 与大家分享

有没有07-08年排名?

盈婆婆 : 2008-12-08#331
回复: 偶搜集到的一些关于加拿大高中教育的资料 与大家分享

真是郁闷,刚听说私校是没有排名的。

余则成 Liftmaster : 2008-12-09#332
回复: 偶搜集到的一些关于加拿大高中教育的资料 与大家分享

Thank you !

samtan07 : 2008-12-09#333
回复: 偶搜集到的一些关于加拿大高中教育的资料 与大家分享

看帖顶帖。

梁溪香榭 : 2008-12-11#334
回复: 偶搜集到的一些关于加拿大高中教育的资料 与大家分享

给楼上几位都加分分

北美枫 : 2008-12-11#335
回复: 偶搜集到的一些关于加拿大高中教育的资料 与大家分享

给楼上几位都加分分




看到你的帖子就顶:wdb6:

菲碧 : 2008-12-11#336
回复: 偶搜集到的一些关于加拿大高中教育的资料 与大家分享

给楼上几位都加分分
接着顶一把:wdb6::wdb6::wdb6:

梁溪香榭 : 2008-12-12#337
回复: 偶搜集到的一些关于加拿大高中教育的资料 与大家分享

看到你的帖子就顶:wdb6:

想要俺滴分分木问题,岛上滴片片先贡献出来

wucedric : 2008-12-12#338
回复: 偶搜集到的一些关于加拿大高中教育的资料 与大家分享

对于大学申请, 我也曾经留意过. 那些名气大的大学基本上都要参考高考成绩的. 对雅思托福有最低分数规定. 很多热门大学专业要求雅思都在7.0以上.
关于大学申请, 建议你去心仪大学的网站, 那里才能找到最权威的答案.
每个学校都有针对来自不同高中教育体系的毕业生申请不同专业的非常具体的要求.


哦!多谢多谢啦!!!!!:wdb17:

余则成 Liftmaster : 2008-12-14#339
回复: 偶搜集到的一些关于加拿大高中教育的资料 与大家分享

push again!

北美枫 : 2008-12-14#340
回复: 偶搜集到的一些关于加拿大高中教育的资料 与大家分享

想要俺滴分分木问题,岛上滴片片先贡献出来





呵呵 才看到这帖子 :wdb6:, 不过我在岛上的片片全在我老婆那 因为我笨的不会在网上发照片:wdb1:

hhwzr : 2008-12-17#341
回复: 偶搜集到的一些关于加拿大高中教育的资料 与大家分享

赞一个:)

海怡宝贝 : 2008-12-17#342
回复: 偶搜集到的一些关于加拿大高中教育的资料 与大家分享

又来报到了.

梁溪香榭 : 2008-12-17#343
回复: 偶搜集到的一些关于加拿大高中教育的资料 与大家分享

哦!多谢多谢啦!!!!!:wdb17:

push again!

赞一个:)

又来报到了.

谢谢
都刷卡加分分. :wdb6:

梁溪香榭 : 2008-12-17#344
回复: 偶搜集到的一些关于加拿大高中教育的资料 与大家分享

各位有什么资料也贡献点出来吧.

余则成 Liftmaster : 2008-12-17#345
回复: 偶搜集到的一些关于加拿大高中教育的资料 与大家分享

想要什么方面的资料啊?

梁溪香榭 : 2008-12-17#346
回复: 偶搜集到的一些关于加拿大高中教育的资料 与大家分享

有什么就贴什么吧. 谢谢啦.

响铃当当 : 2008-12-17#347
回复: 偶搜集到的一些关于加拿大高中教育的资料 与大家分享

加拿大投资移民

对投资移民最具有吸引力的因素

最重要的因素:子女在加拿大的接受教育:
绝大多数的投资移民的目的都是为了孩子在加拿大的发展,因此我们应该了解移民后让子女在加拿大接受教育究竟有什么好处。这一论题也同样适用于有子女的技术移民和可能移民与教育的关系

1.加拿大有世界上最好的大学。
(1)加拿大拥有世界上最好的大学。毫无疑问,在各种大学排名中,美国的大学的排名和数量都独占鳌头,但由于签证、费用等众所周知的原因,除非申请人有非常出色的考试成绩和很好的运气,对中国学子来说美国大学往往只能是水中月、镜中花。因此,在这里我们将美国学校排除在比较范围之列。那么,加拿大的大学就会赫然出现的领先的位置。

商科院校:

西安大略大学(University of Western Ontario) 的Ivey Business School
北美排名:第15名
全球排名:第18名
约克大学(York University) 的Schulich School of Business
北美排名:第25名
全球排名:第31名
多伦多大学(University of Toronto) 的 Rotman School of Business
北美排名:第25名
全球排名:第31名

这三所加拿大商学院同时都位列美国以外全球商学院的前10名。西安大略名列第3名。此外,在排名中皇后大学商学院居46位。UBC大学第60位。其他上榜的还有卡加利大学 83、麦克马斯特大学88位。在该排行榜上,中国方面两所进入100名的都是香港大学:香港科技大学 (四十七位 )、香港中文大学 (六十七 )。此外,新加坡大学居第一百名,但是台湾、日本及韩国均榜上无名。

自然科学:
2004年根据"校友获诺贝尔奖和菲尔兹奖人数"、"员工获诺贝尔奖和菲尔兹奖人数"、 "高引用率研究人员人数"、"《自然》和《科学》论文数"、"SCI论文数"、 "教职人员平均学术表现"等指标排名中,不包含美国大学和包含美国大学,加拿大大学排名分别是:
多伦多大学名列第6名和第24名
UBC大学名列第8名和第36名
而中国大学排名最靠前的是台湾大学第153名,大陆地区排名最靠前的北京大学则在200名以外。

滑铁卢大学:
世界上最好的数学和计算机专业。她拥有世界上最大的数学院,在北美她是最早采用计算机教学的学府。从这里毕业的学生始终是北美IT公司追逐的对象,微软公司曾经出资包机邀请滑铁卢大学的应届毕业生和他们的家人到微软的美国总部参观,以求他们毕业后微软工作。

麦克马斯特:
拥有北美著名的工程学院,坐落于加拿大工业制造中心汉密尔顿,几乎所有的工程专业排名都可以进入北美前10名。学生就业率高达97%。麦克马斯特甚至保存着爱因斯坦的部分大脑。

麦吉尔大学:
位于蒙特利尔,素有“北方哈佛”之称,商学院、医学院、法学院闻名于世。

皇后大学:
有加拿大的普林斯顿之称,与多伦多大学、麦吉尔合称加拿大的长春藤。他的工程物理专业仅次于美国的普林斯顿和康奈尔。

加拿大大学的水平明显高于澳大利亚、日本等中国留学生的热门选择,与法国、德国、瑞士等欧洲国家相比更有主流语言的优势,可以说,加拿大是中国学生除了美国以外的最佳选择。

我们说,加拿大拥有世界上最好的大学还有另外一重含义,就是加拿大大学在美国、乃至世界上有更高的认知度。美国雇主更容易接受加拿大的教育背景,因此在加拿大求学更容易在美国找到一分理想的工作。

加拿大大学教育水平为众多华人富豪认可,香港是大钻石王老五中3人毕业于美国,4人毕业于英国,另外2个人则毕业于加拿大西安大略,还有一个只在美国读到中学3年级。

新世界集团郑裕彤长孙郑之桓:西安大略经济系学士
新鸿基创办人郭得胜外孙黄缵嘉:西安略经济系学士
李泽矩的夫人王富信:毕业于UBC大学工商管理专业

2.移民后子女在加拿大上学可以节省大笔的留学费用。

家长都希望自己的子女能够进入世界上最好的大学,毕业后有一个美好的前途,为此不惜花费大量金钱送子女出国读书。以加拿大为例,如果在中国读完高中后出国读书,通常需要先进入语言学校或者大学预科,补习外语后再进入大学,加上4年的大学学习,大约需要花费50-60万人民币,毕业后还要迅速找到工作以申请工作签证,工作若干年后方有机会申请移民,这一过程十分漫长。如果子女年龄尚小,需要从高中读起,费用则成倍增加。尽管我们都知道从高中开始在国外接受教育可以使孩子更容易适应以后的生活,但限于费用等因素很难实现。

  申请加拿大移民需要花费70万人民币,但取得身份后子女可以享受12年级以下的免费教育和十分低廉的大学教育,节省的费用几乎与申请移民时的花销相当,如果子女从高中甚至更低年级开始在加拿大接受免费教育,节省的费用将大大超过移民申请的付出。更何况,申请人一家已经取得了加拿大的身份,将来上好大学的机会和发展的机会都会远远超过留学生。

如果你计划送两个子女出国读书,那么选择投资移民,其节省的费用将可以在加拿大购置一所不错的住宅。

以下是一个比较表格

假设一个子女从小学开始接受双语教育,中学毕业后进入最好的大学学习商科专业,直到硕士毕业,根据这个假设,我们列出下面的表格进行比较。
学费数据来自以下学校2005年的最新公告学费:
小学和中学数据来自中国著名的大连枫叶学校
中国大学数据来自清华大学
加拿大大学数据来自多伦多大学

(货币:美元)
在中国接受教育 以留学生身份接受教育 移民身份接受教育
小学6年 32000 110000 0
中学6年 60000 110000 0
大学4年 3000 50000 13000
硕士2年 10000 60000 35000
合计 105000 330000 48000

从上面的表格我们可以看出,移民后在加拿大接受教育比留学生节省6-28万美金的费用,甚至比在中国接收双语教育还要节省数万美金。
而我们还没有考虑在加拿大可以得到的贷款、奖学金等因素。在加拿大,大学生以上的学生可以申请每年数千加元的贷款,和金额不等的奖学金,这些足以保证一个大学生在没有经济来源的情况下完成学业。
我们就按照一个子女从高中还是在加拿大接受教育,可以节省12万美金。如果申请加拿大贷款或者奖学金,则可以节省超过16万美金,这个数字是投资移民所需费用的1.5-2倍。

2.取得移民身份,方能实现选择世界一流大学读书的愿望。

尽管到美国和加拿大读书是众所周知的最佳选择,但低得可怜的学生签证成功率使留学申请成了一场人生前途的赌博,根据加拿大驻北京使馆的统计数字:学生签证平均成功率只有区区62%,通过外语考试(IELTS A类6.5分)后直接申请大学的签证成功率平均85%,而需要通过语言项目的学习才能进入大学的成功率更是低至40%。

对技术移民而言

凡是符合加拿大大学录取条件的申请者,几乎无一例外地都具备了加拿大技术移民的申请条件,申请技术移民成功率可以接近100%,而申请学生签证成功率却只有85%,如果得不到奖学金,这一成功率还要进一步降低,就算你宁愿比移民支付更多的学费,宁愿毕业后不在加拿大、美国发展,宁愿得不到任何奖学金或者贷款支持,宁愿不要加拿大移民身份仍然坚持以留学生的身份去加拿大求学,加拿大政府却不一定发给你签证。这的确是一个可笑的逻辑,但事实正是如此。

对投资移民而言

面对40%的签证成功率,望子成龙的父母空有万贯家产却不能给子女的求学的愿望提供任何帮助,因为即使你愿意比移民家庭多支付数十万、上百万甚至更多倍(如果不止一个子女)的费用,但却有高达60%的机会被拒签,而拒签的理由往往是让人哭笑不得:签证官怀疑你是否有足够的经济力量支付学费。

当我们了解了上面的形势,就会知道为什么澳大利亚成了中国学生家长的第一选择?北美学生签证成功率低是造成这种局面的根本原因,由于得不到签证,学生们只好退而求其次,澳大利亚虽然教育整体水平不如加拿大,但毕竟是英语教育,而且签证要比加拿大容易得多,自然成了出国留学家庭的现实选择。

其实,只要申请移民,上面的问题自然迎刃而解,移民签证的平均成功率远远高于留学签证,费用也要便宜很多,我们要对符合移民条件的家庭说:“你们真的很幸运!”

朱三雀 : 2008-12-17#348
回复: 偶搜集到的一些关于加拿大高中教育的资料 与大家分享

有个问题求解。
排名好的中学附近的房子贵的没道理。让人有种冤大头的feeling.(算下来比留学费用可能都多if 买房子的话)
1.想不划片入学有什么途径吗??
2.天才班是小学初中高中都有吗?

梁溪香榭 : 2008-12-17#349
回复: 偶搜集到的一些关于加拿大高中教育的资料 与大家分享

有个问题求解。
排名好的中学附近的房子贵的没道理。让人有种冤大头的feeling.(算下来比留学费用可能都多if 买房子的话)
1.想不划片入学有什么途径吗??
2.天才班是小学初中高中都有吗?

俺听说有这么个法子, 不是效果如何.
先在好学区租房, 等孩子入了学再在别的地区买房. 迂回.
第2问就不清楚了.

梁溪香榭 : 2008-12-17#350
回复: 偶搜集到的一些关于加拿大高中教育的资料 与大家分享

谢谢响铃, 明天送上分分

海怡宝贝 : 2008-12-18#351
回复: 偶搜集到的一些关于加拿大高中教育的资料 与大家分享

加拿大投资移民

对投资移民最具有吸引力的因素

最重要的因素:子女在加拿大的接受教育:
绝大多数的投资移民的目的都是为了孩子在加拿大的发展,因此我们应该了解移民后让子女在加拿大接受教育究竟有什么好处。这一论题也同样适用于有子女的技术移民和可能移民与教育的关系

1.加拿大有世界上最好的大学。
(1)加拿大拥有世界上最好的大学。毫无疑问,在各种大学排名中,美国的大学的排名和数量都独占鳌头,但由于签证、费用等众所周知的原因,除非申请人有非常出色的考试成绩和很好的运气,对中国学子来说美国大学往往只能是水中月、镜中花。因此,在这里我们将美国学校排除在比较范围之列。那么,加拿大的大学就会赫然出现的领先的位置。

商科院校:

西安大略大学(University of Western Ontario) 的Ivey Business School
北美排名:第15名
全球排名:第18名
约克大学(York University) 的Schulich School of Business
北美排名:第25名
全球排名:第31名
多伦多大学(University of Toronto) 的 Rotman School of Business
北美排名:第25名
全球排名:第31名

这三所加拿大商学院同时都位列美国以外全球商学院的前10名。西安大略名列第3名。此外,在排名中皇后大学商学院居46位。UBC大学第60位。其他上榜的还有卡加利大学 83、麦克马斯特大学88位。在该排行榜上,中国方面两所进入100名的都是香港大学:香港科技大学 (四十七位 )、香港中文大学 (六十七 )。此外,新加坡大学居第一百名,但是台湾、日本及韩国均榜上无名。

自然科学:
2004年根据"校友获诺贝尔奖和菲尔兹奖人数"、"员工获诺贝尔奖和菲尔兹奖人数"、 "高引用率研究人员人数"、"《自然》和《科学》论文数"、"SCI论文数"、 "教职人员平均学术表现"等指标排名中,不包含美国大学和包含美国大学,加拿大大学排名分别是:
多伦多大学名列第6名和第24名
UBC大学名列第8名和第36名
而中国大学排名最靠前的是台湾大学第153名,大陆地区排名最靠前的北京大学则在200名以外。

滑铁卢大学:
世界上最好的数学和计算机专业。她拥有世界上最大的数学院,在北美她是最早采用计算机教学的学府。从这里毕业的学生始终是北美IT公司追逐的对象,微软公司曾经出资包机邀请滑铁卢大学的应届毕业生和他们的家人到微软的美国总部参观,以求他们毕业后微软工作。

麦克马斯特:
拥有北美著名的工程学院,坐落于加拿大工业制造中心汉密尔顿,几乎所有的工程专业排名都可以进入北美前10名。学生就业率高达97%。麦克马斯特甚至保存着爱因斯坦的部分大脑。

麦吉尔大学:
位于蒙特利尔,素有“北方哈佛”之称,商学院、医学院、法学院闻名于世。

皇后大学:
有加拿大的普林斯顿之称,与多伦多大学、麦吉尔合称加拿大的长春藤。他的工程物理专业仅次于美国的普林斯顿和康奈尔。

加拿大大学的水平明显高于澳大利亚、日本等中国留学生的热门选择,与法国、德国、瑞士等欧洲国家相比更有主流语言的优势,可以说,加拿大是中国学生除了美国以外的最佳选择。

我们说,加拿大拥有世界上最好的大学还有另外一重含义,就是加拿大大学在美国、乃至世界上有更高的认知度。美国雇主更容易接受加拿大的教育背景,因此在加拿大求学更容易在美国找到一分理想的工作。

加拿大大学教育水平为众多华人富豪认可,香港是大钻石王老五中3人毕业于美国,4人毕业于英国,另外2个人则毕业于加拿大西安大略,还有一个只在美国读到中学3年级。

新世界集团郑裕彤长孙郑之桓:西安大略经济系学士
新鸿基创办人郭得胜外孙黄缵嘉:西安略经济系学士
李泽矩的夫人王富信:毕业于UBC大学工商管理专业

2.移民后子女在加拿大上学可以节省大笔的留学费用。

家长都希望自己的子女能够进入世界上最好的大学,毕业后有一个美好的前途,为此不惜花费大量金钱送子女出国读书。以加拿大为例,如果在中国读完高中后出国读书,通常需要先进入语言学校或者大学预科,补习外语后再进入大学,加上4年的大学学习,大约需要花费50-60万人民币,毕业后还要迅速找到工作以申请工作签证,工作若干年后方有机会申请移民,这一过程十分漫长。如果子女年龄尚小,需要从高中读起,费用则成倍增加。尽管我们都知道从高中开始在国外接受教育可以使孩子更容易适应以后的生活,但限于费用等因素很难实现。

  申请加拿大移民需要花费70万人民币,但取得身份后子女可以享受12年级以下的免费教育和十分低廉的大学教育,节省的费用几乎与申请移民时的花销相当,如果子女从高中甚至更低年级开始在加拿大接受免费教育,节省的费用将大大超过移民申请的付出。更何况,申请人一家已经取得了加拿大的身份,将来上好大学的机会和发展的机会都会远远超过留学生。

如果你计划送两个子女出国读书,那么选择投资移民,其节省的费用将可以在加拿大购置一所不错的住宅。

以下是一个比较表格

假设一个子女从小学开始接受双语教育,中学毕业后进入最好的大学学习商科专业,直到硕士毕业,根据这个假设,我们列出下面的表格进行比较。
学费数据来自以下学校2005年的最新公告学费:
小学和中学数据来自中国著名的大连枫叶学校
中国大学数据来自清华大学
加拿大大学数据来自多伦多大学

(货币:美元)
在中国接受教育 以留学生身份接受教育 移民身份接受教育
小学6年 32000 110000 0
中学6年 60000 110000 0
大学4年 3000 50000 13000
硕士2年 10000 60000 35000
合计 105000 330000 48000

从上面的表格我们可以看出,移民后在加拿大接受教育比留学生节省6-28万美金的费用,甚至比在中国接收双语教育还要节省数万美金。
而我们还没有考虑在加拿大可以得到的贷款、奖学金等因素。在加拿大,大学生以上的学生可以申请每年数千加元的贷款,和金额不等的奖学金,这些足以保证一个大学生在没有经济来源的情况下完成学业。
我们就按照一个子女从高中还是在加拿大接受教育,可以节省12万美金。如果申请加拿大贷款或者奖学金,则可以节省超过16万美金,这个数字是投资移民所需费用的1.5-2倍。

2.取得移民身份,方能实现选择世界一流大学读书的愿望。

尽管到美国和加拿大读书是众所周知的最佳选择,但低得可怜的学生签证成功率使留学申请成了一场人生前途的赌博,根据加拿大驻北京使馆的统计数字:学生签证平均成功率只有区区62%,通过外语考试(IELTS A类6.5分)后直接申请大学的签证成功率平均85%,而需要通过语言项目的学习才能进入大学的成功率更是低至40%。

对技术移民而言

凡是符合加拿大大学录取条件的申请者,几乎无一例外地都具备了加拿大技术移民的申请条件,申请技术移民成功率可以接近100%,而申请学生签证成功率却只有85%,如果得不到奖学金,这一成功率还要进一步降低,就算你宁愿比移民支付更多的学费,宁愿毕业后不在加拿大、美国发展,宁愿得不到任何奖学金或者贷款支持,宁愿不要加拿大移民身份仍然坚持以留学生的身份去加拿大求学,加拿大政府却不一定发给你签证。这的确是一个可笑的逻辑,但事实正是如此。

对投资移民而言

面对40%的签证成功率,望子成龙的父母空有万贯家产却不能给子女的求学的愿望提供任何帮助,因为即使你愿意比移民家庭多支付数十万、上百万甚至更多倍(如果不止一个子女)的费用,但却有高达60%的机会被拒签,而拒签的理由往往是让人哭笑不得:签证官怀疑你是否有足够的经济力量支付学费。

当我们了解了上面的形势,就会知道为什么澳大利亚成了中国学生家长的第一选择?北美学生签证成功率低是造成这种局面的根本原因,由于得不到签证,学生们只好退而求其次,澳大利亚虽然教育整体水平不如加拿大,但毕竟是英语教育,而且签证要比加拿大容易得多,自然成了出国留学家庭的现实选择。

其实,只要申请移民,上面的问题自然迎刃而解,移民签证的平均成功率远远高于留学签证,费用也要便宜很多,我们要对符合移民条件的家庭说:“你们真的很幸运!”
非常有用,很是庆幸当初的选择正确.:wdb10::wdb10:

朱三雀 : 2008-12-18#352
回复: 偶搜集到的一些关于加拿大高中教育的资料 与大家分享

俺听说有这么个法子, 不是效果如何.
先在好学区租房, 等孩子入了学再在别的地区买房. 迂回.
第2问就不清楚了.
哈哈,好!以后try一try.

加纷纷。

家园移民 : 2008-12-22#353
回复: 子女教育信息素材收集帖

顶顶

海怡宝贝 : 2008-12-23#354
回复: 子女教育信息素材收集帖

同感.也建议置顶.我刚发现此贴,非常有用,摘录了很多有用的资料.谢谢!加分分了.:wdb10:

songqiqd : 2008-12-28#355
回复: 子女教育信息素材收集帖

孩子高三,正在等体检。估计明年高考完才会PL.

大饼 : 2008-12-30#356
回复: 偶搜集到的一些关于加拿大高中教育的资料 与大家分享

正需要,好贴,谢谢

tyhme02 : 2009-01-01#357
回复: 偶搜集到的一些关于加拿大高中教育的资料 与大家分享

请教LZ:如何在A-LEVEL和IB之间抉择?
孩子现在初二, 刚递交申请,我想在他中考后选择IB或A-LEVEL,中学毕业后拿到绿卡直接申请大学,不去加拿大读高中了,可以免的转来转去.目前我所在的城市两个学校都有,该如何选择?
看了你前面对两者的介绍好象更建议A LEVEL,我原打算是IB的.A LEVEL学费比IB略高.
迷茫中...

梁溪香榭 : 2009-01-02#358
回复: 偶搜集到的一些关于加拿大高中教育的资料 与大家分享

请教LZ:如何在A-LEVEL和IB之间抉择?
孩子现在初二, 刚递交申请,我想在他中考后选择IB或A-LEVEL,中学毕业后拿到绿卡直接申请大学,不去加拿大读高中了,可以免的转来转去.目前我所在的城市两个学校都有,该如何选择?
看了你前面对两者的介绍好象更建议A LEVEL,我原打算是IB的.A LEVEL学费比IB略高.
迷茫中...

俺之前也很是关注过IB课程的. 目前比较倾向于A-LEVEL
一是俺所在城市没有好的, 信得过的学校开设IB课程.
二是A-LEVEL课程相对来说更适合中国孩子.

个人认为IB课程其实更为先进. 可以说, IB课程原则上是攻读世界名牌大学最好的课程. 很多西方名牌大学都非常乐意接受IB课程学生.
可惜的是IB课程并不太适合中国学校教学理念和中国孩子.
IB课程的重要理念之一是不仅要学生掌握知识技能, 而且要通过培养和发展对知识的批判性思维来发展学生的独立思考能力和创造性.
IB课程注重[FONT=宋体]创新、实践和社会服务,以及拓展论文等这些独特的部分。这些部分,要求学生有很强的独立思考能力,分析批判能力,和组织活动能力。这些都不是中国学生的强项,甚至是中国学生的死穴。而且,独立思考能力和分析批判能力,都需要通过语言和论文来体现,对语言(英语)运用的要求非常高。[/FONT]
[FONT=宋体]另外IB课程也对学校,教师有更高的要求. 能真正贯彻实施IB精髓实在是不容易. [/FONT]
[FONT=宋体]相比而言, A-LEVEL课程对于中国孩子就比较小菜一碟了. 特别是数学强的孩子. [/FONT]
[FONT=宋体]当然以上考虑是比较急功近利一点. 但我也咨询过一些孩子读IB课程的家长, 感觉其实国内的IB课程没有真正发挥出IB的特点. 也是很应试滴说. 而成绩往往不如A-LEVEL.[/FONT]
[FONT=宋体][/FONT]
[FONT=宋体]一家之言哈.
[/FONT]

pwjnd : 2009-01-02#359
回复: 偶搜集到的一些关于加拿大高中教育的资料 与大家分享

俺之前也很是关注过IB课程的. 目前比较倾向于A-LEVEL
一是俺所在城市没有好的, 信得过的学校开设IB课程.
二是A-LEVEL课程相对来说更适合中国孩子.

个人认为IB课程其实更为先进. 可以说, IB课程原则上是攻读世界名牌大学最好的课程. 很多西方名牌大学都非常乐意接受IB课程学生.
可惜的是IB课程并不太适合中国学校教学理念和中国孩子.
IB课程的重要理念之一是不仅要学生掌握知识技能, 而且要通过培养和发展对知识的批判性思维来发展学生的独立思考能力和创造性.
IB课程注重[FONT=宋体]创新、实践和社会服务,以及拓展论文等这些独特的部分。这些部分,要求学生有很强的独立思考能力,分析批判能力,和组织活动能力。这些都不是中国学生的强项,甚至是中国学生的死穴。而且,独立思考能力和分析批判能力,都需要通过语言和论文来体现,对语言(英语)运用的要求非常高。[/FONT]
[FONT=宋体]另外IB课程也对学校,教师有更高的要求. 能真正贯彻实施IB精髓实在是不容易. [/FONT]
[FONT=宋体]相比而言, A-LEVEL课程对于中国孩子就比较小菜一碟了. 特别是数学强的孩子. [/FONT]
[FONT=宋体]当然以上考虑是比较急功近利一点. 但我也咨询过一些孩子读IB课程的家长, 感觉其实国内的IB课程没有真正发挥出IB的特点. 也是很应试滴说. 而成绩往往不如A-LEVEL.[/FONT]

[FONT=宋体]一家之言哈.
[/FONT]
溪溪,有没有了解到学A-LEVEL[FONT=黑体] [/FONT]的孩子移民加拿大之后如何衔接相关课程?加拿大有没有高中开设A-LEVEL课程?我们家孩子正在学A-L,根据现在的进度可能要高二下或高三上才能拿到签证,不知道怎办。

tyhme02 : 2009-01-02#360
回复: 偶搜集到的一些关于加拿大高中教育的资料 与大家分享

溪溪,有没有了解到学A-LEVEL的孩子移民加拿大之后如何衔接相关课程?加拿大有没有高中开设A-LEVEL课程?我们家孩子正在学A-L,根据现在的进度可能要高二下或高三上才能拿到签证,不知道怎办。

我家孩子拿到签证也跟你差不多时间 握个手先!
另,谢谢楼主的解答!

pwjnd : 2009-01-02#361
回复: 偶搜集到的一些关于加拿大高中教育的资料 与大家分享

我家孩子拿到签证也跟你差不多时间 握个手先!
另,谢谢楼主的解答!
荣幸荣幸!还真头痛孩子上学的事,在国内申请,怕申请不到好学校,听说可能还会被拒,过去了又不知如何衔接。:wdb7::wdb7:

梁溪香榭 : 2009-01-02#362
回复: 偶搜集到的一些关于加拿大高中教育的资料 与大家分享

溪溪,有没有了解到学A-LEVEL的孩子移民加拿大之后如何衔接相关课程?加拿大有没有高中开设A-LEVEL课程?我们家孩子正在学A-L,根据现在的进度可能要高二下或高三上才能拿到签证,不知道怎办。

我没有看到加拿大高中有开设A-LEVEL课程的. 只看到有AP和IB.
估计无缝衔接是没戏的. 但毕竟孩子已经接触了英式高中教育理念, 语言上也会提高得快一些.
其实即使读IB, 表面上看来是全球统一的, 但实际上衔接还是有问题的. 特别是母语和第二语言. 在国内母语一科肯定是语文, 第二语言基本是英语. 到了加拿大母语变成英语, 第二外语...... 怎么衔接?

pwjnd : 2009-01-03#363
回复: 偶搜集到的一些关于加拿大高中教育的资料 与大家分享

我没有看到加拿大高中有开设A-LEVEL课程的. 只看到有AP和IB.
估计无缝衔接是没戏的. 但毕竟孩子已经接触了英式高中教育理念, 语言上也会提高得快一些.
其实即使读IB, 表面上看来是全球统一的, 但实际上衔接还是有问题的. 特别是母语和第二语言. 在国内母语一科肯定是语文, 第二语言基本是英语. 到了加拿大母语变成英语, 第二外语...... 怎么衔接?
语言上我听说第二外语就是汉语,并且有加分。

朱三雀 : 2009-01-03#364
回复: 偶搜集到的一些关于加拿大高中教育的资料 与大家分享

ZT--转帖一个关于中西学生差别的文章

【渥京华人网】新浪博客|中国孩子和美国有什么不同吗?当然有了。
根据笔者在美国生活十余载的生活经历,在我眼中,我能看到的中美孩子有两大不同:中国孩子比美国孩子用功,美国孩子比中国孩子会白活。

毛泽东说过,没有调查研究就没有发言权。就这个问题,我特意走访了几位美国重点学校的中国学生。出乎意料却又在意料之中,孩子们几乎不假思索地回答道:“中国孩子比美国孩子用功。”

的确,在孩子的眼里,中国孩子总是比同龄的美国孩子要忙。一大堆的课外功课和各种各样的文体活动,几乎占满了中国孩子们的课后时间。我女儿六岁时,我们第一次买房,也是第一次和美国人民做邻居。每天孩子放学后,在我们小区,户外是跑跑跳跳的美国同龄儿童,屋内则是学中文练钢琴补算术的女儿。每当美国小朋友上门找我女儿玩儿时,小丫头总会无可奈何地说,对不起,我要写作业,然后我还要练钢琴呢。我站在孩子身边,看到了美国孩子的一脸失望和我家孩子的一肚子不满。说实话,我心里也不是滋味。

也许你会问,美国的中国孩子为什么非要这么用功呢?我本人在美国的三个州生活过,见识过许许多多的华人家庭。我家女儿的无奈,在很多华人家庭里,都可以看到类似的影子。你信不,即使是美国学校里最优秀的中国孩子,回到家里,还是离不开课外功课。也许你又会问了,中国孩子本来就不错了,为什么还要课后给他们加码呢?

我在美国中部城市圣路易斯生活时,结识一位来自广州的博士妈妈,她的育儿观点应该很有代表性。她说,美国小学课外时间这么多,我看到孩子在家里没事晃来晃去的,我心里闹得荒。如果不给他们加些课外作业,我觉得对不起他们啊。

这就是望子成龙的中国妈妈,这就是中国妈妈的育儿观。这也是解释为什么中国孩子要比美国孩子用功的一大原因。

记得多年前我在国内读初中时,背过英文课本里的几句话,good, better, best. Never let it rest. 这就是说,好了还要更好。当年我在国内接受的教育理念,正在不知不觉中影响着我对孩子们今天在美国的教育。我和那位广州妈妈一样,也见不得孩子们在放学后无所事事的样子。孩子们哪,对不住了,不管你在学校怎么样,你还是要继续加油,好了还要好上加好,懂吗?不用功,你能最好吗?

作为三个孩子的母亲,在我美国三个州的十几年移民生活中,我在观察自己和其它华人孩子的成长过程时,我发现,中国孩子和美国孩子还有另外一个不容忽视的区别,那就是公众口头语言表达能力问题。

先以我们成人为例。旅居美国的中国人在谈论他们的美国同事时,时常会这样说,我们单位的那个美国人没什么真本事,就是会白活。靠那张嘴,他把老板哄得团团转。会白活的人吃香,这世界太不公平了。上面这样的牢骚在美国的大学,医院,公司里都可以听得见。这里所谓的白活,就是我想说的口头语言表达能力。

早在我家孩子上幼儿园时,我就看到了美国和中国小朋友在这方面的不同。每天接送孩子时,我总能听到美国孩子和老师在滔滔不绝地讲东讲西。比如,家里的小狗,路上的汽车,公园的湖水,美国孩子总能没事找事地说得条条是道儿。可班上的中国孩子们呢?中国孩子大多是当听众啊,或者是躲在妈妈的身后,安静地四处张望着。是我们中国孩子天生不会表达?肯定不是。比如,林妙可这小姑娘,她不仅会表达,还表达到奥运会上去了。是美国孩子的过份自信压制了中国孩子的表达欲望?这很值得探讨和深思。

幼儿园里的中国孩子,大多来自双职工家庭。在大多数情况下,中国妈妈把孩子往托儿所一扔,马上就会匆匆赶去上班。相比之下,那些和老师特别爱说的美国孩子,除了他们的语言天赋因素,他们的全职妈妈肯定也给了他们更多无形的鼓励。比如,我儿子班上有一个叫托米的男孩。每天,他妈妈总会在学校和老师有意多聊一会儿。日子久了,托米在托儿所变得越来越爱说,最后变得和他妈妈一样地健谈。

众所周知,美国妇女做全职妈妈的比中国妇女要多得多。妈妈全职在家里,直接的受益者就是孩子。很多美国全职妈妈,为了让自己的孩子尽早地接触群体,她们通常会把孩子送到半天制的托儿所里。比起全职工作的中国妈妈来说,这些美国全职妈妈的时间会富裕得多。她们不仅会在接送孩子时,和老师悠哉悠哉地闲扯,还会去托儿所做义工帮忙。按照简单的推理,有妈妈在托儿所里帮忙,孩子的自信心就会大增。有自信心的孩子,肯定就爱说。越说越自信,越自信越爱说。这种正反馈,会让那些美国孩子越来越会白活,就像我上面提到的美国男孩托米一样。

和全职妈妈起类似功效的,还有一个不可忽视的因素。在美国的中小学里,都有由学生家长自愿组建的家长会(PTA)。这些家长会,主要负责学校内部各种活动的协调和管理。笔者做过一个有趣的观察,我发现,活跃在家长会里的成人们,大多是美国人。虽然有少量的中国人介入其中,但中国人所起的作用可谓是冰山一角。可想而知,美国孩子在耳濡目染的过程中,他们不仅学会了他们父母的管理能力,也学会了怎么与人沟通,怎么口头展示自己的本事。

笔者在俄亥俄大学读博士时,也亲身体会到了中国同学的用功和美国同学的会白活。当时,在我们的生化课上,书面考试得A的几乎全是中国学生,而课堂口头报告名列前茅的,几乎都是美国人。这种巨大的反差,其实也就是中国孩子和美国孩子的主要区别。除去第二语言的因素,我们中国学生口头表达能力不如美国同学,和我们所受的教育大有关系。

记得我在国内受中小学教育时,除了啃书本念课文,我几乎没有过课堂表达的其它机会。而在美国小学,从三年级时起,孩子们就有读书报告的机会。可以想象,当孩子怯生生地第一次站在讲台前,害羞是难免的。但这样的语言能力训炼,对孩子未来的成长,则是十分宝贵的。对于中国学生来说,只要用功啃书本,得A的机会就会大增。但那种夸夸其谈的口头演讲并非一日之功,绝对是个临阵磨枪磨不出来的硬功夫。

最近,我在和美国马里兰州唯一的华裔小学校长孙凤仪博士访谈时,这位深谙美国中小学教育的孙校长,也提醒华人孩子一定要在美国学校加强口头语言表达能力的训练。她重点提出,在我们中国文化中,大人们常常会教育孩子要三思而行,不要中断或者打扰别人说话。她说,如果我们让孩子刻守这样的古训,在美国日后的成长中,华人孩子与老师和同学们交流时一定会有挫折感。当然,如何使用技巧,如何让自己融入对方的交谈,则是门很大的学问。

以我家自己的孩子为例,我再来思考一下这个问题。我家的三个孩子,功课都不错,但他们的公众语言表达能力都有待加强。有时想想,公开站在众人面前讲话,即使是我们成人,都会觉得紧张,甚至会害羞,更不用说是孩子了。为了祢补孩子在这方面的不足,对家里的小学生,我会尽量抽出时间去学校做义工。每次我去学校帮忙,我六岁的儿子总会自豪地告诉同学,瞧,那是我妈妈。然后,儿子还会和同学们哇拉哇拉顺便讲讲其它杂事。对我家的中学生,我则鼓励她参加学校的演讲比赛班,有意锻炼孩子在公众面前的表达能力。这种训练班通常每周二活动一次,周末会有一整天的演讲训练。虽然大人和孩子都投入了很多时间和精力,我却觉得很值。理由就像前面提过的孙校长所讲的那样。

美国著名的演讲大师戴尔。卡内基在他的那本风靡全球的“语言的突破”书中前序提到,一个人事业的成功,只有15%取决于他本人的智力和技巧,而另外85%则取决于沟通的能力,讲话的技巧以及说服他人的能力。由此可见,“会白活”对一个人的成长是多么的重要。

假如我们能把中国孩子的用功和美国孩子的“会白活”结合在一起,那培养出来的孩子该是什么样呢?不用说,这样的孩子一定会很棒!乐观地想象一下,左手用功,右手会白活,这样的孩子,只要一出手,说不定就是打破天下无敌手啊。

梁溪香榭 : 2009-01-03#365
回复: 偶搜集到的一些关于加拿大高中教育的资料 与大家分享

语言上我听说第二外语就是汉语,并且有加分。

有加分? 额外的加分? 不会吧.
本来第二外语就有其相应的分数可加的.
但如果母语一科变成英语, 其要求就高了不少的.

其实个人确实认为IB比A-LEVEL课程设置先进得多. 特别是IB核心课程. 关键是国内高中能不能真正发挥出其优越性.

梁溪香榭 : 2009-01-03#366
回复: 偶搜集到的一些关于加拿大高中教育的资料 与大家分享

ZT--转帖一个关于中西学生差别的文章

【渥京华人网】新浪博客|中国孩子和美国有什么不同吗?当然有了。
根据笔者在美国生活十余载的生活经历,在我眼中,我能看到的中美孩子有两大不同:中国孩子比美国孩子用功,美国孩子比中国孩子会白活。

毛泽东说过,没有调查研究就没有发言权。就这个问题,我特意走访了几位美国重点学校的中国学生。出乎意料却又在意料之中,孩子们几乎不假思索地回答道:“中国孩子比美国孩子用功。”

的确,在孩子的眼里,中国孩子总是比同龄的美国孩子要忙。一大堆的课外功课和各种各样的文体活动,几乎占满了中国孩子们的课后时间。我女儿六岁时,我们第一次买房,也是第一次和美国人民做邻居。每天孩子放学后,在我们小区,户外是跑跑跳跳的美国同龄儿童,屋内则是学中文练钢琴补算术的女儿。每当美国小朋友上门找我女儿玩儿时,小丫头总会无可奈何地说,对不起,我要写作业,然后我还要练钢琴呢。我站在孩子身边,看到了美国孩子的一脸失望和我家孩子的一肚子不满。说实话,我心里也不是滋味。

也许你会问,美国的中国孩子为什么非要这么用功呢?我本人在美国的三个州生活过,见识过许许多多的华人家庭。我家女儿的无奈,在很多华人家庭里,都可以看到类似的影子。你信不,即使是美国学校里最优秀的中国孩子,回到家里,还是离不开课外功课。也许你又会问了,中国孩子本来就不错了,为什么还要课后给他们加码呢?

我在美国中部城市圣路易斯生活时,结识一位来自广州的博士妈妈,她的育儿观点应该很有代表性。她说,美国小学课外时间这么多,我看到孩子在家里没事晃来晃去的,我心里闹得荒。如果不给他们加些课外作业,我觉得对不起他们啊。

这就是望子成龙的中国妈妈,这就是中国妈妈的育儿观。这也是解释为什么中国孩子要比美国孩子用功的一大原因。

记得多年前我在国内读初中时,背过英文课本里的几句话,good, better, best. Never let it rest. 这就是说,好了还要更好。当年我在国内接受的教育理念,正在不知不觉中影响着我对孩子们今天在美国的教育。我和那位广州妈妈一样,也见不得孩子们在放学后无所事事的样子。孩子们哪,对不住了,不管你在学校怎么样,你还是要继续加油,好了还要好上加好,懂吗?不用功,你能最好吗?

作为三个孩子的母亲,在我美国三个州的十几年移民生活中,我在观察自己和其它华人孩子的成长过程时,我发现,中国孩子和美国孩子还有另外一个不容忽视的区别,那就是公众口头语言表达能力问题。

先以我们成人为例。旅居美国的中国人在谈论他们的美国同事时,时常会这样说,我们单位的那个美国人没什么真本事,就是会白活。靠那张嘴,他把老板哄得团团转。会白活的人吃香,这世界太不公平了。上面这样的牢骚在美国的大学,医院,公司里都可以听得见。这里所谓的白活,就是我想说的口头语言表达能力。

早在我家孩子上幼儿园时,我就看到了美国和中国小朋友在这方面的不同。每天接送孩子时,我总能听到美国孩子和老师在滔滔不绝地讲东讲西。比如,家里的小狗,路上的汽车,公园的湖水,美国孩子总能没事找事地说得条条是道儿。可班上的中国孩子们呢?中国孩子大多是当听众啊,或者是躲在妈妈的身后,安静地四处张望着。是我们中国孩子天生不会表达?肯定不是。比如,林妙可这小姑娘,她不仅会表达,还表达到奥运会上去了。是美国孩子的过份自信压制了中国孩子的表达欲望?这很值得探讨和深思。

幼儿园里的中国孩子,大多来自双职工家庭。在大多数情况下,中国妈妈把孩子往托儿所一扔,马上就会匆匆赶去上班。相比之下,那些和老师特别爱说的美国孩子,除了他们的语言天赋因素,他们的全职妈妈肯定也给了他们更多无形的鼓励。比如,我儿子班上有一个叫托米的男孩。每天,他妈妈总会在学校和老师有意多聊一会儿。日子久了,托米在托儿所变得越来越爱说,最后变得和他妈妈一样地健谈。

众所周知,美国妇女做全职妈妈的比中国妇女要多得多。妈妈全职在家里,直接的受益者就是孩子。很多美国全职妈妈,为了让自己的孩子尽早地接触群体,她们通常会把孩子送到半天制的托儿所里。比起全职工作的中国妈妈来说,这些美国全职妈妈的时间会富裕得多。她们不仅会在接送孩子时,和老师悠哉悠哉地闲扯,还会去托儿所做义工帮忙。按照简单的推理,有妈妈在托儿所里帮忙,孩子的自信心就会大增。有自信心的孩子,肯定就爱说。越说越自信,越自信越爱说。这种正反馈,会让那些美国孩子越来越会白活,就像我上面提到的美国男孩托米一样。

和全职妈妈起类似功效的,还有一个不可忽视的因素。在美国的中小学里,都有由学生家长自愿组建的家长会(PTA)。这些家长会,主要负责学校内部各种活动的协调和管理。笔者做过一个有趣的观察,我发现,活跃在家长会里的成人们,大多是美国人。虽然有少量的中国人介入其中,但中国人所起的作用可谓是冰山一角。可想而知,美国孩子在耳濡目染的过程中,他们不仅学会了他们父母的管理能力,也学会了怎么与人沟通,怎么口头展示自己的本事。

笔者在俄亥俄大学读博士时,也亲身体会到了中国同学的用功和美国同学的会白活。当时,在我们的生化课上,书面考试得A的几乎全是中国学生,而课堂口头报告名列前茅的,几乎都是美国人。这种巨大的反差,其实也就是中国孩子和美国孩子的主要区别。除去第二语言的因素,我们中国学生口头表达能力不如美国同学,和我们所受的教育大有关系。

记得我在国内受中小学教育时,除了啃书本念课文,我几乎没有过课堂表达的其它机会。而在美国小学,从三年级时起,孩子们就有读书报告的机会。可以想象,当孩子怯生生地第一次站在讲台前,害羞是难免的。但这样的语言能力训炼,对孩子未来的成长,则是十分宝贵的。对于中国学生来说,只要用功啃书本,得A的机会就会大增。但那种夸夸其谈的口头演讲并非一日之功,绝对是个临阵磨枪磨不出来的硬功夫。

最近,我在和美国马里兰州唯一的华裔小学校长孙凤仪博士访谈时,这位深谙美国中小学教育的孙校长,也提醒华人孩子一定要在美国学校加强口头语言表达能力的训练。她重点提出,在我们中国文化中,大人们常常会教育孩子要三思而行,不要中断或者打扰别人说话。她说,如果我们让孩子刻守这样的古训,在美国日后的成长中,华人孩子与老师和同学们交流时一定会有挫折感。当然,如何使用技巧,如何让自己融入对方的交谈,则是门很大的学问。

以我家自己的孩子为例,我再来思考一下这个问题。我家的三个孩子,功课都不错,但他们的公众语言表达能力都有待加强。有时想想,公开站在众人面前讲话,即使是我们成人,都会觉得紧张,甚至会害羞,更不用说是孩子了。为了祢补孩子在这方面的不足,对家里的小学生,我会尽量抽出时间去学校做义工。每次我去学校帮忙,我六岁的儿子总会自豪地告诉同学,瞧,那是我妈妈。然后,儿子还会和同学们哇拉哇拉顺便讲讲其它杂事。对我家的中学生,我则鼓励她参加学校的演讲比赛班,有意锻炼孩子在公众面前的表达能力。这种训练班通常每周二活动一次,周末会有一整天的演讲训练。虽然大人和孩子都投入了很多时间和精力,我却觉得很值。理由就像前面提过的孙校长所讲的那样。

美国著名的演讲大师戴尔。卡内基在他的那本风靡全球的“语言的突破”书中前序提到,一个人事业的成功,只有15%取决于他本人的智力和技巧,而另外85%则取决于沟通的能力,讲话的技巧以及说服他人的能力。由此可见,“会白活”对一个人的成长是多么的重要。

假如我们能把中国孩子的用功和美国孩子的“会白活”结合在一起,那培养出来的孩子该是什么样呢?不用说,这样的孩子一定会很棒!乐观地想象一下,左手用功,右手会白活,这样的孩子,只要一出手,说不定就是打破天下无敌手啊。

:wdb10::wdb10::wdb10:
中国教育太讲究谦逊. 其实在西方这个中国式好品德成了其发展的极大障碍. 俺以前也曾读过一个帖子, 大意是说一个中国人在国外公司受印度同事排挤. 那个印度同事的技术能力比这个中国同胞差很多, 但胜在其嘴皮子厉害. 一开口就如恒河之水滔滔不绝. 虽然满口恒河味英语, 但他们敢说. 中国人就成了没嘴的葫芦了.

pwjnd : 2009-01-04#367
回复: 偶搜集到的一些关于加拿大高中教育的资料 与大家分享

有加分? 额外的加分? 不会吧.
本来第二外语就有其相应的分数可加的.
但如果母语一科变成英语, 其要求就高了不少的.

其实个人确实认为IB比A-LEVEL课程设置先进得多. 特别是IB核心课程. 关键是国内高中能不能真正发挥出其优越性.
我将的是在加报考大学,汉语是第二外语。溪溪,能说说IB比A-L的先进之处吗?

pwjnd : 2009-01-04#368
回复: 偶搜集到的一些关于加拿大高中教育的资料 与大家分享

“假如我们能把中国孩子的用功和美国孩子的“会白活”结合在一起,那培养出来的孩子该是什么样呢?不用说,这样的孩子一定会很棒!乐观地想象一下,左手用功,右手会白活,这样的孩子,只要一出手,说不定就是打破天下无敌手啊。”
很对!中国的教育往往强调含蓄、内敛,其实这个时代还要学会沟通、主动和推销自己。鼓励孩子们多说吧!

梁溪香榭 : 2009-01-04#369
回复: 偶搜集到的一些关于加拿大高中教育的资料 与大家分享

我将的是在加报考大学,汉语是第二外语。溪溪,能说说IB比A-L的先进之处吗?

上面说过了呀

pwjnd : 2009-01-04#370
回复: 偶搜集到的一些关于加拿大高中教育的资料 与大家分享

上面说过了呀
溪溪,抱歉,本人学习不认真吧:wdb8::wdb8:,我又将你原来的帖子看了一遍,受益了。纷纷伺候了。

梁溪香榭 : 2009-01-04#371
回复: 偶搜集到的一些关于加拿大高中教育的资料 与大家分享

溪溪,抱歉,本人学习不认真吧:wdb8::wdb8:,我又将你原来的帖子看了一遍,受益了。纷纷伺候了。

谢谢 等俺解冻了就回礼哈 :wdb6:

梁溪香榭 : 2009-01-04#372
回复: 偶搜集到的一些关于加拿大高中教育的资料 与大家分享

ZT 大多伦多地区IB高中简介

1. 什么是IB学校?
IB即International Baccalaureate,是由一个国际非盈利机构IBO管理的初等教育联盟。该机构成立于1968年,现有126个国家的2261所学校被认可授予IB课程,可分别用英语,法语和西班牙语授课。

IB 的特点, 是采用规范化的课程,以国际化的视野,培养全面发展人才。IB 的规范化体现在各个层面上。申请设立IB课程的学校,要经过IBO对学校的师资,设施,信誉等情况的全面考查后,才能认证批准;课程设置是全球统一规划;老师要经过认证才能任课;课程考题是统一的,学生考卷往往由其它国家的老师来审评。这种国际规范化的程序,保证了IB 的信誉和质量。

IB课程分为三种:IB早期课程,IB中期课程,和IB高中课程。一所IB学校可以开设其中一种或数种课程,但只有完成IB高中课程后才能授予IB文凭。大多数IB学校都是高中,家长学生主要关注的也是高中,故本文集中在IB高中的介绍上。

IB高中文凭得到世界上大多数著名大学的认可。这种认可包括两方面,一是在录取时的考量。大多数西方的大学(如哈佛大学)都把 IB, AP, Abitur 以及 A-levels 等四种考试看作说明学生学术兴趣和能力的附加指标。二是入学后的学分的转化。如果IB 高级课程 (HL) 成绩在6分以上,许多大学(包括耶鲁大学)认可将该课程转化为大学学分,即学生可免修此课。如果免修课程较多,学生进而可能提前毕业。有的大学甚至承认部分IB标准课程(SL)的学分。

由于IB高级课程(HL)与大学课程难度相当,经过IB训练的学生在高中最后一年已经预演了大学的学习,进大学后普遍适应较快。麻省理工学院1999年对该校一年级新生微积分(Calculus II )课程的成绩统计表明,有IB文凭的学生的平均成绩为4.58分,大大超过所有学生的平均成绩4.04分,甚至超过在AP Calculus BC中考过5分的学生的平均成绩 (4.21 分)。

梁溪香榭 : 2009-01-04#373
回复: 偶搜集到的一些关于加拿大高中教育的资料 与大家分享

2. 大多伦多地区的IB高中

现有126个国家拥有IB学校。最多的三个国家是美国(580 所),英国(122所),加拿大(118所)。中国现在也有37所。加拿大的IB学校增长很快。举例说安大略省在1997年只有12所,其中有6所公立学校,但迄今总数已经增加到53所,其中大部分是公立学校。另外还有十几所学校正在申请审批之中。大多伦多地区最早一所IB公立学校是Victoria Park CI(1987)。大多伦多地区其它授予IB学校见下表。 其中用法语教学的4所,其余都是英语学校。公立学校22所,其中属于天主教教育局的有6所。能授予IB高中文凭的共有23所。

公立高中的IB文凭课程只有两年,即11和12年级。但往往从9年级开始上 IB 预备班(Pre-IB ),经过9和10两个年级的强化训练之后,正式进入11年级的IB 文凭课程。IB 预备班一般在8年级时的年底报名,在教育局所辖区内不受家庭地址的限制。有兴趣的学生应主动向自己的老师了解申请程序,寻求老师的支持。申请材料包括填写有关表格,附上最近两年的成绩,老师的推荐信(有的学校不需要信,只要求老师的电话),有的学校还要求学生写一篇对IB Program的认识的essay。如果自己有特殊的成就(如竞赛得奖,音乐体育绘画有特长),也可用附件来说明。由于申请人数往往大大超过实际招生人数,不同的学校采用不同的方式来选拔学生。有的学校采用面试,有的学校采用书面考试,有的学校则考试和面试相结合,也有的学校二者都没有,只看成绩单和推荐信。书面考试的形式因学校而异,例如Bayview SS用自己设计的考题(Bayview Assessment Test),范围涉及英文,法语,数学和科学;而St Roberts High School采用Canada Achievement Tests,内容主要包括英文和数学。如果尚有名额,许多Pre-IB也接受9年级学生的申请。

一般有IB的公立高中同时也有普通班,但IB Program是封闭性的,即IB学生单独分班,绝大多数课程与普通班的学生分开上。IB学生如果在毕业之前想回到普通班,通常要求回到离自己家较近的Home school。 由于采用国际统一课程标准,IB学生转学一般没问题,跨地区甚至跨国转到别的IB学校后,过去的学分都将被承认。IB学生可以在暑假和周末到别的IB学校选修自己学校没有设置的IB课程。另外有的IB学校有校车,有的没有。

公立学校的Pre-IB通常不收费。由于11和12年级正式的IB课程要参加国际统考,试卷要送到国外评分,花费较大,为此每个学生每年要花费1000元左右。有的教育局要学生自己出这笔钱,但许多教育局统一出这笔钱。

IB 学生毕业时将同时获得国际统一的IB文凭和本地区教育局的文凭。IB每门课满分是7分,而加拿大高中一般课程的满分是100分,所以存在分数转化问题。不同的课程的转换因子不一样,分数如何转化各省教育厅有相应的标准。以下是IB分数向百分制转化的大致情况:7 (96分), 6 (90分), 5 (86分), 4 (76分), 3 (70分)。

梁溪香榭 : 2009-01-04#374
回复: 偶搜集到的一些关于加拿大高中教育的资料 与大家分享

以下表列是大多伦多地区现有的IB学校简况(2008年2月)
注: 表中的PYP为IB早期课程,MYP为IB中期课程,Diploma为高中文凭课程。

大多数 IB 学校都有自己的网站,详细介绍该校IB Program的情况。目前信息收集最全面的是Bayview SS的网站(见附录)。虽然IB Program都重视类似Liberal Arts的通才教育,各个学校都有自己的特点和侧重,如有的在科学方面强一些,有的人文方面强一些,有的在Business方面强一些。在一个教育局有多所IB高中的情况下(如多伦多教育局),通常允许学生同时报考几所IB学校,学生也可以同时申请所在区域内的公立教育局和天主教教育局的IB学校,在录取后由学生自己选定想去的学校。

附件


梁溪香榭 : 2009-01-04#375
回复: 偶搜集到的一些关于加拿大高中教育的资料 与大家分享

3. IB program 适合什么样的学生

前面提及,IB是采用类似大学文理学院(Libral Arts)的通才教育理念,用国际统一标准来安排课程和活动。它十二年级的高级课程,与大学一年级课程难度相似,这些特点保证了IB文凭的含金量,得到教育界广泛的认可。几位今年毕业的IB学生对IB的优点的总结如下:
1) 老师通常都很好,学校给IB提供的设施(如实验设施)也很好;
2) 学习风气好。不管情不情愿,大家都在努力学习。这训练了自己良好的学习习惯和合理安排时间的能力;
3) 课程难度接近大学程度,课程进度与考试安排与大学相似,进大学后适应较容易;
4) 最重要的一点是IB班上集中了很多精力充沛,野心勃勃的学生。他们除了学习外,在各项社会活动和课外活动中都想领先。据说York Region的一所IB高中,IB学生只占了1/5,但学校学生会,话剧队的主演,学生报纸,几个最大的与社会活动有关的俱乐部基本上都由IB学生把持。这种同学之间的互相促进,对孩子的一生也许都会有良性的影响。

但缺点也是很明显的。IB Program占用学生时间很多,课程缺少灵活性。从因材施教的原则出发,IB只适合一部分学生。
1) IB适合学习能力强,做事有条理,善于安排时间的学生。IB的课程量大,进度快,学生每天需要投入大量时间才能完成作业和projects。学生再聪明,如果不善于安排时间,会觉得穷于应付。
2) IB适合全面均衡发展的学生。IB program在科学,人文,外语,艺术,社会工作等各方面注重平衡发展,不偏科,不侧重某一方面。如果学生在某学科上有特别的兴趣和天份,进入IB Program会感觉自己的特长没有时间得到充分发挥。举例说一个学生在数学上很有天赋,他会很失望没有充分的时间准备和参加大型数学竞赛。如果一个学生很优秀,但某一方面较弱(如法语),会觉得自己强项不能充分发挥,而弱点被放大,进而失去自信心。又如在艺术体育方面有天赋的学生,会因为这些活动占用很多时间,很难在IB program中取得更好的成绩。这类有特殊天赋的学生,最好留在普通班或课程压力尚合理的gifted program。
3) IB适合将来从事社会性的工作的学生。IB重视培养国际视野和社会责任感。IB文凭所要求的社会服务(CAS),毕业论文,和跨学科综合能力(TOK)都远远超过普通中学文凭的要求。理论上它培养的学生责任感较强,综合分析问题的能力较强,写作和交流能力较强,学生最适合将来从事社会性的工作。如果学生对学术的兴趣很大,将来只想做技术性很强的工作,IB对他们的意义就不大。
4) IB不太适合法语或英文基础差的新移民学生。IB课程写作量很大,外语(这里指法语)要求也较高,新移民学生会感到压力很大。

另外一般学校IB课程没有太多的选修课。如果学生的专业兴趣已经确定,希望在这方面多修些课,在IB Program中很难实现。举例说如果某个学校的普通高中课程中开设有微积分,统计学,高等代数等选修课。这些课程不是IB课程。该校的IB学生即使对数学很感兴趣,因为时间冲突而很难被允许修这些课。

基于上述原因和其它一些原因,很多学生在Pre-IB或IB Program完成之前退回普通班。这包括一些优秀学生和部分学习较吃力的学生。以下是多伦多教育局所属的一所IB高中今年毕业班的一个统计:

Pre-IB grade 9: 103 enrolled (120 students were accepted);
Pre-IB grade 10: 84 enrolled (19 students left Pre-IB after grade 9);
IB grage 11: 58 enrolled (26 students left IB after grade 10);
IB grade 12: 35 enrolled (23 students left IB after grade 11).

York Region一所IB高中的情况与此不太相似。那所高中Pre-IB Grade 9大约有60位学生,Pre-IB Grade 10又补充了30位学生。到Grade 12毕业时尚有80位学生。该校退出IB的学生大多数是在Pre-IB grade 10结束之后。

退出IB Program的原因因人而异,学习跟不上的人只是极少数,退出IB的包括许多优秀学生。据了解在上述两个学校最主要的原因包括:

1) 不愿意上IB的法语课,预计自己在法语普通班会容易些。
2) IB的课程安排不符合他们的兴趣。比如有的学生兴趣在人文方面,而该校的IB要求上很多科学和数学课程。
3) 觉得IB课程占用时间太多,作业量大,尤其是写作量很大,不适合自己特长的发展。
4) 自己不适合IB,但先在Pre-IB读两年,培养了良好的学习习惯后再回普通班的11年级,相当于fast-tracking,以后的学习会轻松些。
5) IB学校离家太远,每天往返占用太多时间。
6) 觉得自己在普通班成绩会更好些。

前面五条理由都有合理成份。我采访了几位IB学生,他们对最后一条有的同意有的不同意。有的学生认为IB Program成绩的含金量高,预期成绩确实会低于普通班。有的学生认为虽然课程难一些,付出的努力多一些,但最终的成绩与普通班相比差别不大,而实际上学到的知识比普通班多,培养出的学习方法让自己受益终生。另外还有一种未经查证的说法,加拿大大学录取时对IB学生有特殊考虑。据York Region一位老师说,他们IB的毕业生100%都能进四年制大学。

还有一些孩子本来适合IB Program,但家长盲目相信Fraser所谓高中排名,认为IB Program所在高中不够好。这里姑且不讨论Fraser的排名有多少参考价值。IB Program基本是封闭式的,IB班和其它普通班不在一起上课,IB Program可以说是“校中之校”,所在高中EQAO考多少与IB Program基本不相干。IB Program的师资和设施都是经过IBO论证,并定期检查的,学生都是经过适当程序挑选的,其教学质量不应当被太多地怀疑。

综上所述,IB有优点也有缺点。学生和家长应该根据自己的情况决定取舍。对不适合IB的学生,不要勉强上。对适合IB的学生,不要畏难。

梁溪香榭 : 2009-01-04#376
回复: 偶搜集到的一些关于加拿大高中教育的资料 与大家分享

4. IB与另外几类特殊教育课程的比较

A) IB和 Gifted program的比较

大多伦多地区各教育局的特殊教育(Special Education)除IB Program外还包括另外几种学术强化课程, 如Gifted program,TOPS program,MACS program等等(详见附录中郁苹的文章)。这些课程的共同点,都是选拔学习上有特点的学生,单独分班,与普通班采用不同的教材和不同的进度。其中Gifted program各教育局都有,较有代表性。Gifted program与IB Program有相似点也有不同点:

1) IB Program按国际IBO组织的标准运行,课程设置,教材深度,考试评分等都有国际标准。Gifted program由各个教育局甚至各个学校按各自的标准自行管理;
2) Gifted program一般在小学三年级选拔学生,Pre-IB一般在八年级选拔学生。经9,10两个年级的Pre-IB后,从11年级才正式开始IB Program;
3) Gifted program的学生除了一些学术课单独分班上课之外,一些课程也与普通班分享; IB Program的班级基本是封闭形的,与普通班很少有共享的课程。
4) IB Program毕业后学生将同时获得IB文凭和安省高中文凭。Gifted学生仅获得安省高中文凭。

但以上这些同异都只是表面形式。从学生和家长的角度看,最重要的差异是课程难度和学习强度。

Gifted program本质上仍然是高中的program. 它的教学范围与普通高中没有差异,仍然依据教育局统一的教学大纲,只是难度会比普通高中班深一些。对于很多课程,普通班只是简单地让你知其然,而Gifted试图让你进一步知其所以然。IB是一种高中到大学过渡的Program。它最后一年的高级课程(HL)难度与深度与大学一年级课程相近,这正是许多大学承认IB学分,认可用成绩良好以上的IB高级课程代替大学课程的依据。另外IB特有的TOK训练,以及4000字以上的毕业论文都是普通高中,甚至大学低年级都不会要求的。从这个意义讲,IB program在学术上的要求比Gifted program要高,训练更严格,学生进入大学后适应更快。

但是这种强化学习的代价,是学生的的课程负担很重,减少了学习的灵活性,也没有多少时间体验丰富多彩的高中生活,很难挤时间发展自己的业余爱好。IB学生的压力来自两方面,除了IB program本身的要求外,还有学校校方的期望。由于IB是国际标准,学生的试卷都由外校甚至外国的老师来评判,各个IB学校之间的教学效果在同行中是透明的。主办IB的学校都期望自己不落人后,他们对学生的期望自然转变为学生的学习压力。

与此相对照,Gifted program教学大纲与普通班相似,课程由地方教育局和学校自行制定,不存在学校间的比较。一般来说Gifted学生的学习负担相对于IB学生要轻很多,学生有更多时间做自己想做的事情,包括参与社会活动,发展课外兴趣,也包括自学AP或准备SAT,甚至也包括享受生活。

综上所述,IB program和Gifted program之间不存在谁好谁不好,但存在对谁合适不合适。

梁溪香榭 : 2009-01-04#377
回复: 偶搜集到的一些关于加拿大高中教育的资料 与大家分享

B) IB和 AP的比较

AP (Advanced Placement)是一类高中生可以参加的大学程度的考试(详情见附录的参考文献)。如果考试成绩好,进大学后可以免修相关课程。在这点上AP与IB Program中的高级课程(HL)相似。由于AP和IB的课程深度都涉及到大学程度,两种在北美都很流行。美国一些教育评估机构甚至用学生通过AP和IB的人数,作为高中质量的排名标准。关于AP和IB优劣的争论,一直是美国许多论坛的热点。(见附录华盛顿时报的文章。)以下简单对比分析两者的差别:

1) 本质上AP和IB是不同的概念。 AP是一门一门单独的考试,而IB是一个强化高中文凭的两年的完整学习计划。通常IB课程比复习相应的AP要花更多的时间。美国和加拿大各个大学对AP和IB一视同仁,但一般认为IB难度大一些,含金量高一些。
2) AP可以通过学校学习,on-line学习或自学,进而参加这类考试。IB只能通过学校学习。学生选择考什么AP科目可以根据自己的特长和学校条件自行决定。IB学生课程的选择余地相对很小。
3) IB课程成绩经转换为百分制后列入安省高中的成绩单。在安省AP一般不列入高中成绩单,也不计入高中平均成绩(GPA)的计算,这点与美国高中不一样。
4) IB学生最多只能选四门高级课程(HL),实际上大多数学校只允许三门。所以大学所承认的学分的IB课程最多为四门。AP没有这种限制。近年由于名牌大学入学竞争很激烈,许多学生都用多修AP来证明自己的学习能力。一位学生通过十门以上AP的情况很多见。由于IB Program这种限制,美国许多IB学生不得不自学AP以求参加更多的考试。但加拿大大多数公立高中开设的AP课程不多,即使开设一般只有很少几门。如果学生自学能力不强,选择IB学校至少能有三门大学认可的课程。

惴惴不安 : 2009-01-04#378
回复: 偶搜集到的一些关于加拿大高中教育的资料 与大家分享

溪溪早,嫩太敬业了。又给大家带来好信息。

梁溪香榭 : 2009-01-04#379
回复: 偶搜集到的一些关于加拿大高中教育的资料 与大家分享

惴惴从山上回来啦.
有木有片片啊?

梁溪香榭 : 2009-01-04#380
回复: 偶搜集到的一些关于加拿大高中教育的资料 与大家分享

"中国第一天才少年"出家为僧

宁铂曾是全国瞩目的“神童”,他,曾是全国的风云人物。1977年,他成为中科大少年班破格录取的第一人;他,曾让全国不少父母为“制造”神童着迷,成为无数孩子的榜样,“制造神童”成为一种风气。时至今日,在人们谈到神童教育都离不开他的名字――宁铂。

19岁,宁铂成为全国最年轻的讲师,而他醉心佛学,两度出家,再次引来关注的目光。这个从赣州市走出的天才,成为一个充满争议的天才人物。

  记者巧遇宁铂听其讲课

去年秋天,记者到离南昌不远的某寺采访时无意中得知,素有“中国第一天才少年”之称的宁铂就在该寺院出家,担任该寺佛教学院的讲师。这令记者大感意外。毕竟,此时全国有许多记者都在找寻宁铂,但无人清楚其下落。

经与该寺院的住持沟通,记者被允许到佛教学院听宁铂讲课。

在住持的带领下,记者进入课堂坐在最后一排。几分钟后,一个身高不到一米七,圆脸、微胖的僧人捧着一大摞经书走进教室,放在讲台上。他就是宁铂,赣州市人。

他讲课的语速很快,有着惊人的记忆力,从不翻教材,却能说出哪个内容在教材的哪一章哪一页。昔日神童,今朝高僧,宁铂需要的是一份宁静的生活。

三十年前的“中国第一神童”

1977年9月,教育部召开全国高等学校招生工作会议,决定恢复已经停止了10年的全国高等院校招生考试。此时,一封信引起了高层的注意。这封信是原江西冶金学院教师倪霖写给当时的国务院副总理方毅的推荐信。信中举荐的正是宁铂。

1964年,宁铂出生在赣州市一个普通的知识分子家庭,两岁半便能背30多首毛泽东诗词,3岁时就能从1数到 100,4岁时认得400多个汉字。宁铂看什么会什么,6岁的他看完《中医学概论》就能替人开药方,看完围棋书后就可以与高手对弈,看完唐诗宋词,便能吟诗作对,宁铂的一系列行为让父亲宁恩意识到,这个孩子不寻常。

倪霖是宁铂父亲的朋友。11月3日,方毅将倪霖的举荐信转给中国科技大学,且批示,如属实,应破格收入大学学习。经过考察,宁铂成为中科大少年班第一人。

此后,全国的报纸、杂志、电视都在报道这位少年天才,极大地激励了各个家庭对孩子的教育,这在当时被称为“宁铂现象”。在中科大之后,众多高校的少年班风起云涌,最后的成功者屈指可数,中科大是少数的成功者之一。

宁铂电视上炮轰“神童教育”?

2002年,宁铂离开中科大前往五台山出家,被校方找回;2003年,宁铂再次出家,这次他成功了。

据媒体报道:宁铂入校一年后告诉班主任汪惠迪:“科大的系没有我喜欢的。”汪惠迪打了一份报告给学校,希望学校根据宁铂的兴趣转至南京大学天文系。学校的答复是,既来之,则安之。宁铂被安排选择了自己在赣州八中时就不喜欢的理论物理专业。1982年,宁铂本科毕业留校,19岁的他成为中国最年轻的讲师。在中国科大任教期间,宁铂并没有将时间花在物理研究上,而是开始了对宗教、气功的研究。1993年,他开始学习佛教。

1998年,宁铂参加中央电视台《实话实说》栏目一期有关神童教育的讨论,针对神童教育,他表达出不予肯定的态度。这期节目,被传为是宁铂向神童教育开炮。

“宁铂现象”及“神童教育”留下的争议

宁铂及属于他的时代已经过去,而宁铂及“制造神童”,留给人们的思考很多。

省社会学学会会长王明美谈到,“神童教育”在本质上是违反教育规律和人的智力发展规律的。一般情况下,人的受教育过程都要经历幼儿园、小学、中学到大学的过程,这是正常的一般的教育规律,而“神童教育”却贸然提前了学习时间,导致违背教育规律。人在违背规律的情况下,受到来自各方的压力,在沉重的压力和过高的外界期望之下是难以取得较好的学习效果的。

但也有不同观点者。江西师大的李教授说,著名画家李叔同是在出家后,方才找到艺术创造的崭新天地的。宁铂何以不能成为第二位李叔同?或者成为一名高僧呢?

关于宁铂及神童教育的争论并不会很快结束,或许永远没有结束。不管如何,宁铂及神童教育,均在社会留下了一道深刻印记。

dianalee : 2009-01-06#381
回复: 偶搜集到的一些关于加拿大高中教育的资料 与大家分享

谢谢楼主。太有用了。有空还要再细读...

惴惴不安 : 2009-01-06#382
回复: 偶搜集到的一些关于加拿大高中教育的资料 与大家分享

惴惴从山上回来啦.
有木有片片啊?
拍了很多片子就是传不上来,我把头像换成长白山天池了,这就是这次拍的。

天池就像一个脸盆,那天天气好,都看到底了,比我想像的小多了。

长白山的风景不错,踏雪很好玩。

梁溪香榭 : 2009-01-06#383
回复: 偶搜集到的一些关于加拿大高中教育的资料 与大家分享

拍了很多片子就是传不上来,我把头像换成长白山天池了,这就是这次拍的。

天池就像一个脸盆,那天天气好,都看到底了,比我想像的小多了。

长白山的风景不错,踏雪很好玩。

最近的确是不大容易上传照片. 网速好慢.
不过就嫩这一张小小滴头像已经很美啦.

朱三雀 : 2009-01-06#384
回复: 偶搜集到的一些关于加拿大高中教育的资料 与大家分享

以下表列是大多伦多地区现有的IB学校简况(2008年2月)
注: 表中的PYP为IB早期课程,MYP为IB中期课程,Diploma为高中文凭课程。

大多数 IB 学校都有自己的网站,详细介绍该校IB Program的情况。目前信息收集最全面的是Bayview SS的网站(见附录)。虽然IB Program都重视类似Liberal Arts的通才教育,各个学校都有自己的特点和侧重,如有的在科学方面强一些,有的人文方面强一些,有的在Business方面强一些。在一个教育局有多所IB高中的情况下(如多伦多教育局),通常允许学生同时报考几所IB学校,学生也可以同时申请所在区域内的公立教育局和天主教教育局的IB学校,在录取后由学生自己选定想去的学校。
太好了,灰常有用啊!灰常感谢!
补充一下下, 这个网站可以查查所有的IB学校。
http://www.ibo.org/country/CA/index.cfm

梁溪香榭 : 2009-01-06#385
回复: 偶搜集到的一些关于加拿大高中教育的资料 与大家分享

太好了,灰常有用啊!灰常感谢!
补充一下下, 这个网站可以查查所有的IB学校。
http://www.ibo.org/country/CA/index.cfm

是的. 这里可以查全世界IB学校.
http://www.ibo.org/school/search/index.cfm?programmes=&country=CN&region=&find_schools=Find

朱三雀 : 2009-01-06#386
回复: 偶搜集到的一些关于加拿大高中教育的资料 与大家分享

B) IB和 AP的比较

AP (Advanced Placement)是一类高中生可以参加的大学程度的考试(详情见附录的参考文献)。如果考试成绩好,进大学后可以免修相关课程。在这点上AP与IB Program中的高级课程(HL)相似。由于AP和IB的课程深度都涉及到大学程度,两种在北美都很流行。美国一些教育评估机构甚至用学生通过AP和IB的人数,作为高中质量的排名标准。关于AP和IB优劣的争论,一直是美国许多论坛的热点。(见附录华盛顿时报的文章。)以下简单对比分析两者的差别:

1) 本质上AP和IB是不同的概念。 AP是一门一门单独的考试,而IB是一个强化高中文凭的两年的完整学习计划。通常IB课程比复习相应的AP要花更多的时间。美国和加拿大各个大学对AP和IB一视同仁,但一般认为IB难度大一些,含金量高一些。
2) AP可以通过学校学习,on-line学习或自学,进而参加这类考试。IB只能通过学校学习。学生选择考什么AP科目可以根据自己的特长和学校条件自行决定。IB学生课程的选择余地相对很小。
3) IB课程成绩经转换为百分制后列入安省高中的成绩单。在安省AP一般不列入高中成绩单,也不计入高中平均成绩(GPA)的计算,这点与美国高中不一样。
4) IB学生最多只能选四门高级课程(HL),实际上大多数学校只允许三门。所以大学所承认的学分的IB课程最多为四门。AP没有这种限制。近年由于名牌大学入学竞争很激烈,许多学生都用多修AP来证明自己的学习能力。一位学生通过十门以上AP的情况很多见。由于IB Program这种限制,美国许多IB学生不得不自学AP以求参加更多的考试。但加拿大大多数公立高中开设的AP课程不多,即使开设一般只有很少几门。如果学生自学能力不强,选择IB学校至少能有三门大学认可的课程。
还是AP灵活一些。
找到了这个AP学校目录
http://www.ap.ca/pages/03_AP_schools.htm

梁溪香榭 : 2009-01-06#387
回复: 偶搜集到的一些关于加拿大高中教育的资料 与大家分享

In AP-vs.-IB Debate, A Win for the Students

By Jay Mathews
Washington Post Staff Writer
Thursday, December 9, 2004; Page VA20


National surveys show that U.S. high schoolers on average spend no more than an hour a day on homework but three hours watching television or playing with computers. At Woodson and George Mason, most students smirk at the idea of such a light school load.

According to The Washington Post's annual Challenge Index ratings, George Mason is No. 4 and Woodson No. 6 in the region as measured by their students' participation in college-level courses and tests. Nationally, they rank among the top two dozen public schools. At George Mason and Woodson, about 70 percent of students take at least one college-level course, which is more than twice the national average.

What is most interesting about the two schools to many education specialists is that they have achieved the same impressive results with two different programs , Advanced Placement (AP) at Woodson and International Baccalaureate (IB) at George Mason. At times in Fairfax County and Falls Church, parents and students have argued bitterly over whether AP or IB would be better for their schools. In 1999, for instance, a committee of teachers, parents and students at Woodson voted 15 to 10 to reject a planned IB program because it meant losing AP.

梁溪香榭 : 2009-01-06#388
回复: 偶搜集到的一些关于加拿大高中教育的资料 与大家分享

But George Mason's and Woodson's results seem to suggest that either program, if done right, is fine.

Students in AP "bring their critical thinking skills and disciplined approach to study in all their classes and raise the discussion and often general performance levels," said Susan H. Shue, who teaches AP government and politics to seniors at Woodson.

Richard Peloquin, who teaches IB 20th-century history at George Mason, said, "To engage in higher-level thinking with my students on a daily level is very stimulating and rewarding."

About 14,000 American schools have AP and about 450 have IB, much larger numbers than specialists expected when the programs began. AP was initiated by the College Board in 1956 as a program for a few elite public and private high schools, at which seniors and juniors were given college credit for some high-level courses so they would not be bored by having to cover the same material in college. IB, the brainchild of teachers at the International School of Geneva, started in 1968 as a high-level standard curriculum for schools that catered to the children of diplomats and international business executives.

梁溪香榭 : 2009-01-06#389
回复: 偶搜集到的一些关于加拿大高中教育的资料 与大家分享

But in the 1980s and 1990s, many average and below-average schools in the United States -- such as Garfield High School in East Los Angeles, portrayed in the movie "Stand and Deliver" -- found that the programs helped their students, too.

New research by the National Center for Educational Accountability shows that even students who fail AP examinations in high school are twice as likely to graduate from college in five years as students who never try AP. Hispanic and African American students are three times as likely to graduate from college in five years if they try AP. Other research shows similar results for students who took IB courses.

No school districts responded to the need for college-level courses more enthusiastically than Falls Church and Fairfax County. Of the county's 24 high schools, seven use mostly IB courses; 16, including Woodson, use AP; and one, the 2,842-student Robinson Secondary School, offers large doses of both. The Falls Church district has just one high school, George Mason, which is mostly an IB school. All 25 schools in Fairfax and Falls Church, according to The Post's Challenge Index, have ratings of at least 1.000, meaning they give as many college-level tests as they have graduating seniors. Only 5 percent of schools nationwide achieve this level of participation.

梁溪香榭 : 2009-01-06#390
回复: 偶搜集到的一些关于加拿大高中教育的资料 与大家分享

Robert Elliott, principal of Woodson, said these achievements are the result of hard work. "Students taking their first AP course sometimes experience the need to adjust to the pace," he said. Shue has created a summer program to prepare students for the challenge, he said, "but even with this, our teachers and counselors work with students the first few weeks to get them over the hump."

In most high schools, AP courses are reserved for 11th- and 12th-graders, but at Woodson, "almost half of our tenth-graders take an AP course and . . . do very well on the AP examination," Elliott said.

Robert W. Snee, who was an IB teacher for 11 years before taking over as George Mason's principal in 1992, said the program is a "rising tide lifting all boats." His school has 40 IB courses -- "in every department where an IB curriculum can exist," he said. "The experience of teaching an IB class affects the way a teacher approaches his or her other classes, and the tendency is to add more rigor to those classes and to apply some of the same types of assessments. Those are changes for the better."

Hannah McBride, a senior at Woodson, said getting college credit in high school "enables me to skip some basic, beginner college courses" when she arrives at the university of her choice. "AP classes have saved me time and money," she said.

Andrew Roller, a senior at George Mason, said "the IB program has taught me how to manage my work better. I have learned to prioritize work and have developed healthy work schedules so that I prevent myself from procrastinating and burning out."

Both programs have their downsides, students said. Sean Douglass, a Woodson senior, said "despite these fantastic teachers' dedication to the program, they lose some of their natural enthusiasm and ability to think outside the box because they feel as if they are forced to 'teach to the test.'

"Additionally, some of the teachers I've spoken with dislike the strictness of the AP curriculum because it inhibits creativity and does not allow them to have fun with the class."

Students and parents at George Mason complained that it is harder to get college credit for IB courses than for AP courses because the international program is smaller and less familiar to colleges.

梁溪香榭 : 2009-01-06#391
回复: 偶搜集到的一些关于加拿大高中教育的资料 与大家分享

Usually students can get credits if they earn the full IB diploma, which includes six college-level exams plus a 4,000-word paper. But for individual courses, colleges often will give credit for the AP version but not for the very similar IB version, with no other reason than that is their policy.

"It is really sad to note the discrimination IB students are faced with against AP students," Krish and Indi Namboodiri , parents of recent George Mason graduate Arya Namboodiri, said in an e-mail. "All American universities still consider IB students as 'foreign' and hence not equal."

But guidance counselors at both schools said IB and AP students share a great advantage when seeking admission to selective colleges, who want students that have taken the most demanding courses.

And in the course of pursuing those college goals, both high schools become better places to learn and better places to augment skills as a teacher, said students and teachers.

"Teaching the IB courses has strengthened my knowledge of the subject matter," said Mary McDowell, Science Department chairwoman and biology teacher at George Mason. "I don't think I always realized before how important a certain concept might be in a more advanced course of study."

Paula Spencer, an AP American history teacher at Woodson, said that through the AP network, "we exchange books, readings, teaching techniques and ideas, which have helped me enrich the lessons in my classroom."

tyhme02 : 2009-01-07#392
回复: 偶搜集到的一些关于加拿大高中教育的资料 与大家分享

感谢LZ!

liuliling : 2009-01-08#393
回复: 偶搜集到的一些关于加拿大高中教育的资料 与大家分享

感处很深。。。
THANK YOU

stephencar : 2009-01-09#394
回复: 偶搜集到的一些关于加拿大高中教育的资料 与大家分享

这份资料太好了。谢谢LZ!!!

风华绝代 : 2009-01-10#395
回复: 偶搜集到的一些关于加拿大高中教育的资料 与大家分享

快登陆了,信息好有用!多谢分享!

wh7010 : 2009-01-12#396
回复: 偶搜集到的一些关于加拿大高中教育的资料 与大家分享

才看到这么好的帖子,溪溪辛苦了。

梁溪香榭 : 2009-01-12#397
回复: 偶搜集到的一些关于加拿大高中教育的资料 与大家分享

嘿嘿 谢谢表扬
大家喜欢俺就 :wdb6:

都市骑兵 : 2009-01-13#398
回复: 偶搜集到的一些关于加拿大高中教育的资料 与大家分享

太感谢了,正好我儿子能用上

梁溪香榭 : 2009-01-16#399
回复: 偶搜集到的一些关于加拿大高中教育的资料 与大家分享

推荐一个学习英语的网站.
www.usalearns.org
很不错哦.

FredHe : 2009-01-16#400
回复: 偶搜集到的一些关于加拿大高中教育的资料 与大家分享

多谢分享!

theresawu521 : 2009-01-17#401
回复: 偶搜集到的一些关于加拿大高中教育的资料 与大家分享

好贴!

梁溪香榭 : 2009-01-18#402
回复: 偶搜集到的一些关于加拿大高中教育的资料 与大家分享

今天听了首好听的歌, 记录一下.

I'm dreaming of a white Christmas,
just like the ones I used to know.

Where the treetops glisten,
and children listen to hear sleigh bells in the snow.

I'm dreaming of a white Christmas,
with every Christmas card I write.

May your days be merry and bright,
and may all your Christmases be white.

梁溪香榭 : 2009-01-19#403
回复: 偶搜集到的一些关于加拿大高中教育的资料 与大家分享

ZT: 中国孩子到美国读书很轻松吗?

无论是文学作品,还是新闻报道,多年来,美国确实给无数望子成龙望女成凤的中国父母一个画饼充饥的机会。不知有多少人坚信,中国孩子到美国读书很轻松。在这样的意识支配下,于是就有了下面这样的说法:如果在中国玩儿不转啦,孩子们,咱就投奔美国去!如今的各类媒体把这种说法演绎得几乎全民皆知。



中国孩子到美国读书很轻松吗?不知为什么,当我要细细思考这个问题时,我想起了80后小作家郁秀笔下的那对孪生兄妹董海和董丁。他们在15岁时随父母移民美国,他们就像是鱼,越过太平洋之后,”鱼本会游泳,只是河水换成了海水,鱼失去了水性,不会游泳了“。当这对兄妹在苦苦的挣扎中终于融入所谓的美国主流社会之时,他们一家为此付出了沉重的代价。在小作家的眼里,美国学校颓废而富有活力。这部小说的名字叫“不会游泳的鱼”,感兴趣者不妨一阅。

无庸讳言,中国孩子扎实的数学功底举世皆知,就连钢琴才子郎朗来美国读高中,他的数学成绩都可以在班级名列前茅。由此可见,初来美国的中国孩子,确实可以用骄人的数学做为引人注目的第一张牌。可是在美国学校,只靠数学一张牌是远远不够的。而其它的那些牌,既是美国人注重的强项,又是中国孩子相对的弱势。要想拥有学校所有或大部分的王牌,移民来美国的中国孩子一定要花大气力不可。所以,我要说,中国孩子到美国读书并不轻松!

梁溪香榭 : 2009-01-19#404
回复: 偶搜集到的一些关于加拿大高中教育的资料 与大家分享

在拙文“中国孩子和美国孩子的最大区别是什么”中我提到,美国孩子比中国孩子会白活。这种白活的本事,在美国生活中至关重要。白活的能力,除了和孩子们的性格有关,它与中国孩子的英文表达能力密切相连。不管中国孩子在国内的英文水平如何,到了美国,中国课本里那些古板的语法和非美式的口语表达方式,会让孩子们无所适从。就连我们成人留学生,即使能以高分通过托福考试,初来乍到时和美国人交流,还是经常会找不到北。笔者认为,为了尽快和美国孩子们打成一片,中国孩子一定要先从抓英文做起,而不是用自己的数学成绩沾沾自喜而裹足不前。



美国前劳工部长赵晓兰八岁从台湾来美国时,英文水平烂得一塌糊涂。她补习英文的方式是,不管听懂听不懂,把老师写在黑版上的东西,一字不落地抄下来,回家继续消化。在爸爸每晚耐心的辅导下,一年后的赵晓兰居然可以参加班上的演讲比赛。在这个故事中,我们可以看到父母的细心辅导教育对孩子的进步是多么的关键。确实,当不识水性的孩子在异国的游泳池里乱扑腾时,父母应该就是托起孩子的救生圈啊。非常遗憾的是,很多新移民家庭,由于父母要在忙忙碌碌的日子中谋生,常常会忽视对孩子必要的培养。当孩子长大成人一无所获时,大人悔之不及。

梁溪香榭 : 2009-01-19#405
回复: 偶搜集到的一些关于加拿大高中教育的资料 与大家分享

我来简单说两个例子吧。

八岁来美国的比尔,现在被美国某大学勒令退学。来美国前,比尔在国内某重点小学就读。同时,教英文的姥姥还天天给比尔吃小灶。按理说,八岁来美国的中国孩子,又有国内的英文基础,即使上不了哈佛,怎么也能到美国重点大学走一遭吧。可实际呢,高中毕业那年,比尔勉强上了最一般的大学。在大学校园不到两年,他因多门功课考试不及格被学校清理出门。孩子出了这么大的问题,到底怪谁呢?比尔初来时,他的父母因为工作太忙,又采用了某些美国人对孩子的大撒把政策,不仅比尔的英文水平进步不大,就连孩子的交友和课外活动都大打折扣。这些,就是我前面所言的除了数学之外的其它“牌”。当美国孩子把花花绿绿的几张牌握在手中游刃有余时,中国的新移民孩子显得是那样的孤单无助。


十岁的汉森来美国时,他的父母正在美国大学做最廉价的工作。为了节省开资,他们住进了租金便宜的公寓。在汉森就读的学校,有一个专门欺负外国孩子的小团伙。下课后,这帮恶作剧的美国小男孩一拥而上,把汉森拉到厕所里,强行把汉森的头硬往便器里推压。你想想,十岁的孩子怎么能受得了这样的欺辱?儿时在小学的痛苦经历,给汉森的心里留下了很深的阴影。上中学后,他变得时而暴怒时而沉默寡言,功课更是一泻千里。高中毕业时,他连大学都没上,最后离家出走。你看,汉森的经历和小作家笔下的董海,到底谁悲惨?

梁溪香榭 : 2009-01-19#406
回复: 偶搜集到的一些关于加拿大高中教育的资料 与大家分享

当我们讨论中国孩子来美国读书是否轻松时,别忘了,读书并不是中国孩子在美国学校唯一的内容。当华人家长瞪大眼睛关注孩子的功课时,别忘了美国学校也有欺负人现象,更有无法言喻的潜规则。说的好听一点儿叫同学间的压力“peer pressure”, 。更严重的那些事件,如果我们打开天窗说亮话,这种学生中的恶霸现象叫bully。中国孩子来美国读书,不仅仅要应付功课,更要和周围的环境打成一片。这些硬件建设,远比功课要难得多!

笔者一家去年经历了一次跨州搬家。搬家最受影响的不仅仅是工作升迁的成人,也会影响面临转学的孩子们。刚来新城市不久的一天,在清理孩子们的房间时,我在地上捡到一张纸团。我打开一看,纸上写的几个字让我顿时泪流满面。一向嘻嘻哈哈的已经上初中的女儿在纸上写道,”我来这里都三天了,一个朋友都没有。就连中国孩子都不理我。我好孤独!!!“。孩子的几个惊叹号让我吃惊不已。在我们原来城市的学校,女儿可谓是人见人爱的小领袖。不管是美国孩子还是中国孩子,女儿都可以毫不费力地把她们指使的团团转。没想到,横跨两州搬家,孩子就有了如此大的震撼和不解。这让我这个成人都觉得有些不可思议。女儿在美国出生长大,换个学校她都感觉不适应。以此推理,飘洋过海从中国来美国读书的孩子们是多么的不容易!他们确实像不识水性的孩子,被父母直接推倒游泳池里乱扑腾。为人父母者,应该充分理解孩子们的不易,而不是盲目的乐观。

梁溪香榭 : 2009-01-19#407
回复: 偶搜集到的一些关于加拿大高中教育的资料 与大家分享

记得多年前,我在国内学过一段伟人的语录说,外国语是人生斗争的武器。现在,我很想把这句话送给有志来美国读书的中国孩子们。为了在他乡异国好好地生存,孩子们,你们一定要把英文先学得滚瓜烂熟。只有这样,当外国孩子欺负你的时候,你才可以尽情地述说你的愤怒。当别人讥笑你的肤色时,你才可以用流利的英文回应他们的目光短浅。

学好英文,并不像愚公移山月中折桂那么难。前面提到的赵晓兰英文学习法很值得借鉴。勤能补拙,终会让梦想开花结果。另外,几乎在美国的所有城市,都有一个很有名的叫KUMON的英文补习班。这个学习班既可以为美国孩子拔高儿,也可以为初来乍到的中国孩子们做英文充电练习。笔者在美国土生土长的孩子,现在也在这个英文班里学习拼写和阅读。短短的几个月下来,我六岁孩子的阅读和拼写已经有了长足的进步。高中生来美国读书,不妨借鉴一下钢琴才子郎朗补习英文的经验。在他的自传”千里之行“里,郎朗提到,他来美国读高中时,专门辅导他的英文教授建议他从苦读莎士比亚剧本下手。一个以弹钢琴为生的孩子,要啃下莎士比亚,这得多大的耐力啊。郎郎能做到,别的孩子能做到吗?对于美国高年级的学生来说,提高英文水平不仅仅是为了听说,而是为了应付大大小小的作文。就像郎朗提到的那样,”英文课对我很难。当我读到其它同学的作文时,我们的想象力让我很惊讶“。美国孩子的想象力,就来自他们多年阅读的积累。

对任何事情,我们都应该全面的一分为二来对待。归根结蒂,中国孩子来美国读书轻松吗?笔者的拙见是,如果中国孩子想在美国全面开花力争上游,来美国读书并不轻松。如果只是来应付学校的功课,中国孩子来美国可能会觉得比国内轻松。

在美国,除了父母,美国老师肯定不会像中国老师那么苦口婆心地管孩子,更没有像家访什么的这种事。养不教,父之过。如果移民家长对孩子大撒把,你的孩子多年以后就有可能成为我文中的比尔和汉森。我说这些并不是危言耸听,类似的例子还有很多,在这里暂不列举。此外,即使是在美国土生土长的中国孩子,他们的教育也是华人父母除了工作之外的头等大事。

可怜天下父母心。中国的孩子们,不管在哪里,谁都不容易啊!

maggie6688 : 2009-01-20#408
回复: 偶搜集到的一些关于加拿大高中教育的资料 与大家分享

孩子也不容易,即使到了国外也是一样的。关键是中国人了理念不同于西方人。

梁溪香榭 : 2009-02-14#409
回复: 偶搜集到的一些关于加拿大高中教育的资料 与大家分享

群星灿烂不如一轮皓月当空

美国名牌大学的入学竞争很激励。学生有好的高中成绩和SAT考试成绩只是必要条件, 但还不是录取的充分条件。大学招生时还要考虑学业之外的其它因素。许多家长和学生也明白这一点,但不知从何处着手。有的学生参加了学校的多种兴趣俱乐部,但没有重点。有的学生参加社区服务,但只是为了完成教育局规定的40小时的义工任务。有的聪明的学生学习很轻松,但不知怎么利用好空余的时间。有的家长催着孩子同时学习多种乐器,却没有明确的目的。这些现象的主要问题,是忽略了孩子在全面发展的基础上,必须有突出的亮点。对申请美国大学而言,群星灿烂不如一轮皓月当空。名牌大学录取学生不拘一格。一个学生在任何一方面有抢眼的特点往往会成录取时的幸运者。

一个高中生的亮点可以体现在学业,社会活动,社区服务,体育或艺术表现等许多方面。尺有所短,寸有所长,应避免面面俱到的一般性的参予。 每个人首先要发现自己的特点和兴趣,扬长避短,把精力集中在容易出众的一个或几个方面。使自己脱颖而出的机会永远是有的, 但往往需要家长和学生主动去找机会,去创造机会。下面以一些例子来说明这一点。

梁溪香榭 : 2009-02-14#410
回复: 偶搜集到的一些关于加拿大高中教育的资料 与大家分享

性格开朗,社会活动能力强的同学,应多参与学校的社团活动。但不应满足于学校这个小圈子,应尽量在社会这个大舞台上表现自己的领导才能和组织能力。我听说的一位学生,暑假中自己联系在安省的自由党竞选活动中做义工。积极参与了与竞选中有关的各种活动,表现出众。后来照片被登上了一个杂志的封面。试想如果这个学生的大学申请材料中附了这份杂志,并且一位当选议员为其写了一份赞赏有加的推荐信,对于大学的招生人员而言, 是什么样的份量。事实上类似这样的机会很多。 加拿大是一个鼓励公众参与的民主国家。 各种政治团体,慈善机构, 民间组织需要大量的义务人员。中学生只要直接与有关人员联系,通常都会被欢迎参加。只要学生认真参与,一定能从中得到锻炼,并且有所收获。不要斤斤计较于这些活动算多少个小时的义工。教育局要求学生做义工,不是为了增加学生的负担,而是鼓励学生寻找机会,培养公民意识和社会责任感,在贡献社会中提高自己。

梁溪香榭 : 2009-02-14#411
回复: 偶搜集到的一些关于加拿大高中教育的资料 与大家分享

在学业上最容易使自己出众的不是去争门门功课都是高分,而是在自己感兴趣的科目上更深入地学习,通过参加竞赛来提高自己的层次。加拿大为高中生举办的各种学科的竞赛很多,有地区性的,也有全国性的;有文学艺术体育语言等方面的,也有数理化生物电脑领域的。根据自己的特长选择参加一些竞赛,不但能激发自己的学习兴趣,帮助定位自己将来可能从事的专业,还能在申请名牌大学时增加自己的份量。一位学生门门功课都是全班第一固然不错,但某门学科拿过全省的第一名会给人以更深的印象。大学招生人员面对的是来自不同高中甚至不同国家的申请者,对他们而言,跨地区的竞赛成绩往往更具有可比性。

各种竞赛的信息通常可以在高中的科任老师处得到,也可以在网上查到。许多高中并不重视这些竞赛,有些老师也不太鼓励学生参与,大多数学生即使参与也并不刻意去复习准备。故许多竞赛的竞争其实不如想象的那样激烈。 你只要确有兴趣,认真准备,通过课外阅读或培训班,通常都会有不错的成绩。

梁溪香榭 : 2009-02-14#412
回复: 偶搜集到的一些关于加拿大高中教育的资料 与大家分享

产生亮点的方式没有固定的模式,许多机会也许就在你身边,但需要家长和学生有心才能抓住。从平时朋友间的闲谈中, 从报纸电视上, 从网络中,应随时留心各种线索, 并勇于去尝试。比如North York Mirror曾在2002年登广告招聘一位中学生做该报纸的专栏作家。我儿子去应聘后在该报发表过18篇文章。这成了他申请大学时的亮点之一。我相信当时很多家长和学生都读到过该报的招聘启事。许多人或者读后忽略了,或者没敢去尝试。

大陆到加拿大的很多是技术移民。许多人都有同学或朋友在大学的实验室工作。一些实验室在暑假中允许高中生参与做义工。不管在试验室参与做实验还是仅仅帮助清洗器皿,对学生而言都是一个很好的学习机会。因为有机会了解真实的研究环境,旁听各种学术讲座,接触最前沿的学科信息。如果没有在实验室工作的朋友,也可尝试直接用e-mail给教授写信要求做义工的机会。e-mail地址可以到该大学有关系别的网页上找。许多教授也许不回信,但有的教授希望鼓励中学生对该教授的研究领域发生兴趣,也许会提供这种机会。我儿子在高中期间分别在多伦多大学的发育生物学实验室和美国密歇根的Wayne State University医学院做过两个暑假的实验。这对他确定专业兴趣至关重要,对他大学的录取也许也起了重要作用。

梁溪香榭 : 2009-02-14#413
回复: 偶搜集到的一些关于加拿大高中教育的资料 与大家分享

条条道路通罗马。提升高中生的录取身价有成百上千种方式。以上只是用几个实例来说明其重要性和可操作性。有兴趣的学生应尽早认清自己的特长,寻找并创造机会,让自己脱颖而出。

slhd007 : 2009-02-14#414
回复: 偶搜集到的一些关于加拿大高中教育的资料 与大家分享

真是超级好贴子,让准备出去的家长孩子们有所准备,楼主功德无量啊!!!
请教一下温哥华的博域中学和皇家基督学院这两个学校的教学质量能够排名到什么位置?目前中介给的就这2个私立学校,希望大家帮忙把把关,在此先谢了。

惴惴不安 : 2009-02-14#415
回复: 偶搜集到的一些关于加拿大高中教育的资料 与大家分享

加分:wdb10::wdb10:

惴惴不安 : 2009-02-15#416
回复: 偶搜集到的一些关于加拿大高中教育的资料 与大家分享

:wdb17::wdb19:又从头到尾通读一遍,受益。太好了。

再加分感谢!

海怡宝贝 : 2009-02-15#417
回复: 偶搜集到的一些关于加拿大高中教育的资料 与大家分享

分分送上!

小象宝宝 : 2009-02-15#418
回复: 偶搜集到的一些关于加拿大高中教育的资料 与大家分享

真是用心的,领教了,虽然儿子离读高中还差很远,虽然移民还没办下来,但对于我来说都是非常有用的资料,谢谢!

梁溪香榭 : 2009-02-16#419
回复: 偶搜集到的一些关于加拿大高中教育的资料 与大家分享

真是超级好贴子,让准备出去的家长孩子们有所准备,楼主功德无量啊!!!
请教一下温哥华的博域中学和皇家基督学院这两个学校的教学质量能够排名到什么位置?目前中介给的就这2个私立学校,希望大家帮忙把把关,在此先谢了。

SORRY, 不了解呢.
嫩为何要选择私校呢?
据俺朋友说, 好的私校要排队要考试, 容易进的私校好不到哪里去.
这只是俺朋友的意见, 供您参考吧.

梁溪香榭 : 2009-02-16#420
回复: 偶搜集到的一些关于加拿大高中教育的资料 与大家分享


:wdb17::wdb19:又从头到尾通读一遍,受益。太好了。

再加分感谢!

分分送上!

谢谢 :wdb6::wdb19:
明天回礼

盈婆婆 : 2009-02-16#421
回复: 偶搜集到的一些关于加拿大高中教育的资料 与大家分享

说得真好。再来看看。

梁溪香榭 : 2009-02-16#422
回复: 偶搜集到的一些关于加拿大高中教育的资料 与大家分享

在教育子女的过程中教育自己

现在中国国内, 孩子的学习是许多家庭的生活重心, 父母介入了孩子学习的所有环节。但到了北美后, 新移民家长普遍感到对孩子的教育力不从心。一方面是工作和生存压 力大,作父母的自顾不暇。另一方面对北美教育体系和文化背景不熟悉,想辅导也无从下 手。再加上孩子的英语水平很快超过了自己,在很多学科上孩子知道得比自己还多,在孩子面前有自惭形秽的感觉。移民后环境的改变使得中国国内父母们习惯的居高临下的教育模式不再有效。在这种新的环境下,父母在孩子教育中角色依然重要,但父母的角色应该同时是孩子的老师,同学和学生。

父母作用首先是身教, 其次才是言传。身教包括事业, 家庭生活和社会活动。移民对孩子和父母都是一种挑战。移民后成年人都有一个或长或短的调整适应过程,这个过程中各种失望和挫败是不可避免的。在这个过程中父母要牢记, 自己的孩子也同样正在经历对全新环境的痛苦的调整适应。父母是孩子最直接的老师,父母的情绪和人生态度在这时候对孩子有很大影响。父母如果坚韧不拔,积极向上, 孩子也会受到正面的影响。如果父母怨天尤人, 孩子对新环境就会有畏惧感,就会适应得慢。父母打工的感受, 在孩子面前应多报喜,少报忧。有了进展不妨更强调些,让孩子为父母而自豪, 让孩子觉得北美充满机会, 只要努力就有好的前途。在新移民家庭常常能听到父母对孩子说”我们都是为了你才移民北美的. 我们这一辈子算完了,只有靠你了。” 这种讲法的负面作用, 一是让孩子看不起父母, 进一步对父母的教诲置若罔闻,并找到了自己消极的榜样。父母不管是做技术工作还是蓝领工作, 一定要在孩子面前表现自己积极向上的努力。

梁溪香榭 : 2009-02-16#423
回复: 偶搜集到的一些关于加拿大高中教育的资料 与大家分享

移民时间长一些,有稳定工作和收入的父母, 最易懒惰下来。这本来是人之常情,但还是应记住, 自己的行为时时刻刻在影响着孩子。你如果下班回来从不读书, 整晚守着电视, 却训斥孩子不好学习;或者自己在工作上不思上进, 沾沾自喜于既得利益, 却埋怨孩子在班上甘居中游。这些做法都很难让子女真正信服。 还有一些受过很好教育的父母, 自己在事业上安于现状, 把全部心血都花在子女身上, 一心望子成龙。这种不明智的做法浪费了自己的才华。即使孩子将来成功了, 对父母也只有感激, 没有敬佩。每一代人都应首先为自己活着。自己在工作上的努力在无形中会为孩子树立了榜样。要让孩子以父母为荣。

身教还包括社会责任感的培养。 中国在长期的专制统治下,形成了老百姓明哲保身,不关心公众事务的传统。这种意识如果传染给下一代,则我们的下一代将不能真正成为这个国家的负责的公民,不能产生能代表自己族群利益的政治家, 在民主社会中永远作为”沉默的多数“而存在。第一代新移民由于语言,文化背景等隔阂,很难在社会事务中脱颖而出。但我们在鼓励子女参与社会活动的同时,自己也应该以身作则,力所能及地参与各种社区活动,比如在孩子所在学校做义工,并且在各级选举中积极参与投票。我相信许多在孩子的学校做过义工的家长都会有亲身体会,自己所做的事情给孩子带来很大的自豪感。在参加这些社会活动中虚心向孩子求教,能得到孩子更多的敬重。

梁溪香榭 : 2009-02-16#424
回复: 偶搜集到的一些关于加拿大高中教育的资料 与大家分享

身教还应该包括做家务。在一个平等和谐的家庭里,每个人都有责任分担部分家务。父母对待家务的态度,会直接影响子女的生活习惯,甚至与孩子将来的家庭幸不幸福可能有关。

强调身教并不是说父母不能在”言传“上对孩子的学业有所帮助。我们虽然英语不如孩子,对北美文化背景的了解远逊于孩子,但我们所受的教育和丰富的人生阅历对孩子也是一种可资利用的财富。我们也许在具体的课程学习中帮不了孩子,但在把握学习的大方向上,我们仍然可以成为孩子的良师益友。我们的跨国背景,使我们的眼界往往比许多加拿大本地长大的人开阔。下面以帮助孩子选择和申请美国大学为例来说明这点。

孩子将来选择什么样的大学,家长的意见可能比老师的意见更有指导作用。虽然世界最顶尖的大学在美国,但没有父母的引导和帮助,加拿大的孩子很难有申请美国大学的意愿。加拿大大学整体水平高,学费便宜,高中生申请加拿大的大学似乎是天经地义。除了少数私立学校有学生申请美国大学的传统外,大部分公立学校的老师不会主动鼓励学生申请。我相信绝大部分加拿大的优秀学生从来没有去美国读本科的念头。据多伦多市长Miller回忆,当时他申请美国大学让学校所有同学都感到奇怪,因为除了他之外没有那位校友这样做。但Miller做了,去了哈佛。我猜测当时他能这样做是因为他是刚从英国到加拿大没几年的新移民, 视野比当地长大的同学开阔。我问过几个今年从加拿大去美国读书的华人学生,大多数人都承认是父母首先有这种想法,自己才意识到的。

其次是帮助孩子制定申请过程的时间表。 在美国的高中阶段, 大家都在准备考SAT, 选修AP课程.老师和学生每天热心谈论的话题就是如何申请大学。资讯满天飞,各种培训班和家教到处都是。但加拿大高中没有这种环境气氛,老师和学生顾问都没有帮助学生申请美国大学的经验。高中生很忙,家长有责任先行一步,尽量早一些全面了解申请的过程,帮助孩子制定准备工作的计划和申请时间表。对于很多新移民家长,研究大学申请过程也是一个很有价值的学习过程,从中可以提高英文水平,更多地了解北美的文化背景。不要认为自己的英文水平不行就把全过程推给孩子。 但是有一点必须注意, 北美大学注重孩子独立思考和处理问题的能力,不应在申请材料中透露家长参与的痕迹。

新移民父母在子女的教育中是大有可为的。 教育子女的过程同时也是父母自己的学习过程。 通过积极对子女的言传身教,我们自身也会在适应北美社会的过程中提高了一个档次。

slhd007 : 2009-02-16#425
回复: 偶搜集到的一些关于加拿大高中教育的资料 与大家分享

SORRY, 不了解呢.
嫩为何要选择私校呢?
据俺朋友说, 好的私校要排队要考试, 容易进的私校好不到哪里去.
这只是俺朋友的意见, 供您参考吧.

:wdb11:因为已经高二了,过去上一年就考大学,公立学校考英语估计难因此考虑上私校,也是不得已而为之。头痛啊~~~~~有没有什么好的上学方案呢。:wdb14:

梁溪香榭 : 2009-02-16#426
回复: 偶搜集到的一些关于加拿大高中教育的资料 与大家分享

:wdb11:因为已经高二了,过去上一年就考大学,公立学校考英语估计难因此考虑上私校,也是不得已而为之。头痛啊~~~~~有没有什么好的上学方案呢。:wdb14:

这种情况的确头痛.
个人以为先拿到高中毕业文凭, 再加上托福或雅思成绩申请大学.如果孩子成绩好就干脆参加高考, 凭高考成绩加上托福或雅思成绩更好.

海怡宝贝 : 2009-02-16#427
回复: 偶搜集到的一些关于加拿大高中教育的资料 与大家分享

溪溪,你对新移民子女选择在加读大学还是去美读书怎么看?关注!

梁溪香榭 : 2009-02-16#428
回复: 偶搜集到的一些关于加拿大高中教育的资料 与大家分享

溪溪,你对新移民子女选择在加读大学还是去美读书怎么看?关注!

是这样的.
俺儿子年龄大了. 等过去时读高中了.
考虑到移民监的问题, 我是想让他在加国读本科的.
考虑去美国读研. 不过到时候要由儿子作主了.

个人以为, 如果孩子年龄小, 能爬藤就去爬藤.

嫩以为呢? 很想听听嫩滴意见.

梁溪香榭 : 2009-02-16#429
回复: 偶搜集到的一些关于加拿大高中教育的资料 与大家分享

加拿大学生应申请什么样的美国大学?

美国有三千多所大学,其中良莠不齐。加拿大的大学不多,但水平都不错,学费也较低。对于加拿大的学生而言,去美国一定要去最好的学校。比如许多加拿大的学生在Montana State University (MSU)读书。按理说加拿大的大学大部分都比MSU好, 学费也便宜。花大钱去美国读一般的学校,真是冤大头。

美国著名大学有有私立和公立之分。公立名校如UC伯克利,密西根大学等也是世界一流。 但加拿大学生去美国读本科的首选应该是美国的私立名校,尤其是美国的常青藤联盟大学或者一些著名的文理学院。

第一是学业上的理由。公立名校主要由州政府出钱,资源有限,学生一般较多。 著名教授一般只给研究生上课。本科生大班上课较常见, 许多专业课程一个老师教上百个学生。有的学校招生过多,一些课程只能由研究生来教, 教学质量很难保证。而著名私校特别注重教学的信誉, 再著名的教授都得上本科生的课。小班上课较多。比如去年哈佛大学70%的课程每个班的学生都在20个以下。小班上课的好处是教授与学生交流很直接, 学生能很深入地与教授讨论问题。所以加州伯克利虽然各学科的研究均在世界上名列前茅,但在注重教学的美国的大学本科的排名中只能徘徊在第二十名左右。

第二是建立同学关系网的理由。 私立名校的校友对母校的归属感一般都较强。对校友的提携是很多校友会的宗旨。很多学校的校友会甚至具体到帮助在校的学生找暑期的工作。私立名校历史久远,其过去的校友在美国(甚至加拿大)各界中已经占据要津。美国最好的学生通常也愿意去常青藤联盟大学。你今天的同桌也许就是你明天事业上的合伙人;你今天的室友说不定是明天那个州的州长, 对你将来的事业会有帮助。

第三最现实,是家庭经济上的理由。美国的公立学校用的是本州纳税人的钱,花钱受本州居民监督,很难给外州人助学金,外国人就更难了。外州学生得付更多的学费,外国人更是冤大头。而著名私校财源丰富(如哈佛大学2008年的捐赠基金是340亿美元,真可谓富可敌国), 这些钱与纳税人无关。他们只想收到最好的学生,期望这些学生将来挣大钱后回报母校。所以越是名校,助学金越容易拿。

梁溪香榭 : 2009-02-16#430
回复: 偶搜集到的一些关于加拿大高中教育的资料 与大家分享

在私立名校中常青藤联盟大学应是首选。美国的常青藤大学联盟是东海岸八所著名私立大学的总称,包括哈佛,耶鲁,普林斯顿,哥伦比亚,宾州大学,康乃尔大学,布朗大学, Dartmouth。这些学校在美国US News and Word Report的大学历年的排名中,都稳在前二十名甚至更靠前。再加上这些学校历史久远,其校友在各界中占据要津,常青藤大学联盟很自然被人们看成美国名校的象征,也自然成了高中优秀学生们申请学校的首选。除了常青藤大学外,Duke University, Stanford University, MIT, CalTech都是顶尖的私立大学。

喜欢小一点学校的学生,也可考虑一些著名的文理学院(Liberal Arts Colleges),如Williams College,Amherst College。这些学院的贵族味道重一些,在美国社会上有很高的信誉。 比如很多人都听说过的Wellesley College,宋美龄三姐妹和克林顿夫人都毕业于此。但这些学院偏重教学,研究相对要弱一些。将来想从事研究的学生, 还是到著名教授多一些的研究型大学为好。

美国各著名大学各有特点。所以除了大学的总体名气之外,还应注意自己最喜欢的学科的排名。如有的大学总体排名不是一流,但其某些学科却是世界一流。另外大学四年出了学习之外,满足个人兴趣也很重要。有的象牙塔型的大学(如CalTech)虽然学术上非常顶尖, 但学生人数很少(每年新生才二百多人),社团活动和体育比赛都不活跃。 兴趣广泛的学生也许会觉得枯燥乏味。还有大学离家的距离也应是考虑的因素之一。

梁溪香榭 : 2009-02-16#431
回复: 偶搜集到的一些关于加拿大高中教育的资料 与大家分享

美国著名大学有招收加拿大学生的传统, 每个学校都有专人负责加拿大的招生。私立大学招生办公室的人每年九,十月间都会到加拿大各地巡回作招生宣传。有兴趣的朋友应注意各大学网站上的日程表。他们也会定期访问加拿大一些著名高中。 这些招生办公室的人对加拿大教育体系很了解,有助于正确评价加拿大的学生。比如加拿大公立高中普遍没有AP (Advanced Placement)课程, 而加拿大公立高中AP开设不多。对于不了解加拿大教育体系的美国大学,加拿大考生将处于不利的地位。但常青藤大学一般都了解加拿大的具体情况,不会把这当成一个不利因素。 比如2003年哈佛大学在加拿大考生中的录取率是9%,与在美国的10%的平均录取率很接近。

上面讲的是美国好的私立大学的一般情况。但具体申请什么大学还得根据个人情况而定,如个人的特长和弱点,家庭经济情况,对学校的地理位置的偏爱等等。美国的”US News and World Report”杂志每年都有美国大学的排名。这个排名着重于大学的本科生教育,对于选择大学很有参考价值。除了根据学校质量的排名外,它还对各大学的经济资助、外国学生比例等进行排名。有兴趣的朋友可在图书馆找到这本杂志, 也可在网上看到该排名的大致情况(见下面的“小资料”)。

美国各大学独立招生。提前申请(early application)只能选择一所最喜欢的大学。但正常申请(regular application)时每个学生申请几个大学没有限制。最好多申请一些。除最喜欢的大学外,也申请两三所虽然不是特别喜欢,但尚能接受的大学作为保险。如同时被几所同等级的大学录取,在经济资助上更有与学校讨价还价的余地。

梁溪香榭 : 2009-02-16#432
回复: 偶搜集到的一些关于加拿大高中教育的资料 与大家分享

小资料: 申请美国大学几个常用网址

1). http://www.collegeboard.com
College Board Association是由美国3,500所大学和学院参与的非赢利组织。 该网站全面收集了与申请大学有关的资料, 包括各大学的网址, SAT注册考试信息等等。

2). http://www.usnews.com/usnews/edu/college/rankings/rankindex_brief.php
在各种美国大学排名中,US News and World Report杂志的排名最有名。它的分类排名包括本科教育,研究生教育,经济资助,外国学生比例等等。从上面列的大学网址的招生网页上能查到该校的要求,往年招生情况的统计,并能下载申请表。 网站上的内容不够详细,有兴趣的读者可在这网站上购买它出的书。

3). http://www.fastweb.com
这个网站收集了几千个美国的奖学金和助学金的申请程序。读者注册后把自己的特长和背景情况填入,它会定期通知你适合你的情况的奖学金信息。

4). http://www.campusdirt.com
从这个网站可比较各个大学的情况。

slhd007 : 2009-02-16#433
回复: 偶搜集到的一些关于加拿大高中教育的资料 与大家分享

这种情况的确头痛.
个人以为先拿到高中毕业文凭, 再加上托福或雅思成绩申请大学.如果孩子成绩好就干脆参加高考, 凭高考成绩加上托福或雅思成绩更好.

:wdb11::wdb6:梁溪香榭谢谢方案是不错的。但是他(我堂弟)高中成绩也就一般般,考托福或雅思挺困难。不知道上私校后考大学有什么限制没有?可以的话请介绍几个温哥华地区比较不错的私校。
另外现在留学是因为家里刚开始办移民考虑到时间比较长,年龄也已经18了,因此让他先出去上学。

海怡宝贝 : 2009-02-17#434
回复: 偶搜集到的一些关于加拿大高中教育的资料 与大家分享

是这样的.
俺儿子年龄大了. 等过去时读高中了.
考虑到移民监的问题, 我是想让他在加国读本科的.
考虑去美国读研. 不过到时候要由儿子作主了.

个人以为, 如果孩子年龄小, 能爬藤就去爬藤.

嫩以为呢? 很想听听嫩滴意见.
非常赞同你的意见,我也是这么想的。按我们新移民的情况,至少在加国要待个四五年时间,然后才有可能考虑去美国或留着不动,一切都要看小孩本人今后的发展而定。但我还是有想法从美和加两国比较:诸如气候、人口、学习、就业、父母的经商机会等等来说还是美国略胜一筹。

梁溪香榭 : 2009-02-17#435
回复: 偶搜集到的一些关于加拿大高中教育的资料 与大家分享

非常赞同你的意见,我也是这么想的。按我们新移民的情况,至少在加国要待个四五年时间,然后才有可能考虑去美国或留着不动,一切都要看小孩本人今后的发展而定。但我还是有想法从美和加两国比较:诸如气候、人口、学习、就业、父母的经商机会等等来说还是美国略胜一筹

是. 加国比较适合养老滴说.

梁溪香榭 : 2009-02-17#436
回复: 偶搜集到的一些关于加拿大高中教育的资料 与大家分享

:wdb11::wdb6:梁溪香榭谢谢方案是不错的。但是他(我堂弟)高中成绩也就一般般,考托福或雅思挺困难。不知道上私校后考大学有什么限制没有?可以的话请介绍几个温哥华地区比较不错的私校。
另外现在留学是因为家里刚开始办移民考虑到时间比较长,年龄也已经18了,因此让他先出去上学。

SORRY啊
这种情况我没留意过, 不了解不敢瞎说.

jux : 2009-02-17#437
回复: 偶搜集到的一些关于加拿大高中教育的资料 与大家分享

<P>
据我所知是这样的. 凭高考成绩和托福或雅思成绩可以直接申请加国高校. 具体嫩可以去看各个学校的网站. 每个学校都会对招生有明文规定, 很详细, 包括不同专业对托福及雅思的分数要求都有明文分列. 应该谈不上哪个更好. 二者取其一即可. 具体哪个更好考俺就不知道了.
</P>
<P>&nbsp;</P>

jux : 2009-02-17#438
回复: 偶搜集到的一些关于加拿大高中教育的资料 与大家分享

香榭:太感谢你了。我女儿已经18岁了,想在下半年去读高二。年龄可能偏大一些,在哪个省可以读公立高中?先谢谢你了。

jux : 2009-02-17#439
回复: 偶搜集到的一些关于加拿大高中教育的资料 与大家分享

我女儿也在这个学校读书,这个学校的管理很差,如果学生自我管理能力不够,要想学好英语也不容易。

jux : 2009-02-17#440
回复: 偶搜集到的一些关于加拿大高中教育的资料 与大家分享

谢谢 等俺解冻了就回礼哈 :wdb6:
:wdb17:

梁溪香榭 : 2009-02-17#441
回复: 偶搜集到的一些关于加拿大高中教育的资料 与大家分享

香榭:太感谢你了。我女儿已经18岁了,想在下半年去读高二。年龄可能偏大一些,在哪个省可以读公立高中?先谢谢你了。

不客气哈. 俺们是在探讨. 希望大家多多发言呢.

好象是18岁都可以进公立高中的.19岁在BC就不行了.
如果是我, 我会考虑安省, 可以读到22岁呢.

梁溪香榭 : 2009-02-17#442
回复: 偶搜集到的一些关于加拿大高中教育的资料 与大家分享

我女儿也在这个学校读书,这个学校的管理很差,如果学生自我管理能力不够,要想学好英语也不容易。

哪个学校? 说出来给大家提个醒儿吧.

海怡宝贝 : 2009-02-17#443
回复: 偶搜集到的一些关于加拿大高中教育的资料 与大家分享

是. 加国比较适合养老滴说.
好象我们选择养老还太早了些:wdb7:但是,无奈!为了多给孩子一些机会(包括我们)只能选择一个目前还不是挺适合我们的地方~~.还好,我们都作好了充分的思想准备.

梁溪香榭 : 2009-02-17#444
回复: 偶搜集到的一些关于加拿大高中教育的资料 与大家分享

好象我们选择养老还太早了些:wdb7:但是,无奈!为了多给孩子一些机会(包括我们)只能选择一个目前还不是挺适合我们的地方~~.还好,我们都作好了充分的思想准备.

是的. :wdb10:

梁溪香榭 : 2009-02-17#445
回复: 偶搜集到的一些关于加拿大高中教育的资料 与大家分享

多伦多高中阶段的特殊教育程序

在加拿大,高中教育不仅仅是为了上大学,更重要的是为将来的发展奠基。除了学生居住区内的常规高中程序以外,多伦多教育局还开设了一些特殊课程供学生选择。

GIFTED PROGRAM 小多伦多地区有多所高中提供gifted program,它们是: Martingrove CI , WL Mackenzie CI, York Memorial CI, Don Mills CI, Leaside CI, Danforth C& TI, Northern SS, Western Tech &CS, Woburn CI。中国人称gifted program为天才班。该班开始于小学四年级。三年级时由老师推荐参加IPRC(Identification, Placement and Review Committee) 的测试。该测试不是传统的数学及语言测试,主要测试学生的逻辑思维,反应速度以及知识面。四年级以后也有少数学生陆续插入班级中。TDSB对进入天才班的学生每年有一次评估,但该评估不会导致孩子离开天才班,除非父母主动提出。所以进入天才班的学生可以一直读至高中毕业,基本上是终生制了。天才班的课程进度快于常规班级。

INTERNATIONAL BACCALAUREATE仅有三所学校提供IB 课程,它们是:Vanghan Road Accademy, Victoria CI, Weston CI。IB 课程教学及考试全部采用国际标准。学生毕业时同时获得IB毕业证书及OSSDC(Ontario Secondary School Diploma)毕业证书。IB证书为全世界的大学认可。IB课程不但在学习上要求比常规学校严格得多,还要求学生的社区服务时间除了OSSDC所要求的40小时外至少达到150小时。IB学生录取程序是:个人提出申请, 需提供老师或校长的推荐信,申请人平均分数必须在80分以上,写一篇散文,通过interview。IB通常没有人学考试。IB适合发展全面, 学业优秀, 对自己要求严格的学生。也特别适合那些想去加拿大以外去读大学的学生。

SPECIALITY MATH AND SCIENCE提供该课程的学校有:Marc Garneau’s TOPS program, Northview Heights, WL Mackenzie’s MACS program, Runnymede CI。每个学校课程名称不同,但内容差不多。高中期间数理化的学习进度及深度都高于常规课程。想进入该程序学习的学生要特别提出申请, 申请人八年级平均分要求达80分以上,另设有严格的入学考试。考试内容有:数学,科学及写作。通常录取人数有限。如TOPS近年来每年都有近五百人参加考试,只择优录取约60名。MACS约90名。通常九年级后不再接受新的申请。该课程适合那些擅长并爱好数学及科学的学生。

ARTS PROGRAM 提供该课程的学校有:Cedarbrae CI, CW Jeffreys CI, Downsview SS, Earl Haig SS, Etobicoke School of the Arts, Rosedale Hts, Wexford CI。以华人就读较多的Earl Haig为例, Earl Haig艺术课程的学生直接来自于Claude Watson Schoold for the Arts. 通常学生在小学三年级时申请进入Claude Watson,经考核后录取,从四年级进入该校就读直至八年级。该课程的学生要有绘画,音乐,舞蹈及表演的特长。

梁溪香榭 : 2009-02-17#446
回复: 偶搜集到的一些关于加拿大高中教育的资料 与大家分享

CYBER ARTS 提供该课程的学校有Central Commerce, Don Mills CI, Emery CI, George Harvey CI, Northview Heights。Cyber Arts 与上面说的Arts Program的区别在于,Arts Program是传统艺术,而Cyber Arts 不仅局限于传统艺术,还包括电脑及多媒体艺术。申请人要提交作品并经面试后录取。

EXCEPTIONAL ATHLETS 有四所学校提供该课程,分别是:Birchmount Park CI, Northview Heights, Silverthorn CI, Vanghan Road Accademy。如果您的孩子达到省级,国家级运动员标准,这个课程是最合适不过的了。孩子既能坚持训练,参加各种竞赛又不耽误接受高中教育。

EXTENDED FRENCH & FRENCH IMMERSION 总共有25所学校,限于篇幅这里就不逐一列出。多伦多教育局的网址上有很详细的信息。

ADVANCED PLACEMENT 简称为AP。AP课程的学分为大学承认, 提供AP课程的学校有: Lawrence Pak CI, Martingrove CI, North Albion CI, WL Mackenzie CI, York Mills CI, Georges Vanier SS, Leaside CI, Marc Garneau CI.

ONTARIO YOUTH APPRENTICESHIP PROGRAM 该课程的目的是培养熟练技工,适合高中毕业后想直接就业的学生。大多数学校都有这种应用教育程序。选改课程的学生在高中期间就开始学习实用技术课程并算入学分,如:Electricity, Construction Technology, Communications Technology, Plumbing, Auto Body, Fashion, CSCO Networking Program等等。TDSB每年都有相当一部分人选择高中毕业后直接工作。

在学生八年级第一学期结束前, 每一个学校都会向八年级的学生介绍高中的教育情况并提供相关的资料。各高中在每年的十一月至一月都会举行晚会向父母及学生介绍学校情况及课程。如果孩子有特殊的兴趣及特长,家长应利用这段时间参加这些信息介绍会。每个学生可以选两个常规高中及两个特殊高中程序。高中的申请及录取时间在每年的一月至二月。三月一日前学生一定要选定学校并填好九年级的课程选择表。

这里介绍的仅限于小多伦多地区,不包括天主教教育局,也不包括大多伦多的其他教育局,如York区及Peel区。介绍的内容不保证精确, 仅供参考。详细情况可参考多伦多教育局的网址 www.tdsb.on.ca 或向具体学校咨询。

梁溪香榭 : 2009-02-17#447
回复: 偶搜集到的一些关于加拿大高中教育的资料 与大家分享

加拿大高中生学科竞赛(科学类)介绍

加拿大许多不同的大学和行业协会设有各种各样的学科竞赛。 这些竞赛的目的是为了培养高中学生学习科学的兴趣, 帮助社会和大学甄别特殊人才.

这些竞赛有地区性的,也有全国性的和国际性的。全国性的竞赛往往分级进行。 即学生必须首先参加地方的选拔赛, 其优胜者才能参加更高一级的比赛。

学科竞赛分为两大类。 一类以项目研究为主,另一类以考试为主。 第一类竞赛要求学生个人或研究小组独立提出有创新意义的研究方向, 从事必要的文献研究和初步的实验研究。最后用专家集体评议的方法,选出优胜者。这类竞赛中最著名的是Intel International Sciecne and Engineering Fair. 该竞赛由Intel公司主持,有Shell等公司协助, 并得到美国政府和军方的赞助,和世界各地的大学和研究机构的支持. 参加竞赛的内容包括科学和工程的所有领域. 现有四十多个国家的中学生参加. 这类竞赛周期长. 学生在有了创新的想法后,由高中老师推荐,到大学或公司进行初步研究. 研究结果先在各地区评定. 全世界各地区的1500多名优胜者最后到美国参加决赛. 决赛结果由1000多名教授评出. 每年奖金和各种奖励的总金额为3百万. 所有参加决赛的学生都有奖励. 其中一等奖3名, 每人5万美元.

这类竞赛全面衡量了学生的创新能力,动手能力,和表达能力. 其竞赛的优胜项目往往会得到各大学和公司的高度重视.竞赛的优胜者也因此而身价百倍. 由于这类竞赛需要投入的时间很多, 有兴趣的学生应早作准备.如果没有确实新颖且可行的题目, 虽然能得到很好的锻炼, 但也有可能事倍功半.

第二类竞赛以考试为主。参加者在规定的时间内完成试题。由得分来评定名次。这类竞赛有的偏重测试高中生在课程要求的范围内融会贯通的能力和技巧,有的竞赛则超出课程要求,偏重测试高中生的智商和在某学科的特殊天才。比如同是化学竞赛, 加拿大化学化工协会的竞赛(CIC) 偏重基础知识, 而国际化学奥林匹克则偏重智力和技巧测试. 当然基础测验和智力测验常常不能截然分开.

中学生积极参加适合自己的竞赛,能帮助发现自己的特长, 提高学习兴趣. 除此之外,其重要的现实意义是帮助自己在激烈的大学录取和奖学金申请过程中处于有利地位. 每个人都有自己的弱点和强项. 社会上许多重要岗位并不要求全才. 这反映到许多名牌大学的招生上, 在全面考查学生的同时, 往往看重该生超出常人的某方面的能力. 有时甚至一俊可遮百丑. 比如今年毕业的一位学生, 高中平均成绩刚过八十分,但因为他曾获得过几种物理竞赛奖牌, 仍顺利被麻省理工学院录取.

许多竞赛虽然由大学教授或研究机构组织,但具体过程大都在各个高中的协助下进行. 有志参加竞赛的同学, 首先要了解自己的特长. 在仔细阅读该竞赛的相关网站后, 主动与自己学校的科任老师联系. 如果得不到老师的积极支持, 应与校长联系,得到必要的帮助. 许多竞赛都有指定的参考书, 但这往往还不够. 应从诸如 amazon.com 等网站上找到更多的书和练习题. 如果觉得自己准备信心不够,也可在一些专业学校集中培训. 多伦多地区已经有一些帮助竞赛的培训中心, 如奥林匹克学校 ( http://www.geocities.com/scienceolympia ).

梁溪香榭 : 2009-02-17#448
回复: 偶搜集到的一些关于加拿大高中教育的资料 与大家分享

附录: 加拿大中学生科学竞赛常用网址

第一类: 科学项目研究竞赛

1). Intel International Science and Engineering Fair
< http://www.sciserv.org/isef/ >
2). Canada-Wide Science Fair
< http://www.ysf.ca/ >
3). Toshiba - ExploraVision
http://www.exploravision.org/2005/home.htm
4). Virtual Science Fair
< http://www.virtualsciencefair.com/ >
5). Toronto Sci-Tech Fair
< http://pathfinder.scar.utoronto.ca/~scifair/pages/scifair_home.html >
6). DuPont Science Essay Competition
http://www.glcomm.com/dupont/

第二类: 以考试为主的竞赛

一. 数学
1). University of Waterloo math contests
< http://cemc.uwaterloo.ca >
2). American Mathematics Competitions
< http://www.unl.edu/amc >

二. 电脑
1). Canadian Computing Competition
(Also the website for the Canadian Informatics Olympiad program)
< http://cemc.uwaterloo.ca/ccc >
2). USA Computing Olympiad
< www.usaco.org >

三. 生物学
1). University of Toronto National Biology Competition
< http://www.biocomp.utoronto.ca >

四. 化学
1). CIC National High School Chemistry Examination
< http://www.cheminst.ca/outreach/hsexam/cicfrm_index__e.htm >
2). University of Waterloo Chemistry Exams
< http://www.science.uwaterloo.ca/chemistry/chem_hs_exams >

五. 物理学
1). OAPT Grade 11 Physics Contest
< http://www.physics.uoguelph.ca/OAPT/contest/contest.html >
2). AAPT PhysicsBowl
< http://www.aapt.org/Contests/physicsbowl.cfm >
3). Sir Isaac Newton Exam
< http://sin.uwaterloo.ca >
4). CAP Secondary School Prize Exam
< http://www.cap.ca/edu/hsprzex.html >
5). Leonardo da Vinci Competition
< http://www.ecf.utoronto.ca/apsc/davinci/index1.html >

奥林匹克竞赛
1). Unofficial Site of the International Science Olympiads
< http://olympiads.win.tue.nl >
2). Canadian Mathematics Olympiad
< http://www.cms.math.ca/Competitions/CMO >
3). Canadian Biology Olympiad
< http://www.usask.ca/biology/cbo/index.html >
4). Canadian Chemistry and Physics Olympiads
< http://ccpo-occp.ca >

其它
1). Ontario Physics Olympiad Preparation Program
< http://www.physics.utoronto.ca/~poptor >
2). Western Canada Physics Olympiad
< http://www.phys.ualberta.ca/olympiad >
3). Ontario Chemistry Olympiad
< http://www.chem.utoronto.ca/IChO.Ontario >
4). Collection of Previous CAP Exams
< http://www.physics.ubc.ca/~outreach/CAPexams/oldexams_e.htm >
5). Eric Weisstein’s World of Science
< http://scienceworld.wolfram.com >

海怡宝贝 : 2009-02-18#449
回复: 偶搜集到的一些关于加拿大高中教育的资料 与大家分享

拜读了.收藏!

jux : 2009-02-18#450
回复: 偶搜集到的一些关于加拿大高中教育的资料 与大家分享

哪个学校? 说出来给大家提个醒儿吧.
枫华中学

梁溪香榭 : 2009-02-18#451
回复: 偶搜集到的一些关于加拿大高中教育的资料 与大家分享

枫华中学

多谢. 等会儿解冻了就给嫩加纷纷.

坛子里多次听说过这个学校. 嫩做了件大好事呢.

czy168 : 2009-02-19#452
回复: 偶搜集到的一些关于加拿大高中教育的资料 与大家分享

一个难得的好贴。谢谢分享!

海怡宝贝 : 2009-02-19#453
回复: 偶搜集到的一些关于加拿大高中教育的资料 与大家分享

我女儿也曾经上了一年枫华(高一),我感觉进入这个环境读书完全要靠自律,否则,不知在学些什么.我认为国内这些孩子从小学\初中都是在父母身边百般督促下才被动学习的,进入这类高中是很不适合的.如果家长再不关注,那有可能就此毁了.特别这学校确实存在问题.虽然近几年招生情况好象越来越好.但是实质上学校没有注重管理,师资流失大(据说老板夫妻闹分财产)应该也是一个主要原因。
庆幸的是我女儿高二换了环境后补回了那一年的损失.不怕大家笑话,我女儿从现在的老师那里了解到,整个高一她只修到6~8个学分,而且还只有C,而现在高二上学期成绩出来她修的课都达到B,有两门课就是期末没发挥好,差一点得A(为此,女儿拿到成绩单时好伤心的当着老师.同学的面就大哭了一场)老师说,她在高二这一学年预计要修28个学分.说实在的,我女儿不属于认真学习的好学生,都不能于坛子里移友门的优秀孩子比的.主要是自觉学习的习惯几乎没有,需要有人督促的,因为我们以前(小学.初中阶段有没有花时间培养她的学习习惯)又忙于生意,打拼我们的事业,只注重培养她的能力.
进入高中阶段,首先我最担心的是住读以后不要学坏,其次是学习.(高一,这一年我们都很痛苦.)现在很是庆幸,她很阳光,而且,我们沟通更好了

jux : 2009-02-19#454
回复: 偶搜集到的一些关于加拿大高中教育的资料 与大家分享

我女儿也曾经上了一年枫华(高一),我感觉进入这个环境读书完全要靠自律,否则,不知在学些什么.我认为国内这些孩子从小学\初中都是在父母身边百般督促下才被动学习的,进入这类高中是很不适合的.如果家长再不关注,那有可能就此毁了.特别这学校确实存在问题.虽然近几年招生情况好象越来越好.但是实质上学校没有注重管理,师资流失大(据说老板夫妻闹分财产)应该也是一个主要原因。
庆幸的是我女儿高二换了环境后补回了那一年的损失.不怕大家笑话,我女儿从现在的老师那里了解到,整个高一她只修到6~8个学分,而且还只有C,而现在高二上学期成绩出来她修的课都达到B,有两门课就是期末没发挥好,差一点得A(为此,女儿拿到成绩单时好伤心的当着老师.同学的面就大哭了一场)老师说,她在高二这一学年预计要修28个学分.说实在的,我女儿不属于认真学习的好学生,都不能于坛子里移友门的优秀孩子比的.主要是自觉学习的习惯几乎没有,需要有人督促的,因为我们以前(小学.初中阶段有没有花时间培养她的学习习惯)又忙于生意,打拼我们的事业,只注重培养她的能力.
进入高中阶段,首先我最担心的是住读以后不要学坏,其次是学习.(高一,这一年我们都很痛苦.)现在很是庆幸,她很阳光,而且,我们沟通更好了
不知道海怡飞女儿现在去了什么学校,可否分享一下。我现在很头痛女儿学校的问题。我下个月面试,原以为下半年就可到加拿大就读,与中介交流后才知还要近一年左右时间才能登陆。现在女儿还在读枫华,如果去不了,还得想办法换学校,不能再待在枫华了。

david lv : 2009-02-19#455
回复: 偶搜集到的一些关于加拿大高中教育的资料 与大家分享

好文章。太有用了,谢谢

shine : 2009-02-19#456
回复: 偶搜集到的一些关于加拿大高中教育的资料 与大家分享

太谢谢了!有用的帖子!

海怡宝贝 : 2009-02-19#457
回复: 偶搜集到的一些关于加拿大高中教育的资料 与大家分享

换学校其实也是个难抉择的事,我们的孩子已经走了这条路,是无法走回头路的,这里的普高是行不通的,我也曾经问女儿是否愿意回这里的普高重修高一,苦一下为将来打基础.她不愿.我也于她好好的沟通了一下,她表示看的表现.我也感觉了她主观上已经反省了进入高一后因种种原因(主要是自身到了一个寄宿制的环境,一个小社会里,光怪陆离的让他们目不暇接,无法分辩.)没有努力,回首一下觉得好象浪费了一年.我也尊重了本人的选择.
现在,我女儿就读的学校是类似的学校江苏省木渎中学国际班.说实话也不能寄希望太好,因为学习的模式也没怎么改变,只是环境变了,人没那么多了,影响面没那么广了.他们没有国内的课程了,反正一进入学校没有让你浪费半年或一年专修ESL.
这里,我还是要提醒的是学校确实有很大的环境因素,学生之间互相影响.但是总体还是两个字"自律".这么大的孩子是非观有些模糊,家长要有很好很好的耐心去寻找一切机会与之沟通.引领他们走上正道~
个人愚见,敬请各为谅解.

东去春来 : 2009-02-20#458
回复: 偶搜集到的一些关于加拿大高中教育的资料 与大家分享

俺家小伙子也是明年初中毕业. 俺们08年5月的FN, 瞧着加拿大的蜗牛速度, 2010年都不知道能不能登陆. :wdb23::wdb23::wdb23:
关于A-LEVEL课程, 国内一般都是在高一先上O-LEVEL, 作为过渡, 特别是语言方面. 然后高二高三才是A-LEVEL.
反正是全英文授课, 就当是提高语言吧. 啥时候VISA到啥时候走人.
相比中加学校, 俺感觉至少我们这边的A-LEVEL是由本市最好的高中开设的, 就收50个人, 2个班. 开展得还不错, 收的都是好孩子, 家长比较放心.
转学应该不是问题啊. 不可能不收吧. 呵呵.
考托福或雅思是避免不了的. 其实这也没那么难. 不用太担心这个.

想知道,除了托福或雅思外,最好在学A-LEVEL是吗?用考个sat吗?看美国好像认这个和托福成绩!不知加认不认sat.

梁溪香榭 : 2009-02-20#459
回复: 偶搜集到的一些关于加拿大高中教育的资料 与大家分享

想知道,除了托福或雅思外,最好在学A-LEVEL是吗?用考个sat吗?看美国好像认这个和托福成绩!不知加认不认sat.

不能说最好学A-LEVEL. 我只是提出我的看法. 很难说哪个绝对好.
另外没听说上加拿大上大学要求SAT.

梁溪香榭 : 2009-02-20#460
回复: 偶搜集到的一些关于加拿大高中教育的资料 与大家分享

换学校其实也是个难抉择的事,我们的孩子已经走了这条路,是无法走回头路的,这里的普高是行不通的,我也曾经问女儿是否愿意回这里的普高重修高一,苦一下为将来打基础.她不愿.我也于她好好的沟通了一下,她表示看的表现.我也感觉了她主观上已经反省了进入高一后因种种原因(主要是自身到了一个寄宿制的环境,一个小社会里,光怪陆离的让他们目不暇接,无法分辩.)没有努力,回首一下觉得好象浪费了一年.我也尊重了本人的选择.
现在,我女儿就读的学校是类似的学校江苏省木渎中学国际班.说实话也不能寄希望太好,因为学习的模式也没怎么改变,只是环境变了,人没那么多了,影响面没那么广了.他们没有国内的课程了,反正一进入学校没有让你浪费半年或一年专修ESL.
这里,我还是要提醒的是学校确实有很大的环境因素,学生之间互相影响.但是总体还是两个字"自律".这么大的孩子是非观有些模糊,家长要有很好很好的耐心去寻找一切机会与之沟通.引领他们走上正道~
个人愚见,敬请各为谅解.

非常谢谢海怡. 实实在在的经历最珍贵.

海怡宝贝 : 2009-02-20#461
回复: 偶搜集到的一些关于加拿大高中教育的资料 与大家分享

非常谢谢海怡. 实实在在的经历最珍贵.
不用客气,如果我们的经历能给大家以帮助或警示自感非常快乐。

梁溪香榭 : 2009-02-23#462
回复: 偶搜集到的一些关于加拿大高中教育的资料 与大家分享

刚刚看到这么一段, 贴出来供大家参考.

加拿大学校录取主要看高中成绩。事实上各个高中的水平不一, 光看绝对成绩并不科学。我曾经和University of Toronto生物系的一个教授聊过这个问题。 据他说, 其实每个学校的招生办公室都有自己的“黑名单”。名单上的每个高中都有一个加权指数, 用来调整各个学校的水平差别。 如他们认为学校A好,这指数可能会是1.04, 他们认为学校学校B差一点,这指数可能会是 0.99。 在录取时他们会拿这个指数去乘学生成绩。 比如学校A的90分可能作为93.6分看待。据他说这个名单是根据各高中过去的校友进入该大学以后的成绩来估计的。你以前的高中校友在该大学普遍表现好, 这个指数就会稍高一点。这个指数与社会上任何排名或竞赛无关。

据他说各大学都不承认有这样一个名单。 主要是怕有法律方面的麻烦。但实际上都这样操作。

再次申明, 我是听来的。 而且该教授并不管招生。以上信息的准确与否无法查证。大家自己判断。

另外一些好学校的好专业(如University of Toronto的Engineering Science) 除了成绩外还要看其它业绩和成就。

梁溪香榭 : 2009-02-23#463
回复: 偶搜集到的一些关于加拿大高中教育的资料 与大家分享

地球人都知道,美国的医药费昂贵,美国医生收入也比较高。到底有多高? 以下转美国主要临床专科医生的收入情况。

在看这个美国医生工资表以前,先提醒大家一下几点:
(1).在美国,工薪阶层都是要交税的。而且,收入越高,税率越高。下面罗列的医生收入,均为税前收入。换句话说,医生们交了税以后,每个月拿到手里的银子并不是罗列的年收入除以12这么多。
(2)。以下所列工资,为美国医学院毕业生完成住院医甚至专科训练以后的劳动所得。美国医学生毕业后,至少需要三年以上的住院医训练才可以正式行医。美国住院医的工资并不高,年薪大约有四万左右吧。但从二年级的住院医开始,就可以做MOONLIGHTING获得额外收入。
(3)。虽然美国医生工资比很多行业高,但医生并不属于美国的暴富阶层。
(4)。下面的美国医生工资资料,来自美国洛杉矶的一家医疗卫生公司(Allied Physicians, Inc., Los Angeles Times and Rand McNally ) 。此资料的更新准确度截止于本年 (2006年) 的六月份。网络资料,仅供参考。

另外,美国医生的收入有几个潜规则:
1。在美国医学院,一边搞科研一边行医的医生收入没有本专业纯粹行医的医生收入高。
2。美国边远地区的医生收入会比大城市同类医生的收入偏高。美国农村没人愿意去,只能靠高薪诱惑了
3。不同城市的同科医生,收入也会有所不同。
4。专科医生的收入比非专科医生的收入和生活方式会有些优势。简言之,美国专科医生所接受的训练时间,要比普通的内科医生要长。举个例子简单说明一下。比如,内分泌专科医生,虽然属于内科类,但它需要未来的专科医生在完成普通的内科住院医训练之后,再做2-3年的专科(内分泌)训练,英文简称FELLOW。

下面所列的每一科工资,都有三个数目。它们的真正含义是,第一个数字是工作1-2年时的工资,第二个数字是工作3年以上时的工资,第三个数字是本科所能拿到的最高工资数。

1。儿科麻醉(Anesthesiology: Pediatrics)
$ 283,000 $311,000 $378,000
2。普通麻醉(Anesthesiology: General)
$207,000 $275,000 $448,000
3。普通心脏科医生(Cardiology: Noninvasive)
$268,000 $403,000 $599,000
4。皮肤科(Dermatology)
$ 195,000 $308,000 $452,000
5。内分泌(Endocrinology)
$171,000 $187,000 $260,000
6。胃肠病(Gastroenterology)
$265,000 $349,000 $590,000
7。血液病/肿瘤(Hematology/Oncology)
$181,348 $245,000 $685,000
8。普通内科(Internal Medicine)
$154,000 $176,000 $238,000
9。儿科(Medicine/Pediatrics)
$139,000 $168,000 $271,000
10。妇产科(Obstetrics/Gynecology)
$211,000 $261,000 $417,000
11。眼科(Ophthalmology)
$138,000 $314,000 $511,000
12。耳鼻喉科(Otorhinolaryngology)
$194,000 $311,000 $516,000
13。病理科(Pathology)
$169,000 $321,000 $610,000
14。放射科(Radiology)
$201,000 $354,000 $911,000
15。普通外科
$226,000 $291,000 $520,000

梁溪香榭 : 2009-02-23#464
回复: 偶搜集到的一些关于加拿大高中教育的资料 与大家分享

http://www.youtube.com/profile_videos?user=CarletonPhysics

15个中学物理演示实验

海怡宝贝 : 2009-02-23#465
回复: 偶搜集到的一些关于加拿大高中教育的资料 与大家分享

报个到!

梁溪香榭 : 2009-02-24#466
回复: 偶搜集到的一些关于加拿大高中教育的资料 与大家分享

转载:美国阳光学院的智博士教育专栏

美国有 4100 余所从事高等教育各种类型的大学与学院,其中公立学校约有 1200 余所,私立约有 2900 余所。共有各类学生约 1700 万余人。

其中有近 300 所研究型大学,可以进行本科生,职业文凭,及研究生 ( 硕士生,博士生 ) 教育,并可授予最高学位。另有 2200 余所四年制学院,主要进行本科生教育,授予学生学士学位。在这些学院中,排名前列的有阿默斯特学院、波莫纳学院、斯沃斯莫尔学院、韦尔斯利学院、威廉姆斯学院、和史密斯学院。这些学院在本科教育方面,在美国大名鼎鼎。他们的学生本科毕业后也有相当一部分进入美国最著名的研究型大学进一步深造。还有约 1600 余所二年制的社区大学,这些大学只能授予学生 “ 准学士 ” 学位,学费较低,且三分之二是非全时学生。

我们用类比围棋的段位来大致给美国名牌大学与学院 ( 这里仅指本科教育 ) 分级:

十段大学 ( “超”一流大学, 6 所 ) :

哈佛大学、耶鲁大学、普林斯顿大学、斯坦福大学 、 麻省理工学院、加州理工学院

九段大学 ( “很强”一流大学, 15 所 ) :

哥伦比亚大学、宾西法尼亚大学 、 布朗大学、康奈尔大学、达特茅斯大学、芝加哥大学、西北大学、约翰 - 霍普金斯大学、杜克大学、华盛顿大学 (in St.Louis)

以及一些最著名的文理学院:

阿默斯特 (Amherst) 学院、威廉姆斯 (Williams) 学院、斯沃斯莫尔 (Swarthmore) 学院、波莫纳 (Pomona) 学院、韦尔斯利 (Wellesley) 学院。

八段大学 ( “强”一流大学, 11 所 )

卡内基 - 梅隆大学、乔治城大学、埃默里大学、范德比尔特大学、 莱斯大学、 圣母大学。

以及一些文理学院:

卡里顿 (Carleton) 学院、梅德贝利 ( Middlebury) 学院、史密斯 (Smith) 学院、鲍登 ( Bowdoin ) 学院、戴维森 (Davidson) 学院。

七段大学 ( 一流大学, 16 所 )

加州大学 - 伯克莱大学、弗吉尼亚大学、密西根大学 - 安阿伯尔分校、加州大学 - 洛杉矶分校、北卡罗莱纳大学 - 教堂山分校、南加州大学、塔夫茨大学、维克森林大学、威廉 - 玛丽学院、威斯康星大学 - 麦迪逊分校、凯斯西部储备大学、

以及一些文理学院:

哈弗福 ( Haverford) 学院、克莱蒙特 麦肯纳 ( Claremont McKenna ) 学院、威斯丽安 ( Wesleyan) 学院、格林内尔 ( Grinnell) 学院、瓦萨 (Vassar) 学院。

六段大学 ( “稍弱”一流大学, 25 所 )

布兰迪斯 (Brandeis) 大学、里海 (Lehigh) 大学、波士顿学院、 纽约大学、罗切斯特大学、加州大学 - 圣地亚哥分校、佐治亚理工学院、伊利诺大学 - 香槟分校、华盛顿大学 (in Seattle) 、 伦斯勒尔 (Rensselaer) 理工学院、加州大学欧文分校、杜兰 (Tulane ) 大学、叶史瓦 (Yeshiva ) 大学、宾州州立大学 - 帕克分校、德州大学 - 奥斯汀分校、加州大学 - 戴维斯分校、加州大学 - 圣芭芭拉分校。

以及一些文理学院:


哈卫 - 玛德 (Harvey Mudd) 学院、哈密尔顿 (Hamilton) 学院、柯盖特 (Colgate) 学院、美国海军学院、奥伯林 (Oberlin College) 学院。


所谓名校 ( 这里指本科教育 ) ,没有一个明确的标准,只是一个社会较普遍的认识而已,不同的机构发布的名单也有不同。在上述的分级名单中, 8 - 10 段大学与学院可以认为是世界名校。而 6 - 7 段大学与学院可以认为是国家级名校,这些大学的一些专业也享有世界声誉。(在围棋中, 6 段就是很高段的高手了,是国手了! )而且需要强调的是, 美国大学的本科教育并没有大家想象的相差大;在前一二百名的研究型大学或文理学院里读书,都能得到很好的基础训练。若说差别较大的可能是来自于同学水平的差别,一般来说,越是名校,学生的平均水平越高,同学之间的相互影响也越正面。


最后,需特别声明:这里的分级是美国阳光学院指导入学申请专家的划分,内部供学生申请美国名牌大学时参考。现发表在此供感兴趣的人作一个参考。

百合花正开 : 2009-02-26#467
回复: 偶搜集到的一些关于加拿大高中教育的资料 与大家分享

真是好帖,非常佩服楼主这样的好家长。

Luck Li : 2009-02-26#468
回复: 偶搜集到的一些关于加拿大高中教育的资料 与大家分享

这么有用的帖子,咋木看到呢。先占个位置,来一脚再看。

Luck Li : 2009-02-26#469
回复: 偶搜集到的一些关于加拿大高中教育的资料 与大家分享

给楼上各位都+fenfen。

Luck Li : 2009-02-26#470
回复: 偶搜集到的一些关于加拿大高中教育的资料 与大家分享

辛苦了溪美女,好母亲。:wdb17::wdb19:

Luck Li : 2009-02-26#471
回复: 偶搜集到的一些关于加拿大高中教育的资料 与大家分享

功力恢复!挥霍开始!

家园移民 : 2009-02-28#472
回复: 子女教育信息素材收集帖

好帖子顶顶

家园移民 : 2009-02-28#473
回复: 偶搜集到的一些关于加拿大高中教育的资料 与大家分享

梁溪啊

嫩说偶把这个帖子和http://forum.iask.ca/showthread.php?t=171144&page=8
合并下,

然后置顶怎么样啊?

梁溪香榭 : 2009-02-28#474
回复: 偶搜集到的一些关于加拿大高中教育的资料 与大家分享

没问题啊.

Luck Li : 2009-03-04#475
回复: 子女教育信息素材收集帖

终于看到这个帖子了,真是高兴呀。呵呵!

家园移民 : 2009-03-04#476
回复: 子女教育信息素材收集帖

偶也高兴啊,还是个合并后的帖子涅!!

Luck Li : 2009-03-04#477
回复: 加拿大教育情况有用素材收集帖子--梁溪香榭整理

置顶了真好。感谢守法领导,也感谢香榭的辛苦整理。+fenfen

:wdb17::wdb17::wdb19::wdb19::wdb6::wdb6:

cji8 : 2009-03-04#478
回复: Lucia的说法2

“作者: wangxiaoyu-62
LUCIA,你写的很详细,很有帮助.我也是90年的,我是四月的.我现在正读高三,马上准备登录,但我有些问题想请教一下:我原本打算参加完国内的高考,拿到成绩后赴加.我是想到加后能进入象多大这样的大学学习.但听说这些名牌大学对象我这样的新移民申请时需提供TOFEL或者IELTS成绩,而我没有.象我这种情况,朋友给了我几种方案1)不要参加国内的高考,马上赴加来读这里的高三.------这样的话语言上会有很大进步,但我觉得还是不能申请大学,因为我还是没有TOFEL成绩,我上高三的课程那有时间考啊,还有就象你说的,学分也不够啊.其实我已留了一级,很有可能留两级呢有点浪费时间(2)参加完高考再赴加,然后读一年的预科,这期间学校有TOFEL的课程并可以参加考试.然后可以申请大学-----但听说这类预科学校都是私立的,不可靠,会随时倒闭,这又让我很迷惑.所以我很拿不定主意该怎么办? ”



就你的情况,我和儿子讨论了下,他认为,还是尽快过来,因为大学要求很高,如果成绩不好,或者fail课了,就会被踢出来,所以,到了大学首先语言要过关,不然课听不懂肯定fail了。高中过来读的好处呢,就是有时间适应英语,而且学分不是问题, 国内的学分可以转,他一个同学今年圣诞节过来的,到多伦多,她在国内正读高2,上个月学分转了,得到了20个学分,一年半轻轻松松就可以把学分拿够,学分到不是问题;还有你说的TOFEL,不是所有大学都要TOFEL,有的大学有它专门的英语考试,过了就可以上。在高中还有许多大学的竞赛,waerloo的macmaster的都经常在儿子的学校搞数学竞赛、物理竞赛什么的,他还在数学竞赛中拿了个全校第一,题目很简单;这个竞赛中成绩好的,大学录取的时候就会优先考虑。
在预科呢,他也有认识的同学在读,但是就是像你说的不可靠,主要不是怕倒闭,主要是学习环境差,教学质量也差,而且很贵。
我这里有个方案,你现在已经高三,没几个月就高考了,考不考无所谓,因为你考了过来还可以上高中,只要你是21岁以下的都可以上高中,然后就像我前面说的,大学要6门专业课学分,你就想好要学哪个专业,然后到http://www.electronicinfo.ca/en/上面看...难事。然后还有就是弄好国内成绩的翻译和公正,拿过来转学分,你的情况应该可以转24个学分吧。
就看你怎么安排,读一年,辛苦点,早点毕业,读2年,轻松点,可以有更多选择,比如你可以多?藜该鸥咧械目危?ù笱У氖焙蚓筒恢劣谥荒鼙ㄒ桓鲎ㄒ怠?
我儿子是90年10月的,现在才在这里高中读11年级,算正常年龄。你年龄不算大,不用着急的,主要是要能上个好大学,而且要能坚持住,不被T出来。
有参考价值

梁溪香榭 : 2009-03-04#479
回复: 加拿大教育情况有用素材收集帖子--梁溪香榭整理

哇, 心情好比找回了走失的孩子啊!

谢谢领导, 谢谢各位, 谢谢.
俺再接再厉, 掘地三丈, 争取更大进步滴说 :wdb23:


不过, 领导啊, 嫩这个标题有点太那个啦.
咋全变成俺滴功劳啦?
前面大部分都是嫩搜集的呀.

家园移民 : 2009-03-04#480
回复: 加拿大教育情况有用素材收集帖子--梁溪香榭整理

哇, 心情好比找回了走失的孩子啊!

谢谢领导, 谢谢各位, 谢谢.
俺再接再厉, 掘地三丈, 争取更大进步滴说 :wdb23:


不过, 领导啊, 嫩这个标题有点太那个啦.
咋全变成俺滴功劳啦?
前面大部分都是嫩搜集的呀.
给嫩带个大帽子
嫩就没有借口偷懒了
嘎嘎

有得就有失 : 2009-03-04#481
回复: 加拿大教育情况有用素材收集帖子--梁溪香榭整理

不管怎么样,先来捧个场,虽然看得我头晕,哈哈!

梁溪香榭 : 2009-03-04#482
回复: 加拿大教育情况有用素材收集帖子--梁溪香榭整理

给嫩带个大帽子
嫩就没有借口偷懒了
嘎嘎

哈哈. 有领导艺术. :wdb17:
那行吧, 就这儿一亩三分地儿啦.
请领导和同学一起着力耕耘哈
:wdb20::wdb6:

cji8 : 2009-03-04#483
回复: 偶搜集到的一些关于加拿大高中教育的资料 与大家分享

A LEVEL比较容易?绝对不是。A LEVEL体系的教学内容,拿到中国任何一间重点中学,都足以令教师畏缩。

那么为什么读A LEVEL比较容易升入世界名牌大学呢?

这有几个原因。

第一,按A LEVEL课程体系规定,每个学生修读的课程比较少。对留学生,可以把有限的精力集中于这几门课,相对说来更有把握获得好成绩。

作为对比,可以考虑澳洲维多利亚省的11-12年级课程,一般要求学生修读6门课,加拿大安大略省的预科课程,一般也要求学生选读6门课。而英国的A LEVEL,第一年只要求学生选读4门课,第二年可以进一步减少到3门。这对初来乍到、语言文化都相当陌生的国际留学生非常有利。对中国学生更是如此。以中国学生的勤奋,要在三四门课程中取得好成绩,不是很艰难的事。

第二,A LEVEL没有必修课的规定,学生可以根据自己的知识基础、兴趣爱好和特长,选择自己最有希望、最有把握获得好成绩的课程。其他国家的课程体系,一般都有规定的必修课。

第三,A LEVEL的数学课,可以当作两门课、甚至三门课来修读,这对中国学生最有利。

中国学生从小学高年级起,在数学训练上,就比西方国家学生普遍更严格,数学往往是中国学生的优势。世界上所有的中学和预科课程体系,纯美术音乐类的除外,说到底,就是两大支柱,第一是语言,第二是数学。换句话说,在学校中要获得好成绩的关键条件,只有两条:或者语言好(英语好),或者数学好(当然还要加上学生愿意学习)。中国中学生到英语国家留学,英语再好,与本土优秀学生还差了老一大截。因此,中国中学生到英语国家留学,想要在那些极端依赖英语的课程上与本土学生竞争,是不现实的不明智的。中国中学生应该扬长避短,避开英语的劣势,发挥数学的优势和强势。而英国的A LEVEL课程,正好为中国学生发挥数学优势提供了舞台。

中国学生只要在国内时数学属于中等以上,那么读A LEVEL课程时,就可以选读2门数学。数学对英语语言的要求最低。原因很简单:数学使用的语言最规范、意义最精确,没有文学中那些比喻、含蓄、夸张、讽刺等等花里狐骚的东西,数学词汇相对比较少,数学自身的公式和符号世界统一,不仅不依赖特定语言,还可以反过来帮助理解数学教材/试卷中使用的自然语言。

如果中国学生选2门数学,就意味着在A LEVEL第一年,数学成绩占了总成绩的百分之五十,第二年占了三分之二。换句话说,如果数学好,那么去英国读A LEVEL课程,“高考”中三分之二的好成绩已经握在手中,三分之二个身子,已经踏进世界名牌大学的大门啦。

因此,对那些数学比较好的中国中学生,通过读A LEVEL升入世界名牌大学,简直如履平地。

人人都认为:世界上没有简单的“高考”,世界上没有敞开大门的名牌大学。他们错了,A LEVEL课程对中国优秀学生,是非常“简单”的“高考”,也是通往世界名牌大学校门的大道平川。

只是我们不知道,说不定什么时候,A LEVEL课程也会变化,变成不利于中国学生“钻空子”。

第四,A LEVEL课程的考试,是“边读边考”,两年内根据教学进度分四轮考完,如果一轮考试中,某一科某一模块没有考好,下一轮(隔半年)还可以再考。这样就大大减轻了每次考试的复习负担和心理压力,也更有利于学生表现出自己的正常水平。学生不会因为一次考试发挥失常,就丧失人生的重大机会。

第五,A LEVEL课程是两年中分四轮考完,如果一轮考试中,某一科某一模块没有考好,那么学生重新考试时,完全有把握提高成绩。这是因为英国学校的假期很多、很长,学生可以利用假期,针对性地复习和预习,解决掉薄弱环节。英国的中学/预科,一年有三个大假,三个小假。三个大假是:暑假2个月,圣诞节假近一个月,复活节假20多天。三个小假就是每个学期的期中假(half term holiday),每个期中假都是一个星期左右。英国学生自己没有假期补习的习惯。但对于那些渴望读世界名牌大学的中国少年,为了增加把握,不妨利用这些假期来扩大自己学术成绩上的把握和优势。

第六、英国的A LEVEL课程考试,既是一种应试成分很强的知识性课程,很适合应试教育下“成长”的中国学生,但又绝对不会有象中国考试中那种专门设陷阱让学生往里死的怪题偏题。考试中出现的题型,在教科书里都可以找到。只要掌握了教材中的基本概念、原理、定理、定律,和基本类型的题目,在考试中都有把握获得好成绩。在国内重点中学中,经历过“整人坑人算计人”试题训练的小留学生,看见英国“试卷”中考题的简单明了,不禁要仰天大笑:真是天助我哇。
谢谢丰富的资讯.加分了

螺母 : 2009-03-04#484
回复: 加拿大教育情况有用素材收集帖子--梁溪香榭整理

哇, 心情好比找回了走失的孩子啊!

谢谢领导, 谢谢各位, 谢谢.
俺再接再厉, 掘地三丈, 争取更大进步滴说 :wdb23:


不过, 领导啊, 嫩这个标题有点太那个啦.
咋全变成俺滴功劳啦?
前面大部分都是嫩搜集的呀.

恭喜恭喜!失而复得之后更加珍惜!
置顶得好啊!大家查阅起来很方便!:wdb6:
辛苦粱姐姐和守法领导!谢谢你们!:wdb17:
有空慢慢看。。。

盈婆婆 : 2009-03-04#485
回复: 加拿大教育情况有用素材收集帖子--梁溪香榭整理

这帖差点没了,正着急找呢。

nakijin : 2009-03-04#486
回复: 加拿大教育情况有用素材收集帖子--梁溪香榭整理

好贴,顶一下,楼主辛苦了。

梁溪香榭 : 2009-03-04#487
回复: 加拿大教育情况有用素材收集帖子--梁溪香榭整理

如何选择适合自己孩子的中小学

多伦多地区有私立学校和公立学校两大类。私立学校(不是指旨在招收小留学生的私立学校)的共同优点是校风较好, 同学之间的成绩比较整齐, 家长的参与程度高,教师较负责任。有的私立学校偏重于学术, 但有些较贵族化的私立学校目的是培养谦谦君子和淑女。很多家长送孩子进这类私立学校,意在从小建立孩子的社会关系网,培养孩子在上层社会中交流所需要的风度和礼仪。每年各种选美比赛中的优胜者, 大多数都出自这类私立学校。 多伦多地区一些私立学校的情况请见
http://privateschool.about.com/od/schoolstoronto/

偏重于学术的私立学校良莠不齐。 好的私立学校在社会上有很高的信誉。其校友中名人辈出。因此入学竞争很激烈。有的学校(如UTS)平均只能录取10%左右的申请者,其激烈程度不亚于申请名牌大学。但一些名气小的私立学校则没什么竞争,一般学生都可以进去。 各个私立学校自成体系。 但每年在多伦多都会有集中介绍所有私立学校的展览会(Private School Fair)。有兴趣的家长应事先浏览各校的网站,在展览会上得到更多的信息, 然后逐一访问初选出来的学校。有条件的话, 还应与已经在校的孩子的家长交流。

私立学校的学费一般每年从$10000到$30000不等。如果住校其总费用可每年可高达$40000。一些私立学校对低收入的家庭提供经济资助, 在录取时只看成绩,不考虑学生的家庭经济情况(merit based)。在录取后根据家庭收入提供必要的助学金. 如UTS的捐款基金(endowment)达1千2百万. 又如Upper Canada College每年提供的助学金达140万元。 但大部分私立学校需要家长负担全部费用。 过去保守党在安省主政时, 私校的学费可部份免税。这措施已经被现任政府取消。 对大多数家庭来讲,私立学校的高昂费用让人可望不可及。公立学校是大多数学生自然而然的选择。

对于能负担私立学校学费的家庭, 选择公立还是私立学校仍然要因孩子而异。好的公立学校也有自己的优势。 公立学校人数多, 学生社团活动往往更丰富, 能给孩子提高社会活动能力提供更大的舞台。公立学校学生的家庭背景更多样化,能锻炼孩子的适应性和与各种文化背景的朋友交往的能力。与欧洲相比,北美相对是一个“精英社会”,而不是一个“家族社会”,更看重个人奋斗而不是家庭背景。上公立学校能去除一些孩子的家庭优越感, 以平常心面对竞争。公立学校虽然是就近入学, 不能选择学生。 但许多公立学校设有特殊教育课程,在一定程度上也能因材施教。

梁溪香榭 : 2009-03-04#488
回复: 加拿大教育情况有用素材收集帖子--梁溪香榭整理

多伦多地区的公立学校分天主教学校和普通公立学校两部分。天主教学校除了对学生的信仰有一定的要求外, 一般教学内容和程度都与普通公立学校无异。但天主教学校因为宗教原因,管理较严格,校风相对要严谨些。对于有信仰的家庭, 是一种很好的选择. 如果家长本来不信教也不准备信教, 不宜把孩子送入这类学校. 因为孩子在无信仰的家庭和有信仰的学校之间可能感到矛盾和困惑, 产生不必要的心理冲突. 多伦多天主教教育局所属学校的情况, 请见网站 http://www.tcdsb.org/

在选择公立学校时,应当综合考虑孩子的兴趣和特长, 学校的特点,社区情况等等. 由于加拿大的公立学校没有重点非重点之分, 政府对各个学校的经费投入都差不多。再加上没有一个官方认可的学校质量的比较或综合排名,对所谓“名校”的不同认识常常是各个论坛上的热点话题。很多人眼中的名校,只是基于一些房地产经纪人的流行讲法。这些学校也许在过去某个历史时期确实不错。但铁打的营盘流水的兵,时过境迁,需要我们根据最新的资料作新的评估。

每个公立学校都有自己的特点,有自己的传统和文化。其特点可以反映在不同方面:某些特殊课程, 学生参加社区服务,学生参加各类竞赛,学生社团, 特殊兴趣俱乐部(如运动,艺术,音乐, 戏剧, 辩论)等等。在选择学校过程中,这些特点远比EQAO成绩和各类排名更为重要。

在选择学校时,首先要对自己的孩子的兴趣和潜在能力有一个大概的判断。对于有音乐,美术, 体育,法语等特长的孩子, 应该首选设有相应特殊教育课程的学校。大的学区(如多伦多学区)有一些学校专设有这类课程和配套设施。在这些学校孩子的特长能得到较好地发挥。有关多伦多教育局各校的特殊课程,请参见
http://www.coviews.com/viewtopic.php?t=11864

对于社会活动能力强,综合素质好,但主要兴趣不在学术上的学生,应选择学生人数多,课外活动丰富,学生社团活跃的学校。让孩子有更多机会锻炼培养社会活动能力。将来向管理,从政,商业等方面发展。这方面的情况,可从教育局的网站上得到一些。但更重要的是直接向学校询问。有些杂志(如MaCleans)近几年对一些课外活动丰富多彩的学校有重点介绍。该杂志任何图书馆都能借阅。

梁溪香榭 : 2009-03-04#489
回复: 加拿大教育情况有用素材收集帖子--梁溪香榭整理

对于大多数华人家长,选择学校时考虑最多的是学校的学业水平。Fraser Institute 近年来发表了一系列的报告,对各省的中小学校试图作一个全面的评估, 可参考: http://www.fraserinstitute.ca/reportcards/index.asp?snav=rc

对这些报告,各教育局和很多学校的管理人员,由于涉及利益关系,基本上持否定态度。它2005年对安省小学的排名,主要依据于3年级和6年级的EQAO考试成绩。虽然指标单一,但较为客观,有一定的参考价值。它对安省高中的排名,迄今只有2001年的报告。其依据是2001年以前各中学高级课程的设置情况和学生在这些课程中的考试成绩。这种排名方法与美国新闻周刊(Newsweek) 依据IB和AP课程来排名美国27,468 所公立学校的概念类似, 可参考: http://www.fraserinstitute.ca/reportcards/index.asp?snav=rc

但Fraser的排名中所谓的"高级课程"没有统一的标准,且资料来自各中学自己的统计, 许多中学根本没有提供任何资料。这排名只能说是一个尝试,而且早已过时,没有太大的价值。如果想了解各学校9年级的EQAO和十年级的OSSLT考试成绩,可参考http://www.eqao.com/results/?Lang=E

由于依据EQAO成绩的学校排名没能反映社区即学生来源对学校成绩的影响,National Post根据社区因素对EQAO成绩作了加权修正,得出了大多伦多地区学校的另一个排名。但那个排名也没能得到教育界的认可。

这些考试或排名只能作为参考。网上常常有朋友根据EQAO成绩几分的差距来评判两个学校的好坏,这种看法较为片面的。在小学阶段, 孩子的特点和爱好还没成形, 这些考试或排名的参考意义大一些。对于初中以上的孩子, 这些考试只反映一般的教学要求,适合中等学业的孩子参考。如果你的孩子有特殊兴趣才能或者学业水平较高,这些考试对你选择学校的参考意义不大。 另外应该注意这些考试或排名只是相对地说明过去某一年某一次考试的情况。比如EQAO最高只考到九年级。 有些很好的初中最高为九年级。 它们的学生在十年级将转入其它高中。这些好学生的EQAO成绩不会计入所去的高中。 这可说明为什么有的高中(如Don Mills C.I.) 九年级的EQAO不高,却有那么多拔尖的毕业班学生。 还有公立学校的学术水平很大程度上依赖于校长和老师。校长换届,某几个老师退休往往使一个知名度很高的学校从此风光不再。

在学业上优异的孩子,最好能去有特殊教育课程(如 IB, gifted, TOPS, AP 等等)的班级. 如果新移民因为错过了入学考试而不能进入这些班级,也应争取在这类学校的普通班学习。中学阶段孩子受周围的老师同学的影响很大。 在一个优秀的群体中,有高年级同学的榜样,有周围同学的促进, 有老师的帮助,天资好的学生自然而然会有学习的动力。所谓“蓬生麻中,不扶而直”就是这个道理。在论坛上常见有人建议让gifted program的孩子到高中后降回普通班,据说是可以拿到高一些的分数。我认为这种想法并不恰当。且不说舍高求低会伤害孩子的自尊心,降低孩子学习的积极性,就拿分数来说,混合有gifted program和普通班的学校,其gifted program评分也会有适当的调整。

对于在某些学科上有特殊秉质的学生,应该尽可能进入一些有参加学科竞赛的传统的学校。学生参加学科竞赛往往并不作为学校和教师业绩的考核指标,许多公立学校没有鼓励学生参加竞赛的传统, 孩子的天赋因此可能被忽略或者低估,对自己的天赋的发挥可能得不到激发。从近年有关资料分析,有几所中学鼓励和帮助学生参加各种竞赛卓有成效 (http://www.coviews.com/viewtopic.php?t=11525 )。

有关学科竞赛的信息, 请参考http://www.coviews.com/viewtopic.php?t=9450。

综上所述, 选择学校没有固定的模式。首先要考虑孩子自身的特长和弱点。 在参考学校的各种排名和EQAO成绩的同时, 更重要的是亲自到学校了解其课程设置,课外活动情况,种族多样性等等, 帮助自己的孩子作出正确的决定。对于刚到加拿大不久的孩子,除了上面一些因素外,还应考虑学校的语言环境。为了加快提高英语水平,最好选择同学之间只讲英语的学校。对于家长而言, 因公立学校是就近入学,选择学校还不得不考虑家庭的其它因素,但这不在本文的讨论范围之内。

梁溪香榭 : 2009-03-04#490
回复: 加拿大教育情况有用素材收集帖子--梁溪香榭整理

论孩子成长的内环境

影响孩子成长的因素众多。归结起来不外两大方面:即出生前的父母遗传、优育与出生后的生活环境、训练等等。

绝大部分孩子的智能特长与智能水平在 出生前以就不可改变了。也就是说无论后天条件如何优越、训练如何得当,孩子的发展都不可能超过其先天禀赋。各领域的成功人士并非在青少年时期都有优越的环境、系统的训练。条件极其优越后代无所作为,条件极其艰苦后代却成大器,两者都有不少现成的例子。因此华人家长对子女的期望不可超过孩子的先天禀赋的实际能力,更不可简单地将自己没有实现的理想强加在孩子身上。否则会生出许多矛盾、痛苦、甚至造成不可挽回的终生悔恨。

是不是说做父母的就可以对孩子放任自流了呢?非也!在某些特定的情况下后天因素会使先天的良好禀赋大打折扣,甚至扼杀本来非常优秀的先天遗传。也就是说一些孩子的成长受后天因素影响较大。例如孟母三迁的故事广为熟悉,多伦多许多家长为孩子的外在学习环境煞费苦心,导致了某些学校周围的房子价格飞涨。请允许我再强调一次:无论我们为孩子买那里的房子、安排多少课外训练都只能是使孩子的先天禀赋得到充分发挥,即孩子所能够达到的目标不能超过其先天所赋。

后天因素中还有一个不可忽视却常常不被重视的问题,那就是孩子成长的内在环境。家长们希望孩子达到的无论是行为规范方面,还是技能技巧方面都离不开健全的神经系统的工作。不要以为只有神经病的脑子不能正常工作。不健全的神经系统会在很大程度上影响到学习工作。常有的神经运作障碍如失眠、不能集中注意力等等可以发生在任何年龄,也不是只有典型的多动症才会这样。当孩子们被随便的冠以不听话、习惯不好或是遗传不良时是真的是贻误终生。

神经系统是否正常运作与我们的日常饮食正确与否有密切相关。因为不良的食物使人体神经系统的生物化学反应没有办法正常运作,以至于人们对于外界(学校、家庭、朋友等)的刺激比较不能承受应付。只是不同的孩子敏感度不一样罢了(包括成人)。曾获诺贝尔奖提名的Dr. Wallet甚至认为一个孩子在各方面的成就包括音乐、艺术、体能、学术成绩与教育及环境没有绝对关系,而与饮食有绝对关系。他认为许多所谓的坏孩子其实不需要精神科医生或心理医生的咨询。他们所需要的却是正确的营养观。

研究指出:因犯罪进监狱或是吸毒、酗酒的青少年中有60%是因为缺少维他命、矿物质而引起。一个典型的例子:美国的Jennifer Capriati,她在十三岁时就被预测为明日网球巨星。在十四、五岁时就夺得很多的网球锦标赛冠军,但最后她却成为一位吸毒、而且行为怪异的青少年。医生发现她是因为体内严重缺少矿物质,所以她要借着吸毒来使身体舒服。

那么饮食究竟对神经系统有何影响?什么才是神经系统的正确营养观呢?

・糖的过量摄取。孩子食用太多的糖,不但会影响牙齿,更会造成某些孩子神经系统失调,从而导致行为怪异、学习困难。研究发现小孩在食用高糖份后数小时体内会释放出大量的肾上腺素,会有颤抖、焦虑、兴奋及注意力不集中等问题产生。通过减少糖的摄取量,可以使75%以上“问题”孩子恢复正常。

・偏重动物食品、忽视植物食品也会造成神经系统失调。现在孩子食物中含有太多的动物蛋白质和脂肪。调查发现食用太少的植物食品(蔬菜水果)会造成情绪不稳。因为蔬菜中的钾有助于镇静神经、安定情绪;相反动物食品或盐中的钠会使神经兴奋。体内过多的钠会造成焦虑、情绪不宁。与之相比较,蔬菜中的钾则能帮助排除体内多余的钠。此外动物蛋白质呈酸性,食用过多会使体内的钙质和维生素B1减少,造成精神不稳定、头脑活力迟钝、血液循环不好。儿童的情绪不稳定大都是偏食所致。

・各种饮料中含有的咖啡因也是问题因素之一。咖啡因是一种毒品,它不但会产生停用后出现的头痛、肚子不舒服、沮丧、脾气暴躁等“脱瘾现象”(Withdrawal Syndrome),也会使体内的维生素B1降低。咖啡因又是一种很强的利尿剂,能导致体内的钾和锌随尿一起流失掉。

总而言之,当神经系统处于最佳工作状态时先天的禀赋才能充分发挥,后天的训练才能显著奏效!而神经系统达到最佳工作状态需要均衡的营养,需要鼓励孩子多吃新鲜蔬菜水果(而且要尽量生吃)、减少糖和肉类、远离垃圾食品。这对有些孩子来讲是自自然然的事情,但对有些孩子来讲却要花一番心思。千万不要轻易地走药物治疗或使用化学提炼的营养补充品(如维生素丸),那样对孩子的成长不是安全有效,会有很多副作用的。

Luck Li : 2009-03-04#491
回复: 加拿大教育情况有用素材收集帖子--梁溪香榭整理

公司和家里都收藏了,昨天已经看了一部分,记脑子里更踏实。

Luck Li : 2009-03-04#492
回复: 加拿大教育情况有用素材收集帖子--梁溪香榭整理

辛苦了,各位领导和梁溪。:wdb17::wdb17:

朱三雀 : 2009-03-04#493
回复: 加拿大教育情况有用素材收集帖子--梁溪香榭整理

收藏了。You've been a big help!

孩儿她爹 : 2009-03-04#494
回复: 加拿大教育情况有用素材收集帖子--梁溪香榭整理

仔细研读了5个小时才看完,真是太有用了,感谢楼主点火和众位拾柴!:wdb10::wdb10::wdb10:

zxllxz : 2009-03-05#495
回复: 加拿大教育情况有用素材收集帖子--梁溪香榭整理

谢谢!好贴!有用!:wdb17::wdb17:

梁溪香榭 : 2009-03-06#496
回复: 加拿大教育情况有用素材收集帖子--梁溪香榭整理

博士后和民工的区别

联合利华引进了一条香皂包装生产线,结果发现这条生产线有个缺陷:常常会有盒子里没装入香皂。总不能把空盒子卖给顾客啊,他们只得请了一个学自动化的博士后设计一个方案来分拣空的香皂盒。博士后拉起了一个十几人的科研攻关小组,综合采用了机械、微电子、自动化、X射线探测等技术,花了几十万,成功解决了问题。每当生产线上有空香皂盒通过,两旁的探测器会检测到,并且驱动一只机械手把空皂盒推走。

 
 中国南方有个乡镇企业也买了同样的生产线,老板发现这个问题后大为发火,找了个小工来说:你他妈给老子把这个搞定,不然你给老子......。小工很快想出了办法:他在生产线旁边放了台风扇猛吹,空皂盒自然会被吹走。

梁溪香榭 : 2009-03-06#497
回复: 加拿大教育情况有用素材收集帖子--梁溪香榭整理

(跑题) 犹太人的教育风格:朴实无华 讲求实际

在世界民族之林中,至少有4个国家的历史教育应该引起人们的重视。它们分别是以色列、韩国、德国和日本。以色列人能够非常出色地记住自己曾经遭受的苦难,并从苦难中发奋图强;韩国人能够记住日本曾经带给他们的野蛮行径,并从中寻找自己的民族尊严;德国人能够坦诚地面对自己当年的罪恶,并从罪恶中进行彻底的反省;日本不但能够记住自己遭受原子弹的袭击,还能轻易地忘记对别人的蹂躏。

  犹太人是世界上惟一被打散而脱离原居住地,又在2000年后恢复聚集地的民族。这样的历史是世界民族史上是空前绝后的,对以色列来说是非常重要的。以色列人常说“我们的历史就是世界的历史,《圣经》里说得很清楚:从洪荒开始”。

  根据以色列立国时的《义务教育法》,5岁?18岁的以色列人都免费接受义务教育。一般来说,以色列人从9岁开始接受历史教育,10岁开设历史课。图文声像俱全的教育方式,使得这些以色列孩子对2000年前以色列人被罗马人逐出家园、对到二战时遭到大屠杀的历史都没齿难忘。

  中国人从小教育的就是天文地理,而“以色列没有地理,只有历史”。因为以色列是个地域非常狭小的地方,但几乎每一个地方都堆积着非常厚重的历史。比如说,以色列北部有一个叫泰尔(音译)的一个地方,这里的历史大约可以堆积四五十层。这里的统治者曾被推翻、被征服的历史事件特别多。

  在以色列,无论是世俗学校,还是宗教学校,还是官方学校,还包括社会的各个群体,他们都非常重视历史教育。即使没有文化的人,其对自己民族的历史遭遇都记得一清二楚。以色列的学校时常把学生拉到“野外课堂”去接受历史教育,去亲眼感受当年的历史古迹。以色列人在国内旅游所看到的也几乎全是历史。“以色列人非常清楚地知道,如果以色列不记住历史,他们还会落到其祖先那样的下常”

  中国军人有时去烈士陵园凭吊历史,而以色列军官学校则把军官拉到马塞达。在死海边,有一座相对特别高的山,山上有一个大城堡名叫马塞达。公元70年前后,以色列的耶路撒冷等城堡都被攻陷,只剩下马塞达。一万名罗马人的奴隶士兵曾经围困马塞达长达3年之久而无法攻破城池。在城堡行将陷落时后,守城堡的以色列人认为,反正都是死,倒不如为了不当罗马人的奴隶而集体自杀。

  公元73年4月15日,所有守城的人经过抽签只留下10个男人。由这10个男人把所有守城的人都杀死,然后再自杀。这当中,只有3个女人逃跑了,她们遂向罗马人报信,并把这段历史留传了下来。罗马人最后进入城堡时发现,马塞达只剩下一堆废墟。现在的以色列军官学校学生都是步行爬到山顶的马塞达,并庄严地宣誓一句话:“马塞达绝不会再陷落”,意指以色列绝不会再遭到灭亡。由此可见以色列人顽强的作战精神和生存意识。

  殷罡先生还认为,以色列与中国有两个非常重要的相同之处:第一,以色列人重视传统教育,比如孝道等。第二,以色列有比较强的家庭纽带。历史上对犹太人的迫害非常厉害。在欧洲,就有法律明文禁止犹太人不能拥有不动产。这就意味着,当犹太人失去享有不动产的权利后,惟一的财富就是头脑中的智慧。犹太人只好干别人不干的事情。因此,在欧洲历史上,凡是做“抽象概念”事情的人全是犹太人。

  在这种条件下,犹太人非常重视教育,特别是基础教育。因此,在人们的印象中,犹太人走到哪里,他们都是高智商的典型代表。在以色列,开拖拉机的、养鸡的全是大学毕业,最次的也是高等职业学校毕业(类似于中国的大专文凭)。犹太人无论“流浪”到世界什么地方,他们总是带着观念来,带着观念去。他们也非常清楚不动产是无法守住的。中国人也常说,富不过三代。

  以色列教育的根本前提是:一切教育就是为了以色列的民族生存,也就是“爱祖主义”。所以,以色列法律规定,学生高中毕业以后,只要身体没有残疾,只要没有病,只要不是同性恋,只要没有宗教方面的特殊限制,就必须当兵。以色列人口中的70%男女高中生毕业后都必须当兵,服完兵役后再去上大学。因此,以色列的大学生大都显得比较“老成持重”。正是由于其“老成”,他们的学习观念与那些从校门到校门的学生有着本质的区别,他们的创造力也极其出色。

   据悉,以色列教育经费从3岁开始就由政府拨款。现在的以色列工党和利库德集团两党竞争的重点并不在于如何发展高等教育和培养高级人才,而是如何发展幼儿教育和基础教育。很显然,他们的目的就是想永远走在世界民族之林的前列。此外,殷罡还认为,以色列的工程师和高科技人员在以国内“根本就用不完”,而且,他们的政治素质都特别高,随便找一个人侃大山,什么政治、经济、对外关系,人人都有一套,而且都是中国老百姓所侃不出来的。

  特别值得注意的是,犹太人的教育抛弃了很多浮华的东西,比如讲究这个,讲究那个。犹太人并不看重表面的东西。这从以色列的仪仗队就能看得出来。以色列仪仗队个子高矮、胖瘦都参差不齐。以色列总理的卫士长在陪同总理内塔尼亚胡访华时,在人民大会堂东门外广场上穿的就是一件非常休闲的军夹克,而且“眼还不正”。但是,就是这种人能打仗。在达扬任以色列国防部长并去美国参观军校演习时,他对美国的军校生冷眼相看:这样的人能打仗吗?

  这也从另一个方面表明,以色列人把时间都花在特别有效率的事情上,不做废事儿。中国人往往把家弄得特别复杂,而在以色列特别有身份的人家中,家庭内的装饰也非常简单:“徒有四壁,偶有点缀”。以色列总理办公室也不过是两层的小楼,总统府也就是一个单独的小院。因此,中国人要学习以色列只能学习它的一些原则,有些东西是中国人学不来的。

  以色列人分散到世界各地,但形散神不散。中国人的唐人街满世界都是,但都没有对当地政府形成强大的压力集团,而以色列人在美国的院外活动集团的力量却“非常令人恐怖”。

梁溪香榭 : 2009-03-06#498
回复: 加拿大教育情况有用素材收集帖子--梁溪香榭整理

(接着跑题) 培养子女的参政意识

华人在海外有这样一个明显的美德,即守法、勤勉。华人子女的学习一般都比较突出,人生的轨迹基本是,好学生---好职员或好商人。在经济和科技上,华人有较大贡献,但在政治贡献方面却远不符合一个大族裔的地位,分析一下,无非有以下原因,令海外华人的政治兴趣降低。

一、 客居意识。华人对居住国的认同观念差,由于少数民族的地位,常处于被边缘化的地位,再加上语言、文化的差异、许多华人对居住国的政治运作、政坛动态知之甚少,很难产生“天下兴亡,匹夫有责”的使命感,于是,对政治事务就产生了一种疏离感。
二、 厌恶政治,由于受中国历史文化和现实生活的影响,政治留给人的印象,往往是政客间的勾心斗角,尔虞我诈,政治斗争既不正义,又充满险恶,仅是厚黑者假公济私、成王败寇的权术之争,因此,许多人往往不屑于仕途。
三、 政治与我无关。由于移民海外的华人,多为知识分子、商贾、工农,基本是一个对政治不甚熟悉的群体,许多人对政治采取敬而远之的态度,子女受其影响,也是专注于学习,走科技和商务之路,再不济,就做一个工人,也图个三餐无忧,清静本分,久而久之,海外华人中便形成了一种不问政事的文化。

从现代政治观念来讲,淡漠的从政意识无疑是我们海外华人的一大弱点,提高华裔的政治意识并从孩子抓起,其意义将是很长远的。

首先,参政意识的培养,可以使小孩主动将自己与社会联系起来,培养起全局观念及责任感、使命感,大可以治国平天下,小可以修身齐家,对于孩子健康人格的形成至关重要。历史上许多杰出人物的成功,其动力都是将自己与社会国家民族的利益联系在一起的。
其次,作为一个社会的人,其成功不仅在于自己的才干,也决定于对自己是否有利的社会环境,勇敢地表达自己的意愿,并参与到社会规则的制作中,既是自己社会人身份的一种体现,也是自己思想观念的一种表达,更是对自我实际利益的一种保护,如果上升至国家法律和政治运作体系,则更与族裔的长远利益休戚相关,最近加拿大人头税的平反就是华人积极参政的直接结果。
第三、政治贡献是一个民族受人尊重的重要因素,一个民族的地位,不仅取决于自身的文明水平,也取决于其社会贡献,其中政治贡献是尤其重要的,因此华人在居住国的地位,直接受本族裔政治参与度的影响,每个家长应有意识鼓励孩子积极参与到国家的政治建设中去。

至于家长如何培养孩子的参政意识,以下几点无疑是很重要的。

首先要培养孩子正确的参政观念,即政治是每个人共同的事业,每个人都在其中有自己的利益,每个人也都有关心政治的责任。政治本身就是人生的一个重要组成部分,从政不仅是个人事业的一个选项,也是社会的基本需要。政治有时候是很肮脏的,但杰出的政治家在肮脏的政治环境中人生会更有光彩,马英九父亲希望马英九活在历史上的有关名言,值得每个父母亲深思。

参政能力的培养应该从基本的集体活动、社会活动开始,从学校、社区的公益事业做起,养成良好的公共意识、公德意识和公关能力。加拿大现任总理哈帕从政就是从学生时代开始的。

家长的示范作用十分重要。身教重于言教,家长除关心家事之外,也应该多多关心国事、天下事,培养孩子推己及人的社会道德感和以天下为己任的使命感。从孟母择邻、岳母刺字等一系列人人耳熟能详的历史故事中,我们就可以知道家长行为对孩子成长的巨大影响。

梁溪香榭 : 2009-03-06#499
回复: 加拿大教育情况有用素材收集帖子--梁溪香榭整理

美国名校点评

哈佛大学:“先有哈佛,后有美利坚”

哈佛大学:常青藤盟校之首,美国东海岸最富盛名的大学,创办于1636年,位于马萨诸塞州查尔斯河畔的剑桥市,河的对岸是名城波士顿。哈佛大学医学院则在波士顿市内。

哈佛的校训是「AmicusPlato,AmicusAristotle,SedMagisAmicusVERITAS」。它是拉丁文,中文是「与柏拉图为友,与亚里士多德为友,更要与真理为友」。它是哈佛学生学习和为人的准则。

美国素有“先有哈佛,后有美利坚”之说。1775年至1783年的美国独立战争中,马萨诸塞州几乎所有著名的革命者都是哈佛的毕业生,包括美国《独立宣言》起草人之一、美国第二任总统约翰*亚当斯。1775年7月,华盛顿将军在哈佛的华滋瓦斯(Wadsworth)楼
,临时指挥部所在地,领导其军队打败了英军。1776年美利坚合众国诞生后,哈佛学院给独立战争的主要领导人乔治*华盛顿、托马斯*杰斐逊(《独立宣言》主要起草人,美国第三任总统)、约翰*杰伊(美国联邦最高法院首任首席大法官)、亚历山大*汉密尔顿(首届美国联邦政府财政部长)等人授予荣誉法学博士学位。

哈佛有许多影响重大的科学成果,如一百多年前外科麻醉手术;四十年代发现的核磁共振,现已广泛用于化学和医学研究;五十年代首创了器官移植的新方法,并成功地进行了第一次人体肾脏移植;六十年代提出有机合成化学的理论和技巧,并首次人工合成了维生素B12;九十年代发明新的太阳能转化电能材料和二零零零年一种十分有效的抗癌药的合成等等

。哈佛校友中有41名诺贝尔奖获得者、5名图灵奖得主和34名普利策奖获得者,33位教授荣获过国家科学奖章(全美第1),4位教授荣获过Crafoord奖(全世界第1),对美国的政治、经济、科学、文化都起到重要的作用。同时,对世界其它国家也有一定的影响。中国近代有一批著名学者,科学家曾在哈佛学习过,如陈寅恪,竺可桢,杨杏佛,梁实秋,梁思成,赵元任等。哈佛培养出无数优秀的毕业生。有七位美国总统毕业于哈佛,如富兰克林.罗斯福,约翰.肯尼迪及现任总统乔治*沃克*布什等。哈佛还为美国培养了无数的商界领袖:SUN的首席执行官ScottMcNealy,微软首席执行官、盖茨在哈佛的室友
史蒂夫*鲍尔默(SteveBallmer)等等都是哈佛毕业生。在哈佛退学学生中,也有许多杰出人才。如石油大王霍华德.休斯(HowardHughes),继爱迪生之后美国最出名的发明家、 有五百多项专利的波尼.莱特(BonnieRaitt),创立美国历史上第一个乡村乐队「国际潜水艇乐队」的名歌手格兰姆.帕森斯,以及当今社会的风云人物电脑大王比尔.盖茨(BillGates)。哈佛现任教授中,有151位国家科学院院士、14位国家工程院院士和98位美国医学科学院院士,名列全美第一。

除工科外,哈佛的其它学科都属超强。工科之弱也只是相对于其它几所最顶尖的学校如斯坦福、柏克利和麻省理工,在常青藤八校中却是仅次于康奈尔。医学院在美国堪称“独孤求败”,雄倨榜首多年,岿然不动,商学院、法学院也绝对是美国最好的之一。共28个专
业被NRC评为全美前10名。

商学院第1,医学院第1,教育学院第1,法学院第3,工程学院第17,公共事务学院第1, 公共卫生学院第2; 理科类综合第5:生命科学第2,化学第2,计算机第20,数学第2,物理第3,地质学第7;
人文社科类经济学第2,英语言文学第1,政治学第1,历史学第8,哲学第3,心理学第6,社会学第7;
NRC排名:艺术人文类第3,生物科学类第3,数理类第5,社会科学类第4。

梁溪香榭 : 2009-03-06#500
回复: 加拿大教育情况有用素材收集帖子--梁溪香榭整理

美国名校点评(2):斯坦福大学
斯坦福大学:硅谷精神

斯坦福大学:美国精神的代表,随硅谷的兴起而崛起,美国西海岸最声誉卓著的私立大学
,由铁路大王利兰*斯坦福于开始1885年创办,1891年正式开学,占地8800英亩。

来到斯坦福大学,首先看到的是土黄色石墙环绕下的红屋顶建筑,拱廊相接,棕枷成行,
在古典与现代的交映中充满了浓浓的文化和学术气息。这个中心广场是斯坦福的主要部分
,在它的四周,商学院、地学院、教育学院、工学院、法学院、医学院等星罗棋布。再往
外,就是斯坦福科学园区、植物园、高尔夫球场和若干个科学试验场。斯坦福大学中最有
名的建筑是斯坦福纪念教堂。

斯坦福校友中的诺贝尔奖得主有35位,图灵奖有8位,生中比较著名的有第31任总统胡佛、
诺贝尔文学奖获得者J.斯坦贝克和最高法院第一个女法官S.奥康纳等。现任教授中有17位
诺贝尔奖得主(全世界第1),21位国家科学奖章得主(全美第1),120位国家科学院院士、81
位国家工程院院士和48位美国医学科学院院士。

创业是斯坦福大学的一种重要文化,从硅谷开始兴起的早期,斯坦福大学就开设了创业课
程,从理论上给企业家进行指导。一些著名的创业家,如擅长销售的AMD公司创始人桑德斯
(伊利诺依大学香槟分校毕业),不断另起炉灶的“创业狂”安戴尔,电子游戏工业的泰斗
布什内尔等都是在这里起家和成长的。

1955年威廉*肖克利(WilliamShockley)西迁是硅谷半导体开始起步的重要里程碑。
1947年12月23日肖克利和理论物理学家巴丁(JohnBardeen)、实验物理学家布拉坦(Walte
rBrattain)制成了世界第一个晶体管,这项发明有人称之为“本世纪最重要的发明”。1
949年肖克利又提出PN结理论,次年就制成具有PN结的锗晶休管,由于这些意义深远的发明
,他们3人分享了1956年度的诺贝尔(物理)奖金。1955年,肖克利返回帕拉托建立自己的
公司??肖克利半导体实验室(ShockleySemiconductorLaboratories)。在此之前,尚
未成熟的半导体工业一直集中在美国东部的波士顿和纽约长岛等地,肖克利的公司是硅谷
第一家真正的半导体公司。他从东部召来了8位优秀青年,人称“肖克利八杰”,其中包括
诺宜斯(RobertNoyce)、摩尔(GordonMoore)、斯波克(CharheSpork)、雷蒙德(Pierre
Lamond)、尤金*克莱纳等人。1960年肖克利卖掉自己的公司,去斯坦福任教。他创
建的半导体实验室夭折了,但他播下的种子却在硅谷茁壮成长。“肖克利八杰”中的人日
后陆续创办的仙童半导体公司(FairchildSemiconductorCorp),国家半导体公司(Natio
nalSemiconductor),英特尔公司(Intel),AMD公司在世界半导体产业界都是举足轻重,
叱吒风云。1975年斯坦福毕业生梅隆(RogerMellon)建立了克罗门克公司(Cromemco)。
1982年,又一个光辉的名字SunMicrosystems在硅谷出现。 Sun Microsystems原来是斯坦
福大学网络的缩写。三位创始人Andy Bechtolsheim、Vinod Khosla、Scott McNealy都是
风华正茂的青年。担任公司董事长兼总裁的Scott McNealy毕业于哈佛大学经济系,后来又
在斯坦福取得管理硕士学位。CISCO,SGI等公司的创办人,也都是斯坦福教授。斯坦福毕
业生在硅谷的风云人物还包括早期的惠普公司的创办人休利特和帕卡德,近年崛起的
Google创始人谢尔盖*布林和拉里*佩奇,雅虎的创办人杨致远和大卫*费洛,eBay公司的前
副总裁Jeff Skoll,英特尔现任总裁兼首席执行官贝瑞特等等。微软首席执行官史蒂夫*鲍
尔默(Steve Ballmer)也是斯坦福的退学生。斯坦福的师生对硅谷的发展功不可没。

斯坦福各科俱是超强,美国没有其它任何一所学校有如此全面、且每个学科都可排在前10
的非凡实力。医学院稍逊于哈佛,但工学院远胜。其它文理商法等诸科也足以和哈佛分庭
抗礼,两所私立大学一东一西,可谓一时瑜亮,均代表了美国私立大学的最高水准。共32
个专业被NRC评为全美前10名。

商学院第2,医学院第8,教育学院第2,法学院第2,工程学院第2;
理科类综合第3:应用数学第4,生命科学第1,化学第5,计算机第1,地质学第3,数学第
2,物理第3;
人文社科类经济学第2,英语言文学第1,历史学第3,政治学第2,心理学第1,哲学第5,
社会学第5;
NRC排名:艺术人文类第7,生物科学类第1,工程科学类第3,数理类第8,社会科学类第2。

梁溪香榭 : 2009-03-06#501
回复: 加拿大教育情况有用素材收集帖子--梁溪香榭整理

美国名校点评(3):加州大学柏克利分校
加州大学柏克利分校:“让这里光芒闪耀”

加州大学柏克利分校:加州大学的领袖,美国公立学校王中之王,西海岸最声誉卓著的公
立大学,1868年创建于奥克兰市,1873年迁至伯克利市。伯克利占地1230多英亩。它从伯
克利城区一直延伸到林木覆盖的伯克利山麓。校园内终年绿树成荫,草坪如毯,花开不谢


伯克利的校训是“让这里光芒闪耀”。如果说美国形成史是一部由东向西不断开发的历史
,那么,伯克利加州大学就是美国"西部大开发"中的"智力开发"基地。这所公立大学与私
立的斯坦福大学、加州理工学院共同支撑起与美国东部常春藤大学相比肩的西部学术脊梁
。加州伯克利大学历史上共有22位教授或校友获得过诺贝尔奖,8位教授或校友获得过图灵
奖,9位教授荣获过沃尔夫奖(世界第1),25位教授荣获过国家科学奖章,目前任教的教授
中有7位诺贝尔奖得主、3位普利策奖得主、124位国家科学院院士、68位国家工程院院士和
10位美国医学科学院院士。

伯克利校风极其活跃开放,鼓励文化、生活方式和思想的多元,甚至带有强烈的反主流的
倾向。六十年代美国青年的Counterculture运动中,Berkeley就是一个重镇。如今的美国
社会一有风吹草动,伯克利学生立即作出反应,以各种方式表达自己的观点,而且有趣的
是常常与官方导向唱对台戏。

伯克利分校设有许多重要的研究机构,劳伦斯伯克实验研究中心是享誉世界的物理学研究
中心,自1929年开展研究工作以来,该中心发明和发展了三种类型的加速器??螺旋
加速器、直线加速器和同步加速器,先后有8人获诺贝尔奖金,其中创始人劳伦斯因发明
螺旋加速器和研究人造放射性于1939年荣获诺贝尔奖金。另外,绝大部分不稳定基本
粒子和全部已知的14种超铀元素都是在这里发现的。研究中心还第一个解决了光能合成
的基本步骤,是核医学领域的开拓者。这里现已成为核科学和生物、医学、重离子方面的
国际研究中心。1943年创建的阿拉莫斯科学实验室,是美国研制核武器的重要基地。
它对美国第一颗原子弹和第一颗氢弹的研制做出了重要贡献。著名物理学家、美国原子弹
之父J.罗格斯*奥本海默就是这个实验室的杰出科学家。伯克利分校正是以其显赫
的研究成果和拥有大批优秀的科学家而闻名于世。

柏克利没有医学院,可是近在咫尺的加州大学的另一所分校旧金山分校却是以医学和生命
科学为主而闻名遐迩,加大旧金山分校的生物类学科综合名列全美第5,医学院名列第6,
护理学院第2,实力实在可以用“超强”来形容。

除医学院外,柏克利其它各学科可谓百花齐放,且都在极品之列。共36个专业被NRC评为全
美前10名。

商学院第7,教育学院第11,法学院第10,工程学院第3,公共事务学院第4;
理科类综合第2:应用数学第4,生命科学第2,化学第1,计算机第1,地质学第3,数学第
2,物理第3;
人文社科类经济学第6,历史学第3,英语言文学第1,政治学第2,社会学第1,哲学第4,
心理学第6;
NRC排名:艺术人文类第1,生物科学类第5,工程科学类第2,数理类第1,社会科学类第1。

梁溪香榭 : 2009-03-06#502
回复: 加拿大教育情况有用素材收集帖子--梁溪香榭整理

美国名校点评(4):麻省理工学院
麻省理工学院:无私的科学精神

麻省理工学院:理工科,尤其是工科无可替代的龙头老大,天才诞生和成长的天堂。1861
年由杰出的地质学家罗吉斯创办于波士顿,1916年迁到剑桥,在著名的查尔斯河河畔。

麻省理工学院历史上共有55位教授或校友获得过诺贝尔奖,5位教授或校友获图灵奖,21位
教授荣获过国家科学奖章,目前任教的教授中有101位国家科学院院士、108位国家工程院
院士和26位美国医学科学院院士。麻省理工众多诺贝尔奖获得者中,值得一提的是Walfga
ngKetterle和他的导师的故事。1990年Ketterle从德国来到MIT物理系,Pritchard教授
小组做博士后研究,参加了Pritchard主持的玻色爱因斯坦凝聚(BEC)研究工作。为使
Ketterle能专心致力于这项研究项目,Pritchard建议学校给Ketterle助理教授位置。但是
,同一系里不能有完全相同研究项目的教授。于是,Pritchard让Ketterle接手BEC,自己
退出这项重要项目,选择其它研究方向。Ketterle得到Pritchard和MIT其它物理教授的大
力支持,BEC研究工作非常成功,首次在实验上实现了BEC,因此在2001年获得诺贝尔奖。
获奖后,Ketterle将诺贝尔金质奖章送给他的导师Pritchard,一位同样配得上该奖的科学
家。真正的一流科学家的无私和贡献精神,在当今社会中已不多见,庆幸的是它仍在MIT保
留著。

麻省理工学院在科学研究的许多领域,如电子学、核科学、航空学和航天学、计算机和技
术、光谱学、太阳能、生物学、食品工艺学、核工程、造船学、伺服机构、高压电工程、
化学工程等方面,都曾取得重大的成就,特别是对通讯、计算机技术、惯性制导系统方面
的研究成果,对美国军事设施有极大作用。MIT有许多重大科技成果:辐射实验室研制出雷
达,首次合成了盘林西林和维生素A,研制出人造皮肤,被用于烧伤病人的治疗,发现J粒
子,证明了夸克的存在,完成了一系列实验,证实了质子和中子是由夸克组成的理论,首
次鉴定出人的肿瘤基因。开发出导致计算机发展的磁核记忆新技术等等。从97年的一份报
导,可以看出MIT对美国社会经济的影响。1994年,MIT毕业生创办了4千个公司,有110万
名员工,营业额达到2320亿美元。如果将这些公司算成一个独立国家,它的经济在世界上
排第24名,其总产值(GDP)比南非稍少一点儿,而比泰国还要高。

除了理工科外,麻省理工的斯隆商学院、社科、设计类也很厉害。美中不足的是MIT缺少美
国主流社会所崇尚的法科和医科,在学科的综合性上逊于同在麻省剑桥市的哈佛大学。所
拥有的大部分专业都被NRC评为全美前3。

商学院第4,工程学院第1(12项中7个第1);
理科类综合第1(7项中4个第1):应用数学第1,生命科学第2,化学第2,计算机第1,地质
学第2,数学第1,物理第1;
人文社科类经济学第1,哲学第10,心理学第11;
NRC排名:生物科学类第2,工程科学类第1,数理类第2。

梁溪香榭 : 2009-03-06#503
回复: 加拿大教育情况有用素材收集帖子--梁溪香榭整理

美国名校点评(5):加州理工学院
加州理工学院:小而精

加州理工学院:小而精的西海岸杰出代表。有了它,麻省理工学院不再孤独,建于1891年
,座落在加州巴萨迪那市(Pasadena),占地仅一百二十四英亩,宗旨是“为教育事业、
政府及工业发展需要培养富有创造力的科学家和工程师”。

加州理工学院校友中的诺贝尔奖得主有28位,图灵奖有2位,23位教授荣获过国家科学奖章
。现任教授中有近百位美国科学院、工程院及医学科学院院士。1958年其喷气推进实验室
与国家航空航天局合作发射了美国第一颗人造卫星。爱因斯坦是在这儿放弃了他的“宇宙
不变论”而认可了“宇宙扩展论”;物理学家卡尔*安德逊在这里发现了阳电子;也
是在这里,性格外向、诙谐幽默的诺贝尔化学奖得主理察*费尔曼几十年如一日地授
课,成为学生们最崇拜的教授之一,我国著名科学家钱学森也是加州理工的校友。现在,
这里还有诺贝尔化学奖获得者鲁道夫*马尔克斯(RudolphMarcus),诺贝尔物理奖
获得者莫利*吉尔曼(MurryGellmann),和一九九七年荣膺诺贝尔生物奖的艾德&
#8226;路易斯(EdLewis)等学术界的超级大腕。

加州理工最出名的还有该校的“逃学日”―学生们在这天挖空心思搞“恶作剧”,创下了
一项项令人瞠目结舌的记录。通常在春天,一群即将毕业的四年级学生会选定一天大闹校
园:丢下面对空荡荡教室苦笑的教授,结伴到海滩去玩;往房间里灌满水然后把所有家具
都钉在墙上;把一辆老爷车开到睡床边;要不就把房间竟变成一片“沙漠”,还栽上几棵
棕榈树。最经典的莫过于行将毕业的学生与低年级学生“打垒”:前者运来几吨沙、水泥
、铁板把宿舍门堵死;后者也非等闲之辈,他们找来电锯、搬运机,手持式凿岩机等“重
型”工具迅速打开房门。公开承认并容忍学生在逃学日“胡作非为”全美只有两所大学,
那就是麻省理工和加州理工。在恶作剧方面,加州理工和麻省理工堪称比肩而立,傲视群
雄。

加州理工学院理工科超强,学科综合性上稍显薄弱。但其所遵循的本来就是小而精的办学
模式,在西海岸名校中独树一帜。相比起来,麻省理工学院还算是注重人文社会科学与科
学技术二者相结合的教育方针,但在加州理工学院这边看起来是“胆小鬼”的把戏。“我
们是‘理工学院’,科学是这儿的主题。”曾有加州理工学院的学生如是说。使加州理工
学院最出名的系是物理,直到今天这个系还吸引着全国各地的物理高材生趋之若鹜。工程
,尤其是电子工程,以及化学、生物、植物学、天文和地质,都非常引人注目。

工程学院第7;
理科类综合第4:应用数学第3,生物科学第5,化学第2,计算机第10,地质学第1,物理第
1,数学第8;
人文社科类经济学第14;
NRC排名:生物科学类第12,工程科学类第4,数理类第3。

梁溪香榭 : 2009-03-06#504
回复: 加拿大教育情况有用素材收集帖子--梁溪香榭整理

美国名校点评(6): 普林斯顿大学

普林斯顿大学:常青藤盟校之一,小而精的东海岸杰出代表,1746年创办于新泽西州普林
斯顿。

该校位于纽约和费城之间的一座与众不同的乡村都市,那里到处绿地丛丛,洋溢着一派和
平、生气勃勃的气氛。小城普林斯顿位于新泽西州西南的特拉华平原,面积约7平方公里,
东濒卡内基湖,西临特拉华河。这里的景色十分幽雅,四周绿树成荫、绿草从从,清澈的
河湖环绕著这座小城静静地流淌,这里交通方便,距纽约和费城只需1小时车程,加之小城
恬静、安祥的生活,浓浓的文化氛围笼罩下的贵族气息,因而成为美国上层人士最倩睐的
生活和居住地。座落在这座小城镇里的普林斯顿大学因此被认为是全美国最漂亮的大学之
一。

普林斯顿大学的校训:“普林斯顿为了给国家服务” 。在常青藤各校中,普林斯顿是
最重视传统,讲究门第的一所。当自由派思潮在其它的美国大学校园里汹涌的同时,普林
斯顿学生们的政治倾向却是保守的温和派。普林斯顿的费兹兰道夫门(Fitz Randolph Ga
te)据说是永远打开但不得通过的门,每年的新生都列队走入这扇门,加入普林斯顿大学
这个集体,而每年的毕业生们都会列队走出大门,象征着他们离开了学校走向社会。但据
说在入学到毕业的四年中间,任何走过这扇门的学生都会毕不了业。尽管人人都知道这只
是一种迷信,但绝大多数普林斯顿学生还是“宁可信其有”,在四年中不迈过费兹兰道夫
大门一步。

作为“常春藤联盟”大学的三巨头(另两家为哈佛大学和耶鲁大学)之一,这所大学在学术
上享有极高的声誉。在普林斯顿大学258年的建校史上,出过不少星光灿烂的人物,对美国
的社会文明做出过很大的贡献,从这所学校里走出过大批的科学家、文学家和政治家。普
林斯顿的教授和校友中诺贝尔奖得主有31位, 其中物理奖18位,图灵奖得主有7位,15位教
授荣获过国家科学奖章。普林斯顿的教授获过菲尔兹奖的人数居世界第一。著名的相对论
大师爱因斯坦爱因斯坦在这里度过了生命中最后的二十二年时光,当代最著名的大诗人之
一T.S.艾略特本世纪初在此沉溺于冥想玄思,数学大师冯诺依曼、诺贝尔物理奖得主华裔
科学家崔琦等都在这里从事过研究。著名的科学家华罗庚、姜伯驹、中国科学院外籍院士
陈省声、李政道、杨振宁都曾担任过普林斯顿大学的高级研究院研究员。普林斯顿大学还
为美国培养了J.麦迪逊总统和W.威尔逊总统两位总统、有1000多名普林斯顿大学的毕业
生先后担任过美国国会参议员、众议员、联邦政府的高级官员,以及州长和州政府的高级
官员。由此,普林斯顿大学赢得了“美国政治家摇篮”的誉称。

普林斯顿大学的尊崇地位的标志之一就是每逢五十年校庆,一定请到当时在任的总统前来
演讲两百五十年校庆时来此发表演讲的克林顿总统明白:这是普林斯顿大学给在任总
统一个极大的面子,而不是相反。普林斯顿大学的毕业生中,还有小品家F.S.菲茨杰拉
尔德、美国著名作家、《在天堂的另一边》的作者F.斯科特*费兹杰拉德(F.Scott
Fitzgerald),电影《美丽心灵》所演绎的诺贝尔经济学奖得主博弈论大师约翰*
纳什等等。现任教授中有70多位美国科学院、工程院及医学科学院院士,7位诺贝尔奖得主
。著名的历史学家余英时、经济学家邹至庄等都在这里任教。

普林斯顿大学以重质量、重研究、重理论的传统享誉世界,普大坚持学术至上原则,至今
,学校没有开设社会上最热门的学科:法学、商学、医学,这与习俗追求以及社会时尚完
全不同。普大的任何一个专业在全美大学都是名列前茅,你很难找出有哪个系不够好。普
林斯顿大学的研究课题基本上以理论研究为主,甚至连工科各系也都是如此。普林斯顿大
学的等离子实验室是美国惟一的研究核聚变的国家实验室,普林斯顿大学最大的科学研究
机构佛洛斯特研究中心则在喷气推进气体动力学、航天导航、气象和海洋等领域取得过一
系列重大成果。

与同样是小而精的加州理工比,普林斯顿理科堪与争锋,工科较弱,但人文学科远胜。理
科强而工科稍弱(只是相对斯坦福等),几似乎是除康奈尔外所有常青藤都存在的问题。普
林斯顿大学的数学、物理和哲学闻名遐尔,英语、政治、天体物理、历史、古典文学、经
济、美术、音乐、德语和法语等也在学术界备受推崇;工程类以化学工程、机械工程,航
天工程为最佳。威尔逊公共和国际事务学院的宗旨就是培养公共事务方面的领导人才。该
学院承担着众多的民间和官方的相关课题,在人口研究和国际关系研究方面具有相当大的
影响,许多著名的外交家、政治家和政府高官们出自这所精英学院。普林斯顿的写作项目
也非常优秀。一九九三年诺贝尔文学奖获得者,黑人女作家托尼*莫里森(Toni Mo
rrison),和从五十年代起活跃在美国文坛上的著名女作家乔伊丝*卡罗*奥
兹)Joyce Carol Oates),都是普林斯顿英语系的骄傲。1998年,普林斯顿校友、香港建
筑大亨胡应湘给普林斯顿大学工程学院捐赠了一亿美元,使该校工程学院有了更上一层楼
的经济力量。

工程学院第18,公共事务学院第4;
理科类综合第6:应用数学第4,生命科学第9,化学第14,计算机第9,数学第2,物理第3
,地质学第11;
人文社科类经济学第2,历史学第1,哲学第1,心理学第11,社会学第9;
NRC排名:艺术人文类第2,工程科学类第5,数理类第4,社会科学类第8。

梁溪香榭 : 2009-03-06#505
回复: 加拿大教育情况有用素材收集帖子--梁溪香榭整理

美国名校点评(7): 耶鲁大学
耶鲁大学:“光明与真知”

耶鲁大学:常青藤盟校之一,也是美国上流社会最向往的学校之一,1701年创建于康涅狄
格州纽黑文,校园占地面积175英亩。漂亮的歌特式建筑和乔治王朝式的建筑与现代化的建
筑交相互映,把整个校园点缀得十分古典和秀丽。一座座直逼蓝天的钟楼、高塔巍峨坚定
;满园浓艳似火的丹枫,在烘托着人们对于真理、知识的热切向往和执著探求。每当耶鲁
的象征哈克尼斯塔悠扬的钟声和小教堂管风琴的合鸣在纽黑文上空萦绕回旋,一种对
于这座人文科学殿堂的景仰之情就会在人们心头油然而生。

在耶鲁大学的校徽上,以拉丁文写就的“光明与真知”,历经几个世纪的风风雨雨,依然
焕发出幽远的光芒。因为出了7位总统,耶鲁大学被称为美国总统的摇篮。美国最近三任总
统都是耶鲁大学校友,这在美国是史无前例的。1789年以来的美国内阁中,9%的成员来自
耶鲁;十余位美国最高法院大法官都曾在耶鲁学习。担任美国企业领导的耶鲁校友,数量
远远超过其他大学;耶鲁毕业生成为众多著名大学的创始人或第一任校长,如普林斯顿大
学、康奈尔大学、约翰*霍普金斯大学、哥伦比亚大学、芝加哥大学等,并因此将“
美国学院之母”的桂冠奉献给自己的母校。

此外,耶鲁还造就了灿若群星的各界知名人物:电报的发明者摩尔斯,词典编纂学家诺亚
*韦伯斯特,著名影星梅丽尔*斯特里普、保罗*纽曼、朱迪*福斯特,《时代》周刊的著名
专栏作家凯文*翠林等。还有那位著名的美国民族英雄内森*黑尔,他的铜像伫立在耶鲁丹
枫似火的校园,那句广为传颂的名言:“我唯一的憾事是没有第二次生命献给祖国”,据
说是出自他在耶鲁修习的拉丁文课。容闳、著名铁路工程专家詹天佑、经济学家马寅初、
教育家晏阳初、英国语言文学专家李斌宁等一位位闻名遐迩的大师学者都先后负笈耶鲁。

耶鲁大学历史上共有5位教授或校友获得过诺贝尔奖,1位教授或校友获得过图灵奖,9位教
授荣获过国家科学奖章, 目前任教的教授中有超过100位国家科学院、国家工程院和国家医
学科学院院士。耶鲁的教师还包括卓有成就的历史学家、文学批评家、科学家、工程师、
艺术家、诗人等,他们中不少是诺贝尔奖、菲尔茨奖、普利策奖的获得者。

耶鲁以盛产政治家闻名,因为耶鲁学生有关心政治的传统。校内有许多政治组织,持不同
观点甚至对立观点的组织和平相处,如有组织严密的"青年共和党人"和"青年民主党人"以
及最活跃的"耶鲁政治联盟"等。耶鲁学生的这些组织经常展开辩论并积极参与政治活动,
如60年代抗议美国入侵越南、80年代抗议南非的种族隔离政策等。耶鲁的政治辩论非常活
跃,常常引起美国政界和舆论界的高度关注,人们似乎从一场场激烈的学子论战中看到了
未来的美国政坛新星。

耶鲁大学是世界上最早设立人文和艺术学科的大学之一,拥有众多一流的人文科学系和人
文科学研究计划,其英语系和历史系排名位居全美大学之首,哲学的师资力量也相当雄厚
,耶鲁的骄傲惠特尼人文科学研究中心,使耶鲁成为美国高等教育界人文科学研究的
高地。耶鲁特别强调自由的思想和自由的学术空气。这种“自由教育”的原则,使耶鲁能
够包容各种思想流派,保持勃勃生机。

耶鲁大学学科全面,艺术人文学科极强,文学、音乐及戏剧系在美国数一数二,法学院更
是出类拔萃,文科的历史、英语、法语、德语、政治学、音乐专业,理科的生化、分子生
物、数学、物理专业在美国大学中名列前茅。工科较弱,最强的学科是社会科学、人文科
学以及生命科学。其它学科比起哈佛、斯坦福等虽嫌稍弱,但足可与大部分名校一较短长。

商学院第14,医学院第10,法学院第1,工程学院第41,艺术学院第2;
理科类综合第10:生命科学第9,化学第14,计算机第25,地质学第20,数学第7,物理第
12;
人文社科类经济学第7,政治学第5,英语言文学第1,历史学第1,心理学第3,社会学第2
4;
NRC排名:艺术人文类第6,生物科学类第6,数理类第12,社会科学类第6。

梁溪香榭 : 2009-03-06#506
回复: 加拿大教育情况有用素材收集帖子--梁溪香榭整理

美国名校点评: 芝加哥大学
芝加哥大学:“原子能诞生地”

芝加哥大学:美国中西部最富盛名的私立大学,由石油大王J.D.洛克菲勒捐资于1891年
创办,位于美国伊利诺州的芝加哥市美丽的密西根湖湖畔。校园占地约200英亩,其哥特式
建筑古香古色,具牛津,剑桥之风韵。芝大提出的旨在防止学术课程和职业课程过分专门
化的“芝加哥计划”,对其他大学的本科教育计划产生生巨大影响。

芝加哥大学有75位校友曾获诺贝尔奖,为美国大学中最多。2位校友曾获图灵奖,12位教授
荣获过国家科学奖章,现任教授中有超过60位国家科学院、国家工程院和国家医学科学院
院士。2003年 的诺贝尔奖得主中,便有芝加哥大学的教授John Coetzee得诺贝尔文学
奖。

在费密(Enrico Fermi 1901-1954)的领导下,他和助手们在1942年12月2日在芝加哥大学建
成世界上第一座可控原子核裂变链式反应堆,使它达到临界状态,产生可控的核裂变链式
反应。这一成就是原子能时代的一个重要里程碑,为两年后的原子弹诞生奠定了基础,芝
加哥大学也因此被称为「原子能诞生地」。Albert Abraham Michelson(1907年诺贝尔物理
奖得主)首先进行光速的测量,并发展了同位素年代测定法。全美第一次活体肝脏移植的病
例,就是在芝大医院完成的。这项创举,为无数苦于「一肝难求」的肝病病人点燃了一线
生机。 在经济学方面, 芝加哥大学的经济系产生了22位诺贝尔奖得主,因此被誉为“芝
加哥学派”,对经济学的研究有著深远的影响。一九八二年该校的经济学家,同时也教授
MBA课程的George J. Stigler 荣获诺贝尔经济奖,是全球商学院教授获此殊荣的第一人。
芝加哥大学的校训是“提升知识,以便充实人生。”(拉丁文原文:Crescat scientia v
ita excolatur。 英译:Let knowledge grow from more to more; and so be human li
fe enriched)。

芝大对教育观念的「宏观」与实验精神,奠定了它在美国教育史上的重要地位;而它在学
术研究上的地位与贡献,也同样值得称道。 芝加哥大学的人类学、地球科学、经济学、地
理学、历史学、语言学、统计学、社会学等学科专业在美国大学的相应领域排名中均居前
十名,具有较强的学术实力。经济学、社会学实力尤其超强,“芝加哥学派” 赫赫有名。
芝加哥大学的物理系也是闻名遐迩,所有的华裔诺贝尔物理奖得主都和芝大有渊源。除工
科外,各科实力都颇不俗,在一流之列。

商学院第9,医学院第21,法学院第6;
理科类综合第12:生命科学第20,化学第14,计算机第35,地质学第10,数学第6,物理
第 7;
人文社科类英语言文学第5,经济学第2,社会学第3,历史学第5,哲学第7,心理学第
30;
NRC排名:艺术人文类第8,生物科学类第11,数理类第7,社会科学类第5。

梁溪香榭 : 2009-03-06#507
回复: 加拿大教育情况有用素材收集帖子--梁溪香榭整理

美国名校点评(9): 哥伦比亚大学
哥伦比亚大学:世界之都的心脏

哥伦比亚大学:常青藤盟校之一,也是世界之都纽约最值得骄傲的大学,成立于1754年。
哥伦比亚大学出了近代中国不少名人,如早期的胡适之、陈公博、宋子文、顾维钧,近期
的倪文亚、李焕、张京育、吴舜文等,不但如此,美国心理学家桑待克、进步主义先驱柯
普居、实验主义教育大师杜威都出自哥伦比亚大学教育学院,纽约市有十四位市长、纽约
州十位州长也都是哥伦比亚大学校友。

共有64位诺贝尔奖得主,1位图灵奖得主是哥伦比亚大学校友。10位教授荣获过国家科学奖
章。诺贝尔奖得主物理学家R.A.米 利肯(1923),化学家I。兰米尔(1932),生物学家T.
H.摩尔根(1933) 等都是哥大校友。镭射、激光、调频广播的发明,都是哥伦比亚大学的
杰作;普立兹,美国报坛名人,是哥大新闻学院的创办人。据统计哥伦比亚大学毕业生,
几乎每四个人就有一个留在纽约地区工作,法学院的校友也是纽约法律界的天之骄子,新
闻系的毕业生也大多是三大电视网的中坚份子。

哥伦比亚大学各科发展均衡,医、法、商出类拔萃,都在前10之列,化学、生物、计算机
、地理等,排名名列前茅,文、史、哲、管理、新闻、国际关系等的名声也较大,工科稍
弱。哥伦比亚大学教育研究生院是世界上最大的教育学、应用心理学和心理健康学方面的
综合研究生院,1997年全美教育研究生院排名第一。它拥有众多的研究中心,是美国上述
学科最好的研究生院之一。

商学院第6,医学院第7,教育学院第4,法学院第4,工程学院第26;
理科类综合第11:生命科学第14,化学第8,计算机第20,地质学第5,数学第13,物理第
10;
人文社科类经济学第14,历史学第5,哲学第26,心理学第11,社会学第14;
NRC排名:艺术人文类第4,生物科学类第8,工程科学类第30,数理类第14,社会科学类
第 10。

梁溪香榭 : 2009-03-06#508
回复: 加拿大教育情况有用素材收集帖子--梁溪香榭整理

美国名校点评(10): 康奈尔大学
康奈尔大学:平民化的贵族

康奈尔大学:常青藤盟校之一,东部最璀璨的明珠之一,1865年由联合电报公司创始人、
慈善家埃兹拉.康乃尔和外交家安德鲁.怀特创建。公私合营的特殊制度使得康乃尔大学成
为美国长春藤名校中贵族色彩最淡的一所。

康奈尔大学几乎从它一诞生起,就以其创新精神影响了整个美国高等教育,也为世界各国
培养了不少有影响的人物。据统计,目前先后有29位诺贝尔奖得主曾经是康奈尔大学教授
,学生或研究人员,其中文学奖得主1 位,和平奖得主1位,物理学奖得主6位,化学奖得
主5位,医学和生物学奖得主5位。此外,还有2位图灵奖得主是康奈尔教授,7位教授荣获
过国家科学奖章。1931年毕业于康奈尔大学并获得博士学位的威尔斯]比德尔是著名的遗
传学家,获诺贝尔奖,还曾任芝加哥大学校长。因“对中国农民生活史诗般的描述”而获
得1938年诺贝尔文学奖的赛珍珠 (Pearl S. Buck) 则是在康奈尔大学拿到艺术硕士学位的
。语言大师,白话运动的发起者胡适也是从这里毕业的。在康奈尔大学培养出来的3名普利
策奖得主中,伍洁芳(Sheryl WuDunn) 因报道1989年北京天安门大屠杀事件而闻名世界。
从康奈尔大学毕业的名人还有:格鲁曼飞机工程公司(Grumman Aircraft Engineering C
o.)的创始人格鲁曼(Leroy Grumman),世界最大的金融集团花旗集团总裁山福德.威尔(S
anford I. Weill) ,2002年世界杯带领美国足球队冲入八强的主教练布鲁斯.阿瑞纳(Bru
ce Arena),前密西根大学法学院院长, 新任康奈尔大学校长杰夫.黎曼(Jeffrey S.
Lehman) ,现任白宫首席经济顾问史蒂芬.弗里德曼(Stephen Friedman),现任国防部副部
长鲍尔.沃尔夫兹(Paul D. Wolfowitz)、华尔街高盛证券首席投资策略员科恩(Abby
Cohen)、著名作家柯特*威讷格特、美国最高法院法官露丝*金斯堡等等。

康奈尔校区约为745英亩。校区内建筑物精致典雅,颇具欧洲迷人风情,拥有美丽的瀑布及
登山步道, 为理想的读书研究环境,被公认为是长春藤八所大学中最美、最大的学校。康
乃尔大学的宗旨是:充满责任心,追求自由的理想。她的校色是大红,象征著热情奔放。
似乎,任何人探索科学和生命的激情都能在这里燃烧。

康奈尔大学各科发展均衡,都在一流之列,航空工程,植物学、化学、土木工程,计算机
科学、地球科学、电气工程、英语、法语、德语、微生物学、音乐、统计学,动物学等学
科在美国大学相应学科排名中名列前茅。旅馆管理学院是被公认全美同专业中最好的,农
业学院在全美排名也是数一数二。其工科为常青藤八校中最好的。

商学院第16,医学院第12,教育学院第24,法学院第10,工程学院第11;
理科类综合第8:生命科学第14,化学第8,计算机第6,地质学第11,数学第10,物理第7;
人文社科类经济学第17,历史学第10,哲学第9,心理学第15,社会学第14;
NRC排名:艺术人文类第5,生物科学类第19,工程科学类第6,数理类第6,社会科学类
第 14。

梁溪香榭 : 2009-03-06#509
回复: 加拿大教育情况有用素材收集帖子--梁溪香榭整理

美国名校点评(11): 密歇根大学安阿伯分校
密歇根大学安阿伯分校:“美国公立大学典范”

密歇根大学安阿伯分校:“十大” 盟校之首,美国中西部最富盛名的公立大学,1817年建
校,向称“美国公立大学典范”。

密大校园占地2,842英亩,其中安阿伯本部占地2,600英亩,位于美国中北部五大湖地区。
密歇根大学历史上共有7位教授或校友获得过诺贝尔奖,包括著名华裔科学家丁肇中,1位
获得过图灵奖,4位教授荣获过国家科学奖章, 多位校友获颁普立兹新闻奖。目前任教的教
授中有26位国家科学院院士、17位国家工程院院士。前美国总统福特也是密歇根大学出身。

密歇根大学学科发展均衡,几乎没有弱项,水准均属一流。美国公立大学中可与柏克利一
争锋芒的,恐怕舍此其谁了。与柏克利、伊利诺依香槟并称美国公立大学三巨头。

商学院第13,医学院第8,法学院第7,教育学院第8,工程学院第6,图书馆学学院第3,
音乐学院第4,护理学院第3;
理科类综合第13:生命科学第14,化学第21,计算机第14,地质学第5,数学第8,物理第
13;
人文社科类经济学第11,历史学第5,政治学第2,心理学第2,社会学第3,哲学第8;
NRC排名:艺术人文类第12,生物科学类第17,工程科学类第8,数理类第24,社会科学
类第3。

梁溪香榭 : 2009-03-06#510
回复: 加拿大教育情况有用素材收集帖子--梁溪香榭整理

美国名校点评(12): 伊利诺依大学香槟分校
伊利诺依大学香槟分校:“理工翘楚”

伊利诺依大学香槟分校:“十大” 盟校之一,建立于1867年,一直名列全美最优秀的理工
大学之一,理工科在“十大”(The Big Ten)中更是排名第一。

伊利诺大学香槟分校占地1450亩,位于伊利诺伊南部安静幽雅的双子城-厄巴纳及香槟市
。该州的密歇根湖是闻名世界的美国五大湖之一,自然景象令人叹为观止。

截至2003年,该大学共有20位教授及校友荣获过诺贝尔奖,2位校友荣获过图灵奖,1位教授
荣获过菲尔兹奖,21位教授及校友荣获过普利策奖,11位教授荣获过美国国家科学奖章,
现任教授中有2位普利策奖得主、26位国家科学院院士、29位国家工程院院士。

伊利诺大学香槟分校教授巴丁(John Bardeen)发明晶体管、提出低温超导理论(BCS理论),
历史上第一次在同一领域(固态理论)中两次获得诺贝尔奖, 教授哈肯与阿佩尔合作在计算
机完成了四色定理的证明,名列世界历史上4大化学家的校友卡罗瑟斯(Wallace Hume Car
others, 化学博士, 1924)在1934年发明尼龙,.....2003年,物理系教授安东尼.莱格特(
Anthony J. Leggett)和化学系教授保罗.劳特布尔(Paul C. Lauterbur) 分别荣获诺
贝尔物理奖和诺贝尔生理学和医学奖,给该大学再添殊荣。在其它方面,伊利诺伊大学香
槟分校也造就了无数名人。网络神童马克*安德森(Mark Anderson)在伊利诺大学香槟分校
设计了因特网浏览器软件Mosaic(莫塞克)及Netscape(导航者), 为人类进入互联网时代打
开了大门, 校友Jack Kilby首创的集成电路(IC)是信息时代最为重要的一项发明...... 花
花公子杂志的创始人Hugh M. Hefner毕业于此,备受中外观众推崇的中国电影《卧
虎藏龙》的导演李安毕业于伊利诺伊大学香槟分校戏剧系,获得学士学位。华罗庚曾任伊
利诺大学香槟分校教授, 敢讲真话的清华大学水利系教授黄万里是伊利诺大学香槟分校博
士, 参与研制人工合成牛胰岛素的北大化学系教授邢其毅是伊利诺大学香槟分校博士, 竺
可桢早年曾在伊利诺大学香槟分校攻读农学, 台湾中央研究院院长(19701983
年), 台湾大学校长(1951-1970年)钱思亮是伊利诺大学香槟分校化学博士, 菲律
宾前总统拉莫斯是伊利诺大学香槟分校土木工程硕士, 身价一度超过比尔.盖茨成为世界首
富的甲骨文公司(Oracle) 董事长兼首席执官(CEO)拉里.埃里森(Larry Ellison)曾在伊
利诺大学香槟分校读书, AMD创始人兼首席执官杰里.桑德斯(Jerry Sanders)桑德斯是伊利
诺大学香槟分校电子工程学士, 通用电气(GE)董事长兼首席执官杰克.韦尔奇(Jeck Welch)
是伊利诺大学香槟分校化工博士...... 这样的杰出校友还有很多。

伊利诺依大学香槟分校的理工科尤称翘楚,商、法等专业学院却较弱,缺少医科。伊利诺
伊大学香槟分校排名全国前10名的研究生专业有物理学,化学,计算机科学, 心理学, 教育
学,工程学,会计学,大众传播学,图书馆科学,音乐,数论,代数,逻辑学,微生物学。工程学院
在全美闻名遐迩,其电子工程, 计算机工程, 土木工程, 材料科学与工程, 机械工程, 原子
工程,农业工程,环境工程等系科都排在全美前5位,化学工程,航空航天工程排在全美前10位。

商学院第27,法学院第25,教育学院第18,工程学院第4,图书馆学学院第1;
理科类综合第7:生命科学第24,化学第6,计算机第5,数学第16,物理第8;
人文社科类心理学第3,经济学第26,历史学第22,新闻传媒第6,哲学第33,社会学第28

NRC排名:艺术人文类第25,生物科学类第26,工程科学类第7,数理类第15,社会科学
类第17。

梁溪香榭 : 2009-03-06#511
回复: 加拿大教育情况有用素材收集帖子--梁溪香榭整理

美国名校点评(13): 宾州大学
宾州大学:"我们定会找到办法,否则就创造出办法"

宾夕法尼亚大学:常青藤盟校之一,于1740年由美国著名科学家和政治家、独立宣言起草
人之一本杰明*富兰克林创办。

宾大不同于依照英国模式开设老式课程的殖民地学院,它标志着新的高等学府模式在北美
洲的诞生,开创了现代美国教育制度的先河。她不仅首先设立了科学课程,同时还是第一
个开设历史、数学、农学、英语和现代语言等课程的美国大学。

在其它许多领域,宾大也作出了很多开创性贡献。如:18世纪时曾任大学副校长的天文学
家大卫*里顿豪斯创建了几座著名的机械天文台,其中的一座保留至今,成为校园的
景点之一。这种被称为“太阳系仪”的机械装置实际上就是一种机械模拟计算机。1946年
,宾大莫尔电子工程学院又设计出了世界上第一台全电子数学计算机“ENIAC”,开创了
计算机的新时代。宾夕法尼亚大学校门上镂刻着"我们定会找到办法,否则就创造出办法"(
We will find a way or we will make one)。这集中体现了宾夕法尼亚大学勇于创新的
精神。

该大学共有15位教授及校友荣获过诺贝尔奖,4位教授荣获过美国国家科学奖章。现宾大全
校共有41名美国医学科学院院士,和近40名国家科学院院士。

宾州大学的商、医、法等学院无不是众多人梦寐以求的目标,文理各科也多在一流,工科
稍弱。在理工科方面,宾夕法尼亚大学的生物工程学、考古学、基因工程学、计算机科学
和医学研究在全美高校中处于领先地位。在社会科学和人文科学方面,宾夕法尼亚大学的
人类学、心理学、社会学、妇女学和城市研究等学科都在学术界相当有名。

商学院第2,医学院第4,法学院第7,教育学院第6,工程学院第29,护理学院第3;
理科类综合第17:生命科学第23,化学第18,计算机第17,数学第16,物理第20;
人文社科类经济学第9,历史学第13,新闻传媒第33,哲学第24,心理学第15,社会学第1
1;
NRC排名:艺术人文类第9,生物科学类第14,工程科学类第17,数理类第31,社会科学
类第11。

梁溪香榭 : 2009-03-06#512
回复: 加拿大教育情况有用素材收集帖子--梁溪香榭整理

美国名校点评(14): 威斯康星大学迈迪逊分校
威斯康星大学迈迪逊分校:“十大” 盟校的大哥大

威斯康星大学迈迪逊分校:“十大” 盟校之一,创校于1848年。校园面积有933英亩,校
园环境风光更是一等一。

威斯康星大学历史上共有17位教授或校友荣获过诺贝尔奖,20位教授或校友荣获过普利策
奖,9位教授荣获过国家科学奖章,目前任教的教授中有46位国家科学院院士和19位国家工
程院院士。

威斯康星大学理工人文俱强,商、法、医等专业学院却较弱,但学术声望之高足可列美国
名校前20之列。

商学院第46,法学院第31,医学院第29,教育学院第9,工程学院第15;
理科类综合第9:生命科学第12,化学第8,计算机第10,地质学第17,数学第13,物理第16;
人文社科类社会学第1,经济学第10,历史学第10,新闻传媒第9,哲学第20,心理学第9;
NRC排名:艺术人文类第24,生物科学类第9,工程科学类第16,数理类第16,社会科学
类第9。

梁溪香榭 : 2009-03-06#513
回复: 加拿大教育情况有用素材收集帖子--梁溪香榭整理

美国名校点评(15): 加州大学洛杉矶分校
加州大学洛杉矶分校:西海岸明珠

加州大学洛杉矶分校:加州大学的明珠之二,其前身自1919年成为加州大学南方分部并于
1927年改为现名。二战前后,加大洛杉矶分校迅速发展,今天的加大洛杉矶分校与柏克利
、圣地亚哥分校鼎足而立,组成加州大学的中坚,公立大学中无可与之匹敌者。其校园占
地约419英亩。

加大洛杉矶分校历史上共有5位教授或校友获得过诺贝尔奖,7位教授荣获过国家科学奖章
, 目前任教的教授中有32位国家科学院院士、14位国家工程院院士。

加大洛杉矶分校学科发展均衡,没有一科落在20名以外,人文学科更是少有10名以外的。

商学院第14,医学院第14,法学院第16,教育学院第3,工程学院第19;
理科类综合第13:应用数学第4,生命科学第20,化学第11,计算机第14,地质学第11,
数学第10,物理第16;
人文社科类经济学第11,历史学第9,哲学第6,心理学第6,社会学第7;
NRC排名:艺术人文类第11,生物科学类第16,工程科学类第19,数理类第13,社会科学类第7。

梁溪香榭 : 2009-03-06#514
回复: 加拿大教育情况有用素材收集帖子--梁溪香榭整理

美国名校点评(16): 约翰霍普金斯大学
约翰霍普金斯大学:医学圣地

约翰霍普金斯大学:著名私立大学,由富商、慈善事业家J.霍普金斯捐资于1876 年创办
,校址在马里兰州的巴尔的摩。这座城市濒临大西洋,是著名的港湾。

约翰霍普金斯大学以医科驰名于世,其医学院仅次于哈佛。该校所有跟生物沾亲带故的专
业,都在美国名声斐然,其中包括生物医学工程、化学和生物化学等。大学的医院是学生
们搞临床研究的好去处,约翰*霍普金斯医院几乎年年被医学杂志评为最好的医院。
其它科目如英语、历史、美术、国际关系、政治科学、政治经济也相当出色。其它文、理
、工也算不错。

与约翰霍普金斯大学有关的诺贝尔奖得主有27位。4位教授荣获过国家科学奖章。

约翰霍普金斯大学医学院第2,工程学院第22,公共卫生学院第1,护理学院第5;
理科类综合第19:生物科学第5,化学第27,计算机第29,地质学第11,数学第26,物理
第 20;
人文社科类经济学第24,历史学第10,哲学第29,心理学第24,
社会学第19;
NRC排名:艺术人文类第13,生物科学类第15,工程科学类第25,数理类第20,社会科学
类第23。

梁溪香榭 : 2009-03-06#515
回复: 加拿大教育情况有用素材收集帖子--梁溪香榭整理

美国名校点评(17): 西北大学
西北大学:商界的“西点军校”

西北大学:“十大” 盟校中唯一的私立大学,建立于1851年,位于伊利诺州密歇根湖岸的
小城埃文斯顿(Evanston),校园占地250公顷。

埃文斯顿在芝加哥以北,适合居住,是芝加哥收入相对较高的地区。西北大学虽然校园面
积不太大,但风景极为优美,有长达数英里的湖滨沙滩和浴场。

西北大学的校友中许多人事业有成,声誉卓著,其中包括诺贝尔文学奖获得者贝娄(Saul
Bellow),托尼奖获得者海德里(Heather Headley)和齐默曼(Mary Zimmerman),卡
夫食品CEO霍尔顿(Betsy Holden),美国最高法院大法官斯蒂文斯(John Paul Stevens
), 以及美国航空航天局前局长维勒(Edward J. Weiler)和著名影星辛迪-克劳芙(C
indy Crawford)等等。西北大学校友中的诺贝尔奖得主有8位。

西北大学凯洛格商学院绝对是全美最灸手可热的商学院之一。在《商业周刊》总共十六年
的八次评比当中,凯洛格是唯一的一所从来没有离开三甲的商学院;而自始至终没有离开
前五名的学校分别是:凯洛格商学院,哈佛商学院和沃顿商学院。莫迪尔新闻学院是全美
公认的最好,最出色的新闻学院。这所新闻学院不但设备先进,师资雄厚,出过将近20
位普利策奖得主。在工程学方面,西北大学的材料科学研究在全美乃至全世界独树一帜。
在上一个世纪50年代,西北大学率先成立了全球第一个材料科学系,整合了冶金,陶瓷
和高分子科学等多个研究领域。90年代以后,西北大学又成立了由联邦资助的材料研究
中心。此类中心全国只有三个。2002年,美国国家级纳米科学中心落成开幕,西北大
学的材料研究如虎添翼。西北大学的音乐学院建立于1895年,是美国授予音乐学位历
史最悠久的学校之一。音乐学院的铜管乐器和吹奏乐器表演课在全美颇负盛名。闻名世界
的芝加哥交响乐团及Lyric歌剧团,更常是音乐学院学生们的良师益友。

西北大学的其它学科发展均衡,工科略强于理科。

商学院第4,医学院第21,法学院第12,教育学院第9,工程学院第19;
理科类生命科学第36,化学第12,计算机第42,数学第21,物理第28;
人文社科类经济学第8,历史学第15,新闻传媒第8,哲学第23,心理学第17,社会学第
9;
NRC排名:艺术人文类第22,生物科学类第18,工程科学类第13,数理类第25,社会科学
类第13。

梁溪香榭 : 2009-03-06#516
回复: 加拿大教育情况有用素材收集帖子--梁溪香榭整理

美国名校点评(18): 德州大学奥斯丁分校
德州大学奥斯丁分校:南方之最

德州大学奥斯丁分校:美国南部最富盛名的大学。德克萨斯大学奥斯汀分校是德克萨斯大
学的主校,也是美国最大的大学,始建于1883年。

德克萨斯大学奥斯汀分校位于美国第二大州德克萨斯州的首府奥斯汀,该市名列美国
5座最佳居住城市,为美丽的山川湖泊环抱,景色宜人,也是美国的政治中心之一。奥斯汀
还被誉为美国IT业的“硅山”,是全球最大的计算机系统公司戴尔的摇篮。另外,奥斯汀
的街头音乐和夜生活也为它在民间赢得“音乐之都”的称号。

德克萨斯大学校友中既有诺贝尔奖得主、也有普利策奖得主和图灵奖得主,还有3位是国家
科学奖章获得者,现任教授中有13位国家科学院院士、40位国家工程院院士。

德州大学奥斯丁分校没有医学院,位於达拉斯的德州大学西南医学中心生物学综合排名第
14,医学院排名第16。奥斯丁各科都很强,工程和传媒等学科尤负盛名。

商学院第17,法学院第15,教育学院第13,工程学院第9;
理科类综合第15:生命科学第29,化学第12,计算机第7,地质学第11,数学第15,物理
第 13;
人文社科类经济学第21,历史学第22,新闻传媒第1,哲学第25,心理学第17,社会学第1
6;
NRC排名:艺术人文类第16,生物科学类第25,工程科学类第12,数理类第11,社会科学
类第18。

梁溪香榭 : 2009-03-06#517
回复: 加拿大教育情况有用素材收集帖子--梁溪香榭整理

美国名校点评(19): 杜克大学
杜克大学:“南方哈佛”

杜克大学:美国东南海岸的又一所私立名校,由烟草大亨詹姆斯.杜克建立于1924年,其
进取精神和迅速发展引人侧目,俨有「南方哈佛」之称。

杜克大学地处北卡州杜兰市(Durham, North Carolina),与人杰地灵的教堂山 (Chapel H
ill)及北卡州首府洛丽市(Raleigh),正好鼎足而立,构成人文与地理上著名的"北卡三角
区"(North Carolina Triangle)高科技、尖端医学研究、与文教地带。

以杜克的校园建筑来说,整个东校园保持着乔治亚式古典建筑,而较新的西校园,一眼望
去,简直就是英国牛津大学的翻版。建筑虽然外型非常古典精致,所有建材却是美东南地
区的坚固花岗岩。一百七十栋古典建筑,彷佛一一隐藏在秀丽的山林中一般。

杜克大学不但是一流学术殿堂,更是修身养性、术德兼修的好地方。美国南部地域广阔,
气候温暖而舒适。杜克花园(Sarah P. Duke Gardens) 风景优美,是远近闻名的景点。杜
克的学生体育好是出了名的,而他们也自豪地宣称“杜克有全世界最好的篮球队”。
杜克大学工科较弱,理科除生物外,难称一流,但医、法、商实力不俗,足可使杜克挤身
美国最佳大学之列。

商学院第7,医学院第4,法学院第12,工程学院第33;
理科类综合第20:生命科学第12,化学第43,计算机第20,数学第25,物理第33;
人文社科类经济学第21,历史学第15,哲学第39,心理学第24,社会学第16;
NRC排名:艺术人文类第10,生物科学类第10,工程科学类第45,数理类第35。

梁溪香榭 : 2009-03-06#518
回复: 加拿大教育情况有用素材收集帖子--梁溪香榭整理

美国名校点评(20): 加州大学圣地亚哥分校
加州大学圣地亚哥分校:加州明日之星

加州大学圣地亚哥分校:加州大学的明珠之三,建校于1960年,位于加州圣地亚哥。地
牙哥是全美第7大的城市,有最美的太平洋海岸光及4位加大圣地亚哥分校教授荣获
过国家科学奖章。现在加大圣地亚哥分校的研究人员中有五位诺贝尔奖获得者, 此外它拥
有国家科学院院士的数目在全美排名第七。(61位国家科学院院士、12位国家工程院院士)
加大圣地亚哥分校文、理、医、工俱强,发展迅速,前程远大。

医学院第16,工程学院第11;
理科类综合第18:生命科学第14,化学第22,计算机第20,地质学第11,数学第21,物理
第16;
人文社科类经济学第17,历史学第36,哲学第19,心理学第17,社会学第34;
NRC排名:艺术人文类第19,生物科学类第4,工程科学类第9,数理类第9,社会科学类
第 12。

梁溪香榭 : 2009-03-06#519
回复: 加拿大教育情况有用素材收集帖子--梁溪香榭整理

卡耐基梅隆大学:计算机专业学生的“麦加” ,1900年成立。卡耐基梅隆大学校友中的图
灵奖得主有6位,2位教授荣获过国家科学奖章。其计算机科学及其它工科、美术、艺术及
工业管理为世界公认一流专业。该校教授中有2名诺贝尔奖得主。其它除商学院、心理学和
经济学等尚堪一提之外,乏善可陈。

商学院第17,工程学院第11;
理科类计算机第1,生命科学第54,化学第51,数学第34,物理第28;
人文社科类经济学第19,历史学第36,心理学第9;
NRC排名:工程科学类第15,数理类第19。

梁溪香榭 : 2009-03-06#520
回复: 加拿大教育情况有用素材收集帖子--梁溪香榭整理

纽约大学:世界之都纽约的另一颗明珠,建立于1831年,是全美最大的私立大学之一。纽
约的社会局势、政治环境无时不对纽约大学的学生产生影响,使得该学校与纽约市一样,
充满着紧张感和机遇。

纽约大学法、商超强,医学院也属准一流。美术是全美第1名,新闻学、法语列前3名,而
运筹学、财务金融学、市场学、会计学、俄语、工商管理都在前10名之列,工业心理学、
计算机科学、英语、经济,临床心理学、语言学,社会学、古典文学、德语、物理学,生
物化学、心理学、人类学都是排在30名前后。理工科除数学及应用数学超强外,其它都太
弱。

医学院第28,商学院第12,法学院第5,教育学院第14;
理科类应用数学第1,生命科学第68,化学第86,计算机第29,数学第10,物理第52;
人文社科类经济学第19,历史学第24,心理学第30,社会学第22;
NRC排名:艺术人文类第21,生物科学类第31,数理类第26,社会科学类第22。

梁溪香榭 : 2009-03-06#521
回复: 加拿大教育情况有用素材收集帖子--梁溪香榭整理

西雅图华盛顿大学:西海岸公立名校之一。医学院一流,理科、工科还不错,其余乏善可
陈。与之有关的诺贝尔奖得主有4位。 2位教授荣获过国家科学奖章。

医学院第11,商学院第35,法学院第45,教育学院第27,工程学院第29,护理学院第1;
理科类综合第16:生命科学第20,化学第27,计算机第7,地质学第18,数学第26,物理
第 16;
人文社科类经济学第31,历史学第27,新闻传媒第15,心理学第17,社会学第16;
NRC排名:艺术人文类第32,生物科学类第7,工程科学类第21,数理类第10,社会科学
类第16。

梁溪香榭 : 2009-03-06#522
回复: 加拿大教育情况有用素材收集帖子--梁溪香榭整理

明尼苏达大学双子城分校:“十大” 盟校之一,始建于1851年。明尼苏达大学双城分校包
括明尼阿波利斯校园和圣保罗校园,又称东校园和西校园,其中明尼阿波利斯校园位于密
西西比河东西两岸。双城分校风景秀丽,建筑风格各异,是美国大学中传统的十大美丽校
园之一。明尼苏达大学处极寒之地但有相当水准的学术水平及声望。文理工法商都是在一
流和准一流之间。在2001年和2002年,明尼苏达大学双城校园被评为美国公立科研大学的
三甲之一。明尼苏达大学的化工系、机械工程系、经济学系、应用经济学系、心理系、教
育系、应用数学、人力资源管理、信息管理系统、森林、药学及医疗系统管理等多门学科
在全美始终名列前茅。与该校有关的诺贝尔奖得主有15位。校友中还出过1位前美国首席大
法官,两位美国前副总统,以及多位排名美国财富500家(FORTUNE 500)的企业巨子。
医学院第37,商学院第26,法学院第19,教育学院第12,工程学院第22; 哈姆弗瑞公共事
务研究所(Hubert H. Humphrey Institute of Public Affairs)以前美国副总统哈姆弗
瑞(Hubert H. Humphrey)命名,是明尼苏达大学的一个专业研究生院。该学院是闻名全
美的公共政策问题研究中心,在全国公立大学的公共事务学院中一直列于前十名。该学院
以影响和帮助制定当地,州,美国和国际各种公共事务政策而著名。在非盈利组织管理问
题上,该学院更是独树一帜。

理科类生命科学第29,化学第22,计算机第35,数学第16,物理第24;
人文社科类心理学第3,经济学第11,历史学第19,哲学第31,心理学第11,社会学第
22;
NRC排名:艺术人文类第33,生物科学类第28,工程科学类第11,数理类第27,社会科学
类第15。

梁溪香榭 : 2009-03-06#523
回复: 加拿大教育情况有用素材收集帖子--梁溪香榭整理

弗吉尼亚大学:美国最具贵族精神的大学之一,由美国第三任总统杰佛逊在1819年创建,
占地1千零50亩,位于美国弗吉尼亚州中部的夏洛茨维尔(Charlattesville)市。这所大
学的杰出校友不计其数,纵横各界。著名的政治家有美国第28任总统威尔逊、参议员罗伯
肯尼迪(Robert Kennedy)及爱德华肯尼迪(Edward Kennedy),文艺界名人包括Louis
Auchincloss,Lewis Allen(百老汇歌剧Annie的制片人),Henry Taylor(1986普利茨奖
诗歌获奖者)和国际名导演Mark Johnson(电影「雨人」,《早安,越南》等的制片人)
等等。

弗吉尼亚大学商学院、法学院绝对一流,医学院可称准一流,理工科较弱。弗吉尼亚大学
的健康科学中心是美国有名的学术医疗中心。美国新闻与世界报导评述:这个学术医疗中
心有美国最优秀的医生及良好的医疗环境,不仅是美国最佳的医院之一,并且也是顶尖的
教学医院之一。

商学院第11,医学院第27,教育学院第21,法学院第9,工程学院第38;
理科类生命科学第36,化学第47,计算机第27,数学第42,物理第38;
人文社科类经济学第26,历史学第15,心理学第17,哲学第38,社会学第38;
NRC排名:艺术人文类第15,生物科学类第32,工程科学类第35,数理类第36,社会科学
类第24。

梁溪香榭 : 2009-03-06#524
回复: 加拿大教育情况有用素材收集帖子--梁溪香榭整理

北卡罗莱纳大学:美国东海岸最耀眼的公立大学之一,也是全美国最早的州立大学,成立
于1789年,主校园有729英亩。缺少工科,其它各科发展均衡,水准也较高,但缺乏堪称
顶尖的带头学科。教授中有1位图灵奖得主。

商学院第21,医学院第21,教育学院第31,法学院第28,图书馆学学院第1,公共卫生学
院第2,护理学院第5;
理科类生命科学第27,化学第14,计算机第17,数学第32,物理第38;
人文社科类经济学第30,历史学第13,心理学第17,哲学第15,社会学第5;
NRC排名:艺术人文类第26,生物科学类第23,数理类第33,社会科学类第19。

梁溪香榭 : 2009-03-06#525
回复: 加拿大教育情况有用素材收集帖子--梁溪香榭整理

布朗大学:常青藤盟校之一。文理都算不弱,本科教育绝对一流。
医学院第43,工程学院第47;
理科类应用数学第4,生命科学第45,计算机第14,地质学第18,数学第21,物理第28;
人文社科类经济学第21,历史学第15,哲学第13,心理学第30,社会学第28;
NRC排名:艺术人文类第17,生物科学类第43,数理类第17,社会科学类第29。

梁溪香榭 : 2009-03-06#526
回复: 加拿大教育情况有用素材收集帖子--梁溪香榭整理

圣路易斯华盛顿大学:创立于1853年,校园坐落在圣路易斯市西面与市中心相距七英里的
地方,占地面积2,267 英亩,学校建筑多为欧式风格,环以大面积的绿树草坪,加之校园紧
靠著著名的森林公园,环境十分幽雅。教授中有22名美国国家科学院院士,16名国家科学
院医学科学院院士,1名全国工程院院士,2位教授荣获过国家科学奖章。与华盛顿大学有
联系的诺贝尔奖获得者有21人之多,其中有10人的得奖成果是在华大期间完成的,包括亚瑟
.H.康普顿(1927年诺贝尔物理奖),约瑟夫.厄兰格与希尔伯特.S.加塞(1944年
诺贝尔生理学医学奖),卡尔.F.柯内与格蒂.T.柯内(1947年诺贝尔生理学医学奖)等
。华盛顿大学现任教授中有1993年诺贝尔经济学奖得主道格拉斯.诺斯(Douglas North)
和1986年诺贝尔生理学医学奖得主丽塔.纳维-蒙塔西尼(Rita Levi-Montalcini)。华盛
顿大学最突出的是医学。华盛顿大学医学院多次被评为全美最佳五所医学院之一。1944年
,1947年,1959年和1986年的诺贝尔生理学医学奖得主都曾在华大医学院任教。

总体来说圣路易斯华盛顿大学除排名第2的医学院绝对顶尖,生物科学、本科教育及环保,
社会学,计算机和网络通信技术等少数学科属一流之外,乏善可陈。

梁溪香榭 : 2009-03-06#527
回复: 加拿大教育情况有用素材收集帖子--梁溪香榭整理

佐治亚理工学院:除了排名第5的超强工科外,毫无可称道之处。

梁溪香榭 : 2009-03-06#528
回复: 加拿大教育情况有用素材收集帖子--梁溪香榭整理

南加州大学:坐落于美国西岸洛杉矶市中心,成立于1880年。本科排名一直在上升,现在
已经进入前30名,研究生院不少专业和学院的专业排名在全美都进入了前10位,如工学院
今年排第8位,电影学院一直第一,政策、发展与规划学院一直在前5名,总体说来工科一
流,商学院、法学院在一流、准一流之间,其它乏善可陈。一说南加大大腕教授越来越多
,得全国性大奖的教授也越来越多了。2002年筹款达28亿美圆,甚至超过了哈佛!著名的
校友有电影「星际大战」的导演乔治鲁卡斯等。

梁溪香榭 : 2009-03-06#529
回复: 加拿大教育情况有用素材收集帖子--梁溪香榭整理

莱斯大学:位于得克萨斯州休斯敦市。本科第16名。物理系不错,英语、历史和考古学系
也非常受欢迎。工程系和医学预科是全校竞争最激烈的。莱斯大学建筑系是全美最好的建
筑系之一,其它乏善可陈。

盈婆婆 : 2009-03-06#530
回复: 加拿大教育情况有用素材收集帖子--梁溪香榭整理

偷偷进来,夸一句好妈妈。再偷偷溜走看资料了。

cji8 : 2009-03-07#531
回复: 加拿大教育情况有用素材收集帖子--梁溪香榭整理

等着看加拿大大学,分分容后补

Luck Li : 2009-03-07#532
回复: 加拿大教育情况有用素材收集帖子--梁溪香榭整理

博士后和民工的区别

联合利华引进了一条香皂包装生产线,结果发现这条生产线有个缺陷:常常会有盒子里没装入香皂。总不能把空盒子卖给顾客啊,他们只得请了一个学自动化的博士后设计一个方案来分拣空的香皂盒。博士后拉起了一个十几人的科研攻关小组,综合采用了机械、微电子、自动化、X射线探测等技术,花了几十万,成功解决了问题。每当生产线上有空香皂盒通过,两旁的探测器会检测到,并且驱动一只机械手把空皂盒推走。

 
 中国南方有个乡镇企业也买了同样的生产线,老板发现这个问题后大为发火,找了个小工来说:你他妈给老子把这个搞定,不然你给老子......。小工很快想出了办法:他在生产线旁边放了台风扇猛吹,空皂盒自然会被吹走。

这个小故事俺也听过,就是主角不是香皂。呵呵!:wdb6:

梁溪香榭 : 2009-03-08#533
回复: 加拿大教育情况有用素材收集帖子--梁溪香榭整理

加拿大大学点评

文科专业里, 最吸引人的莫过于商科. 大学里的商学院一般是相对独立的一个大系, 常以其赞助发起人的名字命名. 如西安大略大学的商学院的全称是Richard Ivey School of Business, 以其发起人叫做Richard Ivey之故. 商学院建立的初衷是教授管理经验, 推广北美管理理念. 但随着管理类成为一个学科渐渐成熟, 以及工作市场供求的变化, 商科已经发展出了许多分支, 形成了一套完整的专业体系. 其中比较常见的专业有:
管理 General Management
金融 Finance
会计 Accounting
市场营销 Marketing
国际金融 International Trade
组织行为学 Organizational Behavior
经营策略 Business Strategy
管理信息系统 Management Information System
等等......

梁溪香榭 : 2009-03-08#534
回复: 加拿大教育情况有用素材收集帖子--梁溪香榭整理

在众多商学院里, 以西安大略大学的Ivey最为著名. 在业界最著名的杂志Business week的非美国商学院排名中, Ivey力排众多世界著名商校, 跻身于前十强, 在加拿大商学院排名中, 更是无可争议的第一名. Ivey坚持传统的案例教学, 学生在入学伊始就会收到厚厚的十几本案例. 与之相应, 课堂教学则强调参与和讨论, 其课堂参与和表现可占到该课程评分的40%左右. 这对于母语不是英语, 又持东方思维方式的华裔学生来说, 可不是什么好消息, 无法发挥擅长考试的优势.
但另一方面, 这也强迫你提高英语水平, 接受加拿大文化. 而这对于今后在加国的生活和工作都是百利无一害的.
Ivey的另一个特点是其专业设置只有管理(General Management)和经营策略(Business Strategy)两个大方向. 对于那些有志于金融, 财会的学子, Ivey可能并不适合.
Ivey的招生非常严谨, 但会在给出拒绝信的同时, 也会给出Appeal Committee的详细联系方式. 如果你上诉, 他们会认真地重新审查你的申请, 给你第二次机会. 所以即使申请Ivey被拒绝, 也千万不要轻易放弃.

梁溪香榭 : 2009-03-08#535
回复: 加拿大教育情况有用素材收集帖子--梁溪香榭整理

除了Ivey, 多伦多大学的Rotman, 约克的Schulich, 女皇大学商学院, 以及BC大学商学院都非常不错.
多伦多大学的Rotman在金融方面一向独领风骚. 而约克的Schulich则是加拿大商学院中的新贵, 在过去三年里, Rotman和Schulich在不同来源的排名中"此起彼伏", 显示出Schulich的强大竞争力.
Schulich的会计专业公认是加拿大最强的, 其金融专业近年来也蒸蒸日上.
女皇大学在本地人心目中还是最好的, 但其MBA是理工和技术类的, 对有些人可能不大合适.
BC大学的MBA对于有志向西岸发展的学子可谓首选. BC大学环境优雅, 风景如画, 是读书的好去处.

次一等的商学院里其实也有许多不错的选择, 如卡尔加里大学的商学院, Brock University等, 不妨选择一二作为保底.

梁溪香榭 : 2009-03-08#536
回复: 加拿大教育情况有用素材收集帖子--梁溪香榭整理

码字好累. 下次再谈谈理工科.

Luck Li : 2009-03-08#537
回复: 加拿大教育情况有用素材收集帖子--梁溪香榭整理

码字好累. 下次再谈谈理工科.

辛苦了梁溪,我更喜欢你的签名。:wdb17::wdb17:

hecjing : 2009-03-08#538
回复: 加拿大教育情况有用素材收集帖子--梁溪香榭整理

好帖慢慢看!

梁溪香榭 : 2009-03-09#539
回复: 加拿大教育情况有用素材收集帖子--梁溪香榭整理

偷偷进来,夸一句好妈妈。再偷偷溜走看资料了。

谢谢盈儿一路的鼓励和支持. :wdb6::wdb19:

有时候俺也挺矛盾的. 勤快的老妈会养出懒儿子啊. 俺把这些个事情都做了, 儿子干什么捏 :wdb2::wdb23:

梁溪香榭 : 2009-03-09#540
回复: 加拿大教育情况有用素材收集帖子--梁溪香榭整理

等着看加拿大大学,分分容后补

慢慢道来 :wdb6:

梁溪香榭 : 2009-03-09#541
回复: 加拿大教育情况有用素材收集帖子--梁溪香榭整理

辛苦了梁溪,我更喜欢你的签名。:wdb17::wdb17:

:D

俺也稀饭嫩滴签名 :wdb20:

梁溪香榭 : 2009-03-09#542
回复: 加拿大教育情况有用素材收集帖子--梁溪香榭整理

不管怎么样,先来捧个场,虽然看得我头晕,哈哈!

谢谢丰富的资讯.加分了

恭喜恭喜!失而复得之后更加珍惜!
置顶得好啊!大家查阅起来很方便!:wdb6:
辛苦粱姐姐和守法领导!谢谢你们!:wdb17:
有空慢慢看。。。

这帖差点没了,正着急找呢。

好贴,顶一下,楼主辛苦了。

公司和家里都收藏了,昨天已经看了一部分,记脑子里更踏实。

辛苦了,各位领导和梁溪。:wdb17::wdb17:

收藏了。You've been a big help!

仔细研读了5个小时才看完,真是太有用了,感谢楼主点火和众位拾柴!:wdb10::wdb10::wdb10:

谢谢!好贴!有用!:wdb17::wdb17:

等着看加拿大大学,分分容后补

好帖慢慢看!

谢谢各位鼓励支持 :wdb19:

cji8 : 2009-03-09#543
回复: 加拿大教育情况有用素材收集帖子--梁溪香榭整理

你昨知道我要的就是加拿大的商科大学呢?好TZ啊

wliu88 : 2009-03-09#544
回复: 加拿大教育情况有用素材收集帖子--梁溪香榭整理

领导能不能在1楼建个直达电梯啊,否则看起来累啊

家园移民 : 2009-03-09#545
回复: 加拿大教育情况有用素材收集帖子--梁溪香榭整理

梁榭可以写个目录啊

偶可以改一楼的,改几楼都行啊

梁溪香榭 : 2009-03-09#546
回复: 加拿大教育情况有用素材收集帖子--梁溪香榭整理

梁榭可以写个目录啊

偶可以改一楼的,改几楼都行啊

遵命!

梁溪香榭 : 2009-03-09#547
回复: 加拿大教育情况有用素材收集帖子--梁溪香榭整理

加拿大大学点评, 理工类

在专业设置方面, 多伦多大学现有14个院系, 300个学士点, 148个硕士点和95个博士点. 其专业设置涵盖基础科学(如化学, 物理), 应用科学(如教育学, 保险精算), 人文科学(如艺术, 音乐), 经济商科(如金融, 管理), 医学法律等各个方面.

约克虽然没有医学, 但优势在于其许多专业都是一年两次招生, 分别是秋季(九月)和冬季(一月)入学, 因而给学子们更多的选择.

梁溪香榭 : 2009-03-09#548
回复: 加拿大教育情况有用素材收集帖子--梁溪香榭整理

多市以外的大学, 如女皇大学, 麦克马斯特大学和西安大略大学都离多伦多不远, 分别在Kingston, Hamilton和London.

女皇大学创立于1839年, 一向以其浓厚的贵族气息在本地人心目中占据无比尊崇的地位. Kingston地处多伦多, 渥太华和蒙特利尔三市交界处, 因而女皇大学的地理位置相当不错. 她所招收的本科学生的平均入学成绩历来都是加拿大最高的. 她所提供的专业包括: 应用科学, 艺术类, 商科, 教育, 医学和法律等.

梁溪香榭 : 2009-03-09#549
回复: 加拿大教育情况有用素材收集帖子--梁溪香榭整理

阿尔伯特大学始建于1908年, 为政府公立. 学校在教学, 研究和社区服务方面蜚声加拿大. 作为在加拿大见就工作最前沿的学校之一, 在2001年到2002年度, 阿尔伯特大学所获的来自各方的研究基金超过3亿加币. 学校坐落在阿尔伯特省的省府Edmonton. 该城市2001年的经济增长率位居加拿大第一并连年保持同样的增长势头. 通过与企业政府合作, 阿尔伯特大学在Edmonton的高速增长中扮演了重要角色. 目前该校设置有200多个本科专业, 170个研究生专业, 在校学生多达32,000名. 根据著名的麦克林的加国大学指南(Mclean's Guide to Canadian University), 2002年阿尔伯特大学的综合排名为第五, 其"明日领袖"单项排名为第三, 仅在多伦多大学和滑铁卢大学之后.

Luck Li : 2009-03-09#550
回复: 加拿大教育情况有用素材收集帖子--梁溪香榭整理

领导能不能在1楼建个直达电梯啊,否则看起来累啊

非常好的建议!:wdb6::wdb6:

Luck Li : 2009-03-09#551
回复: 加拿大教育情况有用素材收集帖子--梁溪香榭整理

梁榭可以写个目录啊

偶可以改一楼的,改几楼都行啊

太好了,感谢领导和梁溪。:wdb19::wdb19::wdb17::wdb17:

Luck Li : 2009-03-09#552
回复: 加拿大教育情况有用素材收集帖子--梁溪香榭整理

遵命!

很多有用的东西收藏了。:wdb19::wdb19:

Luck Li : 2009-03-09#553
回复: 加拿大教育情况有用素材收集帖子--梁溪香榭整理

给楼上各位+SW.

Luck Li : 2009-03-09#554
回复: 加拿大教育情况有用素材收集帖子--梁溪香榭整理

别人都能加上,就是LZ的加不上?记账。

moon-beach : 2009-03-10#555
回复: 加拿大教育情况有用素材收集帖子--梁溪香榭整理

博士后和民工的区别

联合利华引进了一条香皂包装生产线,结果发现这条生产线有个缺陷:常常会有盒子里没装入香皂。总不能把空盒子卖给顾客啊,他们只得请了一个学自动化的博士后设计一个方案来分拣空的香皂盒。博士后拉起了一个十几人的科研攻关小组,综合采用了机械、微电子、自动化、X射线探测等技术,花了几十万,成功解决了问题。每当生产线上有空香皂盒通过,两旁的探测器会检测到,并且驱动一只机械手把空皂盒推走。

 
 中国南方有个乡镇企业也买了同样的生产线,老板发现这个问题后大为发火,找了个小工来说:你他妈给老子把这个搞定,不然你给老子......。小工很快想出了办法:他在生产线旁边放了台风扇猛吹,空皂盒自然会被吹走。

moon-beach : 2009-03-10#556
回复: 加拿大教育情况有用素材收集帖子--梁溪香榭整理

很认真地学习了两天,非常感谢梁溪

cji8 : 2009-03-10#557
回复: 加拿大教育情况有用素材收集帖子--梁溪香榭整理

多市以外的大学, 如女皇大学, 麦克马斯特大学和西安大略大学都离多伦多不远, 分别在Kingston, Hamilton和London.

女皇大学创立于1839年, 一向以其浓厚的贵族气息在本地人心目中占据无比尊崇的地位. Kingston地处多伦多, 渥太华和蒙特利尔三市交界处, 因而女皇大学的地理位置相当不错. 她所招收的本科学生的平均入学成绩历来都是加拿大最高的. 她所提供的专业包括: 应用科学, 艺术类, 商科, 教育, 医学和法律等.

西安大略大学捏?:wdb17:

Luck Li : 2009-03-10#558
回复: 加拿大教育情况有用素材收集帖子--梁溪香榭整理

给LZ欠账还钱来了,呵呵!

zhuolh : 2009-03-11#559
回复: 偶搜集到的一些关于加拿大高中教育的资料 与大家分享

握手握手 :wdb6::wdb6::wdb6:
俺也准备让儿子上A-LEVEL.
他现在初三, 高一就准备读A-LEVEL啦.
我的女儿现在也在上A_LEVEL,高一年级。明年读完高二后就应该有A-LEVEL成绩了,按理可以申请国外大学了。你们是否知道:
1。如果她申请加拿大的大学,这边的大学会把她以国际学生的名额录取?还是加国学生的名额录取?二者的难易程度是不同的。据说国际学生的名额一般只有8%,竞争更加激烈。当然学费据说是按加国学生收的。
2。这边的大学好象都是读完十二年级,才凭十二年级成绩申请大学,而读到有A-LEVEL成绩,是高二,一进入高三就可以申请了,作为国际学生加的大学也这样在预录取(条件是高三成绩达到一定标准),但若是作为加国学生名额录取,不知是否会有问题?
打扰名位了,谢谢!

zhuolh : 2009-03-11#560
回复: 加拿大教育情况有用素材收集帖子--梁溪香榭整理

前面的内容花了一个晚上、一个下午看完了,领略了大量的信息,受到很大的启示。特别为梁溪的热心、认真、细致所感动,也佩服她良好的综合素质,真的为网站增色不少。还有版主“守法”,似乎是位女士吧?看名字总以为是个男士呢。还有其它的好人们。很为去加途中有这样的移友而高兴!希望今后有机会相识,作更多交流。虽说已经在移民了,但这里国内的工作、生活还是有些放不下。孩子现读高一(ALEVEL),成绩也还可以,中上吧,有时更好些。我们正常的话7、8月份该取签了。有人说孩子越早登陆入学加国学校越好,但我先生现在是肯定走不开的,我也有份还不错的工作,放弃去陪读,家庭生活整个影响太大。不去又担心孩子升大学会有遗憾。让孩子独自去,这么大的孩子自我管理能力,心理承受能力还不强,只怕出了岔子可就后悔晚了。

家园移民 : 2009-03-11#561
回复: 加拿大教育情况有用素材收集帖子--梁溪香榭整理

前面的内容花了一个晚上、一个下午看完了,领略了大量的信息,受到很大的启示。特别为梁溪的热心、认真、细致所感动,也佩服她良好的综合素质,真的为网站增色不少。还有版主“守法”,似乎是位女士吧?看名字总以为是个男士呢。还有其它的好人们。很为去加途中有这样的移友而高兴!希望今后有机会相识,作更多交流。虽说已经在移民了,但这里国内的工作、生活还是有些放不下。孩子现读高一(ALEVEL),成绩也还可以,中上吧,有时更好些。我们正常的话7、8月份该取签了。有人说孩子越早登陆入学加国学校越好,但我先生现在是肯定走不开的,我也有份还不错的工作,放弃去陪读,家庭生活整个影响太大。不去又担心孩子升大学会有遗憾。让孩子独自去,这么大的孩子自我管理能力,心理承受能力还不强,只怕出了岔子可就后悔晚了。
偶很娘娘腔吗?
偶受伤了
嘎嘎

zhuolh : 2009-03-11#562
回复: 加拿大教育情况有用素材收集帖子--梁溪香榭整理

没有啊,大男人呢!只是我好象在哪看到你的性别更正呢,也许是我记错了,哈哈,不好意思啊。说真话,我还心想,这个女的风格好象很阳刚的么。哈哈

jux : 2009-03-11#563
回复: 子女教育信息素材收集帖

嘎嘎

这个帖子也算历经了劫难了,丢了又重新找回来了。

主要整理的是关于教育的一些素材

从147楼开始,主要就是梁溪香榭搜集的一些的,推荐推荐!

推荐本书--从普通女孩到银行家,http://forum.iask.ca/showthread.php?t=233637

另外,本帖子打算建立个目录了,呵呵

等梁溪了
我也来一个,女,18岁,高二。

梁溪香榭 : 2009-03-11#564
回复: 偶搜集到的一些关于加拿大高中教育的资料 与大家分享

我的女儿现在也在上A_LEVEL,高一年级。明年读完高二后就应该有A-LEVEL成绩了,按理可以申请国外大学了。你们是否知道:
1。如果她申请加拿大的大学,这边的大学会把她以国际学生的名额录取?还是加国学生的名额录取?二者的难易程度是不同的。据说国际学生的名额一般只有8%,竞争更加激烈。当然学费据说是按加国学生收的。
2。这边的大学好象都是读完十二年级,才凭十二年级成绩申请大学,而读到有A-LEVEL成绩,是高二,一进入高三就可以申请了,作为国际学生加的大学也这样在预录取(条件是高三成绩达到一定标准),但若是作为加国学生名额录取,不知是否会有问题?
打扰名位了,谢谢!

偶查过很多大学的网站对A-LEVEL生的录取的相关规定. 很详细啊. 按照俺的理解, 他们只依据A-LEVEL的成绩以及托福或雅思成绩录取.

这个是滑铁卢大学对A-LEVEL生的录取条件. 按照不同专业分列.
http://findoutmore.uwaterloo.ca/admissions/international.php?id=2

对于这个结果偶没有完全的把握. 偶也看到嫩在这个坛子的别的版块问过. 其实要得到权威答案, 只有直接去问那些大学. 先仔细看嫩心仪的大学的网站, 再发MAIL过去问就是了. 或者请孩子的老师帮着问.

一般来说A-LEVEL生还可以有学分转. 具体参考学校网站.

梁溪香榭 : 2009-03-11#565
回复: 加拿大教育情况有用素材收集帖子--梁溪香榭整理

前面的内容花了一个晚上、一个下午看完了,领略了大量的信息,受到很大的启示。特别为梁溪的热心、认真、细致所感动,也佩服她良好的综合素质,真的为网站增色不少。还有版主“守法”,似乎是位女士吧?看名字总以为是个男士呢。还有其它的好人们。很为去加途中有这样的移友而高兴!希望今后有机会相识,作更多交流。虽说已经在移民了,但这里国内的工作、生活还是有些放不下。孩子现读高一(ALEVEL),成绩也还可以,中上吧,有时更好些。我们正常的话7、8月份该取签了。有人说孩子越早登陆入学加国学校越好,但我先生现在是肯定走不开的,我也有份还不错的工作,放弃去陪读,家庭生活整个影响太大。不去又担心孩子升大学会有遗憾。让孩子独自去,这么大的孩子自我管理能力,心理承受能力还不强,只怕出了岔子可就后悔晚了。

如果换作是偶, 偶会毫不犹豫选择陪孩子去加国继续读高中. 高一结束过去是来得及的. 毕竟由加国高中毕业进入大学, 衔接方面更好, 语言环境就更是没得比了.
当然各家情况不同. 偶认为如果父母不能陪过去, 除非孩子自律和自理能力强, 否则风险有点高. 还不如保守一点, 等孩子A-LEVEL结束后直接申请大学.
另外如果父母不陪过去的话, 孩子在那边是要找个监护人才可以的.

梁溪香榭 : 2009-03-11#566
回复: 加拿大教育情况有用素材收集帖子--梁溪香榭整理

偶很娘娘腔吗?
偶受伤了
嘎嘎

:D:D

说明领导耐心好啊, 又仔细又认真还不厌其烦啊! :wdb17::wdb6:

BTW, 目录还真不容易搞. 很多资料相互穿插.
唉. 怪俺没条理性. 乱呢.

梁溪香榭 : 2009-03-11#567
回复: 加拿大教育情况有用素材收集帖子--梁溪香榭整理

西安大略大学捏?:wdb17:
前面不是已经讲过了吗?

梁溪香榭 : 2009-03-11#568
回复: 加拿大教育情况有用素材收集帖子--梁溪香榭整理

别人都能加上,就是LZ的加不上?记账。

谢谢支持 :wdb19:
回礼啦 :wdb6:

rwx1378 : 2009-03-11#569
回复: 加拿大教育情况有用素材收集帖子--梁溪香榭整理

溪,谢谢分享!!给你加分分了!!!

梁溪香榭 : 2009-03-11#570
回复: 加拿大教育情况有用素材收集帖子--梁溪香榭整理

溪,谢谢分享!!给你加分分了!!!

看到老朋友好开心. 最近一切安好?
回礼! :wdb6:

rwx1378 : 2009-03-11#571
回复: 偶搜集到的一些关于加拿大高中教育的资料 与大家分享

我的女儿现在也在上A_LEVEL,高一年级。明年读完高二后就应该有A-LEVEL成绩了,按理可以申请国外大学了。你们是否知道:
1。如果她申请加拿大的大学,这边的大学会把她以国际学生的名额录取?还是加国学生的名额录取?二者的难易程度是不同的。据说国际学生的名额一般只有8%,竞争更加激烈。当然学费据说是按加国学生收的。
2。这边的大学好象都是读完十二年级,才凭十二年级成绩申请大学,而读到有A-LEVEL成绩,是高二,一进入高三就可以申请了,作为国际学生加的大学也这样在预录取(条件是高三成绩达到一定标准),但若是作为加国学生名额录取,不知是否会有问题?
打扰名位了,谢谢!


请问你的女儿在哪读A-LEVEL,我家女儿和你女儿的情况一样,高一上的,现读G2,上海的。

rwx1378 : 2009-03-11#572
回复: 加拿大教育情况有用素材收集帖子--梁溪香榭整理

看到老朋友好开心. 最近一切安好?
回礼! :wdb6:

:wdb6:每天都想着看一下溪溪的贴子,好在大家都是妇人,否则:wdb8:

梁溪香榭 : 2009-03-11#573
回复: 加拿大教育情况有用素材收集帖子--梁溪香榭整理

一楼更新有目录了

Luck Li : 2009-03-11#574
回复: 加拿大教育情况有用素材收集帖子--梁溪香榭整理

一目了然,太详细了。辛苦了领导和香溪。

家园移民 : 2009-03-11#575
回复: 加拿大教育情况有用素材收集帖子--梁溪香榭整理

一目了然,太详细了。辛苦了领导和香溪。
偶就是转帖
嘎嘎
都是梁榭辛苦啊

梁溪香榭 : 2009-03-11#576
回复: 加拿大教育情况有用素材收集帖子--梁溪香榭整理

:wdb6:每天都想着看一下溪溪的贴子,好在大家都是妇人,否则:wdb8:

一目了然,太详细了。辛苦了领导和香溪。

偶就是转帖
嘎嘎
都是梁榭辛苦啊

梁溪, 香榭, 梁榭, 香溪, 溪溪,
哈哈, 还有吗? :wdb6:

Luck Li : 2009-03-11#577
回复: 加拿大教育情况有用素材收集帖子--梁溪香榭整理

o(∩_∩)o...哈哈,反正都是你,都好听。

Luck Li : 2009-03-11#578
回复: 加拿大教育情况有用素材收集帖子--梁溪香榭整理

:wdb6:每天都想着看一下溪溪的贴子,好在大家都是妇人,否则:wdb8:

否则如何?抢家去。呵呵!:wdb6::wdb6:

Luck Li : 2009-03-11#579
回复: 加拿大教育情况有用素材收集帖子--梁溪香榭整理

我们大家都爱你,溪,o(∩_∩)o...哈哈!

ltxs119 : 2009-03-11#580
回复: 加拿大教育情况有用素材收集帖子--梁溪香榭整理

太有帮助了,谢谢!

pikexima : 2009-03-13#581
回复: 加拿大教育情况有用素材收集帖子--梁溪香榭整理

受益匪浅,太好太好了!
隆重谢谢梁溪(名字真好听),还有斑竹守法(听着像法律板块哦)!
前阵子在网上溜达,看到这个贴,马上收藏了,后来就打不开了,再后来被斑竹费劲捞回来了,好感动哦。
继续关注中。。。

梁溪香榭 : 2009-03-13#582
回复: 加拿大教育情况有用素材收集帖子--梁溪香榭整理

受益匪浅,太好太好了!
隆重谢谢梁溪(名字真好听),还有斑竹守法(听着像法律板块哦)!
前阵子在网上溜达,看到这个贴,马上收藏了,后来就打不开了,再后来被斑竹费劲捞回来了,好感动哦。
继续关注中。。。

谢谢支持. 嫩滴第一贴啊! :wdb19:

梁溪香榭 : 2009-03-13#583
回复: 加拿大教育情况有用素材收集帖子--梁溪香榭整理

滑铁卢大学 University of Waterloo

滑铁卢大学在加拿大虽然不是第一, 但滑铁卢大学在创新, 前景和名声方面堪称无冕之王. 尽管建校不过几十年, 滑铁卢大学崛起速度之快确十分惊人. 今天的滑铁卢仅全职本科生就有近18,000名, 规模堪与多伦多大学这样的老牌名校相媲美. 借者八,九十年代高科技的发展, 滑铁卢在软件工程方面吸引了大量优秀的研究和教学人员. 另一方面, 该校重视实践, 许多专业都设有实习机会, 使得学生在毕业前就已经拥有一定的工作经验和社会实践, 在激烈的市场竞争中更具优势. 其毕业生中涌现了一大批成功人士, 使得该校在业界拥有良好的口碑.

与此相对应, 要申请滑铁卢大学也是非常之难. 安省高中毕业生的平均成绩要达到85分以上, 其最好的软件工程及工程专业的平均分要求更是达到90分以上. 托福要求250分(相当于纸考600分), 是所有加拿大和美国大学最高档要求. 即使入学以后, 也会有一大批学生被淘汰. 但作为滑铁卢大学毕业生的光明前景, 还是使得该校具有不可抗拒的魅力和诱惑力.

梁溪香榭 : 2009-03-13#584
回复: 加拿大教育情况有用素材收集帖子--梁溪香榭整理

安省高中教育咨询问答

1,安省高中哪一年级的成绩最重要?
加拿大高中的12年级是高中学习生涯中的最重要阶段,12年级的成绩直接决定了学生所进入的大学和专业。加拿大没有高考,加拿大大学录取学生是根据高中最后一年成绩,也就是12年级6门主要课程的平均分。所以抓好高中最后一年的成绩是将至关重要。

2,哪些课程是高中必须抓好的重点科目?
在安省高中课程中,英语、数学、科学(物理、化学、生物)仍然是高中的最重要的课程,加拿大大学所要求的12年级6门课程中,几乎都会要求英语、至少两门数学以及两门科学课程,可见这些课程的重要性,与中国的教育一样,“学好语文数理化,走遍天下都不怕”!

3,安省高中12年级数学做了什么重大的改革?
原有的12年级的数学课程有3门:Data management(简称MDM4U)、Advanced Function& Introductory to Calculus(简称MHF4U) 和Geometry &Discrete math(简称MGD4U)。新的MDM4U在总的内容上没有太大的变动,只是对学生的要求降低,简单化了,这对选修这门课的学生是个好消息。另外两门数学则做了较大的调整:取消了MGD4U;将MHF4U分成Advanced Function和 Calculus& Vectors。另外,新的数学课程更加注重Technology,把Technology融入到图表 、设计和应用中。

4, 安省为什么要对高中12年级数学改革?
数学课作如此大的调整主要有以下几原因:一、一直以来MCR4U和MGD4U两门课程都比较难,而工程和理科专业又对这两门要求很高,造成很多女生为此放弃报考工程和理科专业。二、MHF4U和MGD4U与11年级数学在内容上差距较大,所以把Advanced Function单独列出来作为一门从而与11年级的数学接轨。三、Calculus& Vectors组成一门新的难度较大的数学,为那些数学较好的学生选修。另外以前的Calculus由于较简单,使一些学生进入大学以后Calculus的基础不扎实,知识不够用,影响其他大学课程的学习,所以新的12年级Calculus& Vectors比以前的课程难度增加了许多,以便适应大学的课程。

5,为什么数学在高中阶段是最重要的课程?
在高中毕业所需要的30个学分课中,每个年级都要求学生修一个数学学分,四年高中数学可达到6个学分。如果你是一个数学高手,在大学要求的12年级6门成绩中,学生有3门数学(MHF4U、MCV4U、MDM4U)课可以选择,那么将大大提高学生12年级成绩平均分,增加绝对的竞争力。加拿大的名牌大学以及名牌专业几乎都会要求12年级至少两门的数学成绩,没有一个好的数学功底将会直接影响你的名牌大学梦想。
数学也是大学一年级的必修课,一个好的数学基础将保证你在高淘汰率的大学生活中立于不败之地。所以,只有一个好的数学基础才会有一个好的数学成绩,一个好的数学成绩将是进入名牌大学有力保证!

6,如何有效的提高12年级的在校成绩?
所谓的提高12年级在校成绩实际上是指提高12年级6门主要课程的成绩。除了在12年级更加努力学习之外,更应尽早的确定自己将来的发展方向,根据所学专业的大学要求,明确自己高中必须要修的课程,然后有的放矢重点抓好这些课程。例如:如果学生将来的专业定位是Life Science,学生一定要学好要求的有效课程:ENG4U、MHF4U、MCV4U、SBI4U、SCH4U。而不能选修一些大学的Life Science专业并不要求的课程,即使是成绩再好也不会对你的大学申请有任何的帮助,反而浪费了学生的精力。

大多数中国过来的学生,通常英语成绩并不象成绩始终不能像数学那样满意和自信。对于12年级的学生,在进入大学前的最后冲刺的关键时期,最有效的大幅度提高平均分的方法就是扬长避短,充分发挥数学优势,除了努力提高英语成绩的同时,12年级有可能的话选修3门数学课程,3门优异的数学成绩一定会大幅度提高你12年级的平均分,将为进入名牌大学的大门奠定有力的基础。

梁溪香榭 : 2009-03-13#585
回复: 加拿大教育情况有用素材收集帖子--梁溪香榭整理

这里有些帖子是俺以前在别的地方发过的. 一并归总到这里.

梁溪香榭 : 2009-03-13#586
回复: 加拿大教育情况有用素材收集帖子--梁溪香榭整理

(转) 毕业之际谈我的大学:滑铁卢大学

我是上期刚从滑铁卢毕业的数学院学生。在毕业之际,我想谈谈我上的这个大学。我并不打算写一篇系统的完整的有关滑铁卢的介绍,我只想从一个本科生的角度说一些只有我们本校学生才会有的感受。

  我认为,对于学生们而言,这种性质的述说比一切外校人的评论和大学排名榜更有意义。

  首先说说大学综合排名的问题。综合排名这个东西是很无聊的,学校关注综合排名是出于宣传的目的,其实是利益驱使。因为排名高的学校容易获得资助,容易获得生源。对于单个的学生而言,你本专业的地位远远重过你这所学校的综合排名。假如你是MIT学中文的,没错,MIT好啊,第一流啊,可对你的院系在中文研究领域里算哪根葱啊?MIT综合排名哪怕全世界第一又关你屁事?专业领域里谁也不尿你又有什么用?所以你们学校的本专业的地位才是最重要的。综合性大学固然好,但并不等于技术类大学就不行。这就好比军队,有的是集团军,编制庞大种类齐全,但未必每个兵种都好;有的象特种兵,专而精,也许不完善,但总有一两支拳头部队、一两项看家本领。硬要比较谁比谁好是可笑而幼稚的,因为这根本就是两种性质的军队。在合适的领域发挥自己的能力才是关键所在.

拿滑铁卢来说,滑铁卢一般不被当作综合性大学来看待。由于没有医学院法学院什么的所以无法与多伦多大学这样的学校拼综合实力,也没有那个必要。滑铁卢的长项在于数学院,主要包括Accounting, Actuarial Science, Applied Math, Computer Science, Cobinatorics & Optimization, Math Business, Pure Math, Satistics等几个系。以本科教育而论,无论是什么学校都不能在以上领域拍胸脯保证我一定比滑铁卢强。这就够了。比如你是学电脑的,滑铁卢电脑名满天下,IT业界人人首肯就可以了,就足以为你的IT前途打下良好基础了,滑铁卢没有法学院对你有什么影响吗?行外人士知不知道滑铁卢对你的职业生涯有影响吗?名声是假的,事业和事业带来的财富才是属于你自己的。你说滑铁卢Arts不强,对啊,一点儿没错,可你要想学Arts干吗来这儿啊?没谁逼着你上滑铁卢,对不对?多伦多大学的Arts很棒,你可以去那儿读呀?如果你没有能力进更好的Arts院系,或者因为选择失误而不得不来滑铁卢,那么对不起,这是你自己的问题,滑铁卢在数学与工程界的名气也帮不了你。需要说明的是,滑铁卢工程院的学生在素质和成就上胜过数学院的学生。当初招生的时候工程院的要求就高很多。可是滑铁卢的数学太有特色了,所以名气响一些。世界上把数学单独设成院,颁发数学学士学位的学校据我所知只有三所而已,而美洲大陆上仅滑铁卢一家。

  然而滑铁卢大学真正的王牌不是任何院系,是他的Co-op系统。那些连篇累牍讨论排名高低的人,还有那些眼光只盯在论文数量上的人,都是些秀才,他们的讨论对于即将展开职业生涯的学生而言是毫无意义的,除非你把自己定位在科研领域。Co-op简单的说就是半工半读吧。一个学期上课一个学期工作,上8个学期课工作6个学期。一年3个学期每个学期将近4个月,没有寒暑假(你可以给自己放假,休息个把学期,那是你自己的事情),一年到头连轴转,顺利的话4年零8个月本科毕业。工作由学校组织Employer来招工,能不能找到工看你自己的本事。不过你要是有两个学期没有工作,你的Co-op资格就完蛋啦,学校会把你赶出这个系统的。工作必须与本专业相关,你学工程的去刷盘子可不算的。每个学期工作结束后都要准备一份报告,报告在学校或者教授那儿通不过也不能拿学分的。一个Co-op的本科生毕业的时候就拥有两年的工作经验,而且这个经验远远胜过你在一家公司干两年。因为多数学生在6个工作学期(部分专业为4个学期)里会选择4到6个不同的职位,从而适应和体验4到6份不同的工作,收获不是固守一份工作可比的。虽然专心与一份工作可以让你更加深入,但在本科阶段,广阔的视野与多样的尝试显然更为重要。除非你认定自己一辈子的事业就是眼前的这家公司,否则死盯住一个职位连干6个学期是没有出息的做法,是没有胆量尝试改变的表现。Co-op工作是全职工作,一天8个小时,一星期5天。你可以用赚的工资补贴学费生活费。我的工资只能算学生里的平均数,也有20多块一小时,安排的好的话足够你搞定上学开销的。我在进大学后就再也没从爸妈那儿拿过一分钱,全靠自己赚钱完成本科学业,这是我迄今为止最自豪的两件事之一。

梁溪香榭 : 2009-03-13#587
回复: 加拿大教育情况有用素材收集帖子--梁溪香榭整理

Co-op给学生带来的好处众多。一是眼界,眼界对于事业的成败是决定性的,这个道理已经被一再地证明,请众多象牙塔里的学生哥不要再忽视这一点了;二是思考,有了眼界又有了实践的经验再加上学术知识才可以做实用意义上的思考。Co-op对我的意义在于:总算弄明白了我适合干什么、应该怎么干、能干出什么前途。有的人一辈子都没搞清楚这三个问题。本科生在毕业的时候具备对职业生涯的深度思考与可行的规划是罕见而且可贵的。而这在Co-op学生而言是基本要求;三是人脉,你从前的老板、同事都是你未来职场可用的Network。别的不说,一封推荐信,一个内部推荐绝对具有举足轻重的价值;四是经验,找工、面试、升迁、办公室政治,哪样不要经验?虽然两年的Co-op还远远不足,可比起一般的本科生来讲就强太多太多了;五是适应能力,一个公司雇你一个学期,总共4个月的时间,你要花3个半月适应谁还雇你啊?通常我是上班3到5天后就开始和其他员工承担相同的业务,不懂的边做边学,反正必须在最短时间内理解工作的内容与环境。而且你可以在两年里适应6次!如果你毕业后再这么两年适应6次,看以后谁还敢雇你?这种适应能力在未来的职业生涯里会带给你巨大的回报。你问我为什么这么说?你问达尔文去吧;六是债务减少,工资可以大量补贴学费与生活费。看看那些毕业后花几年甚至十多年辛苦还贷款的兄弟们你就明白这一条的意义了;七是生活能力,每个学期找一次工作,找到工作多半还要搬家,近的往多伦多、魁北克搬,中等的往美国和温哥华搬,远的还有往日本、欧洲、中国搬的。工作做完了再搬回来。很多学生害怕的找工作、找房子、搬家对我们而言是基本功,每个学期玩儿一到两次,生活能力当然强很多。

  滑铁卢目前在校的Co-op学生估计在5000人以上吧,包括工程院全院与多数数学院学生,其他院系相对少一些。可以想象当这些学生毕业后杀向就业市场的时候,他们占据了多大的优势。当他们与同年毕业的普通学生竞争的时候,总体看来基本就是一群人精和没长大的孩子们在比拼。这种Co-op教育的意义就在于让学生在面对社会之前先一步走向成熟与冷静,准备好了以后再真正踏上舞台,在这之前,你可以彩排多次。也就是说这种教育一旦错过就终生不可弥补。滑铁卢的学生素质主要来源与此,生源好还在其次。任何学校想要建立这个系统都必须付出巨大而艰苦的努力。滑铁卢作为Co-op的首创者,在全球拥有接近3000家雇用我们在校生的Employer,这样的规模是我们的工作人员用50年的时间辛苦努力得来的,是我们的学生用50年的时间一点点积累起来的。如果你是老板,你会放心把一个学生和其他员工一样使用吗?不经过长期检验,你敢把重要的工作交给在校的学生吗?可是你看,连国防、核电、军队研究所这样的要害部门也愿意雇用我们一、二年级的本科生参与项目开发,这种信任与名誉可不是一年两年就能建立的,那是几代人的出色表现赢来的。这就是为什么人人都知道Co-op好,可全世界没有一所大学可以把Co-op做得和滑铁卢一样成功的原因。现在有些大学开始认识到Co-op的优势,他们做的也很努力也很出色,不过还需要时间建立品牌。

梁溪香榭 : 2009-03-13#588
回复: 加拿大教育情况有用素材收集帖子--梁溪香榭整理

所以滑铁卢的本科教育质量根本不是什么综合排名或者学术论文可以决定的。在这些东西上硬钻牛角尖,以本科生的实际需求看,非常无聊、非常幼稚。我个人不认为滑铁卢是什么世界名校,但他的Co-op毕业生的素质相比名校毕业生只高不低,在数学与工程领域更加突出。很多人奇怪这个学校那么多人称赞,可用传统的评价标准看他并不怎么样,原因就在于Co-op。我可以这么说,滑铁卢之所以是滑铁卢就因为他的Co-op。没有了这个,滑铁卢只能说是优秀的理工科大学,但没有什么鲜明的特点与绝对的优势。所以我时常奉劝想要考滑铁卢的高中生:如果你不想做Co-op,就不一定非选滑铁卢。

关于滑铁卢的校风,我压根儿没听过还有校训这一说。什么“严、勤、实、活”之类的屁话在这里没听讲过。我对滑铁卢校风的总结是“务实”。非常现实的一群人聚在一起就是这个样子的。每个人有自己的爱好和规划,你做的好大家夸一声,,然后继续干自己的事情。你要做衰了,大家表示一下遗憾,你要还继续衰着,也就没人理会你了,你就完蛋了。你要是个书呆子,爱当电脑狂人,那就去吧,祝你好运,我会在适当的时候嘲笑你一下,拿你解个闷,然后就忙我自己的事业去了。这里没有主义之争,没有激烈的哲学思辩,这儿就是一伙为自己的将来不停打算不懈努力的人,是一群把读书当作过日子的人,非常非常的现实。我接触过的所有大学生里只有滑铁卢学生的精神状态最接近现实世界里的人,或者说,最不象学生。这种精神状态和Co-op有重大关系。它的优点体现在成熟、体现在积极的职业准备、体现在创业的热情、体现在踏实的作风。而他的缺点在于缺乏人文的内涵,有的时候甚至现实到冷漠。

  关于这两天热炒的ACM竞赛,滑铁卢的学生们不会明白这有什么好炒作的。滑铁卢是ACM的老主顾了,成绩从来就很棒。我记得至少在1994年与1999年拿过冠军,后来有没有拿冠军就不知道了,因为我进了大学后就不再关心这些事了,周围也没人关心。每次滑铁卢在ACM得奖我都是从报纸上知道的。我自己在电脑系呆到三年级才转专业,可我从没听说过有什么ACM校内选拔赛之类的东西。每年都得奖,前三前五跟玩似的,可都谁得奖了,我们系里没人打听,还是从学校报纸上看到的名字,具体是谁不晓得,谁耐烦打听这种三八事情?得奖的也没听讲有谁到处招摇的,因为在大家看来,ACM奖项不过是滑铁卢学生众多荣誉的一个,不是什么了不起的大事。ACM对于个人的现实意义无非是下一次找工作更有竞争力,可竞争力的加强有多种方式,何况编程不过就是份IT苦工罢了。对于没得奖的多数学生而言,更加没有价值。雇主要看的是我的经验与技能,以此决定我的地位与报酬,与那几个得奖的人有什么关系?ACM的名次对我们多数人而言就是偶尔碰上有人聊到ACM的时候,可以地说一句:“我的大学在这个竞赛里成绩很好。”吹嘘一番,洋洋得意一下。对我个人的素质培养与职业发展没有任何好处与坏处。我认为这是滑铁卢校风的体现。

梁溪香榭 : 2009-03-13#589
回复: 加拿大教育情况有用素材收集帖子--梁溪香榭整理

有趣的是很多Co-op学生结合自己的经验并不认为ACM有多大的价值。我的一个朋友前两天问大伙关于上海交大拿ACM冠军的看法。有人说:“将来都是一群去美国留学然后打IT工的Cheap Labour。”这个话说的有点尖刻,但我们的共识是:编程序就是苦工,而且会越来越苦,属于蓝领职位,如果是单纯为了钱就不必了。当然这是个人意见,可比较明确的是职业发展必须符合个人的条件,会编程充其量只能说你多了一项工具,编程水平的提高对于IT产业的发展也只是有帮助而已,离现代化的IT产业还有重大差别,这就是为什么印度严格的说不是高科技发达,只是软件发达而已。炒作出那么大动静是误导学生误导社会,是中国媒体的傻B行为。我对于这位兄弟的话有另一番意见:“只怕将来他们留学去美国之后连Cheap Labour都没的做,都外包到中国和印度去了。”这个话,一半算是开玩笑吧。

  在这里我要为我们滑铁卢的小留学生们说两句话。我认为他们都是好样的。我这个人比较喜欢交朋友,我比较亲近的中国小留学生至少有五、六十个。我可以负责任地说:“他们中99%都是优秀的人才,出色的学生,值得交往的朋友。”你去看看我们数学院里那些成绩拔尖的学生,平均分靠近和超过90%的那些牛人,有几个不是中国大陆来的小留学生?我只见过三个平均分过95的牛人,全是我们的小留学生。95分在滑铁卢是什么概念?我来告诉你:滑铁卢数学院聚集了全加拿大最好的数学苗子。可是在我入大学的时候,这些顶尖的数学苗子们平均毕业率只有一半,四年级课程的考试75分就能拿全班第一。四年下来,四五十门课平均超过95分是什么概念,您自个儿掂量吧。坦白地说,留学生们读书比我们这些本地学生要刻苦,成绩也更好。为什么?他们一个学期的学费将近9000加元,差不多是我们的三倍,法律还不允许他们打工挣钱,全靠家里提供,而且还是父母在国内拼命工作把人民币换成加元!谁他妈的在这种情况下还不努力读书,丫儿就是一杂种!读的好不好要看天分和方法,好不好好读就看你有没有良心了。有良心好好读书的人,人品也不会太离谱的。

  滑铁卢的学生被加拿大各行各业投票选为“明日的领袖”。我对此持保留意见。滑铁卢未来的声望也许要靠一群特殊的学生来维护。这些人用一个简单的词来概括就是“混混”。混在滑铁卢是一门学问。混混们成绩都不高,马马虎虎一般话,基本大概勉强行。课基本是翘,作业基本是抄,考试基本靠突击,最牛的不毕业就跑。这些人以他们的智慧绝对可以把分数搞得惊世骇俗,引来无数惊叹。可他们不屑于此。他们有自己的哲学:大学教育的目的不是把书本知识变成习题和考试,大学教育在于培养人的各种能力。书本知识只是个基本要求,能毕业就是了,学生有权力根据自己的特点打造自己的能力,并不一定要以学校的标准来衡量自己的得失,学校设置的毕业标准只是一个最基本的要求。混混们通常善于交际,善于领导,善于组织,善于结交朋友,善于谈判,善于分析判断,善于打破常规,善于放眼未来,善于纵观全局,善于总结他人,善于检讨自己,就是不善于考高分。不过关于分数,这些人也有一绝:善于在不得不读书的时候勉强自己做自己不喜欢的事,并且达到标准。没达到标准的全被淘汰了,留下来的混混们才是真正的人才,而且都是大材。据说网易的丁磊就属于这号学生。所以我更加欣赏这些人。学校里教的东西有多少是你将来在工作中肯定会遇到的?有哪门课能证明差二十分这个人的职业就没发展希望了呢?绝大多数情况下,课上的东西与实际工作的关系并不是特别密切的,领略到基本思路具有自学的能力就好了。因为Co-op的原因,滑铁卢的学生对于这个问题发现得更早、更多,所以混混们就更多。混混们的业余时间通常花在学生组织上,因为这些地方练人。关于这个我的体会特别深,收获也特别大,但我不打算多说,因为不管我怎么说,自己不去做是不会理解的。需要说明的是那种不好好读书只顾打游戏的不在我说的混混之列。

梁溪香榭 : 2009-03-13#590
回复: 加拿大教育情况有用素材收集帖子--梁溪香榭整理

我们的小留学生们应该适当地加强自己在学生社团里的活动,滑铁卢上百个学生社团,值得你们贡献与学习的有很多。分数高并不等于能力高,更不等于学到了精髓。你要只想学那些公式,何必出国留学呢?那些东西国内大学也教的嘛!我们的小留学生们,恕我说句冒犯的话,总体来看,你们的英语水平实在太差太差了。固然有个别好的,但总的来说实在不像话。打个电话都不敢还要人代劳,这不是个别现象吧?一听说作演讲就紧张得出汗,这不是我胡诌的吧?成天打听那门副课语言要求高,能躲则躲,这不能算有出息吧?你们这个样子哪怕平均分100也不能给你们带来好的职业生涯。学校专门为语言不好的新生设立了辅导课程,简称ELAS。这个课程让你在一个学期里专攻英语,专人辅导,手把手地教。口语课的作业录在磁带上,老师听了以后照着原稿给你一个单词一个单词地标出发音和语气的错误,发下来后还让你订正了再交上去,直到过关。演讲课教你怎么连读、怎么弱读、怎么强调、怎么使用肢体语言,抠细节抠到每个音节、每个动作。除此之外还教你们写论文、课堂速记、民间俚语、文史背景知识。这么好的课程,只要多花一个学期就能完成,你们怎么就一口咬定是骗你们的钱?怎么就混个及格呢?你们可以把数学考到100分,却不愿意在最要害最关键的地方下功夫,怎么这么糊涂呢?你们不学语言出国干什么呀?你们知不知道这有可能是全世界最实用最细致的英语教育课程,你们从此以后再也不会遇上其他的人这样努力提高你们的英语了。我也是从前ELAS的学生,4个月的课程让我的英语脱胎换骨,最重要的是突破了瓶颈,赢来好大的提高空间。我可以说我在滑铁卢学过的最有价值的课程就是ELAS,我将为此终生受益匪浅,而你们,很遗憾,已经没有这样的可能了。希望你们在以后的日子里想方设法加强语言能力,也希望后面的师弟师妹们看了这篇文章能吸取教训。

  接下来我要说的话也许是滑铁卢的学生们最不想听的,也许这是网上对于滑铁卢最严厉的批评。但我还是要说,因为这总比那些不懂装懂的外路人故作专业地胡说八道要强得多。

梁溪香榭 : 2009-03-13#591
回复: 加拿大教育情况有用素材收集帖子--梁溪香榭整理

第一,滑铁卢没有精神。务实不是精神,是人面对现实世界的自然反映。过分现实的人通常是冷漠自私的小人。滑铁卢的学生们不算冷漠自私,可有些冷淡。伟大领袖他老人家有句话“人是要有些精神的”。人文内涵的缺乏使得这个学校的毕业生更象是谋生者或者逐利者,不停地为利益所驱策,眼光短浅,循规蹈矩,不具备开创世界的勇气与智慧。这就是为什么我对“明日领袖”这些话持保留意见的原因。比尔盖茨说滑铁卢是他最中意的学校,他最喜欢滑铁卢的毕业生。在为这句话欢欣鼓舞之前,我必须提醒你们,说这句话的人在说这句话的时候不是什么IT界的泰斗,而是一个不折不扣的资本家。知道他为什么说这句话吗?我要是资本家我也喜欢没有思想只认利益能苦干肯拼命又循规蹈矩的苦力!滑铁卢的名气很响,但是在他建立起属于自己的独立人格之前,我不认为他是个名校。

  也许你要问我,人文涵养与技术专业有什么关系?这就是滑铁卢众多学生的误区。没有精神内涵你就是一个机器,只配给别人当枪使,还是一把想用就用想扔就扔的枪。CIA在调查美国科学家为什么在武器研发上总是落后于苏联科学家的时候作出一个结论:双方在各方面都极为接近,包括技术基础与政府支持等等,唯一重大的差别在于苏联科学家们在古典音乐上造诣非常高,而美国科学家通常只听些个流行摇滚。CIA认为这是原因所在。滑铁卢的学生们应该仔细体会一下。

  第二,走向迷茫。滑铁卢究竟是技术类大学还是综合性大学?这个问题这些年在领导层一直摇摆不定。所以走出来的路扭扭捏捏浑浑噩噩,很难从这些年滑铁卢的建设里看出清晰的发展思路。似乎是想搞成综合性大学,可又把资源主要配置在数学与工程。今天说要保持数学的优势,明天要打算让工程院独步天下,过些日子被综合排名一刺激又觉得要搞好Arts,还要建立自己的MBA学院和医学专业。发展的路上既缺乏明确的目标,又完全没有自己的主见,更没有坚定不移的信念,好像一个色鬼来到女儿国,看看这个也想追,瞧瞧那个也想泡,魂不守舍头晕目眩。这个样子是搞不出好结果的,和中国那些疯狂并校的理工科大学犯的是同一个傻B毛病。

  这些年搞扩招。好,我同意,大学应该尽可能地搞平民教育,提升全民素质,不是培养个别几个精英就成了的。可你扩招之后该有相应的策略吧?从前课程难度过高,因为那是为少数精英设计的课程。现在学生中精英还在,可毕竟平均素质下降了,还用过去的教学方法,你想让毕业率掉到30%啊?这样以后谁来啊?放松难度吧,又怕毕业生素质下降。既要扩招,素质必然下降,这是必须面对的现实。有胆扩招多收钱,没胆承担必然的后果,实在是很没出息的做法。在这个问题上,各大院系表现得象个娘儿们一样犹犹豫豫,最后糊弄出一个保持难度大家加分的办法。考砸了就大家一起加分,好像这样就又能保持难度又能维持毕业率了。掩耳盗铃不知到底想骗谁。

梁溪香榭 : 2009-03-13#592
回复: 加拿大教育情况有用素材收集帖子--梁溪香榭整理

第三,狂傲与唯技术论。骄傲是可以的,每个人都为自己的团体所骄傲。狂傲是不能接受的。举个例子:2002年还是2003年的CUTC(Canadian Undergrad Technology Conference),我们的学生参加一个讲座。因为是高科技讲座,所以现场滑铁卢的学生占绝大多数。开始之前滑铁卢的学生高喊Waterloo为自己的母校扬威。可当会场里唯一的三个Western Ontario的学校高喊他们母校名字的时候,我们的学生利用人数优势大喊“Sucks”!这就过了!大街上流氓打架还给人留面子呢,你们这些一流大学的学生比小痞子还不如。从这件小事上可以看出滑铁卢的学生在专业领域里的狂妄。仅此一点我就断定这些人没有出息,都是些没有容人之量的井底之蛙。这件事背后的根源是滑铁卢学生里由来以久的唯技术论。老是以为只要我的技术出色就可以包打天下,就可以横着走,就可以颐指气使,尤其是白人学生和我们本地的中国学生。这种短浅的见识是将来失败的伏笔。我还是觉得混混们才是滑铁卢最有希望的一群人。可是多数混混们和我一样并不十分认同滑铁卢的作风。

  第四,对于政治与社会的漠视。滑铁卢的学生们不关心政治与社会是出了名的。比如说学费,年年涨。我进大学的时候电脑系5门课要$2600,现在呢?奔$3600去了吧?更别说那些留学生的学费了,他们的涨幅可是从$6000到$9000啊!可我们的学生们除了背后嘀咕之外有过任何抗争行动吗?多伦多大学的TA可以为了学费涨价而罢工,并且赢得了胜利。而我们的学生会在干什么?有谁出面组织过抗议?为什么没有?我们的学生会主席虽然是选的,可他的工资是学校发的!所以面对高涨的学费学生会象阳萎一样闷不作声。当面不敢抗争,背后窝窝囊囊地抱怨,这就是一群非常现实的人聚在一起时的典型镜头,没种!

  你也许会说:“MIT也是理工科大学,他们从政的也不多,他们不一样名满天下吗?”你知道MIT和滑铁卢最大的差别吗?MIT是私立学校,而我们是公立学校。知道什么叫公立学校吗?就是说你是靠政府赏饭给你吃的。在政府里没有自己的代言人会是什么下场,你们自己想吧。工人们要组织工会是为什么?忽视政治与社会的力量必然会给滑铁卢的学生们带来苦果。而且,如果一所大学里的学生缺乏对社会的关心与责任感,我不明白社会为什么要供养这样的大学。

  总的说来,我觉得滑铁卢现在处在转型期。他目前既不在下降也不在上升,是爬上一定高峰后走的短暂平路。至于以后是上升还是下降,就看他现在怎么走。目前的步伐很乱也很慢。滑铁卢必须尽快找出一条保持传统又能添加新活力的路子。Co-op是灵魂,千万不可忽视。

  最后我想说,刚刚讲的这些缺点肯定不是滑铁卢所独有,差不多每个学校都有不同程度的表现。但是用滑铁卢务实的眼光看,其他学校有什么毛病那是他们自己的事情,跟我们无关。一个处在发展中的机构应该把眼睛盯在自己的缺点上,少去八卦别人的短处.

abc66 : 2009-03-14#593
回复: 加拿大教育情况有用素材收集帖子--梁溪香榭整理

好贴,好妈妈,:wdb17:,加声望

梁溪香榭 : 2009-03-15#594
回复: 加拿大教育情况有用素材收集帖子--梁溪香榭整理

受益匪浅,太好太好了!
隆重谢谢梁溪(名字真好听),还有斑竹守法(听着像法律板块哦)!
前阵子在网上溜达,看到这个贴,马上收藏了,后来就打不开了,再后来被斑竹费劲捞回来了,好感动哦。
继续关注中。。。

好贴,好妈妈,:wdb17:,加声望

刷卡 :wdb6:

Luck Li : 2009-03-15#595
回复: 加拿大教育情况有用素材收集帖子--梁溪香榭整理

溪溪又更新了怎么多,呵呵!辛苦了。

pikexima : 2009-03-17#596
回复: 加拿大教育情况有用素材收集帖子--梁溪香榭整理

更新速度好快,不愧是网络时代,梁溪辛苦了啊!

滑铁卢在我孩子他们班上(小学)名气好大,喜欢计算机的孩子都说将来一定要去滑铁卢。提到哈佛,他们会毫不犹豫地回答,不!???!%¥#@

梁溪香榭 : 2009-03-20#597
回复: 加拿大教育情况有用素材收集帖子--梁溪香榭整理

(转) 我在麦克马斯特


  2003年我以优异的成绩被麦克马斯特大学商学院录取,并且获得了入学奖学金。大一,大二出来商业学专业规定很多的必修课课程,选修课程则相对较少。必修课包括微积分,心理学,商业学,市场营销学,会计学,人力资源学,经济学,计算机科学,地理学等。其他的选修课,学生们可以从多大100多门的课程里面选择自己喜欢的课程进行学习。在加拿大的大学里面大一和大二的淘汰率是最高的,学校通过规定一些难度很大的必修课程来淘汰一些虽然已经入学但是成绩不达标的学生,这些学生被淘汰到其他系里面,如果学生在该系里面也没有达标,学校则会勒令其退学。因此,当我每升一年的时候,都发现原来的一些同学都已经转系了,后来我从一位老师那里了解到了,麦克马斯特商学院大一的淘汰率是30%,大二的淘汰率是15%,大三的淘汰率是8%,而大四的淘汰率则降低到了3%。麦克马斯特商学院之所以这样做是为了保证其毕业生自身的学术质量。通过大一和大二的学习,学生可以很明显的感觉到麦克马斯特大学是一所以group work著称的大学,几乎所有的课程都设置了group work,学生被要求以团队的名义完成report,lab research,presentation等课程作业,在每次group work后,所有组员都要互相评分,老师会依据各组员的得分和该组总成绩给每个学生打分的,group work评分的重要准则是劳者多得,不劳不得。

  到了大三,学生可以选择就读科系的方向了,比如金融,会计,市场营销等。我当时没有选择一个明确的方向发展,我自己的想法是关于商业的所有学科我都要学习,所以就选择了generalized commerce(如果选择某一专业,则叫做major in XXX)。作为商业学,大三和大四的必修课很少,两年的必修课一共只有6门,international business,game policy and strategic thinking,marketing,corporate finance ,cost accounting和intermediate financial accounting。此时,最大的变化就是必修课的小班授课,小班授课对于中国学生最大的挑战就是class participation(课上参与),class participation考验的是学生瞬间分析的能力和反应的速度。对于英语为第二语言的中国学生来说,我们在讲英文的时候总是先要自己默默在心理组织一下语言,然后在张口说出。就是这个在中国人看起来司空见惯的习惯,却耽误了我们很多回答问题的机会,因为当地学生是没有组织语言的这个程序的,所以我自己总结出来的经验是,如果听到一个问题,而且自己对这个问题有独特的见解,第一时间抢答,当你拥有了第一时间对这个问题的回答权利时,你可以大胆的说出你的想法。切记,在课上讲英文的时候千万不要胆怯,千万不要害怕有语法错误,在这方面,加拿大的教授们考虑的只是你对某些事务的分析能力,而不是使用多么华丽的语句。

  从大三到大四的感觉又有了新的变化,大三每天泡在图书馆里面啃书的场面没有了,而是换成了以group为单位的几个学生在校园各个角落嬉戏和学习的场景了。我之前讲过McMaster是一个以group work闻名的大学。到了大四更是如此,大四很多课程评分标准都是以group为准的,所以就没有以前那么多的test了,从另一方面来说,以group为准的演讲和网上模拟却大幅度的增加了。Game policy and strategic thinking有一个模拟,这个模拟是5个人一组负责一个公司的研发,生产,人力资源管理,财会和市场策划,也就是说没人扮演这个公司的一个部门经理和其他group的公司进行竞争。这个模拟让我们知道了一个公司运作和组织协调的重要性。我们group是在争辩和争吵中结束了这次模拟的,因为在一个公司来讲,每个部门都有自己的预算和计划,这些预算和计划都是互相关联的,有些时候还是互相矛盾的,所以这就不能避免的导致了各个部门之间的争辩。然而识大体顾大局才是这次模拟的真正目的,就像古人说的,1根筷子很好折断,可是当你把5根筷子攥到一起时,这股团结起来的力量是很难被打破的。

  4年的大学生活让我感受最深刻的就是要想到达目标就必须付出巨大的努力,其中的艰辛是常人想象不到的,但是这些磨练也是我自己人生中最难忘的。

梁溪香榭 : 2009-03-20#598
回复: 加拿大教育情况有用素材收集帖子--梁溪香榭整理

2005年度全球十大商学院

1.哈佛商学院美国

  1.宾州大学沃顿商学院美国

  3.哥伦比亚商学院美国

  4.斯坦福大学商学院美国

  5.伦敦政经学院英国

  6.芝加哥大学商学院美国

  7.达特茅斯学院塔克商学院美国

  8.欧洲工商管理学院法国、新加坡

  9.耶鲁管理学院美国

  9.纽约大学斯特恩商学院美国

梁溪香榭 : 2009-03-20#599
回复: 加拿大教育情况有用素材收集帖子--梁溪香榭整理

世界大学金融专业的排名

1 UNIVERSITY OF PENNSYLVANIA (Wharton) :
宾夕法尼亚大学(沃顿商学院) 特色:属于PHD课程, 学制四年,仅有秋季入学. 只接受在线申请. 众多的学生俱乐部和各种学术会议,并且经常有机会跟全球著名商业领袖互相交流,
条件:学士学位,专业没有明确限制,GPA平均3.5分(3.1--3.9),需要TOEFL/GRE或者GMAT(各部分都至少在90%以上);简历;面试;两封推荐信;平均5--6年的工作经验。

2 University of Chicago
芝加哥大学 特色:属于PHD课程,学制五年.是美国最早培养潜在的学者成为全球有创新能力的研究者的大学. 在整个课程中始终贯穿实际调研和实习。
条件:学士学位,GPA平均3.5分(3.0--3.9),托福600/250/100,接受IELTS成绩,GRE/GAMT(更愿意接受GRE成绩),没有分数的限制;小论文;面试;推荐信;平均5--6年的工作经验;简历。

3 New York University (Stern)
纽约大学(斯特恩商学院) 特色:属于PHD课程,是全世界最好也最具竞争力的课程之一,内容主要涉及全球金融管理和金融市场. 是被认可的最早开设金融课程的院系之一,也是全世界最大最著名的金融课程的学院.学制五年。 需要同时进行在县申请和材料寄送.要求申请人具有很好的金融分析、会计和定量分析的能力. 课程的学习可以帮助学生在金融领域、财政机构和政府或非赢利机构找到适当的工作。
条件:学士学位,没有最低的GPA、TOEFL和GMAT的要求;提供小论文;面试;两封推荐信;但是非常看中申请人的学术潜力、职业历史和目标、个人和职业素质等。

4 Columbia University (NY)
哥伦比亚大学 特色: 属于PHD课程,只有秋季入学,学制4(硕士)--4.5年(学士学位,需要先上一部分准备课程.
条件: 四年学士学位,相关专业背景, 但是更注重学术成绩;TOEFL:85分以上; 只接受GRE(定量部分平均793分),两封推荐信,ESSAY,简历;面试;能根据成绩提供奖学金。

5 Stanford University (CA)
斯坦福大学 特色: 属于PHD课程,学制4年,适合杰出的学者或者在商学院的研究领域任教的学者;只有秋季入学,只接受在线申请
条件: 四年学士学位,虽然没有明确具体的专业背景和学术要求, 但是更注重定量分析的能力; TOEFL:600/100/250-300; 更愿意接受GRE,没有最低的GRE/GMAT成绩要求,不要求工作经验;面试;一旦录取提供奖学金

6 Massachusetts Institute of Technology (Sloan) :
麻省理工学院(斯隆管理学院) 特色:属于PHD课程,学制四年,毕业生基本上在全球著名的大学里任教;
条件:申请人在经济学\行为科学\统计学和应用数学方面有很强的学术实力,要求提供各种考试成绩,推荐信,有关自己详细的陈述,兴趣爱好和职业目标;TOEFL:577/273/90--91;要求ETS寄送成绩单;学士学位; GRE/GMAT,

7 University of California?Los Angeles (Anderson)
加州大学洛杉矶分校(安德森商学院) 特色:属于PHD课程,学制四年,毕业生基本上在全球著名的大学里任教或者在著名的大企业里担任重要职位;
条件:至少学士学位,;平均GPA3.7(本科)--3.9(硕士,不要求工作经验),TOEFL:不低于560/220/87,GRE:平均650/790/780/5.0; GMAT:平均700; 三封推荐信;不需要面试

8 Harvard University
哈佛大学 特色:属于PHD课程,学制四年--五年,
条件:至少学士学位,;TOEFL/GRE/GMAT,没有明确分数要求, 三封推荐信;

9 Northwestern University (Kellogg) (IL)
西北大学(凯洛格商学院) 特色: 属于PHD课程, KELLOGG的金融系在金融经济学领域中的研究处于领导地位,研究和实践的领域非常广泛.而且给有资格的学生提供各种工作机会.(如助教和研究助理等):
条件: 四年学士学位,要求GRE/GMAT,需要金融\经济等背景,或者相关的工作背景,同时也接受数学背景的申请人.

10 University of California?BerkeLey (Haas)
加州大学伯克利分校(哈斯商学院) 特色: 提供MFE(金融工程)专业,学制一年,只有春季开学;是商学院赞助的唯一的一个金融工程课程。: [& C. a: Q1 b
条件: 四年学士学位,没有明确的专业背景要求, 要求相关数学理论和工作经历(平均4年);面试; TOEFL:570/230/68以上,需要GRE(要求数学专项考试)/GMAT,其中定量部分至少90%以上;GPA3.0以上; 两封推荐信; 要求ESSAY。
总结:大多数属于PHD课程,对于申请人的学术背景要求严格,尤其是数学分析计算能力,普遍具有相关工作经验;不适合本科应届毕业生申请.毕业生一般在国际上著名大学里任教.

pikexima : 2009-03-21#600
回复: 加拿大教育情况有用素材收集帖子--梁溪香榭整理

再道梁溪辛苦了!

滑铁卢那篇分析得尤为到位,拥有自己独特的精神----“魂”,才是一所大学繁荣昌盛之本。

梁溪香榭 : 2009-03-21#601
回复: 加拿大教育情况有用素材收集帖子--梁溪香榭整理

再道梁溪辛苦了!

滑铁卢那篇分析得尤为到位,拥有自己独特的精神----“魂”,才是一所大学繁荣昌盛之本。

是啊, 俺也喜欢那篇分析.

梁溪香榭 : 2009-03-21#602
回复: 加拿大教育情况有用素材收集帖子--梁溪香榭整理

(转) 在加拿大IVEY商学院读MBA的日子

IVEY 商学院的教学楼被阳光拥抱着。 灰色的城堡式建筑散发着沧桑的味道。 在学校的教室里,初春的温柔的阳光,穿过米色的百页窗,落在我的手上。 手指偶尔从笔记本电脑的键盘上抬起。 我的目光也随着抬起,望一下窗外清澈的蓝天和天空下灰色的树。

来到加拿大已有两年了。 我是来加移民中的幸运儿。 在踏上加拿大土地的二十天后,就开始了在这里最好的商学院读书的日子。 而现在,二十天后,我就要离开这里,转迁多伦多。 开始将要在那里的工作。 回首在IVEY的日日夜夜,我的心里百感交集。

一、来到加拿大

1999年7月15日,当我所乘的飞机在多伦多上空盘旋降落时,地面上无尽的灯海向我的眼前扑来。 这是我第一次出国,第一次看到地面上有这么多无边无际的灯火。 我拉着坐在我身边的先生的手,我问他,我们就要融进这个城市,我们会是其中的哪一盏灯火呢! 兴奋,好奇,焦虑一起涌上心头,而更多的是迷惘,不知前方等待我的是什么。 我想这是每一个新来的中国移民的真实心态。

到欧美读MBA是我多年的夙愿。 选择来加拿大读MBA实在是一举几得。 首先是免去了申请签证的痛苦过程。 我的一位老友曾连续三年被美国优秀MBA学校录取,却连连惨遭拒签。 每逢提起,她都不堪回首。 第二,在加拿大读书,学费便宜。 第三,在读书的同时,兼坐“移民监”。 毕业后,只需要一年,就可稳拿公民。

IVEY的排名无疑是我选择她的主要原因。 IVEY在加拿大商界是排名第一的老牌商学院。 她始建于1921年,位多伦多东南的伦敦市,是该市西安大略大学的一部分,IVEY完全按照哈佛商学院的教学模式,百分百采取案例教学。 我们在学校期间,共研究了七百多个案例。 内容涵盖了公司经营的每个方面,包括市场决策,宏观决策,融资战略,财务会计,统计,人力资源管理,等等。 IVEY出版的案例是除哈佛以外第二个在北美被广泛采用的案例。 在英国金融时报1999年及2000年的全球商学院排名中,IVEY名列第19名。 是前二十名中唯一的加拿大商学院。

梁溪香榭 : 2009-03-21#603
回复: 加拿大教育情况有用素材收集帖子--梁溪香榭整理

(续) 在加拿大IVEY商学院读MBA的日子

二、来到伦敦市 初建家居

  1999年7月20日,在离开多伦多两个小时后,我们搭乘的汽车离开了401高速路,驶入了伦敦市。这个清凉安静的小城给我的第一印象是荒芜偏僻的。没有北京的林立的高楼,只有茂密的绿树掩映的小屋;没有多伦多的摩天大厦,只有零星的孤单的公寓楼。唯有涓涓的太唔士河和水里成群的喧闹的野鸭给小城添一点人气。后来我才知道夏天是伦敦一年中最安静的时间。八九月份回校的学生将会让这 个城市变得生气勃勃。

  和刚来的新移民一样,没有汽车,没有工作记录,没有社会信用,没有银行户头,没有驾驶执照,没有社会保险号,也没有医疗保险。而这些却是生活在加拿大所必需的。我的身上唯有的只是一本中国护照,一张移民纸,和几十张百元美钞。而这些在我们进入这个国家后,就变得毫无用处。

  尽管万事开头难,但当生活从零点开始时,每向前进一步,都是一分意外的惊喜。比如说,当在超市里终于明白,那个身上没刺,个头很大的原来就是黄瓜;当在酷日下徒步几小时找房子,突然感受冰冻可乐透心凉的过人妙处;当骑着簇新的自行车,在上下起伏的居民区里寻找家具时,发现两个轮子确实比两条腿省力而且走的远;当用三块钱买到洋人老妈留下的硕大台灯,一块钱买到四个美仑美奂的玻璃酒 杯时的窃喜;当卷起硬地板上的被褥,开心的想象着刚搬进的床垫的舒适和柔软;还有当在新居里第一次吃到白菜猪肉饺子,手赶面条,大米饭和京酱肉丝的狂喜……………。。

  在孤身来到一个陌生的城市,没有朋友和亲人时,生活给人感觉是冰冷的。一个朋友告诉我,当他在伦敦市中心热闹的十字路口驻立时,身边熙熙攘攘的人群,只让他感到孤单;旁边咖啡馆里金发碧眼的人们的谈笑,只让他感到别人的欢笑与自己无关。在那一刻,他突然感到自己像是一颗无端被抛出的石子,不知道为什么来到这里,到这里要做什么,怎么做……。

  在举目无亲时,朋友是我的亲人。要是没有二年级RICHARD的帮助,我是没有办法找到朋友的。我还清楚地记得,第一次在小熊家吃饭,听小熊用那口上海味浓厚的普通话,向我们介绍他的超级迷你电饭锅;还有第一次去图书馆,碰到NATHAN操着正宗北京话,热情的在伦敦地图上给我们指点迷津……。我感到我在伦敦的生活,因为有着越来越多的朋友,而慢慢温暖起来。

  在经历种种磨难后,我们终于在加拿大伦敦市快乐地安家了。

梁溪香榭 : 2009-03-21#604
回复: 加拿大教育情况有用素材收集帖子--梁溪香榭整理

(续) 在加拿大IVEY商学院读MBA的日子

三、来到IVEY

  1999年8月10日,我参加的PRE-IVEY PROGRAM开始了。这个为期两周的课程是专为国际学生设计的,目的在于帮助国际学生迅速适应新环境,为即将开始的学习做好准备。这个课程告诉我们如何克服文化障碍,如何适应加拿大寒冷气候。课程还介绍了学校历史,案例教学法等等。组织这个课程的ELLA STRONG女士,也曾在北京,上海组织过IVEY MBA介绍会。她善良助人,对国际学生非常热心,深受大家的爱戴。

  国际学生的比例高低是国际商学院排名的重要因素。IVEY的国际学生高达30%。同学来自墨西哥,秘鲁,委内瑞拉,日本,新加坡,香港,台湾,中国大陆。我惊奇的发现,来自大陆的新移民占了国际学生的大多数。在IVEY当年招收的240人中,来自中国的就有将近50人。这个数字远远超出了上届的15人。这无疑是和最近几年的大陆移民潮密切相关的。

  ORIENTATION WEEK开始时,越来也多的本地同学出现了。这一周的活动非常丰富:白天是热闹的户外运动竞赛,头脑奥林匹克,晚上则是载歌载舞的卡拉OK,蒙特卡罗博彩之夜和魔幻之夜。本地同学表现得非常踊跃。在激烈的运动赛后,同学们在运动场旁搭起的大舞台上以小组为单位表演自编自演的歌舞,争取最后一轮的好分数。小组成员的安排是随机的。我的组里有银发碧眼的SANDRA,说话特别快的AM Y,还有极为淑女的ANDRA。我和另外一个中国同学因为文化不同,没有办法参与讨论,只能等她们商定表演的歌曲,再现学现唱。

  原以为自己在外语学院和外企混了这许多年,应该不会有文化冲击(CULTURE SHOCK)。可现在深入其中,才明白文化冲击是深层次,全方面的。比如说做为一个从小生长在中国的人,是没有可能知道本地同学在小学读字母课的趣事,也不会了解他们从小唱到大的儿歌。做为一个从小看排球,羽毛球长大的中国人,是没有机会了解加拿大人疯狂热爱的冰球,CURLING。而解决文化冲击的唯一办法是多读,多问,多看,多听。而这一点,说到容易,做到却很难。

  1999年9月3日,IVEY教学楼一层的大厅坐满了MBA2001的新同学。这个新生的群落共有240人,平均年龄29岁,平均GMAT成绩660分。他们肤色各一,语言不同。国际学生超过30%,所用语言超过40种。他们的工作背景更是十分丰富,从投资银行分析员,商业银行专员,股票经纪,到工程师,药剂师,医生,还有地质学家,戏剧作家。

  坐在这样一个多姿多彩的群落里,我又是兴奋,又是激动,又是好奇。回想为申请移民所经历的漫漫等待,回想在酷暑里苦读GMAT的日子。我深切感到我能来到加拿大,来到IVEY,是多么的幸运。我还隐隐感到,读MBA的日子将会比我以前所经历的更有挑战性,更艰苦,也更有回报。

梁溪香榭 : 2009-03-21#605
回复: 加拿大教育情况有用素材收集帖子--梁溪香榭整理

(续) 在加拿大IVEY商学院读MBA的日子

四、第一个月

  早晨八点,我急急地钻进我的教室,坐在早已安排好的座位上。这是一个阶梯式的环形教室,淡淡的藕荷色的墙壁和谐地衬托着深色的藕荷色的CHAIR。教室中央的墙上有9块可移动的白板,白板顶上还有可以自动降下的幻灯幕布。白板前的仪器操作台,有各样的控制按钮,可以播放录像,电脑幻灯片,控制所有的灯光。教授们可以瞬间把教室变成小型电影院。

  66位MBA的新同学神色庄重地坐在自己的座位上。每位同学的面前的桌子上,都插放着自己的名牌。为了方便教授们为同学参与课堂讨论的程度打分,同学们每节课都必须放好名牌。这块黑底白字的塑料牌,将会陪着我走过未来的20个月。

  IVEY的课程设置100%沿袭HARVARD。这渊源于IVEY在1921年建校时的全面移植HARVARD的教学法的构想。所以IVEY也和HARVARD一样,不设FINANCE, MIS,或ACCOUNTING的专向,而全部集中于GENERAL MANAGEMENT。 MBA课程共两年。每年两个学期。 MBA的第二年,学生可以根据自己的兴趣,略有侧重地选修课程。而MBA的第一年里,所有的学生上同样的课程,不能自由选课。 FINANCE, ACCOUNTING, OPERATION, MACRO ECONOMIES, MARKETING, STRATEGY, INFORMATION SYSTEM。这些课将为二年级的学习打下基础,但功课的强度和难度非常大,是整个MBA过程中最具挑战性的日子。而第一个月却被称作是难中最难,苦中最苦的阶段。

  这难和苦对国际学生特别是中国学生而言则会更苦。原因有三:

  一是语言困难。尽管所有中国同学都通过面试,被证明有很好的听说能力,但还是很难一下子跟上快节奏的课堂讨论。有时可以听清教授的讲话,却对同学的发言不知所云。而有时可以清楚地听清每一个字,却又因为文化不同,而不能理解。

  二是功课强度大。 IVEY为培养未来经理人对压力的适应力,刻意在第一个月中安排了在有限的时间里不能完成的课程量。除了每天上午八点到下午一点的三节案例讨论课,下午还要和学习小组讨论第二天课上的案例,晚上则要读完第二天小组讨论的另外三个案例和所有的补充材料。来读MBA的同学大部分都在大学毕业后工作了六七年。在离开学校这末多年后,又重回教室,面对这末大的工作量,确实极具挑战性。而中 国同学因为阅读速度慢,又增加了更大的工作量。学习到凌晨两三点钟,确实是家常便饭。由于吃饭时间缩短,睡眠严重不足,大部分同学都会迅速从红光满面变得面色发青。

  三是教学法不适应。 IVEY的案例教学的精髓在于课堂讨论。教授从不给一个最后的正确答案,教授的任务只是利用学生丰富的工作阅历,引导学生开阔思路,建立一个全面客观的思维方式。为鼓励学生参与讨论,每个的学生的成绩有40%到50%决定于课堂讨论的质量和数量。如何在80分钟课上从其他65个从不服输的同学们竞争,抢到发言的机会,对于从没有过举手发言的中国同学来说,极具挑战性。

  在伦敦深秋的灿烂红叶里,我在IVEY的第一个月在混天黑地中飞失而过。多亏有二年级中国同学的指点迷津,我慢慢摸到门路,懂得如何应对这样的挑战,如何鼓励自己举手发言,如何提高阅读速度。如何和学习小组成员协调讨论。走在依然翠绿的山坡上,我望着坡上的古堡式的钟楼,心里十分的怅然。似乎这一个月中,我除了在马不停蹄的读案例,没有很实在的收获。 IVEY到底能教给什么呢?

梁溪香榭 : 2009-03-21#606
回复: 加拿大教育情况有用素材收集帖子--梁溪香榭整理

(续) 在加拿大IVEY商学院读MBA的日子

五、暑期工作

  1999年12月10日,MBA的第一学期终于结束了。我依然不能回答我在IVEY收获了什么的问题。可是暑期找工已经热闹的开始了。

  有人说,读MBA的过程就是找工作的过程。而这个过程的第一阶段就是找暑假的实习工作(SUMMER INTERN)。每年的五月到八月,学校不安排课程,学生可以自己决定选择到公司里实习工作或是在家渡假。如果没有特别情况,MBA的学生还是希望能在暑假里找到心仪的工作,好好表现,争取尽早拿到PERMENANT OFFER。但是由于暑期的工作机会有限,到校招聘的公司数量有限,竞争十分激烈。

  此时的伦敦已是漫天飞雪,白雪皑皑。我站在积雪的公寓阳台上,看外面雪花飞舞,心里却只有焦虑和迷茫。面对陌生的加拿大市场,我似乎有点手足无措。不管在中国还是在加拿大找工作,关系是非常重要的。关系不仅可以帮助我们找到招聘信息,还可以帮我们增加接近招聘公司的机会。本地学生似乎有遍布加拿大的小学同学,中学同学,大学同学,父母,兄妹的朋友。可对于只身来到加拿大不到一年的 我来说,只有眼前漆黑一片的感觉。尽管学校的CAREER MANAGEMENT安排了每周一次的找工培训,但洋人的办法是否适合中国学生呢?这时候,二年级的中国同学再次伸出援助的手。他们召开经验交流会,热情地为我们介绍找工的途径和技巧。分享他们的成功经验和失败教训。

  找工的途径无非只有两类:校内招聘和校外招聘(ON-CAMPUS AND OFF-CAMPUS)。校内招聘就是公司上门招聘而校外招聘就要靠学生自己各显身手,通过中介公司,网络,和朋友找工作。

  圣诞和新年刚过,新学期伊始,各大公司就蜂拥而至了。最早来的北美最著名的投资银行和顾问咨询公司,比如BCG,UBS AG,GOLDMAN SACHS,MERRILL LYNCH,ANDERSON,MCKINSEY,DELOITTE,ERNST & YOUNG,KPMG等等他们旨在最早抢到最有进取心(AGGRESSIVE),成绩最好,分析能力最强,最想挣很多钱,最可能成为工作狂的MBA学生。这两个行业的高薪对MBA学生来说极具吸引力。大部分的本地学生的兴趣都集中在这两个行业。对中国学生来说,大家都会勇于尝试,但同时也深知成功的可能性会很低。这是和这两个行业对语言的较高要求有关的。尽管在IVEY的历史上, 没有一个来自大陆的中国学生进入过这两个行业,我的同学里终于出现了一位被五大著名顾问咨询公司之一录取的骄骄者。

  随后而来的公司是加拿大顶尖的企业和著名的跨国公司,NORTEL,LUCENT,FORD CANADA,GM CANADA,DELPHI,MAPLE LEAF FOOD,GENERAL MILLS,BELLBOMBARDIA。这些公司大都本着长期培养的目的,不但注重学生以前的背景,更看重学生是否有可培养的潜质。这对中国学生来说,无疑是找工作的重点。很多来自中国的同学都在跨国公司工作多年,或从事对外贸易的工作,或在国内经营自己的企业。这些国际背景对跨国公司来说很有吸引力。很多中国同学幸运地被这些公司录取了。

  学校的招聘部门会随时更新IVEY内部网站的招聘通知,还会帮助到校的招聘公司安排公司介绍午餐会,晚餐会(INFO SESSION)。同学们笑言,就是找不到工作,这几个月的饭钱确实能省下不少。一旦收到面试通知,学生可以通过IVEY内部招聘网,上网选择自己的面试时间。虽然学校的招聘部门每年有上百万加元的预算,可是对学生的找工作的帮助还是不能尽如人意。很多学生还是要通过校外招聘(OFF CAMPUS)找到暑期工作。

  校外招聘或言自己找工,确实是大海捞针。自己找工不过只有三个办法:注册招聘网站,联络IVEY校友,联络自己的朋友。这对本地同学是非常有效的办法,但对中国同学来讲,要靠自己的运气。还是有一些同学通过这几种办法找到暑期工作。

  暑期找工重要的不是结果,而是过程。也许在每天挤出几小时的时间来找工作,而努力了几个月后,依然是两手空空。可是这其间我们学会如何写好COVER LETTER,个人简历。如何做好面试准备,如何做好面试,如何和校友联络,如何FOLLOW UP。而这些就是找工的最大收获,因为我们为找毕业后的工作做好了准备。由于当年经济形势较好,在我离开伦敦时,大半中国同学都已尘埃落定,准备开始在加拿大的暑期工作。而我在上下求索四个月未果的情况下,我有幸拿到了加拿大最大的私人金融管理集团在北京办事处的暑期工作。登上回京的飞机,回家了。

梁溪香榭 : 2009-03-21#607
回复: 加拿大教育情况有用素材收集帖子--梁溪香榭整理

(续) 在加拿大IVEY商学院读MBA的日子

六、学习

  经过了一个暑假,我在IVEY能收获什么的问题也慢慢有了答案。我在暑假里某一天猛然悟到,IVEY对我思维方式的改变。对所有遇到的问题,我会自动把它当作是一个案例,用IVEY教给我的方法,一步步分析,找到多个可行的解决方案,进行比较,然后确定行动计划,和调整方案。我忽然意识到这种思维方式的变化不是一朝一夕发生的,而是通过分析上百个不同的商业案例,潜移默化地移植到 我的头脑里。我终于明白了在IVEY的第一个月的困惑。IVEY也许不能让我记住如何进行数据统计分析,建立数据金融模型,如何精确计算财务平衡表和盈亏表,但IVEY赋予了我一个清晰的全面而客观的思维方式。这种高位的战略思维方式(STRATEGIC THINKING)正是一个企业经理人所必须具备的。正如我在暑假北京遇到的93年毕业于IVEY的校友,万维网总经理,董先生所言,这种战略思维方式,让IVEY的毕业生能受用终生。

  在IVEY读书,还要面临学习小组的挑战。IVEY要求每个学生必须参加一个五到六个人的学习小组。小组每天必须讨论第二天课上将要讨论的案例。这个办法自然也是来自HARVARD,旨在培养团队合作技巧和提高上课讨论的质量。小组成员在开始是硬性分配的,两个月后,学校认为同学间有了足够了解,就可以自然组合。这时候,大部分国际学生就开始处于尴尬境地。由于很多本地学生组合时 ,只吸纳一个或两个国际学生,很多国际学生只好自己组合。由于语言和文化差别,以及性格差一,有些小组在开学几个月后就寿终正寝了。而有的小组却能活过整整一年。小组的成员,因为朝夕相处,日夜厮守,将会成为终生的朋友。

  考试是MBA生活中最刺激的事情。IVEY的考试通常有三种,4小时堂考,48小时报告和回家考试(TAKE HOME TEST)。考试全部开卷,只是要求的时间,地点略有不同。我在考试期间有严重的焦虑失眠症。记得一年级最后一周的五门考试期间,我的有效睡眠时间逐日递减,直到最后一天的运作考试时,我直等到伦敦的鸟儿开始在凌晨五点为我唱歌时,方才睡去。而刚刚休息了两个小时的脑袋,在早晨八点就开始运作。四个小时考试之后,自然只有手脚冰凉,四肢发颤,眼睛发直。写48小时报告的过程也非常 有趣。五六个小组成员,领了考试案例,聚在同学家里。打开啤酒,可乐,要了比萨饼外卖,就开始了两个通宵的苦战。在交报告的前十分钟,最好不要站在教学楼前。因为随时都有风驰电掣的汽车疾停楼前,接着车里的同学立即蹦出车门,直奔楼立交报告的箱子,把自己小组的苦熬两昼夜的报告准时塞进箱子。

  IVEY的学习负担很重。在IVEY的一天是这样的。早晨七点钟迷迷糊糊地从温暖的被窝里爬起,匆匆早饭后,又匆匆赶到学校。八点钟,第一节课开始了。很多同学在这时候依旧是半梦半醒。直到第二节课,一杯杯浓郁的咖啡终于把所有的人唤醒了。第三节课总是饥肠辘辘中度过。因为结束的时候已经是接近一点钟了。可是不管脑子的清醒程度如何,也不管肠胃的饱和程度如何,我们必须要集中精神 跟上课堂讨论,还要在手臂如林的教室抢到发言的机会。下课后,在简短的午餐后,各个小组选择不同的时间,地点,开始小组讨论。如果小组讨论效率不高,讨论时间会拖到四五个小时。匆匆回家后,胡乱吃点东西,又要开始准备第二天的三个案例。我准备一个案例至少要两个小时。等到功课完成时,一定是又过了子夜了。在IVEY两个月后,所有人都感到一种源自骨髓的疲惫,却不能停止工作。经过 这样高强度的训练,每个人最大的潜能都被发掘出来,足以应付未来工作的挑战。

  苦中作乐的事情,自然是每周五下课前的最佳”葛话儿”(CHICKEN WORDS)评选。就是列出本周课堂讨论中最可笑的发言。由于人的大脑经常出轨,发言时经常词不答意,言不由衷,或是自己说的话,自己都不知所云。比如说:

  - Starbucks should expand globally,because the world is a much bigger market than the United States.

  - If the system gets stuck,you need to lubricate it to get going。It needs an injection (of money).

  - You need to find a good source of reproductive capability.God knows,we’re all looking for that!

  - I don’t know how many people live in Germany,but I know there are quite a few

  - As 18 month employees,they had only been there a year and a half

  - 24% of a small number is a small number

  而第一年评选的年度最佳“葛话儿”是:I have needs,and I’m willing to pay to have them filled。”获得每周最佳的同学每天必须携带那只光皮塑料鸡到校。我们班的DARCY还在她抱鸡的那一周每天热心地给鸡换上漂亮的白裙子,戴上紫色的小帽子。

梁溪香榭 : 2009-03-21#608
回复: 加拿大教育情况有用素材收集帖子--梁溪香榭整理

(续) 在加拿大IVEY商学院读MBA的日子

七、工作!工作!

  2000年9月4日,MBA二年级的课程在平静中开始了。这种平静来自于对MBA生活的完全适应和对将要开始的新挑战的充分准备。二年级的学习压力相比一年级小了很多,可以自由选课,加上每星期五没有课程安排,多了很多时间。大家的主要精力基本放在找工作上。

  MBA的日子走到这个这里,同学们基本对自己未来的方向有了明确的选择,再加上在暑期找工时积累了很多经验教训,所以这时候找工比一年级时更有方向,更有策略,更加得法。我把找工作的过程比作完全自我包装的过程。如何让招聘公司了解到自己对公司有价值的方面,就要做好每一个环节。

  COVER LETTER和RESUME自然是第一步。几乎所有的同学都为不同行业准备了不同版本的简历。而送给不同招聘公司的COVER LETTER更是度身打造。为了能让招聘公司一见钟情,很多中国同学请本地同学帮忙较对,检查语法错误。即使有著名公司背景的同学也不能放松。对于好不容易拿到的第一轮面试机会,大家更是紧张备战,除了自己把面试的故事编好,还要请同学帮忙做面试演习。虽然第二和第三轮面试可以住在公司提供的舒服的酒店,可是第二天的五六个经理人员的车轮大战确实能让人筋疲力尽。当一个个希望升起 又破灭的时候,心情也在期待和沮丧之间轮回。拿到OFFER的同学自然是欣喜若狂,暂时没有得到好消息的同学在毕业的日子渐渐临近时,心情就越发沉重起来。

  2000年圣诞节前,已经有不少同学拿到了OFFER。中国同学也战果辉煌,居然有16位同学拿到了工作。这些公司都是一些国际上的著名企业,比如CITIBANK,GM,CIBC,NORTEL,FORD,ROYAL BANK,AT KERNEY,MANULIFE等等。这一年的圣诞和新年对他们来说是轻松和快乐的。可正当大家为新年孕育着新希望时,北美股票市场却风云突变,各大公司噩耗频传,NASTAQ更是跌破了二千点。大家心里的希望突然变得沉甸甸起来。从2000年1月开学起,无论是加拿大还是美国的MBA学校突然门庭冷落起来。IVEY内部招聘网上每天只挂着七八条招聘启事。没有OFFER的同学面 临着毕业即失业的危险,心里怎是一个急字了得;而有OFFER的同学心里也是惴惴不安,唯恐公司收回OFFER。在毕业典礼前一周,我的一个土生华裔同学就惨被一家美国著名高科技公司收回了OFFER。

  尽管在如此极为困难的情况下,很多中国同学的努力还是慢慢有了结果。截至到2001年4月底,超过大半的中国同学在加拿大找了工作。他们所加盟的公司遍及各个领域,银行,保险业,风险投资,顾问咨询,高科技,汽车制造,电子设备制造,医药业等等。他们的新生活将会在多伦多,渥太华和蒙特利尔开始。还有的同学在此时移民监已满,将要移师美国或是在众人羡慕的眼光中,回国创业去了。

  有人形容MBA找工作的过程,就象是背负着沉重的包袱,在黑暗的隧道里前行。尽管每走一步都付出了巨大的努力,可依然不知道何时能到达终点。其间每个人所承受的心理压力是别人难以想象的。这种压力不仅会压抑人的情绪,更会摧毁人的自信。能够经得起这种磨练,不被压力摧垮的人是让人佩服的铮铮铁汉。

惴惴不安 : 2009-03-21#609
回复: 加拿大教育情况有用素材收集帖子--梁溪香榭整理

梁妹妹真勤奋,加分鼓励。

Luck Li : 2009-03-21#610
回复: 加拿大教育情况有用素材收集帖子--梁溪香榭整理

梁妹妹真勤奋,加分鼓励。

9494! +SW 都加都加。:wdb17::wdb17:

梁溪香榭 : 2009-03-22#611
回复: 加拿大教育情况有用素材收集帖子--梁溪香榭整理

梁妹妹真勤奋,加分鼓励。

9494! +SW 都加都加。:wdb17::wdb17:

谢谢! 今天分分派发完了. 明天补.
送捆秋天滴菠菜先 :wdb6:

pikexima : 2009-03-22#612
回复: 加拿大教育情况有用素材收集帖子--梁溪香榭整理

怎么给梁溪加分啊?就是发表回复时的主题评分吗?

主人这么勤奋,应该有重奖!

海边小舞 : 2009-03-22#613
回复: 加拿大教育情况有用素材收集帖子--梁溪香榭整理

发自内心的感谢梁溪香榭!辛苦了!

redsea2008 : 2009-03-22#614
回复: 加拿大教育情况有用素材收集帖子--梁溪香榭整理

thanks for sharing!

梁溪香榭 : 2009-03-22#615
回复: 加拿大教育情况有用素材收集帖子--梁溪香榭整理

怎么给梁溪加分啊?就是发表回复时的主题评分吗?

主人这么勤奋,应该有重奖!

点击俺滴头像下面的银色天平

发自内心的感谢梁溪香榭!辛苦了!

thanks for sharing!

谢谢鼓励 :wdb6:

梁溪香榭 : 2009-03-28#616
回复: 加拿大教育情况有用素材收集帖子--梁溪香榭整理

ZT 加拿大几个名牌大学比较之我见

这个帖子时没有要贬低哪个学校的意思,只是想澄清一些误解。我说UBC的本科教学质量可能不如SFU,并非要说UBC的本科比SFU差很多,只是差一点点而已。其实我感觉加拿大的公立大学前20名的本科质量都很好,读本科,到哪里都差不多,关键是你看重学校的名气还是更看重上课有所收获。再说,UBC 的本科教学质量可能没有SFU好,并不意味着UBC的本科毕业生不如SFU. 我只是认为UBC的本科生在课堂上得到教授的帮助比较少,很多靠自学,所以会特别辛苦,我做TA,经常会有本科生一头雾水地跑到我办公室问一些 question, 蛮可怜的,呵呵。UBC的官方数据显示有30%的本科生被 kick off, 最终拿不到学位。

无疑UT是加拿大大学中研究实力最强的一所,据说可以和U. of Michigan比一比(这点我很怀疑),但是UBC/McGill和UT比起来差别其实很小,加拿大前几名大学的研究实力确实差不了多少。很多PhD 如同时拿到UT和UBC的faculty offer,反而会选择UBC,因为是加拿大最舒适的城市。象我supervisor,他是MIT的PhD, 在卡内基梅隆(CMU)一直做到副教授(tenure),后来还是跳槽到UBC,我问过他为什么,他说因为喜欢温哥华。3年前,在选择UT和UBC的 PhD program, 我也有过犹豫,但是今天,如果再让我选择一次,我会依然选择UBC. UBC和UT比起来最主要的问题是,BC省的工业不如安省发达,因此UBC从工业界得到的资助没有UT多,这一定程度上影响了它的研究。

如果我指出了waterloo的一些不足,会搞得有些waterloo的同学不开心,但确实是实话,你在waterloo呆久了就会明白。waterloo和 SFU, Guelph,确实是在同一个level, 共同特点是:都是很年轻的大学,规模小,锐气十足,课程改革的力度也很大,如waterloo的co-op,课程质量很好,所以读本科,这些大学是很好的选择。但是它们的总体研究水平确实差一点,以这类大学中风头最健的waterloo为例,它确实有一个很好的计算机系和数学系,但是一个大学不仅仅只包含一个计算机系,计算机系和数学只是理学院中的一小部分,而一个综合性大学应该有法学院、文学院、理学院、工学院、艺术学院、医学院、农学院等等,只有这些都有研究实力,才可成为一所一流名校。此外,即使在计算机和数学系,waterloo教授中好象没有图灵奖、菲‘尔兹奖或者wolf奖的获得者, 以这样的实力,它的数学和计算机的研究水平又何以敢自称北美前10? 在加拿大,至少UT的计算机要远胜waterloo. 而McGill自称它的Intelligent Machine Center的研究排名世界前10,它似乎也不把waterloo放在眼里。另外, waterloo也从来没有得过nobel奖,而UT, UBC和McGill得过多次Nobel奖。

说waterloo的计算机,在加拿大大学中本科教学水平排第一,研究生教育和研究水平 top 5,而整个学校的学科综合排名 top 10, 是一种比较中肯的说法。

归纳一下,waterloo的计算机的本科教学水平在加拿大数一数二,确实很优秀,但是它最优秀的地方也就在这里。还是建议你本科读waterloo、SFU和Guelph, 等你想读研究生时再跳到UT,UBC, McGill或者美国名校.

当然,有些同学认为本科学校的名气和校友网络要比上课的收获更重要,而舍弃 SFU/waterloo 去选择 UT/UBC,这也是一种明智的选择。

个人观点:不要太看重排名,要看重你真正需要的是什么,个人的成功归根到底要靠个人的努力,好学校对你有帮助,但是不是你想象的那么大.

评价一个大学,是要区分,你是来读本科还是做研究生。如果读本科,我认为U. of Calgary都比UBC好,这两个大学我都读过书,深有体会,因为UBC拥有太多名教授,他们专注于研究,根本不在乎上课质量,我在做TA,发现那些本科生根本学不到什么东西。因为UBC很多教授很有名,即使课上得很差,学生complain, 学校拿他们也没有办法,而在U.of Calgary, 教授都很年轻,没有名气,如果课上得不好,学生complain,他们可能饭碗不保。所以,SFU和Waterloo的本科教育质量应该比UBC好,如果去读本科,我推荐waterloo, SFU, U.of Victoria这些小巧型的大学,班级size小,教育质量高,教授又认真负责。UBC本科生,有时120个人在一个教室里,教授上课又不认真,你能学到什么?

其实,本科教育质量和教授是否有名没有什么关系。我们系上课上得最好的老师却是一个只有master degree的lecturer, 而系里那些牛气冲天的教授上课却奇烂无比。教授研究搞得好,对你本科生没有什么太大关系。

但是研究生教育、研究声誉和学术水平,SFU无法和UBC比。如果读研究生,尤其PhD,当然是UT,UBC, McGill这些规模大、国际学术声誉高、大牌教授云集的大学。导师的学术眼光和声誉,对你的学术生涯非常重要,象SFU和Waterloo这些中等规模的学校,在个别专业上领先UBC和UT完全可能,但是总体学术水平,后者高出不止一个档次。

当然国际上的声望也是这些学术水平高、历史悠久、规模大的学校要高得多,出了加拿大相信没有几个人会知道SFU,但有不少人知道UBC,上次我去美国开会,对这点很有体会。

代表加拿大研究实力和科研资助强度的是加拿大国家研究委员会的NSERC Chair Professor的数量,我在NSERC网站查了一下, UT有170位,UBC 120几位, McGill97位,U. of Alberta 70几位,而Waterloo, SFU , Guelph, U of Calgary之类只有40-50位而已,研究水平的差距可见一斑。至于诺贝尔奖的数量和高水平论文数量, UBC、McGill和UT都远超Waterloo和SFU。

国内对加拿大大学误会很大,认为waterloo是加拿大最好的大学,很可笑。微软招了很多waterloo的学生去做程序员,只能说明它的CS本科教育很有特色,微软很喜欢,不代表研究实力。微软在Waterloo招的学生比在MIT招的多,难道waterloo比MIT还强?我们系有几个从 waterloo本科考到UBC读master的学生,我问他们waterloo那边的研究水平和实验室设备这么样,他们都连连摇头,说无法和UBC比。学术声誉,最终要看这个学校在国际顶尖学术期刊和学术会议上发表的论文数量。

做研究,UBC, UT和McGill, 我认为差别不大,以我的观察, UBC的CS的研究实力甚至在UT之上。不是吹牛,UBC的CS和EE确实很强,比较两个大学网上的教授个人主页,可以得出结论。

我这么说,并非说SFU不是好大学,相反,我认为它是很优秀的大学,我也很喜欢,只是它比较侧重于本科教学,不是很合适我而已。

总结一下,如果去读本科,建议读SFU,Waterloo之类,如果读研究生,推荐UT, UBC, McGill, U of Alberta之类。

盈婆婆 : 2009-03-28#617
回复: 加拿大教育情况有用素材收集帖子--梁溪香榭整理

辛苦了。

梁溪香榭 : 2009-03-28#618
回复: 加拿大教育情况有用素材收集帖子--梁溪香榭整理

ZT: 加拿大的商学院MBA排名分析

很多朋友都希望知道加拿大的MBA排名,其实就在几年前我联系商学院的时候也是这样的心情。本人代表个人意见参考有关网站和杂志,结合最近时间的排名(不是最新资料)说说加拿大的商学院。

第一,2002年Financial Times MBA Ranking,在世界范围内排了前100名商学院,其中加拿大仅仅有8所入围。但是本人很喜欢这个排名,因为很多人都说在加拿大不值得都MBA,但是这个排名至少有5所进入前50名了。另外一个原因就是本人读的MBA在着排行榜中排名最高,比很多加拿大的国内商学院排名好

排名 学校

18 University of Western Ontario: IVEY
33 University of Toronto: Rotman
34 York University: Schulich
36 McGill University
46 Queen’s School of Business
60 UBC
83 University of Calgary
88 McMaster University: DeGroote</FONT></P>

第二,看看最权威的MBA排名:BUSINESS WEEK
与众多名校相比,加拿大的商学院底气不足。但是除了美国之外的世界上的其他商学院,加拿大的商学院应当是一流的.
最新的排名,美国以外的MBA
第一档次的商学院有7所,其中加拿大的IVEY第5和ROTMAN第7。第二档次有8所,没有排名,加拿大商学院有QUEENS, YORK, McGill。(补充:本人绝对认为UBC也是这个档次的,强烈要求排名的人有来学校看看)
第三,加拿大本国排名。没有找到最新的在Canadian Business杂志的排名,只是在为客户"http://www.visawang.com/apply/申请索要的York的材料中发现这个杂志关于加拿大商学院2000年和2001年的排名,请参考。并且希望能找到最新排名的朋友告诉我,一定请你喝酒。

Rank         University        
2001  2000

1      2     Schulich
2      1     Queen's
3      -     McGill
4      5     Simon Fraster
5      3     UBC
6      15    Saint Mary's
7      12    McMater
8       6    Victoria
9       -    New Brunswick
10      -    New Brunswick-SaintJoin
11      -    Calgary
12     13    Memorial
13      9    Alberta
14      7    Manitoba
15      -    Montreal
16     10    Dalhousie
17      8    Wilfrid laurier
18     14    Concordia
19     11    Windsor
20      -    Laval
21      -    Ottawa
22      -    Saskatchewan 
-      -     Rotman
-      -     Moncton
-      4     Ivey 

大体上是这个排名,但是其中我们发现有的学校由于某种原因没有参加排名
最近几年Queen's连续排名第一,Rotman的GMAT平均分与美国名校不相上下

第四,本人的个人见解
选择商学院,名气固然重要,但是最重要的要看自己的背景还有就是将来的发展方向。否则,选择了学校后悔不算,累也累死了。
如果你是金融背静,也特别希望进入银行,保险公司等Finance 行业,那么Rotman绝对首选.
如果你善于辩论,勇于挑战自我,有意图到世界各地发展,并且对于General Management感兴趣,毫无疑问去IVEY。世界上最典型的案例教学的三个学校有Harvard,Darden和IVEY.我的感觉是那句话:如果你恨一个人,就让他到IVEY;如果你爱一个人,也让他去 IVEY.
York的IMBA据说非常不错,能有世界各地工作实习机会。Queen's非常注重工科背静的申请者.12月紧张学习压力很大。UBC作为后起之秀发展迅速。15个月的项目很牛。

McGill在北美这个学校很有名气,老美特喜欢。据说是加拿大的“哈福”,其它的学校也就不罗索了。

Luck Li : 2009-03-29#619
回复: 加拿大教育情况有用素材收集帖子--梁溪香榭整理

ZT 加拿大几个名牌大学比较之我见

这个帖子时没有要贬低哪个学校的意思,只是想澄清一些误解。我说UBC的本科教学质量可能不如SFU,并非要说UBC的本科比SFU差很多,只是差一点点而已。其实我感觉加拿大的公立大学前20名的本科质量都很好,读本科,到哪里都差不多,关键是你看重学校的名气还是更看重上课有所收获。再说,UBC 的本科教学质量可能没有SFU好,并不意味着UBC的本科毕业生不如SFU. 我只是认为UBC的本科生在课堂上得到教授的帮助比较少,很多靠自学,所以会特别辛苦,我做TA,经常会有本科生一头雾水地跑到我办公室问一些 question, 蛮可怜的,呵呵。UBC的官方数据显示有30%的本科生被 kick off, 最终拿不到学位。

无疑UT是加拿大大学中研究实力最强的一所,据说可以和U. of Michigan比一比(这点我很怀疑),但是UBC/McGill和UT比起来差别其实很小,加拿大前几名大学的研究实力确实差不了多少。很多PhD 如同时拿到UT和UBC的faculty offer,反而会选择UBC,因为是加拿大最舒适的城市。象我supervisor,他是MIT的PhD, 在卡内基梅隆(CMU)一直做到副教授(tenure),后来还是跳槽到UBC,我问过他为什么,他说因为喜欢温哥华。3年前,在选择UT和UBC的 PhD program, 我也有过犹豫,但是今天,如果再让我选择一次,我会依然选择UBC. UBC和UT比起来最主要的问题是,BC省的工业不如安省发达,因此UBC从工业界得到的资助没有UT多,这一定程度上影响了它的研究。

如果我指出了waterloo的一些不足,会搞得有些waterloo的同学不开心,但确实是实话,你在waterloo呆久了就会明白。waterloo和 SFU, Guelph,确实是在同一个level, 共同特点是:都是很年轻的大学,规模小,锐气十足,课程改革的力度也很大,如waterloo的co-op,课程质量很好,所以读本科,这些大学是很好的选择。但是它们的总体研究水平确实差一点,以这类大学中风头最健的waterloo为例,它确实有一个很好的计算机系和数学系,但是一个大学不仅仅只包含一个计算机系,计算机系和数学只是理学院中的一小部分,而一个综合性大学应该有法学院、文学院、理学院、工学院、艺术学院、医学院、农学院等等,只有这些都有研究实力,才可成为一所一流名校。此外,即使在计算机和数学系,waterloo教授中好象没有图灵奖、菲‘尔兹奖或者wolf奖的获得者, 以这样的实力,它的数学和计算机的研究水平又何以敢自称北美前10? 在加拿大,至少UT的计算机要远胜waterloo. 而McGill自称它的Intelligent Machine Center的研究排名世界前10,它似乎也不把waterloo放在眼里。另外, waterloo也从来没有得过nobel奖,而UT, UBC和McGill得过多次Nobel奖。

说waterloo的计算机,在加拿大大学中本科教学水平排第一,研究生教育和研究水平 top 5,而整个学校的学科综合排名 top 10, 是一种比较中肯的说法。

归纳一下,waterloo的计算机的本科教学水平在加拿大数一数二,确实很优秀,但是它最优秀的地方也就在这里。还是建议你本科读waterloo、SFU和Guelph, 等你想读研究生时再跳到UT,UBC, McGill或者美国名校.

当然,有些同学认为本科学校的名气和校友网络要比上课的收获更重要,而舍弃 SFU/waterloo 去选择 UT/UBC,这也是一种明智的选择。

个人观点:不要太看重排名,要看重你真正需要的是什么,个人的成功归根到底要靠个人的努力,好学校对你有帮助,但是不是你想象的那么大.

评价一个大学,是要区分,你是来读本科还是做研究生。如果读本科,我认为U. of Calgary都比UBC好,这两个大学我都读过书,深有体会,因为UBC拥有太多名教授,他们专注于研究,根本不在乎上课质量,我在做TA,发现那些本科生根本学不到什么东西。因为UBC很多教授很有名,即使课上得很差,学生complain, 学校拿他们也没有办法,而在U.of Calgary, 教授都很年轻,没有名气,如果课上得不好,学生complain,他们可能饭碗不保。所以,SFU和Waterloo的本科教育质量应该比UBC好,如果去读本科,我推荐waterloo, SFU, U.of Victoria这些小巧型的大学,班级size小,教育质量高,教授又认真负责。UBC本科生,有时120个人在一个教室里,教授上课又不认真,你能学到什么?

其实,本科教育质量和教授是否有名没有什么关系。我们系上课上得最好的老师却是一个只有master degree的lecturer, 而系里那些牛气冲天的教授上课却奇烂无比。教授研究搞得好,对你本科生没有什么太大关系。

但是研究生教育、研究声誉和学术水平,SFU无法和UBC比。如果读研究生,尤其PhD,当然是UT,UBC, McGill这些规模大、国际学术声誉高、大牌教授云集的大学。导师的学术眼光和声誉,对你的学术生涯非常重要,象SFU和Waterloo这些中等规模的学校,在个别专业上领先UBC和UT完全可能,但是总体学术水平,后者高出不止一个档次。

当然国际上的声望也是这些学术水平高、历史悠久、规模大的学校要高得多,出了加拿大相信没有几个人会知道SFU,但有不少人知道UBC,上次我去美国开会,对这点很有体会。

代表加拿大研究实力和科研资助强度的是加拿大国家研究委员会的NSERC Chair Professor的数量,我在NSERC网站查了一下, UT有170位,UBC 120几位, McGill97位,U. of Alberta 70几位,而Waterloo, SFU , Guelph, U of Calgary之类只有40-50位而已,研究水平的差距可见一斑。至于诺贝尔奖的数量和高水平论文数量, UBC、McGill和UT都远超Waterloo和SFU。

国内对加拿大大学误会很大,认为waterloo是加拿大最好的大学,很可笑。微软招了很多waterloo的学生去做程序员,只能说明它的CS本科教育很有特色,微软很喜欢,不代表研究实力。微软在Waterloo招的学生比在MIT招的多,难道waterloo比MIT还强?我们系有几个从 waterloo本科考到UBC读master的学生,我问他们waterloo那边的研究水平和实验室设备这么样,他们都连连摇头,说无法和UBC比。学术声誉,最终要看这个学校在国际顶尖学术期刊和学术会议上发表的论文数量。

做研究,UBC, UT和McGill, 我认为差别不大,以我的观察, UBC的CS的研究实力甚至在UT之上。不是吹牛,UBC的CS和EE确实很强,比较两个大学网上的教授个人主页,可以得出结论。

我这么说,并非说SFU不是好大学,相反,我认为它是很优秀的大学,我也很喜欢,只是它比较侧重于本科教学,不是很合适我而已。

总结一下,如果去读本科,建议读SFU,Waterloo之类,如果读研究生,推荐UT, UBC, McGill, U of Alberta之类。

:wdb11::wdb11:辛苦了,香香。:wdb19::wdb19:

梁溪香榭 : 2009-04-10#620
回复: 加拿大教育情况有用素材收集帖子--梁溪香榭整理

加拿大本科申请指南

1、学校的选择
加拿大中学12年级毕业后,可直接申请进入大学,没有大学入学考试制度。在魁北克省规定学生完成11年级后,再转读该省的普通和职业教育学院读一年或两年预科方能升入大学。

加拿大约有70所高等院校,其中6所用法语,6所用英语、法语双重语言教学,其余均用英语教学。高等教育的结构中共分十个等级:博士学位、哲学硕士学位、硕士学位、执照(指律师或医生合格开业证明)、研究生毕业证书、学士学位、大学毕业证书、特别证书、结业证书和证书。普通学士学位一般修读3年。荣誉学士学位一般修读4年。

加拿大的大学多数采用学分制。


全日制高等院校每年有两个学期,有的大学分三学期。除大学外,加拿大还有社区学院,约250多所,教学内容侧重于应用:如美术、工科、商科及其他专业训练。一般期限为1至3年。这些学院只发证书或文凭,不颁发学士学位,但可以选择一些以后可转读大学深造的课程,修业期1至2年,程度相当于大学一二年级,学完后转入正规大学二三年级,继续攻读学士课程,所以一般留学生都愿意读这些学校。但是时间比较长。

查询加拿大大学信息可以参见加拿大大学与学院联合会网站:

http://www.aucc.ca/can_uni/index_e.html

梁溪香榭 : 2009-04-10#621
回复: 加拿大教育情况有用素材收集帖子--梁溪香榭整理

2、选校注意事项

第一,要询问学校是英语还是法语授课,免得语言不通而影响学习。

第二,加拿大不同地区,特别南北之间气温相差甚大,因此中国南方的学生较好适应。

第三,加拿大多伦多和温哥华人口多,学校多,而大学(学院)又分布在市区或郊区。在那里出现不少营业性质的“学店”,校舍简陋,设备不全,师资不足,教学质量差甚至有的卖学分和学位。这样的学校不要选。

3、入学资格
加拿大入大学主要取决于申请人的中学毕业成绩,有些留学生先去加拿大插班中学,中学毕业后直接上大学,学校可能会要求托福考试的成绩。

4、申请学校

申请人可于开课前9个月,去函向自己选中的学校索取申请表格及有关资料,或者直接从学校网站下载表格。在截止申请期限前填好表格,连同校方所要求的文件及申请费一齐寄回该校,校方收到该文件、表格后,会进行查核及考虑,如决定录取,便寄出入学证明书(Letter of Accepatance)。但部分学校则先发临时录取通知书,待申请人交纳了全年费用后,才发正式的入学证明书。

梁溪香榭 : 2009-04-10#622
回复: 加拿大教育情况有用素材收集帖子--梁溪香榭整理

下面以UNiversity of New Brunswick 为例说明如下:

UNB 诞生于1785年,是加拿大历史悠久的英语大学之一,同时也是北美历史悠久的公立大学之一。200多年的师资力量造就了高素质的人才,现在的UNB有着55000个学生,他们来自世界各地将近70个不同的国家,这使我们大学成为加拿大多元化的国际留学生学校之一。
UNB 阅历
你将找到国际留学生怎样评价UNB。
两个校区
UNB 福来:将近10000学生,福来有50000人,是纽宾省省会。
UNB 圣约翰:大约3000学生,圣约翰人口超过125000,是一个历史文化发展迅速的城市,有着小城镇的亲切感和大城市的现代气息。
申请的最终期限
对于国际留学生,UNB没有申请的最终期限。*但是,学生签证需要很长的过程。建议在办理之前至少提前6个月申请UNB。



9月入学,在6月1日之前申请。

1月入学,在10月1日之前申请

5月入学(仅限于能够选课),在2月1日前申请
申请费用
申请费为$45加币或者38$美金
奖学金
基于学术课程的成就,UNB授予多于500$大学生奖学金-1500$给大学一年级学生。
英语要求
UNB圣约翰 / UNB福来
转学分
为了便于UNB评估之前学术课程的的学分,请从之前的学校提供详细的课程说明和成绩单。2年级的学生必须完成UNB(大约60个学分)才能拿到UNB的学位。
校外工作
国际留学生在UNB完成第一年学业并且保持学术的要求将允许在校内和校外每周工作20个小时。毕业之后,纽宾省的国际留学生在回国之前可以在纽宾省全日制工作两年。

运动
UNB的运动项目被认为在一定区域内有最好的提供。UNB有着优秀运动项目于游泳,摔跤,跨越国家,曲棍球,以及足球,篮球,野外曲棍球,羽毛球和排球。
学生服务
学生健康与支持服务位于校圆内,也有数学,写作,和学习技巧等实验室。每个国际留学生都可以要求接机但他们到达时。
体育俱乐部,社会俱乐部和社团给你们的孩子提供更广泛的活动以加强他们在UNB的经历,给予他们机会在生活中建立人际关系。
留宿
校园宿舍有电视休闲室,学习室,厨房和洗衣室。在圣约翰和福来你可以选择校外居住。

梁溪香榭 : 2009-04-10#623
回复: 加拿大教育情况有用素材收集帖子--梁溪香榭整理

录取要求:
Students must submit a TOEFL score or equivalent test score before they will be considered for admission to the Fredericton campus.

学生必须提供托福成绩或者相关的语言成绩。

Test Name

Score

TOEFL iBT 92 or higher TOEFL (paper-based)580 or higher
TOEFL (computer-based)


237 or higherCAEL70 or higherIELTS6.5 or higherMELAB82 or higherCanTEST4.75 or higher
托福580分以上直接录取。少于580/237,但是高于500/173的学生,需要参加密歇根语言测试当他到达UNB以后。少于500/173,不予考虑录取。

护士专业要求托福600分,商务管理专业最低成绩580。

Students who submit a TOEFL score of 580 (237 computer based) may be admitted directly to an academic program. Students who score less than 580/237, but higher than 500/173 will be required to take the Michigan English Language test when they arrive on campus and, depending on the score, may be required to take intensive English or an English language support course in conjunction with academic courses.*
Students who score below 500/173 will not be considered for admission at the Fredericton campus. All students wishing to enter an academic program should apply directly to UNB using UNB's standard application for admission.
* These ranges apply to all faculties except Nursing (BN) and Business Administration (BBA). Our Nursing program has a TOEFL requirement of 600/250 and our Business Administration program has a TOEFL requirement of 580/237. English Language Support and Intensive English are not options for either the BN or the BBA.

梁溪香榭 : 2009-04-10#624
回复: 加拿大教育情况有用素材收集帖子--梁溪香榭整理

填写申请表:

https://www.unb.ca/sweb/application/unb_int_application.pdf

提供11-12年级的成绩单原件与翻译件

你参加的大学的官方成绩单,包括ESL学校

你在加拿大以外参加大学的详细的课程列表

语言成绩单

如果需要从大学退学,如果该学生从离开到申请之日少于12个月,本校不予考虑。

如果该学生从某大学退学,需要提供:
个人解释信,说明他在原来大学的表现以及如果被本校录取的学习计划。

一封推荐信,说明该生的良好的潜在的学习能力。

最后需要申请费45加元(38美元),支票或者信用卡。

从500-1500元不等的奖学金可以向学校直接申请,根据学生的表现学校判定。

完成所有的文件,寄到相关的地址。

bigblackapple : 2009-04-11#625
回复: 加拿大教育情况有用素材收集帖子--梁溪香榭整理

太好了,真是太好了。谢谢

tramp : 2009-04-11#626
回复: 加拿大教育情况有用素材收集帖子--梁溪香榭整理

好贴

梁溪香榭 : 2009-04-11#627
回复: 加拿大教育情况有用素材收集帖子--梁溪香榭整理

太好了,真是太好了。谢谢


谢谢鼓励

家园移民 : 2009-04-11#628
回复: 加拿大教育情况有用素材收集帖子--梁溪香榭整理

偶也鼓励下

梁溪香榭 : 2009-04-11#629
回复: 加拿大教育情况有用素材收集帖子--梁溪香榭整理

偶也鼓励下

谢谢 也T嫩一脚 :wdb6:

Luck Li : 2009-04-12#630
回复: 加拿大教育情况有用素材收集帖子--梁溪香榭整理

收到,喜欢LZ溪溪。

小桥流水 : 2009-04-14#631
回复: 加拿大教育情况有用素材收集帖子--梁溪香榭整理

先谢谢LZ溪溪,顶完慢慢看。:wdb10:

bg1rco : 2009-04-17#632
回复: 加拿大教育情况有用素材收集帖子--梁溪香榭整理

太全面了!感谢!

梁溪香榭 : 2009-04-17#633
回复: 加拿大教育情况有用素材收集帖子--梁溪香榭整理

感谢网友yhyxll提供以下信息.

我是加拿大移民,想申请UBC请问需要什么条件,难度大否
我07年在国内参加高考,文科成绩610+,目前未参加托福考试。08年3月移民来加拿大,想尽快申请UBC学习文学,但不知道UBC的申请条件,也不知道难度,希望能有方家指正。
问题补充:【补充】第一,我想学习文学,不知道这所以医学著称的大学文学科类的竞争是不是会比较小;第二,本人英语高考成绩130+,有独立阅读英语文学书籍和文学教材能力,不知道更适合考托福还是雅思;第三,如果回国学习托福或雅思课程,是否会取得比在国外学习更好的效果(因为据说新东方的托福课程很厉害~)~

我是UBC大二的~针对你的情况:
高考610+:足够,UBC要求是80%,达到就OK,再高也没有意义,外国人这点就是好~

托福雅思:建议雅思,因为雅思相对容易。UBC的相关要求是:雅思6.5,各科不能低于6;托福86,阅读,写作>22,听力口语>21。PS:各科的成绩不一定要一次达到。比如:你第一次雅思6。5,但是写作是5.5,你可以先寄这次的成绩单,下次考了7,写作是6.5,但是口语又变5.5了,寄这次的成绩单,UBC是算你通过的~总之就是你只要有各项大于6的成绩,总分在6.5的基础上就算通过。托福各项也是这样计算

文学专业:首先,文学属于UBC的Arts Faculty, 根据你所选的具体类别来决定program。申请Arts Faculty的难度比其他院系都要小。其二,你若是纯文学类,那就是Fine Arts,在这一类别中亚洲学生非常少,学校为了保持各个系的学生多样性,会优先录取一定的亚洲学生。在我周围的,还没有一个亚洲的Fine Arts学生。。。不过可能因为我是SCIENCE的原因

考试:建议考雅思。就我的经验来看,雅思各个部分比较平均,拿中等偏上的成绩很轻松,我是高中水平,去上了新东方的补习班后第一次考6.5,阅读听力7,口语写作6。但是我本人的托福考的很糟糕,阅读29,口语23,写作26但是听力是个位数(惭愧),原因就是听力本事我强项,就没有复习,后来上考场就傻了。。。

如果是为了通过考试,我觉得在国内上更有效果。因为这边的老师更擅长托福和LPI,而你要攻克的是(我推荐)雅思。

UBC真的很漂亮,赶快过来吧~


你所说的不要高考成绩,大概是指预科。再给你转如下:


英属哥伦比亚大学(UBC)本科双录取


项目优势:
本项目为UBC首次与本省内教育局直接合作,有条件招收国际学生;
学生一经被公立费南教育局录取,将会同时收到一份由UBC国际学生申请及注册处发出的有条件录取信;
UBC在其有条件录取信中明确指定学生在费南教育局学习,并完成相关课程,达到费南教育局的要求,即可申请进入UBC;
申请本公立UBC预科课程及通过此课程进入UBC的学生,都将无需提供托福或雅思成绩,也无需提供中国高考成绩;
UBC是否接收某个申请,取决于:
1,高中三年平均成绩;2,预科成绩,其中预科中的英语成绩不能低于70分;
中国学生申请UBC预科课程要求:高中平均成绩85分以上,省重点中学学生及能够提供雅思5.5分以上将被优先考虑录取。


上面这种,商业性很强,事实上,很多人最后进不了UBC,进去了也不一定能毕业,但愿能帮到您!

梁溪香榭 : 2009-04-17#634
回复: 加拿大教育情况有用素材收集帖子--梁溪香榭整理

感谢网友yhyxll提供以下信息

高中毕业生如何选择加拿大大学?

  在咨询当中,尤其是最近,刚刚结束高考之后,很多打算到加拿大留学的中国学生或其家长都会对我们提出这个问题,根据自身情况的不同,我们建议学生可以考虑以下三种类型:

  标准型:直接申请大学--适合于成绩优秀的高中毕业生(或高三在读生)

  申请条件:高中成绩75%以上,会考成绩优良,TOEFL580或IELTS6.5, 没有合格英语成绩加拿大教育中心可以帮助申请英语+专业课双录取

  担保金额:人民币40万以上,其中至少一半以上有一年以上历史,家庭收入状况良好

  推荐学校:西蒙菲莎大学、阿尔伯塔大学、西安大略大学、萨省大学、圭尔夫大学等

专家点评:

  除了上述优秀大学外,还有很多高中毕业生希望能够进入多伦多、滑铁卢、麦吉尔等加拿大名校就读,但是由于这样的学校专著于硕士和博士研究,对本科生多采取大班授课
(有的多达3-400人),而且没有语言中心,所以只接受达到以下条件的中国学生:高考成绩至少70-80%以上,TOEFL580或IELTS6.5以上,高中三年平均成绩80-90%以上,会考成绩优秀。即使达到上述条件,建议学生还是要慎重考虑,因为这类学校淘汰率很高,尤其对于刚刚从中国教育体制下出国的学生来说,不是每一个都能在短期内适应并生存的

  实用型:申请加拿大学院适合于成绩一般的高中毕业生或中专、职高毕业生

  申请条件:中学成绩70%以上,TOEFL550或IELTS6.0以上,没有合格英语成绩可以申请双录取

  推荐学校:兰加拉学院,圣力嘉学院、尼亚加拉学院、马拉斯比那大学学院、昆特兰大学学院,北阿尔伯塔理工学院

  担保金额:人民币30-35万以上,父母工作收入状况良好

专家点评:

  因为名校直接申请难度较大,而且淘汰率高,而加拿大学院与大学之间均有协议,更有学院提供2+2大学联办课程,所以学生可以选择先进入学院就读,2-3年后再转入大学完成本科,这样既可以拿到只有学院才能提供的职业执照(在加拿大就业必备的文凭),又可以获得名牌大学的学位,而且按照加拿大移民法的规定,三年制大专所获移民分数(22分)高于学士学位所获移民分数(20分),从移民角度考虑也非常有利,所以,现在有越来越多的学生选择了这样的申请方式

  预科型:申请加拿大预科适合于希望就读最优秀加拿大大学的高中学生

  申请条件:21岁以下,高三以上学历,平均成绩75%以上;最好有良好的英文基础,以保证学生在预科半年- 1 年的时间内能完成英文和文化课的课程;

  担保金额:人民币60万以上,至少有一半能提供一年以上的历史,家庭经济条件优裕

  推荐学校:哥伦比亚国际学院,宝迪学院、博裕学院等

专家点评:

  这种方案是那些希望到加拿大就读最优秀大学但不想参加中国高考或成绩不理想的高中学生设计的。因为加拿大很多顶级名校如多伦多、UBC、麦吉尔等大学除了很高的英文成绩外(IELTS6.5或TOEFL580),还要求学生提供优秀的高考成绩,学生申请读中学或预科,获得加拿大高中毕业文凭后再申请大学,这样无需提供中国的高考成绩,大学英语入学要求业会相应降低。


【本文为艾迪国际原创内容 如需转载请注明来源及作者 或致电67083592】

盈婆婆 : 2009-04-20#635
回复: 加拿大教育情况有用素材收集帖子--梁溪香榭整理

溪溪,零九年费萨的排名出来了,你有连接吗?

Luck Li : 2009-04-20#636
回复: 加拿大教育情况有用素材收集帖子--梁溪香榭整理

溪溪、TX们和领导,大家早上好!

梁溪香榭 : 2009-04-20#637
回复: 加拿大教育情况有用素材收集帖子--梁溪香榭整理

溪溪,零九年费萨的排名出来了,你有连接吗?

www.fraserinstitute.ca

梁溪香榭 : 2009-04-20#638
回复: 加拿大教育情况有用素材收集帖子--梁溪香榭整理

溪溪、TX们和领导,大家早上好!

Luck早啊!

Luck, 嫩家宝贝滴学业如何安排的呀?

zv2008 : 2009-04-21#639
回复: 加拿大教育情况有用素材收集帖子--梁溪香榭整理

好帖,很多信息非常有用,支持一下,慢慢再看

新月儿 : 2009-04-21#640
回复: 加拿大教育情况有用素材收集帖子--梁溪香榭整理

这些素材太实用了,我女儿现在国内高一,2008年8月递交材料,等面试中,有好的建议吗?关于孩子就学问题,多谢!

锦叶知秋 : 2009-04-29#641
回复: 加拿大教育情况有用素材收集帖子--梁溪香榭整理

http://book.idoican.com.cn/Detail/DefaultView.aspx?BookId=m20071023m000w005206 比尔这家伙 的电子版

梁溪香榭 : 2009-04-29#642
回复: 加拿大教育情况有用素材收集帖子--梁溪香榭整理


谢谢. 分分送上.
看过这本书, 绝对推荐!

家园移民 : 2009-04-29#643
回复: 加拿大教育情况有用素材收集帖子--梁溪香榭整理

谢谢. 分分送上.
看过这本书, 绝对推荐!
偶也绝对推荐啊

Luck Li : 2009-04-29#644
回复: 加拿大教育情况有用素材收集帖子--梁溪香榭整理

谢谢. 分分送上.
看过这本书, 绝对推荐!

俺也看过绝对推荐。:wdb9::wdb9:

zhangxuan119 : 2009-04-30#645
回复: 加拿大教育情况有用素材收集帖子--梁溪香榭整理

一口气看完LZ的大作,受益匪浅,强烈致谢!好人好报!

梁溪香榭 : 2009-05-02#646
回复: 加拿大教育情况有用素材收集帖子--梁溪香榭整理

感谢网友王晶的推荐.
特在此推荐好书 " 美国语文".

美国孩子的语文课分为3大类. 语法, 写作和文学. 本书属于文学范畴.
该书选编的课文以美国历史的发展为线索,取材于不同时代的具有广泛社会影响及文学代表意义的文章,完整体现了语文教育的人文性、综合性及开放性,尤其课后问题的设置、编配极具趣味性、现实性,对学生文学综合素质的培养有很强的启发意义。
 教程很好地平衡了知识的教学和学生综合能力的发展,在强调学生多元化思维的同时,更加注重其个性思维的独特经验,既是语文教程,也是历史人文教程、思想实践教程

举个例子:
《美国语文》选用了亨利大卫梭罗的《论公民的不服从》。这是一篇名作,列入中学教程,无疑是有冲击力的文字。有意思的是,文章背后的思考题很带挑战性。三道题是:
  1、梭罗暗示谁应该对墨西哥战争负责任?
  2、根据梭罗的观点,为什么一小部分人可以滥用政府而免受惩罚?
  3、根据梭罗的观点,什么时候美国人将会获得在可能范围内的最好的政府?
这样的设问,在我看来都是中国的成年人才有的,不料洋人的教育中早已种下了这粒种子。胡适留美的时候,感叹过中西教育的不同。他执教北大后,终身倡导的就是梭罗式的思想……

家园移民 : 2009-05-02#647
回复: 加拿大教育情况有用素材收集帖子--梁溪香榭整理

感谢网友王晶的推荐.
特在此推荐好书 " 美国语文".

美国孩子的语文课分为3大类. 语法, 写作和文学. 本书属于文学范畴.
该书选编的课文以美国历史的发展为线索,取材于不同时代的具有广泛社会影响及文学代表意义的文章,完整体现了语文教育的人文性、综合性及开放性,尤其课后问题的设置、编配极具趣味性、现实性,对学生文学综合素质的培养有很强的启发意义。
 教程很好地平衡了知识的教学和学生综合能力的发展,在强调学生多元化思维的同时,更加注重其个性思维的独特经验,既是语文教程,也是历史人文教程、思想实践教程

举个例子:
《美国语文》选用了亨利大卫梭罗的《论公民的不服从》。这是一篇名作,列入中学教程,无疑是有冲击力的文字。有意思的是,文章背后的思考题很带挑战性。三道题是:
  1、梭罗暗示谁应该对墨西哥战争负责任?
  2、根据梭罗的观点,为什么一小部分人可以滥用政府而免受惩罚?
  3、根据梭罗的观点,什么时候美国人将会获得在可能范围内的最好的政府?
这样的设问,在我看来都是中国的成年人才有的,不料洋人的教育中早已种下了这粒种子。胡适留美的时候,感叹过中西教育的不同。他执教北大后,终身倡导的就是梭罗式的思想……


偶也去买看看

梁溪香榭 : 2009-05-02#648
回复: 加拿大教育情况有用素材收集帖子--梁溪香榭整理



偶也去买看看

呵呵, 领导也是位书虫哈. :wdb20:

minxw : 2009-05-02#649
回复: 加拿大教育情况有用素材收集帖子--梁溪香榭整理

太好了

家园移民 : 2009-05-05#650
回复: 加拿大教育情况有用素材收集帖子--梁溪香榭整理

感谢网友王晶的推荐.
特在此推荐好书 " 美国语文".

美国孩子的语文课分为3大类. 语法, 写作和文学. 本书属于文学范畴.
该书选编的课文以美国历史的发展为线索,取材于不同时代的具有广泛社会影响及文学代表意义的文章,完整体现了语文教育的人文性、综合性及开放性,尤其课后问题的设置、编配极具趣味性、现实性,对学生文学综合素质的培养有很强的启发意义。
 教程很好地平衡了知识的教学和学生综合能力的发展,在强调学生多元化思维的同时,更加注重其个性思维的独特经验,既是语文教程,也是历史人文教程、思想实践教程

举个例子:
《美国语文》选用了亨利大卫梭罗的《论公民的不服从》。这是一篇名作,列入中学教程,无疑是有冲击力的文字。有意思的是,文章背后的思考题很带挑战性。三道题是:
  1、梭罗暗示谁应该对墨西哥战争负责任?
  2、根据梭罗的观点,为什么一小部分人可以滥用政府而免受惩罚?
  3、根据梭罗的观点,什么时候美国人将会获得在可能范围内的最好的政府?
这样的设问,在我看来都是中国的成年人才有的,不料洋人的教育中早已种下了这粒种子。胡适留美的时候,感叹过中西教育的不同。他执教北大后,终身倡导的就是梭罗式的思想……
嘎嘎
偶买了2套,今天到了

偶仔细看看

梁溪香榭 : 2009-05-05#651
回复: 加拿大教育情况有用素材收集帖子--梁溪香榭整理

嘎嘎
偶买了2套,今天到了

偶仔细看看

比较震撼. 两者的教育思路有若云泥!

家园移民 : 2009-05-05#652
回复: 加拿大教育情况有用素材收集帖子--梁溪香榭整理

比较震撼. 两者的教育思路有若云泥!
嘎嘎
这个不用看偶就知道

梁溪香榭 : 2009-05-05#653
回复: 加拿大教育情况有用素材收集帖子--梁溪香榭整理

嘎嘎
这个不用看偶就知道

看过感受更深

家园移民 : 2009-05-06#654
回复: 加拿大教育情况有用素材收集帖子--梁溪香榭整理

看过感受更深
偶看了,觉得不错
又下了个大单子
估计当当又要断货了

梁溪香榭 : 2009-05-06#655
回复: 加拿大教育情况有用素材收集帖子--梁溪香榭整理

偶看了,觉得不错
又下了个大单子
估计当当又要断货了

嫩公司厉害啊! :wdb17:
管昨天还说要寄比尔这家伙给我的, 呵呵,

王晶 : 2009-05-06#656
回复: 加拿大教育情况有用素材收集帖子--梁溪香榭整理

偶看了,觉得不错
又下了个大单子
估计当当又要断货了
比尔,这家伙 已经断货。还好,有了电子版,就在电脑上阅读。
听梁溪香榭说你推荐
今天你又下大单子,明天《美国语文》一定也缺货了。
下次有什么好书多推荐。倒一杯茶谢谢你了。

家园移民 : 2009-05-06#657
回复: 加拿大教育情况有用素材收集帖子--梁溪香榭整理

比尔,这家伙 已经断货。还好,有了电子版,就在电脑上阅读。
听梁溪香榭说你推荐
今天你又下大单子,明天《美国语文》一定也缺货了。
下次有什么好书多推荐。倒一杯茶谢谢你了。
呵呵

比尔这家伙

偶公司多了

嫩需要吗?送嫩本?

家园移民 : 2009-05-06#658
回复: 加拿大教育情况有用素材收集帖子--梁溪香榭整理

嫩公司厉害啊! :wdb17:
管昨天还说要寄比尔这家伙给我的, 呵呵,
最近各个阶段的等待时间比以前都加长很多
打乱了很多客户的计划
对客户偶公司都特别特别不好意思
只能在多服务上想办法了

Luck Li : 2009-05-06#659
回复: 加拿大教育情况有用素材收集帖子--梁溪香榭整理

看过感受更深

俺也当当订一本去。:wdb6::wdb6:

梁溪香榭 : 2009-05-06#660
回复: 加拿大教育情况有用素材收集帖子--梁溪香榭整理

俺也当当订一本去。:wdb6::wdb6:

抓紧时间啊, 别全让守法抢去了

王晶 : 2009-05-06#661
回复: 加拿大教育情况有用素材收集帖子--梁溪香榭整理

呵呵

比尔这家伙

偶公司多了

嫩需要吗?送嫩本?
太激动了,我一定带上礼物到你公司取书来。需要什么礼物呢?:wdb6:

家园移民 : 2009-05-06#662
回复: 加拿大教育情况有用素材收集帖子--梁溪香榭整理

太激动了,我一定带上礼物到你公司取书来。需要什么礼物呢?:wdb6:
嘎嘎
发送个短消息给偶
注明地址姓名电话

偶安排个快递就可以了

梁溪香榭 : 2009-05-06#663
回复: 加拿大教育情况有用素材收集帖子--梁溪香榭整理

太激动了,我一定带上礼物到你公司取书来。需要什么礼物呢?:wdb6:

嘎嘎
发送个短消息给偶
注明地址姓名电话

偶安排个快递就可以了

王晶, 俺们这个环球大家庭温暖吧? :wdb19:

王晶 : 2009-05-07#664
回复: 加拿大教育情况有用素材收集帖子--梁溪香榭整理

王晶, 俺们这个环球大家庭温暖吧? :wdb19:
非常后悔太晚认识环球,否则早就加入你们的行列。

王晶 : 2009-05-07#665
回复: 加拿大教育情况有用素材收集帖子--梁溪香榭整理

嘎嘎
发送个短消息给偶
注明地址姓名电话

偶安排个快递就可以了
不太好意思,下次有朋友要移民,我一定拖他/她到你们公司办理,顺便认识家园领导,再得到你的书作为奖励。:wdb6::wdb23:

Luck Li : 2009-05-07#666
回复: 加拿大教育情况有用素材收集帖子--梁溪香榭整理

抓紧时间啊, 别全让守法抢去了

您已选购以下商品 (,根据您登录后的身份,我们为您显示了商品的价格。) 商品名 单品积分 市场价 当当价 价 数量 删除
商家:当当网
20278301 美国语文(上下册/中英对照)/美国著名中学课文... 637 ¥98.00 ¥63.70 (65折) ---- 删除
您还没有挑选商品
继续挑选商品>> 再逛逛暂存架>>商品金额总计:¥63.70您共节省:¥34.30( 其中享有优惠:¥0.00 )
可获商品积分:637

梁溪香榭 : 2009-05-07#667
回复: 加拿大教育情况有用素材收集帖子--梁溪香榭整理

您已选购以下商品 (,根据您登录后的身份,我们为您显示了商品的价格。) 商品名 单品积分 市场价 当当价 价 数量 删除
商家:当当网
20278301 美国语文(上下册/中英对照)/美国著名中学课文... 637 ¥98.00 ¥63.70 (65折) ---- 删除
您还没有挑选商品
继续挑选商品>> 再逛逛暂存架>>商品金额总计:¥63.70您共节省:¥34.30( 其中享有优惠:¥0.00 )
可获商品积分:637

怎么好象比俺买得便宜!
守法买的是多少钱啊?

家园移民 : 2009-05-07#668
回复: 加拿大教育情况有用素材收集帖子--梁溪香榭整理

也是63多点啊

梁溪香榭 : 2009-05-07#669
回复: 加拿大教育情况有用素材收集帖子--梁溪香榭整理

您已选购以下商品 (,根据您登录后的身份,我们为您显示了商品的价格。) 商品名 单品积分 市场价 当当价 价 数量 删除
商家:当当网
20278301 美国语文(上下册/中英对照)/美国著名中学课文... 637 ¥98.00 ¥63.70 (65折) ---- 删除
您还没有挑选商品
继续挑选商品>> 再逛逛暂存架>>商品金额总计:¥63.70您共节省:¥34.30( 其中享有优惠:¥0.00 )
可获商品积分:637

也是63多点啊

哈哈, 刚刚查了, 我买的"美国语文"是53.90元.
怎么相差这么大?

商品名称单品积分市场价当当价发货数量小计 美国语文(上下册/中英对照)/美国著名...539¥98.00¥53.901¥53.90  商品金额合计:¥53.90
您已经为订单支付了:¥53.90

家园移民 : 2009-05-07#670
回复: 加拿大教育情况有用素材收集帖子--梁溪香榭整理

哈哈, 刚刚查了, 我买的"美国语文"是53.90元.
怎么相差这么大?

商品名称单品积分市场价当当价发货数量小计 美国语文(上下册/中英对照)/美国著名...539¥98.00¥53.901¥53.90  商品金额合计:¥53.90
您已经为订单支付了:¥53.90
嘎嘎

偶定的是时候显示的也是53
但是到货变成了63
现在也改成了63

欺负偶大户?

梁溪香榭 : 2009-05-08#671
回复: 加拿大教育情况有用素材收集帖子--梁溪香榭整理

嘎嘎

偶定的是时候显示的也是53
但是到货变成了63
现在也改成了63

欺负偶大户?

恩, 看来就是嫩啦, 扰乱市场哈 :wdb6:

Luck Li : 2009-05-08#672
回复: 加拿大教育情况有用素材收集帖子--梁溪香榭整理

嘎嘎

偶定的是时候显示的也是53
但是到货变成了63
现在也改成了63

欺负偶大户?

俺也是一开始显示53,点击购买就变成63了,这不是欺负人嘛,呵呵!:wdb23::wdb23:

Luck Li : 2009-05-08#673
回复: 加拿大教育情况有用素材收集帖子--梁溪香榭整理

恩, 看来就是嫩啦, 扰乱市场哈 :wdb6:
估计是看守法领导大户来了,一律涨价,o(∩_∩)o...哈哈。:wdb20::wdb20:

Luck Li : 2009-05-08#674
回复: 加拿大教育情况有用素材收集帖子--梁溪香榭整理

正在读(15人), 已读过(62人) | 放入书架 什么是书架?
点击查看大图作  者: 张健鹏,胡足青 主编
出 版 社: 中国妇女出版社
出版时间: 2008-7-1 字  数: 版  次: 1 页  数: 全两册 印刷时间: 开  本: 16开 印  次: 纸  张: I S B N : 9787802035829 包  装: 平装 所属分类: 图书 >> 中小学教辅 >> 拓展读物
定价:¥98.00 当当价:¥63.70 折扣:65折 节省:¥34.30 钻石vip价:¥60.60
送积分:637 积分说明

现在显示都是63了,呵呵!:wdb23::wdb23:

梁溪香榭 : 2009-05-08#675
回复: 加拿大教育情况有用素材收集帖子--梁溪香榭整理

估计是看守法领导大户来了,一律涨价,o(∩_∩)o...哈哈。:wdb20::wdb20:

恩. 领导是风向标, 他一来准是涨价木商量! :wdb20:

王晶 : 2009-05-08#676
回复: 加拿大教育情况有用素材收集帖子--梁溪香榭整理

哈哈, 刚刚查了, 我买的"美国语文"是53.90元.
怎么相差这么大?

商品名称单品积分市场价当当价发货数量小计 美国语文(上下册/中英对照)/美国著名...539¥98.00¥53.901¥53.90  商品金额合计:¥53.90
您已经为订单支付了:¥53.90

送货方式:送货上门
付款方式:货到付款
需要支付:¥58.00 请在收货时向送货员支付您的货款
怎么一样书,一样在当当网买,价格怎么不同?

Luck Li : 2009-05-08#677
回复: 加拿大教育情况有用素材收集帖子--梁溪香榭整理

送货方式:送货上门
付款方式:货到付款
需要支付:¥58.00 请在收货时向送货员支付您的货款
怎么一样书,一样在当当网买,价格怎么不同?

:wdb2::wdb2:

请看我的

您订单1782977399中的商品已于 2009-5-8 21:05:16 从当当网发出,请注意查收。
送货方式:送货上门
付款方式:货到付款
需要支付:¥63.70 请在收货时向送货员支付您的货款
快递公司:北京城际―A,电话 010-88099667、88099665
您可以点击此处查看订单的配送时间标准。
您也可以随时进入“订单详情页面”查看此订单的详细信息。

Luck Li : 2009-05-08#678
回复: 加拿大教育情况有用素材收集帖子--梁溪香榭整理

您已选购以下商品 (,根据您登录后的身份,我们为您显示了商品的价格。) 商品名 单品积分 市场价 当当价 价 数量 删除
商家:当当网
20278301 美国语文(上下册/中英对照)/美国著名中学课文... 637 ¥98.00 ¥63.70 (65折) ---- 删除
您还没有挑选商品
继续挑选商品>> 再逛逛暂存架>>商品金额总计:¥63.70您共节省:¥34.30( 其中享有优惠:¥0.00 )
可获商品积分:637

俺和领导是个啥身份呀?是买书越多越贵身份吧?呵呵!:wdb1::wdb1:这当当也太会看人下菜碟了,在北京买本地送货到贵了,大家不在这里交流还真不知道呢。:wdb2::wdb2::wdb18::wdb18:卓越发北京要近10天,当当又这么办事,做生意哪有这样的呀?我们买书多应该适当便宜一点点才对呀,让俺们也心里平衡平衡,高兴呀。:wdb8::wdb8:

梁溪香榭 : 2009-05-08#679
回复: 加拿大教育情况有用素材收集帖子--梁溪香榭整理

就是呀, 不交流还真不知道呢.
守法是VIP钻石级别了吧? 投诉他们 :wdb23:

willam131 : 2009-05-10#680
回复: 加拿大教育情况有用素材收集帖子--梁溪香榭整理

首先谈a-level:转发我在2006年刚离开中国内地第一批A-Level学校的时候写的一个总结作为回答,希望可以对你有所帮助:

“一位ALevel学校A-Level经济学、IGCSE商务研究教师兼升学指导老师

现在小留学生现象越来越普遍了,而且,随着美国签证的难度加大,越来越多的留学生把眼光瞄向了英国。小留学生在英国主要学习的是GCSE(General Certificate of Secondary Education,一般译作普通中等教育证书)课程,以及以后的GCE A-Level (General Certificate of Education,Advanced Level,一般译作普通教育高级水平证书)课程。尤其是这之后的A-Level课程成绩,往往在很大程度上决定了这些小留学生在英国可以升读什么类型的大学,其重要性几乎可以等价于中国的高考!

然而,近来,不时地听到一些留学的悲剧,原因有信息的不对称、小孩子心理生理等方面的不成熟、当初做出留学决策的鲁莽等等,不一而足。听后深感痛心!

作为CIE(Cambridge International Examinations)剑桥大学国际考试院中国首批A-Level学校之一的某学校A-Level Economics经济学教师,IGCSE Business Studies 商务研究教师,学院UCAS (Universities and Colleges Admissions Service,一般翻译作英国高等教育招生服务中心,其作用类似于中国的大学招生录取委员会)Coordinator和学院留学申请指导老师,(去年为学院29名毕业生申请到4份剑桥大学的Conditional Offer预录取通知书,1份牛津大学的预录取通知书,10份帝国理工的预录取通知书,5份伦敦政治经济学院的预录取通知书,14份华威大学的预录取通知书,7份布里斯托大学的预录取通知书,9份巴斯大学的预录取通知书,9份约克大学的预录取通知书,17份诺丁汉大学的预录取通知书等英国著名大学的预录取通知书,还有加拿大多伦多大学、麦克基尔大学、滑铁卢大学等大学、澳大利亚墨尔本大学等大学、美国密歇根大学等大学多份录取或与录取通知书),在这个家长又忙着为小孩决定何去何从的时刻,把自己在这两年内对英国教育体制(课程体系,考试体系,大学升学申请程序等)的了解与实践与大家共享,希望能为大家的决策起到一些参考的作用。刚好趁着学校在临近期末组织各个年级家长会,本人整理了一些作为家长会发言稿,指导学生在学校如何走好每一步并让家长放心。考虑到其他的许许多多的同样的心情的学生和家长,就想到把他们发表出来并放到了网上,希望能让更多的人获益。今天开个头,以后希望有机会陆续地把每一个留学的关键的环节,比如牛津剑桥申请表格的填写,学院的选择(牛津剑桥是学院制),申请材料的准备,面试的真实过程,笔试的考题,如何准备,A-Level、IGCSE课程的选择、考察的范围及提纲及评分标准、学习的方法、应试的技巧(只要是考试,中国人都会有考试的技巧),雅思和托福的准备以及应试的技巧等等再分别以不同的主题的形式与大家探讨,敬请关注。

GCSE(英国之外的GCSE课程叫做International GCSE,简称 IGCSE课程)学制两年,共有61门课程可以选择,其中包括中国学生比较擅长并且熟悉的Mathematics数学(数学在IGCSE和A-Level的阶段都有两门,强烈建议中国学生都选,这样可以充分发挥中国学生的数学方面的天赋和比较优势)、Physics物理、Chemistry化学、Biology生物、history历史(但与我们平时学习的历史课的内容相差很大,而且考察方法也区别很大)、Music音乐(可能很多学生会思维定势地误以为音乐课并不重要,其实国外许多著名大学非常看重像Music音乐、Drama戏剧等比较活泼的学科的成绩,因为他们觉得在学好相关课程的情况下又能学好这些课程的学生会给他们的校园带去积极向上丰富多样的校园文化,笔者的一位学生收到剑桥大学预录取通知书的学生就学了AS-Level的音乐)等,还有中国学生倍感陌生但应该会很感兴趣的Business Studies商务研究(对英语的要求极高,考题中没有选择题,全须学生论述展开,还需阅读案例,强烈建议英语不够好的学生选择的时候慎重,哪怕对这个学科很感兴趣,这是我教了这门课后的真实体会,一些英语不够好的学生学到最后了还没把教材完全看懂;但反过来,如果学生接受体挑战把这门学好了,那么英语水平就会直线上升,而且对雅思考试的高分都会有直接的帮助;更重要的是,也是笔者在课堂上经常告诉学生的,学好管理学,不一定就是要去管理别人,做到很好的自我管理也是同样很重要的), Accounting 会计学、Economics经济学(非常有用的一门学科,它不光教授学生大体的经济运行的规律,更重要的是教给学生一种全新的成本收益分析和有限资源最优选择效益最大化的考虑问题的思维方法。去年笔者的4位得到剑桥大学预录取通知书的学生中有两个选学了经济学,而且其中一位学生选读的大学专业与经济学无关。还有一点需要指出的就是,经济学一点都不难学,相反,是一门非常容易拿到A的学科,至少,根据笔者的教学经验,笔者已经确实得让学生掌握了它的应试方法), (商务研究、会计学和经济学是经典的商科方向的三门课程)Art and Design艺术与设计, Information Technology 信息技术,Travel and Tourism旅游与观光等, 还有既陌生又可能不会有任何兴趣的Religious Studies宗教研究、Arabic 阿拉伯语、Portuguese葡萄牙语等,甚至有让中国学生感到非常扬眉吐气(但对留学申请不是非常有用的)的Chinese汉语,还有两种汉语,Chinese(Mandarin) 汉语(普通话)和 Chinese - first language 汉语-本国语(可惜由于反而对中国学生的申请不是很有关系,所以笔者也没有做太多相关研究)。

好在英国的教育体系是非常人性化的,你可以自由选择并组合课程,所以你可以尽管选择自己又优势、熟悉并且感兴趣的课程,以达到最好的成绩和最与大学专业紧密相关的学术背景,以获得大学申请时最强的比较优势。一般选课控制在每学年4-5课。据剑桥大学的Prospectus介绍资料介绍,学5门A-Level课程比没有比学4门占优势。所以,得A才是最重要的!

相对于A-Level课程成绩来说,IGCSE就显得次要了很多,举一个极端的例子,甚至你没有IGCSE的成绩而只有A-Level的成绩,也丝毫不影响小孩的大学申请。打一个熟悉的比方,IGCSE类似于中国的会考,只有A-Level成绩才是高考成绩,那大学录取当然是只看高考成绩了!我们去年开始留学申请的学生,都是在学院直接入读A-Level课程而没有读任何IGCSE课程的,而且他们甚至都没有国内的高中毕业证书,因为他们是在高一或高二(英语基础和学科基础好一些的学生甚至可以跳一级在高一的时候就读A-Level课程,这样可以为小孩节约一年的时间;当然,小孩也可以在高三结束之后再读A-Level,只是这样小孩相对来说会多花一年时间学习,其实这未尝不是一好事情,因为这样小孩基础打得越扎实,就更有把握在A-Level考试时取得好成绩。而这一些都是托英国教育体制灵活而不拘泥于形式的福!)的时候来的我们学校。而他们最后几乎都申请到了英国甚至美国、加拿大、澳大利达、香港等国家和地区的最好的大学。

A-Level 课程与IGCSE课程相比,其重要性当然要大得多。因为这是英国的“高考”!A-Level课程选择的科目种类比IGCSE还要多,共有76门,大体与IGCSE类似。(这也正常,因为A-Level是IGCSE的延续)选课的原则也一样:充分的发挥自身的优势,并结合自己的兴趣,选最容易的A的课程来学。由于A-Level课程的选择与大学专业的申请有很大的关系,所以,从某种意义上来说,开始选A-Level课程的时候差不多就开了选大学的专业,这是对的。选得准,就是优势;选得不准,到时候要换,就是劣势。

刚才介绍了一些英国IGCSE/A-Level课程体系的课程门类,听上去很有趣,很有选择的余地,但这里不得不有一个不好的消息告诉大家:没有哪一个IGCSE或者A-Level学校或开齐这六七十门课程的。一般规模的学校会开设10门左右比较主流的课程。如果学生想学的课程这个学校没有开设,这就比较遗憾了。所以,学生在选择国外或国内的A-Level学校时,首先就应该问自己想学什么课程,然后去看这个学校开不开,否则,学校再好可能也不适合你。

A-Level的考试安排,这是与国内的高考有很大的差异的,而且,利用得好,中国学生可以获得很大的优势。这个,一言难尽。

几个建议:
1. 英语的问题:英语是迟早都必须学好的,所以,既然如此,为什么不马上下定决心学好英语呢?而且,学好英语绝不仅仅体现在足够高的雅思或者托福成绩上,他们还是有区别的,尤其是在通过考试技巧取得高分的情况下;我经常告诫学生的:这个6.5分,或者7.0分怎么得来的,我们都心知肚明,靠的就是技巧;我们帮你达到这个分数为的是什么,就是为了满足国外大学,特别是好大学的录取要求;你们真正的英语水平我们也都清楚,所以,如果你们以为自己的英语实力真正的达到了这个水平,那就是天大的自欺欺人了!其实,真正的把英语掌握后,学生会发现,IGCSE课程也好,A-Level课程也罢,都会变得容易很多,许多难的问题原来就是因为题目没有读懂而变得迎刃而解;自己在老师的指导下写留学申请材料,特别是非常重要的Personal Statement 个人陈述的时候,就不会有想法但又不知从何说起了;有机会参加牛津剑桥的面试的时候就可以畅快淋漓挥洒自如地表现一番了。而最后,你进入世界顶级大学的梦想就会离你近了许多!这就是英语的重要性!
2. 对学习的态度:国际教育只是小孩整个求学生涯的一部分,所以,即使已经决定去国外留学,也必须扎扎实实地把眼下的书读好。任何的书,不管是国内读的,还是在国外读的,都是你知识的一部分,都会对你未来的成功产生作用。而且,根据我们的观察,学生原先的基础越好,越容易顺利进入IGCSE/A-Level阶段的学习并取得好成绩。所以,千万不要以为他们是相互独立的,更不要以为学了国外的课程中国学生就一定会自动地变得聪明,这些观点都是站不住脚的。
3. 为留学申请的准备:坦白说,IGCSE成绩,A-Level成绩,雅思成绩,托福成绩再高都是只是中间过程,而申请上什么类型的大学及是否是自己感兴趣的专业,这才是我们所有一切努力的醉翁之意。所以,一切以大学申请为重点,这才是上策!而我们知道,国外的大学,特别是好大学,除了成绩之外,他还看中你的很多其他的方面,比如,相关的专业实践,足够的社会实践,体现奉献社会精神的义工等等,所以,这一些都必须未雨绸缪,不能把这个问题留到开始写留学文书的时候。到那个时候,在抓耳挠腮也是没有用的,因为我们一直不提倡编故事!编的故事在专业而富有经验的国际招生官那里是一眼就会被看出来的。而这样的后果又是很严重的:他们会对你的个人诚信度产生怀疑,可能就即使你再好都不会在考虑你的申请了!
4. 出国留学的时机选择:我经常跟学生说:如果说IGCSE的课程学不好,转眼之间过了暑假就要升读A-Level课程了,凭什么更难的A-Level课程就一定会学好?有一些A-Level的学生学得非常吃力,他们的今天可能就是你们的明天;在国内学不好IGCSE或者A-Level课程,然后就想转学到国外去读,凭什么保证到国外就一定能学好?在中国我们有外方富有经验的老师和努力的中方老师的齐心协力的配合,还有中方老师把所有的往年试卷钻研之后研究出来专门帮助学生取得高分的考试技巧,到了国外,语言又是障碍,老师可能也不会像在国内这么的关心,等等。

以上是笔者在一ALevel学校教A-Level经济学和IGCSE商务研究,并担任升学指导老师和UCAS协调人的工作经验的心得体会,并得到了一些我的英国同事的咨询帮助(再次对他们深表感谢,既感谢他们对我这篇文章的帮助,也感谢他们对我工作的一贯真诚帮助),写出来供大家参考,希望能为大家各自的决策起到一些作用。

英国的教育体系,英国大学的申请,都是不小的话题。今天想尽量简短地给面临选择的学生或者家长介绍一下的时候,发觉就像写论文,大题目难写,小题目好写得多:发觉什么都重要,什么都应该交代,但文章却反而因此被组织得凌乱不堪,有点信手由缰,所以哪儿算哪儿,对此,笔者深表歉意。如有疑问,欢迎大家探讨,笔者常用的邮箱号码是huacaifeng@hotmail.com。最后祝大家一切顺利!”


关于牛津剑桥面试:日前,我们的英国友人提供了我们630多位以往剑桥牛津申请学生的申请回顾,涵盖了几乎剑桥牛津所有学院、所有专业的申请,他们的面试试题回忆和经验或教训总结等。稍微翻看了几个案例后,即感觉如获至宝,练武之人获得了武林秘籍,天助神兵。我们一定充分利用并妥善保管这些宝贵的申请材料,以不辜负大力支持我们的英国朋友的热心帮助。



连同我们以前收集的200多位剑桥牛津和美国常春藤盟校等顶级大学的申请案例和面试反馈,如此大样本的申请案例必将为我们今年的世界顶级大学申请提供更多经验支持!



在学生寄出申请材料后,我们与剑桥大学密切的合作关系将确保我们的学生百分百获得剑桥大学的面试邀请(其他英国和美国的顶级大学我们也有独特的方法帮助学生顺利获得面试邀请);然后,我们将结合多年来的辅导经验,专门花时间来研究并消化吸收这些申请案例和面试试题库,为学生望而生畏的世界顶级大学面试提供更为专业和精准的辅导,让学生在继SAT或者A-Level、托福或者IELTS及国内学校成绩的高分、展现特长的各种获奖证书、丰富多彩并具社会责任感的课外活动、意味隽永跃然纸上甚至“化腐朽为神奇”的个人陈述、热情洋溢栩栩如生的推荐信之后,精彩出色并早有准备的面试笔试回答更让学生在顶级大学大学申请这堪称最后“临门一脚”的面试中给面试官或招生官留下深刻的影响。



刚好有一篇深圳特区报的采访,原文摘录如下作为答复:

“巧闯世界顶级大学面试关



专家指出,学生能够迅速抓住问题的关键并自信展开论述最为重要



国外大学独特的面试和笔试可能让中国高中毕业生迷惘。眼下,正是国外大学面试即将来临之际,为帮助大家更好地了解海外知名院校的笔试和面试,并作好更充分和更有针对性的准备,记者专程采访了名校申请辅导专家华才锋老师。

华老师每年都辅导众多的学生申请牛津、剑桥、哈佛、耶鲁、港大等世界顶级大学,美国麻省理工等大学的招生官称赞学生的留学申请材料和面试回答是当年他们印象最为深刻的。今年美国常青藤盟校几乎都有华老师的学生报考;凯洛格商学院的西北大学招生官最近来到中国,有幸与其共餐的11位内地高中生中也有3人为华老师辅导的报考西北的学生;上周日,他辅导的学生刚在上海收了普林斯顿大学校友的面试。

申请海外名校“很锻炼人”

记者:很多人认为名校申请很神秘,你是怎样认为的?

华才锋:申请国外名校,高分不是惟一的敲门砖,综合素质才是最重要的。我一直鼓励学生大胆申请,要充满自信。因为在申请过程中可以学到很多东西。学生会认真思考为什么我要申请牛津剑桥、为什么读这个专业、为什么我是一个合格的申请学生、我将如何利用大学这段学习时间、我将如何规划自己的职业生涯等等深刻的问题。这也是一个锻炼和成长的过程。

申请过程是一个系统工程

记者:你认为申请名校最重要的是什么?

华才锋:申请名校是一个系统工程,申请资料、面试印象和笔试成绩最为重要。申请资料要展示一个很精彩的学习方案和个人陈述,要把你的闪光点跃然纸上,吸引老师的注意,要让老师相信你是一个很有潜力且全面发展的学生。其实,名校面试虽然非常严格,但还有可以找到各校各自的一般标准。以剑桥为例,在给学生的面试邀请中写道:希望你们表达你的观点,展现你的能力,体现你的热情。学生能够开动脑筋,把有创意的想法释放出来,呈现一个最真实的自我,往往能够成功申请。

周密准备自信应对面试

记者:面试之前应做哪些准备?应注意什么?

华才锋:第一,口语能力很重要;第二,尽量去了解国外的文化习俗,了解考官的思维方式;第三,仔细阅读要申请大学的书籍,了解这个学校需要什么样的学生。正如我上面所说,利用以往的面试真题在模拟面试的情况下进行针对性的练习,作用巨大。我强调一点,面试绝不仅仅是面试口语,口语只是面试中考察的很基础的东西;最重要的,是学生

在面试过程中展现出来的自信。

记者:在采访中许多家长透露你有辅导大学面试的撒手锏,能与我们的读者一起分享吗?

华才锋:我每年都辅导不少优秀学生申请海外知名学校,由于每个学生都是自己亲自单独教授,与学生的感情很好,所以他们基本上都会在面试一结束就马上上网告诉我面试题目。我是一个有心人,由此收集了一些大学面试的问题库。这些学生的面试过程让我积淀了丰富的经验,而题库的建立也让我和学生能够尽情模拟面试过程,反过来为后来学生积了经验。当然,面试的成功与否,这只是很基本的一个训练,教会学生如何应对任何突发的甚至有些异想天开到匪夷所思的问题的方法,在极短时间内能够迅速抓住问题的关键并展开论述,从而给面试官留下深刻的印象,这才是最后取胜的把握。



原文网址:http://paper.sznews.com/tqb/20061207/ca2522997.htm”
参考资料:http://satkiller66.bokee.comhttp://blog.sina.com.cn/satieltskiller

梁溪香榭 : 2009-05-10#681
回复: 加拿大教育情况有用素材收集帖子--梁溪香榭整理

谢谢william, 分分送上.

rwx1378 : 2009-05-11#682
回复: 加拿大教育情况有用素材收集帖子--梁溪香榭整理

谢谢william,俺家孩子上的是A-LEVEL,现在单有A-LEVEL的成绩想申请北美名校特别是美国前20的学校还是有难度的,学校要求有精力并想申请美国名校的同学加考SAT。
谢谢分享选科分析。

Luck Li : 2009-05-12#683
回复: 加拿大教育情况有用素材收集帖子--梁溪香榭整理

就是呀, 不交流还真不知道呢.
守法是VIP钻石级别了吧? 投诉他们 :wdb23:
俺昨天下午在单位就收到书了,回家书就被儿子抢跑了,呵呵!
到现在还没摸到呢。:wdb19::wdb19::wdb6::wdb6:

wls1wls : 2009-05-13#684
回复: 子女教育信息素材收集帖

嘎嘎

这个帖子也算历经了劫难了,丢了又重新找回来了。

主要整理的是关于教育的一些素材

从147楼开始,主要就是梁溪香榭搜集的一些的,推荐推荐!

推荐本书--从普通女孩到银行家,http://forum.iask.ca/showthread.php?t=233637

目录

tony 3213, PEI地区教育情况 Page 1, 5楼
tiantianxingchen 高中课程简介 Page 1, 6-9楼
守法公民 初三学生多伦多求学记 Page1, 18-22楼
守法公民 安省新移民高中指南 Page 2, 24-30楼
守法公民 安省新移民小学指南 Page 2, 34-36楼
守法公民 高中家长如是说 Page 3, 44-49楼
雨中冷百合 大多伦多地区高中排名 Page 3, 57楼, 59楼
守法公民 温哥华十佳公立中学 Page 4, 65楼
一缕阳光 菲沙关于BC省中学排名及相关简介 Page 4, 69楼
守法公民 部分大学对是否需要雅思或托福成绩的年限规定 Page 6, 101楼
守法公民 教育篇 Page 6, 113-121楼
梁溪香榭 高中留学须谨慎 Page 8, 149-152楼
梁溪香榭 加拿大高中教育感悟 Page 8, 153-155楼
梁溪香榭 根据成绩选择加拿大大学 Page 8, 159楼
梁溪香榭 如何选择最适合的加拿大大学 Page 9, 161楼
梁溪香榭 读高二的孩子去加拿大需要准备什么 Page 9, 164-165楼
梁溪香榭 高中英语课程介绍 Page 9, 166-169楼
梁溪香榭 高中英语课程介绍 Page 10, 188-191楼
梁溪香榭 多伦多著名私立学校收费一览 Page 10, 192楼
梁溪香榭 孩子的英语过关了吗? Page10, 194楼
梁溪香榭 关于A-LEVEL课程 Page 11, 217-228楼
shieley6811 关于A-LEVEL课程及学制 Page 13, 256-257楼
梁溪香榭 2007年安省高中拍名 Page 17, 322楼
朱三雀 安省, BC省, AB省高中拍名, Page 17, 329楼
响铃当当 加拿大教育优势 Page18, 347楼
梁溪香榭 A-LEVEL与IB课程的比较 Page 18, 358楼
朱三雀 中西学生的差异 Page 19, 364楼
梁溪香榭 大多伦多地区IB高中简介 Page 19, 372-375楼
梁溪香榭 IB课程与其他特殊教育课程的比较 Page 19, 376楼
梁溪香榭 IB与AP的比较, Page 19, 377楼
梁溪香榭 美国相关研究机构关于IB与AP的选择比较 Page 20, 387-391楼
梁溪香榭 群星灿烂不如一轮皓月当空-大学录取篇 Page 21, 409-413楼
梁溪香榭 在教育子女的过程中教育自己 Page 22, 422-424楼
梁溪香榭 关于美国大学 Page 22, 429-432楼
梁溪香榭 多伦多高中阶段特殊教育程序 Page 23, 445-446楼
梁溪香榭 加拿大高中生学科竞赛 Page 23, 447-448楼
梁溪香榭 美国大学 Page 24, 466楼
梁溪香榭 如何选择适合自己孩子的中小学 Page 25, 487-489楼
梁溪香榭 论孩子成长的内环境 Page 25, 490楼
梁溪香榭 犹太人的教育风格:朴实无华 讲求实际 Page 25, 497楼
梁溪香榭 培养子女的参政意识 Page 25, 498楼
梁溪香榭 美国大学点评 Page 25, 499-529楼
梁溪香榭 加拿大大学点评(商科) Page 27, 533-535楼
梁溪香榭 加拿大大学点评(理工科) Page 27, 547-549楼
谢谢!太有用了!:wdb17:

wls1wls : 2009-05-13#685
回复: 偶搜集到的一些关于加拿大高中教育的资料 与大家分享

八、课程设置及课程结构

1.课程设置

英国高中课程(A-Level)有60多门课供学生选择,学生可以任意选修其中的3至4门

课程包括:数学、进阶数学、物理、化学、生物学、会计学、商业学、经济学、英国文学、心理学、计算机学等科目。

综合国外大学的录取要求及中国学生的优势和特点,A- Level课程将首先在中国开设数学、进阶数学、物理、计算机学、会计学、商业学、经济学等课程供学生选择。

应该选择哪几门课程是学生和家长都普遍关注的问题,但也是非常难回答的一个问题。英国、加拿大、爱尔兰、澳大利亚、新西兰、新加坡等英语国家没有统一的大学入学标准,虽然它们都认可A-Level证书,但是各所大学、各个专业对学生学过哪几门A-Level课程以及成绩都有不同的要求。所以怎样选择课程并没有一个唯一的答案。

学生选择课程时,一般要考虑现在自己的优势科目和将来的发展方向,即你想选择哪个大学、什么专业,从而根据他们的要求有的放矢地选课。然而,对于16-18岁的学生做这样的选择也是很难的,因为自己可能还没有一个清晰的决定。所以我们建议选择适合大部分大学和专业的课程,给自己今后发展留下比较大的选择空间。

数学、进阶数学和物理是大多数大学和专业招生时要求学过的A-Level科目(只有极专业的学科除外,例如法学),所以我们建议选择这三门课。除了被广泛地接受外,学生还有其他的收益。相对于西方学生,中国学生在数理化方面的训练更为严格,基础扎实;而且学习数理化对英语能力的要求比其他科目较低。所以这样的选择能够体现中国学生的优势。

但是学生如果对将来所学专业有了清晰的选择,那么选课就必须谨慎,因为有的专业是具有特殊要求的,例如:将来学习医学,现在就需要学习化学和生物学。如果学生对将来留学的大学或专业有了选择,可以向中心的工作人员咨询具体大学和专业对A- Level课程的专门要求,以便让学生选课时具有针对性。学生也可以自己上网查询相关信息,以下是一些大学的网站:

澳大利亚:www.dest.gov.au/highered ausiunis.htm

新 西 兰:www.educationnz.org.nz/institutions/universities.html

美 国:www.ed.gov , www.usjournal.com

加 拿 大:www.studycanada.ca

英 国:www.ucas.com/instit/index.html


2.课程结构 [学生可根据自身特点和兴趣,从中选择3至4门课]

基础数学 中国学生在数学学科上有很大的优势,一般学生都会选择基础数学。基础数学的内容涵盖:纯粹数学、概率统计、机械学。考试以笔试的形式,分为六个模块。

进阶数学 在基础数学的基础上,进阶数学在内容的深度和广度上略有提高。那些在理科方面有特长的同学,通常会选择进阶数学。

物理学 如果学生要进入大学的理工类专业,通常要选物理学。物理学的内容包括:普通物理、牛顿力学、物质、振动及波、电学与磁学、现代物理。

商业学 商业学内容包括:商务及环境、人与组织、市场营销、运作管理、商业会计学、决策与支持、信息学等。考试以笔试为主,题型包括:简答、小论文、案例分析等。

经济学 经济学内容包括:经济学基础、价格体系及公司理论、价格体系的政府干预行为、国际贸易、宏观经济学基础、宏观经济学问题、宏观经济学政策。考试以笔试为主,题型有多项选择、数据分析、结构化问题、小论文等。

计算机科学 计算机科学内容包括:计算机系统学、计算机通讯与软件学、结构化实践任务、系统软件技术、数据库理论、程序模块和集成信息系统、计算工程。考试以笔试为主,题型有问答及编程。
:wdb17:

Luck Li : 2009-05-13#686
回复: 加拿大教育情况有用素材收集帖子--梁溪香榭整理

全部是非常非常有用的东东,坛子里的很多TX都受益了。感谢香溪!

梁溪香榭 : 2009-05-13#687
回复: 加拿大教育情况有用素材收集帖子--梁溪香榭整理

全部是非常非常有用的东东,坛子里的很多TX都受益了。感谢香溪!

俺要好好掰掰指头,算算嫩一共给俺起了几个小名. :wdb6:

Luck Li : 2009-05-14#688
回复: 加拿大教育情况有用素材收集帖子--梁溪香榭整理

俺要好好掰掰指头,算算嫩一共给俺起了几个小名. :wdb6:
都是大家对嫩的爱称,请不要拒绝的说。:wdb17::wdb17::wdb19::wdb19:

梁溪香榭 : 2009-05-14#689
回复: 加拿大教育情况有用素材收集帖子--梁溪香榭整理

都是大家对嫩的爱称,请不要拒绝的说。:wdb17::wdb17::wdb19::wdb19:

哪里会拒绝啊:wdb6:. 爱得紧呢! :wdb6:

jessie_f : 2009-05-20#690
回复: 加拿大教育情况有用素材收集帖子--梁溪香榭整理

我家女儿也是读的A-LEVEL,现在高一,夏天升高二开始正式的核心课程,去年(08年)她中考前开始办的魁投,8月的FN,原先听说魁投快的,以为到10年8月办了2年应该可以了,谁想到按现在的进度,现在还在补料中,6月初送出去,到今年9,10月还不一定面试好,到10年的8月满2年时能拿体检就是好的了,可是女儿学校那时就要我们开始申请大学了,虽然加拿大大学截止在11年2月,可是名校最先预录取却是在11月左右,这就意味着申请时不能按移民身份,学费不一样不说,选择的专业会受影响才最重要,真是超级郁闷!不知道像这种情况,拿到取签信时也许已经在被学校预录取后的话,还能不能和学校再更改身份,因为等高三成绩出来被正式录取或开学11年9月都办了3年了,应该已经是移民的了,唉,该怎么办呢 :wdb5:

梁溪香榭 : 2009-05-20#691
回复: 加拿大教育情况有用素材收集帖子--梁溪香榭整理

我家女儿也是读的A-LEVEL,现在高一,夏天升高二开始正式的核心课程,去年(08年)她中考前开始办的魁投,8月的FN,原先听说魁投快的,以为到10年8月办了2年应该可以了,谁想到按现在的进度,现在还在补料中,6月初送出去,到今年9,10月还不一定面试好,到10年的8月满2年时能拿体检就是好的了,可是女儿学校那时就要我们开始申请大学了,虽然加拿大大学截止在11年2月,可是名校最先预录取却是在11月左右,这就意味着申请时不能按移民身份,学费不一样不说,选择的专业会受影响才最重要,真是超级郁闷!不知道像这种情况,拿到取签信时也许已经在被学校预录取后的话,还能不能和学校再更改身份,因为等高三成绩出来被正式录取或开学11年9月都办了3年了,应该已经是移民的了,唉,该怎么办呢 :wdb5:

这种情况的确是很头疼的. 在这个坛子里俺也曾经看到过好几位家长咨询类似的问题但好象没看到过比较肯定的答复.
基本上来说, 是以留学申请, 移民办下来再去学校更改身份是可以的. 但留学会不会影响专业选择, 增加录取难度, 我认为会的.
俺建议, 这种问题还是要直接问学校. 这里谁说了都不算, 搞不好还会误导.
直接写E-MAIL给心仪的学校吧. 多问几家, 看看他们怎么说.

jessie_f : 2009-05-21#692
回复: 加拿大教育情况有用素材收集帖子--梁溪香榭整理

这种情况的确是很头疼的. 在这个坛子里俺也曾经看到过好几位家长咨询类似的问题但好象没看到过比较肯定的答复.
基本上来说, 是以留学申请, 移民办下来再去学校更改身份是可以的. 但留学会不会影响专业选择, 增加录取难度, 我认为会的.
俺建议, 这种问题还是要直接问学校. 这里谁说了都不算, 搞不好还会误导.
直接写E-MAIL给心仪的学校吧. 多问几家, 看看他们怎么说.


谢谢梁溪啊,看来也只有这样办了,唉,麻烦,可是为了孩子也是没法子了,不过,梁溪的帖子一直很认真的看,呵呵,真是有心加用心啊,:wdb17::wdb17: ,等着听你的好消息哦,:wdb10::wdb10::wdb9::wdb9:

sunny-5926 : 2009-05-21#693
回复: 加拿大教育情况有用素材收集帖子--梁溪香榭整理

谢谢,有空看看!:wdb10:

sunny-5926 : 2009-05-21#694
回复: 加拿大教育情况有用素材收集帖子--梁溪香榭整理

梁溪真是个大好人,谢谢你啊!我得到这么多有用的信息真是太有启发了!

梁溪香榭 : 2009-05-23#695
回复: 加拿大教育情况有用素材收集帖子--梁溪香榭整理

谢谢梁溪啊,看来也只有这样办了,唉,麻烦,可是为了孩子也是没法子了,不过,梁溪的帖子一直很认真的看,呵呵,真是有心加用心啊,:wdb17::wdb17: ,等着听你的好消息哦,:wdb10::wdb10::wdb9::wdb9:

谢谢哈 :wdb6:

梁溪香榭 : 2009-05-23#696
回复: 加拿大教育情况有用素材收集帖子--梁溪香榭整理

梁溪真是个大好人,谢谢你啊!我得到这么多有用的信息真是太有启发了!

呵呵, 对大家有用俺就 :wdb6::wdb6::wdb6:

taylor2755 : 2009-06-04#697
回复: 加拿大教育情况有用素材收集帖子--梁溪香榭整理

请问香榭里,你家儿子在哪个中学啊?我们的也是初三

rwx1378 : 2009-06-07#698
回复: 加拿大教育情况有用素材收集帖子--梁溪香榭整理

我家女儿也是读的A-LEVEL,现在高一,夏天升高二开始正式的核心课程,去年(08年)她中考前开始办的魁投,8月的FN,原先听说魁投快的,以为到10年8月办了2年应该可以了,谁想到按现在的进度,现在还在补料中,6月初送出去,到今年9,10月还不一定面试好,到10年的8月满2年时能拿体检就是好的了,可是女儿学校那时就要我们开始申请大学了,虽然加拿大大学截止在11年2月,可是名校最先预录取却是在11月左右,这就意味着申请时不能按移民身份,学费不一样不说,选择的专业会受影响才最重要,真是超级郁闷!不知道像这种情况,拿到取签信时也许已经在被学校预录取后的话,还能不能和学校再更改身份,因为等高三成绩出来被正式录取或开学11年9月都办了3年了,应该已经是移民的了,唉,该怎么办呢 :wdb5:


情况和俺孩子一样,现读G2?在上海?俺是08年7月的FN。

王晶 : 2009-06-08#699
回复: 子女教育信息素材收集帖

嘎嘎

这个帖子也算历经了劫难了,丢了又重新找回来了。

主要整理的是关于教育的一些素材

从147楼开始,主要就是梁溪香榭搜集的一些的,推荐推荐!

推荐本书--从普通女孩到银行家,http://forum.iask.ca/showthread.php?t=233637

目录

tony 3213, PEI地区教育情况 Page 1, 5楼
tiantianxingchen 高中课程简介 Page 1, 6-9楼
守法公民 初三学生多伦多求学记 Page1, 18-22楼
守法公民 安省新移民高中指南 Page 2, 24-30楼
守法公民 安省新移民小学指南 Page 2, 34-36楼
守法公民 高中家长如是说 Page 3, 44-49楼
雨中冷百合 大多伦多地区高中排名 Page 3, 57楼, 59楼
守法公民 温哥华十佳公立中学 Page 4, 65楼
一缕阳光 菲沙关于BC省中学排名及相关简介 Page 4, 69楼
守法公民 部分大学对是否需要雅思或托福成绩的年限规定 Page 6, 101楼
守法公民 教育篇 Page 6, 113-121楼
梁溪香榭 高中留学须谨慎 Page 8, 149-152楼
梁溪香榭 加拿大高中教育感悟 Page 8, 153-155楼
梁溪香榭 根据成绩选择加拿大大学 Page 8, 159楼
梁溪香榭 如何选择最适合的加拿大大学 Page 9, 161楼
梁溪香榭 读高二的孩子去加拿大需要准备什么 Page 9, 164-165楼
梁溪香榭 高中英语课程介绍 Page 9, 166-169楼
梁溪香榭 高中英语课程介绍 Page 10, 188-191楼
梁溪香榭 多伦多著名私立学校收费一览 Page 10, 192楼
梁溪香榭 孩子的英语过关了吗? Page10, 194楼
梁溪香榭 关于A-LEVEL课程 Page 11, 217-228楼
shieley6811 关于A-LEVEL课程及学制 Page 13, 256-257楼
梁溪香榭 2007年安省高中拍名 Page 17, 322楼
朱三雀 安省, BC省, AB省高中拍名, Page 17, 329楼
响铃当当 加拿大教育优势 Page18, 347楼
梁溪香榭 A-LEVEL与IB课程的比较 Page 18, 358楼
朱三雀 中西学生的差异 Page 19, 364楼
梁溪香榭 大多伦多地区IB高中简介 Page 19, 372-375楼
梁溪香榭 IB课程与其他特殊教育课程的比较 Page 19, 376楼
梁溪香榭 IB与AP的比较, Page 19, 377楼
梁溪香榭 美国相关研究机构关于IB与AP的选择比较 Page 20, 387-391楼
梁溪香榭 群星灿烂不如一轮皓月当空-大学录取篇 Page 21, 409-413楼
梁溪香榭 在教育子女的过程中教育自己 Page 22, 422-424楼
梁溪香榭 关于美国大学 Page 22, 429-432楼
梁溪香榭 多伦多高中阶段特殊教育程序 Page 23, 445-446楼
梁溪香榭 加拿大高中生学科竞赛 Page 23, 447-448楼
梁溪香榭 美国大学 Page 24, 466楼
梁溪香榭 如何选择适合自己孩子的中小学 Page 25, 487-489楼
梁溪香榭 论孩子成长的内环境 Page 25, 490楼
梁溪香榭 犹太人的教育风格:朴实无华 讲求实际 Page 25, 497楼
梁溪香榭 培养子女的参政意识 Page 25, 498楼
梁溪香榭 美国大学点评 Page 25, 499-529楼
梁溪香榭 加拿大大学点评(商科) Page 27, 533-535楼
梁溪香榭 加拿大大学点评(理工科) Page 27, 547-549楼
你收集内容非常好,但是怎样才能快速查到这些文章?
我用笨方法,复制标题到百度,确实查到了,但是打不开,因为以前的网被封掉了,所以还是读不到你收集的文章。

梁溪香榭 : 2009-06-08#700
回复: 子女教育信息素材收集帖

你收集内容非常好,但是怎样才能快速查到这些文章?
我用笨方法,复制标题到百度,确实查到了,但是打不开,因为以前的网被封掉了,所以还是读不到你收集的文章。

怎么会呢?

每个内容都标记了在这个帖子的哪一页, 哪层楼了呀.
为什么要去百度找呢? :wdb2:

家园移民 : 2009-06-08#701
回复: 子女教育信息素材收集帖

你收集内容非常好,但是怎样才能快速查到这些文章?
我用笨方法,复制标题到百度,确实查到了,但是打不开,因为以前的网被封掉了,所以还是读不到你收集的文章。
嘎嘎

每一页吗有20个帖子

如果是193楼

9页到180

所以193楼就在第10页

就可以找到了

梁溪香榭 : 2009-06-08#702
回复: 子女教育信息素材收集帖

嘎嘎

每一页吗有20个帖子

如果是193楼

9页到180

所以193楼就在第10页

就可以找到了

恩, 是的. 其实目录里面也标注了页码了.

家园移民 : 2009-06-08#703
回复: 子女教育信息素材收集帖

恩, 是的. 其实目录里面也标注了页码了.
啧啧

嫩想的还真细啊

王晶 : 2009-06-08#704
回复: 子女教育信息素材收集帖

嘎嘎

每一页吗有20个帖子

如果是193楼

9页到180

所以193楼就在第10页

就可以找到了
理解错误,实际上文章就在本贴中!!!:wdb7::wdb7::wdb7:

王晶 : 2009-06-08#705
回复: 子女教育信息素材收集帖

恩, 是的. 其实目录里面也标注了页码了.
我以为文章在另外帖子中,理解性错误,该:wdb35:

sunnytan : 2009-06-12#706
回复: 加拿大教育情况有用素材收集帖子--梁溪香榭整理

谢谢梁溪妹妹,大量的资料真如雪中送炭!不好好研究真是对不起妹妹的辛勤劳动!喜欢你的签名,是BIBLE里的金句,GOD BLESS YOU!也谢谢守法提供的帮助!:wdb17:

willam131 : 2009-06-22#707
回复: 加拿大教育情况有用素材收集帖子--梁溪香榭整理

http://www.fraserinstitute.org/commerce.web/product_files/70BCESC09COMP.pdf大温中学最新排名

梁溪香榭 : 2009-06-23#708
回复: 加拿大教育情况有用素材收集帖子--梁溪香榭整理

谢谢梁溪妹妹,大量的资料真如雪中送炭!不好好研究真是对不起妹妹的辛勤劳动!喜欢你的签名,是BIBLE里的金句,GOD BLESS YOU!也谢谢守法提供的帮助!:wdb17:

God bless you!

梁溪香榭 : 2009-06-23#709
回复: 加拿大教育情况有用素材收集帖子--梁溪香榭整理



分分致谢!

Luck Li : 2009-06-24#710
回复: 加拿大教育情况有用素材收集帖子--梁溪香榭整理

God bless you!

GOD BLESS YOU! :wdb9::wdb9:

梁溪香榭 : 2009-06-24#711
回复: 加拿大教育情况有用素材收集帖子--梁溪香榭整理

GOD BLESS YOU! :wdb9::wdb9:

AND YOU!

jessie_f : 2009-06-24#712
回复: 加拿大教育情况有用素材收集帖子--梁溪香榭整理

情况和俺孩子一样,现读G2?在上海?俺是08年7月的FN。


咱是在你隔壁啊,苏州,08年8月的FN,补料送出去没多久,道路漫长着呢,好在有这里,也不孤单了,大家伙儿相互打气啦,:wdb10::wdb10:,还有梁溪这么多的奉献,:wdb17::wdb17:,真的是家园 :wdb23::wdb23: :wdb6::wdb6:

梁溪香榭 : 2009-06-26#713
回复: 加拿大教育情况有用素材收集帖子--梁溪香榭整理

[转帖]公立学校vs.私立学校

华人对子女教育的重视是不言而喻的,对于大多数人来说,只要条件允许,就会给子女创造最佳的学习环境,比如把孩子送进私立学校。据美国私立教育委员会(CAPE)统计,2009年全美私立学校有六百零四万九千名学生注册,占全美学生人数的11%)。不过,昂贵的学费又使不少家长望而却步,私立学校到底值不值呢?

要弄清楚这个问题,必须对公立和私立学校的一些基本情况做一个对比:

教学质量 。这是人们常常用来作为参照的一个标准,比较深入人心的理念是,私立学校的教学质量高于公立学校。做出这一结论的依据是私立学校学生在各种标准化考试中的成绩高于公立学校的学生。不过,如果把学生的家庭背景考虑进去的话,情况就不一样了。 2006 年,美国教育部资助美国国家教育统计中心( National Center for Education Statistics )所作的一项调查显示,公立学校四年级学生的阅读和八年级的数学与私立学校学生没有显著差异,公立学校四年级学生的数学成绩显著高于私立学校学生,私立学校只是在八年级的阅读上超过了公立学校学生(Department of Education: Comparing Private Schools and Public Schools Using Hierarchical Linear Modeling )。这一报告证实了早先伊利诺大学的一项研究:在等同的家庭社会经济背景下,公立学校的学生成绩略高于私立学校。这一差异在数学学科上尤为显著,因为数学成绩多数取决于学校的教育,家庭影响非常有限,不像阅读那样,家庭文化经济较好的家长会陪同孩子一块阅读。(Christopher and Sarah Theule Lubienski, University of Illinois Charter, Private, Public Schools and Academic Achievement ) 所以,私立学校教学质量高的理论缺乏有效的论证,有时私立学校学生的成绩好,很可能是学生的总体来源好,或者是学生家庭背景的关系,并不是学校的教学在起决定作用。

附件


梁溪香榭 : 2009-06-26#714
回复: 加拿大教育情况有用素材收集帖子--梁溪香榭整理

课程 。公立学校由于接受政府拨款,其课程也严格按照政府标准施行。公立学校课程的特征是涉及面广范,面向每一个学生,在教授传统学科的同时,也兼顾其它课程,如音乐、艺术、家政等。几乎所有公立学校的教学大纲都由政府统一制定,教学质量也会通过标准化测验进行监控。

私立学校在课程设置上有相当的自由度,有的学校偏重文艺,有的强调理化,有的还让学生参与许多户外活动包括参观访问旅游等等。多样化的教学手段为开阔学生视野起到了一定的积极作用。

教师。 公立学校的教师都必须经过专业培训,并获得政府统一颁发的教师执照。而私立学校虽然都具有大学学历,但不一定要求经过正规的师范训练。

学生。 公立学校的学生一般代表了所属社区的文化和经济背景,虽然个体学生在能力、兴趣和爱好上有所不同,但是根本的差异体现在家庭环境的多样性之上。由于私立学校的学生入学时就按照校方的要求进行了筛选,所以学生的整体差异较小。就美国来看,近 80% 的私立学校学生为白种人,公立学校只有 60% 左右。有 14% 的私立学校根本没有少数族裔的学生,而公立学校中的比例只有 4% 。

班级人数。 一般来说,公立学校的班级人数多于私立学校,私立学校的教师 / 学生比例也因此大于公立学校,私立学校为 9:1 ;公立为 17:1 。不过,不是所有孩子都适合小班教学,也有孩子在一个较大的集体里在有更多的同伴的环境下反而学习有更大的积极性。

学费。 这也许是最关键的问题了,在权衡公立和私立学校时,学费是一个重要的参照点。公立学校是免费的,然而私立学校在学费上的差异也很大。根据美国私立学校联合会( NAIS )的统计,该会所属的一万二千多所学校的平均学费是 1.5 万美元左右,而寄宿学校可高达 3.2 万美元。

那么是不是收费高就一定好,免费就一定不好呢?其实,就学生人均占有资源而言,政府在公立学校上的投入比私立学校多,私立学校的学费只抵得上政府在公立学校学生身上投入的三分之一到二分之一,当然,私立学校还有其它的赞助和资金来源。

附件


梁溪香榭 : 2009-06-26#715
回复: 加拿大教育情况有用素材收集帖子--梁溪香榭整理

一般来说,孩子上私立学校除了学费外,还有一大堆杂七杂八的开销,一不留神花上三万五万的也不足为奇。那么,这些花费是不是物有所值?

综上所述,我个人觉得,如果你的孩子是一个智力正常,有一定自学能力且积极向上的人,大可不必花这么多钱上私校。不就是要孩子多学一点吗?这几万块钱能不能用来请家教辅导(如果有必要的话)?或者省下带孩子出去旅游,在接触大自然中学习书本上学不到的知识。有些家长误认为花了钱把孩子就给私校就万事大吉了,反而放松了对孩子的辅导和督促。如果有条件的话,家长少挣这两三万,多花些时间陪伴孩子是不是有更大的效益?

无疑,从总体上看,私校的学习环境,教师的投入,纪律的维持要比公立学校好一些,不过在决定送孩子去私校之前,除了算一算经济账,除了让孩子今后考试成绩更好一些,还要考虑到问题有:

如果是男女分校,是不是对孩子成长有利?女校 / 男校将男女分开,对孩子在和异性交往的能力和观念上的影响是正面的还是负面的?孩子总体社交能力是否受到影响?

附件


梁溪香榭 : 2009-06-26#716
回复: 加拿大教育情况有用素材收集帖子--梁溪香榭整理

亚洲的学校出了什么问题

  最近一期的时代杂志报导探讨:亚洲的学校出了什么问题?针对亚洲各国的高中生的学习状况,做了一些调查报告,很值得我们作家长的重新思考,走出只重视孩子课业升学主义的迷思。  在这份报告中,以一位南韩的高中生为案例,报导他一天的生活,他每天早晨6点50分起床,至清晨2点睡觉。大部分的时间都在学校和补习班(cram school)接受填鸭式的教育,为的就是要升学。虽有较高成绩,但并不享受学习

  在这样的密集冗长的学习下,比较美国和亚洲国家的学业成绩,亚洲国家都名列前茅,然而,有趣的是,这些成绩优异的亚洲学生,在大学毕业后进入多国籍企业当中,却不见得是受企业主满意的员工。归纳其原因,这些亚洲学生虽然有较高的成绩表现,但是他们并不享受学习,他们喜欢做的事都是课堂以外的事。他们在学校时眼神呆滞,脑子里装满了各种信息,但是一旦考试结束就立即遗忘。他们的学习,不是为了想要求知,而是为了要升学。他所学的知识并无法和实际生活互相连结、应用。

亚洲学生所缺乏的学习特质

  相较于亚洲,虽然美国高中生的成绩略逊一筹,但是在美国确有最优秀的大学生和研究生。探究其原因:时代杂志归纳出下列几点,这是美国的高中生拥有的学习特质而亚洲学生所缺乏的。

  第一:独立思考的能力

  美国的学生能够多方面、且具前瞻性地的去思考事情,这是因为他们的学习有多元的刺激。而亚洲学生普遍比较难有自己的看法,或者他们的看法是比较制式、甚至不太确定自己的想法为何。美国的学生对自己的看法,却很有自信。

  第二,社交能力

  美国的教育提倡团队合作精神,鼓励合作做研究报告。他们的学习是经过比较、交换意见,融合不同的观点,经过讨论而成。所以美国的学生,有较优良的表达能力、沟通协调能力。

  第三:与科技的连结

  美国学生具备另一学习优势是,他们的学习和科技相连结。从小学开始,就人手一机利用网际网络来找资料学习。他们对于利用科技,并不生疏也没有畏惧。

超过在课堂里学习的

  综观上述时代杂志的报导,再次提醒各位姊妹:不要太在意孩子的学业成绩,反倒要在意你的孩子是否享受学习,他有无学习的动力。多元化的学习,对他人生的助益可能更胜于课堂里的学习。

梁溪香榭 : 2009-06-26#717
回复: 加拿大教育情况有用素材收集帖子--梁溪香榭整理

路在自己脚下
----- 一位小留学生的自述

出国是很多国内学生梦寐以求的梦想,当我得知我可以到加拿大上学时,天真地认为,那里的世界就好似童话一般,一切都会比国内先进和优越,学习压力也不会象在国内这么大,生活和学习一定会是无忧无虑,丰富多彩的。那时我只有15岁。1年之后,也就是2004年的3月,我来到了加拿大,居住在位于多伦多附近一个叫Cambridge 的小城,城市人口只有10万多。我住在父母朋友的家里,在附近的一所高中读10年级。1年之后,为了更多地了解加拿大风土人情,适应西方语言和文化的差异,我在一个西人阿姨家租了一间房子。在那里我独立生活了将近一年,自己管理自己的学习和生活,与房东阿姨相处也非常好,生活非常愉快,学到了很多知识。

2年过去了。回首一步步走过的历程,一切仿佛还在眼前。这2年,比我以前人生中任何一个时期都要精彩和难忘。

由于父母不在身边,所有的事情都需要自己去打理,加上紧张的学习和外界带来的压力,每天过得都很紧张。首先,每个星期学校大大小小的assignments, projects, quiz和tests是必不可少的。开始的时候,完成这些任务要花费很多时间和精力。这也是加拿大教育和国内不同的地方,学生在完成这些任务的过程中,锻炼自己的综合能力。其次,是生活上许多琐事,做饭,买菜,洗衣服,这些以前不用自己去考虑的事情,现在都要亲力亲为。最沉重的就是精神上的压力。这种压力是从你一登陆就有的,是一种无形的压力,它来自于异国的环境、家人的期盼和自己的孤独无助。

我也曾感到恐慌和畏惧,甚至想过回国。面对新环境给自己带来的各种压力,我不断地鼓励自己:不能退缩,要坚持,自己选择的路是正确的,就一定要走到底,否则永远是失败者。每当我看到一棵参天大树,在惊叹它的高大之余,我更欣赏的是它经历过无数次风雨的考验。如果它仅仅是温室中的一棵小树,就不会有如此震撼人心之美。人与树又有什么不同呢?我们既然来到这个世界上,就不能只寻找温室般的平静。

我开始学会坚强的面对困难。早已不是2年前蹲在汽车站无助地想哭的那个脆弱小女孩。要生存,就必须和自己战斗。我克服一个又一个的困难使自己在陌生的环境中慢慢成长起来,懂得了如何高效率地完成每一天的任务。700多天每天都嘱咐自己:我能行的,再苦再难,也不能放弃!

在一个陌生的城市里,很少会遇到熟悉的人,加上语言的不熟练,怎样才能融入到这个城市中呢?我觉得最重要的是要有勇气,要自信,要有胆量。当你迷路的时候,可以不停的问路人;当你不知道如何用英语表达的时候,要敢于向对方请教。中国人一向很爱面子,在国内学英语时,都不好意思张口说,到了加拿大,要改变过去的那些习惯,对自己不知道、不明白的事情,要多问,多请教,这样可以少走很多弯路。记得刚来的时候,我对打电话和接听电话留言很恐惧,现在已经驾轻就熟了,任何事情都是在日积月累的锻炼中提高的。

我的日常生活是这样安排的:作业一般都争取在学校完成。放学之后,用30分钟到1个小时去买食品。回家后,听电话留言,打一些电话,查看邮件,付帐单。现在又在申请大学,经常要和不同的学校保持联系。

处理完这些个人事务后,我会用30分钟到1个小时的时间做运动。房东家有一个较大的后院,可以在那里跳绳或跑步,然后就可以休息一下了。下午6:00左右开始做饭,我会尽量争取变换我的食谱,做到荤素、营养合理搭配,有时候还电话咨询妈妈和小姨。不过,有时为节省时间,变换口味,我也会买一些现成食品回来,或到餐馆美味一顿。我居住的城市很小,中国人也很少,中国餐馆的口味也不是很正宗。对于西餐,偶尔吃一顿还可以,吃多了,也不习惯。由于我住在西人家里,还学到了很多洋人的菜谱,自己也可以简单地做一些。

晚上的时间最有利用价值,也最有弹性。完成作业后,可以读读英文小说、圣经,看看英文电视,或者和同学打电话,练练英文,也可以在网上写写文章,和父母聊聊天。有时也会和同学出去吃饭,坐在一起,聊聊学校和生活中的趣事,每次大家在一起时就好像一家人一样。

为了开阔眼界,我还去Toronto、Ottawa 和Washington 旅游, 去感受北美都市文化。为得到一份学生工,我四处奔波,几经周折,通过应聘和面试。现在每周我会有10-16个小时的工作。时间不是很多,不过可以体验社会,丰富生活,通过干一些实际的工作,使我增强了在加拿大学习、生活的信心。

我相信大部分留学生的生活都是忙忙碌碌,充满色彩和激情,我们也很期望安逸、清静的生活,不过,人生真的承担不起停下脚步的代价,也许哪天功成名就了,没有欲望了,就可以隐居山林,归隐田居了,希望那时不要因为没有激情而感到茫然和空虚。

回想这2年来走过的点点滴滴,我相信,我的经历只是千千万万中的一个,每一位海外学子都有很多欲说还休的故事。异乡读书不仅仅是风花雪月的浪漫,学成名就的风华,更多的是鲜为人知的辛酸苦辣。重要的是,不论身处何方,身在何时,一定要相信,路就在脚下,请用你的双脚去走出你的人生格言!说句心里话,加拿大拥有美丽的自然生活环境,然而强压在海外学生头上的生活与精神压力很大,到处都是荆棘,充满了挑战,但是――我愿意面对这些挑战!

王晶 : 2009-06-27#718
回复: 子女教育信息素材收集帖子-冲置顶了

温哥华十佳公立中学
排名 校区 学校名称
1 温哥华 University Hill
2 西温 Sentinel
3 本那比 Burnaby North
4 列治文 Steveston
5 温哥华 Prince of Wales
6 温哥华 Point Grey
7 温哥华 Lord Byng
8 温哥华 Magee
9 列治文 Hugh Mcroberts
10 素里 Elgin Park
我的朋友买房子买到白石镇,因为孩子要到加拿大读书,在2年前买房子的时候,在附近的学校排名是第26名,现在孩子在国内初3毕业,孩子的父亲为孩子联系学校,结果这个学校现在排名为76名,目前孩子已经考上重点学校,她不肯去加拿大,因为这学校排名到76名。所以朋友再赶快买房子到排名前十几名的学校。
学校的名次每年在滚动,所以只能到了加拿大再选择学校。

家园移民 : 2009-06-27#719
回复: 子女教育信息素材收集帖子-冲置顶了

我的朋友买房子买到白石镇,因为孩子要到加拿大读书,在2年前买房子的时候,在附近的学校排名是第26名,现在孩子在国内初3毕业,孩子的父亲为孩子联系学校,结果这个学校现在排名为76名,目前孩子已经考上重点学校,她不肯去加拿大,因为这学校排名到76名。所以朋友再赶快买房子到排名前十几名的学校。
学校的名次每年在滚动,所以只能到了加拿大再选择学校。
家长和小孩都有些对排名的误解

建议研读下 比尔这家伙 关于学校排名和选择的说法,个人觉得是目前为止偶读到的最准确的分析

梁溪香榭 : 2009-06-27#720
回复: 子女教育信息素材收集帖子-冲置顶了

家长和小孩都有些对排名的误解

建议研读下 比尔这家伙 关于学校排名和选择的说法,个人觉得是目前为止偶读到的最准确的分析

严重同意! 建议王晶让嫩朋友看看这本书.

故乡的云 : 2009-06-30#721
回复: 加拿大教育情况有用素材收集帖子--梁溪香榭整理

非常有用的帖子

惴惴不安 : 2009-07-22#722
回复: 加拿大教育情况有用素材收集帖子--梁溪香榭整理

移民学生多“断层” 上公校还是私校难倒华人家长(图)
(博讯北京时间2009年7月23日 转载)

来源:《环球华报》

据报道,Gap Student,是加拿大教育界人士对16岁左右抵加移民学生的称谓。中文可以译作“断层学生”。这些学生来加后便要进入开始选科的10年级,由于加拿大与原居地的教育体系,在授课语言、学科选择、课程设置、教学方法及考核制度等方面,都存在不少的差异。所以,移民学生在两地教育系统接轨时都会出现断层现象,从而令他们成为移民家庭中最为负累的一群。而加拿大的公立学校往往不能完全照顾这些移民学生,于是,针对这一群体的私立学校便应运而生了。到底是上公校好?还是选私校好呢?许多移民家长仍是十分彷徨。

移民学生多“断层” 上公校还是私校难倒华人家长


华裔家长望子成龙心切。


难为天下父母心

Daisy和David,同样是两年前随父母从北京来加的移民学生,而且他们在北京时也是中学同班同学。移民加拿大时,他们同是17岁,按加拿大的学制进入11年级。

Daisy的父母本在中国有着薪优粮厚的稳定工作,被称为“打工皇帝”的“金领一族”。2000年,他们下定决心移民加拿大,主要是为了孩子,希望Daisy能够进入世界一流的加拿大公立中学完成高中课程。

“当时想得很简单,加拿大的教育体系良好,而且移民可以享受免费公立教育,就算我们牺牲一点也值得。”Daisy妈妈对记者说,“不过,人算不如天算。移民手续一办就是六、七年,到我们正式登陆时,女儿已经17岁了。”

虽然好不容易才得到加拿大的移民身份,但这张迟来的移民纸却打乱了他们的家庭计划。“几年前,我们已不指望加拿大能批准我们的移民申请,所以,女儿一早便以国际学生的身份申请位于维多利亚的私校圣・迈克尔中学(St. Michaels University School)。”Daisy妈妈说,“女儿中学的录取通知书与移民纸几乎是同时收到的。”她说,当时一家人仍然希望女儿可以上公立学校,甚至已经在卑诗大学附近找房子,准备让Daisy上公立名校大学山中学(University Hill Secondary School)。

“不过,有人提醒我们,虽然住在学校所属地区,但移民学生未必可以马上进入U Hill,”Daisy妈妈表示,当时不少朋友都说,由于太多人想进名校,所以头一、两个学期可能会先被分配到其他学校,尤其是ESL学生。

由于只剩下两年的中学时间,Daisy妈妈觉得女儿已经浪费不起了,于是,她打消了送女儿入读公立名校的念头,下定决心陪Daisy到维多利亚上圣・迈克尔中学。

也许是英语基础好,也许是自身努力,也许是天资聪颖,Daisy以中上的成绩完成了所有高中课程,并已经获取多伦多大学的入学通知书,今年9月便开始大学生涯了。Daisy妈妈终于松了一口气。

Daisy在北京时的同班同学David,与她走的是完全不同的路。David的父亲朱先生是个成功的商人,其家境优于Daisy,完全有能力送 David上任何一间私立名校。但朱太太认为,加拿大投入在公立教育系统中的财力物力占政府财政预算很大比例,对比起私立学校,公立学校的优势更多。她觉得,自己虽是新移民,但也是加拿大的纳税人,应该有权力去享受这里的公立教育服务。

于是,朱太太坚持让David上公立学校。不过,David被编入了ESL班,两年下来,他还未脱离ESL的“苦海”。眼睁睁在看着昔日的同窗即将入读大学,自己却不知什么时候才能完成中学学业,David一家十分苦恼,后悔当初没有让儿子去读私校。


公立学校资源充足

到底新移民学生是上公校好,还是选私校好?我们带着此一问题,请教了资深教育工作者朱婉瑶。

朱婉瑶,曾任公立小学、中学教师,代理校长,拥有校监资格;后担任安省教育厅教育专员,专责函授的中学学分课程并颁发中学文凭;也曾从事3年的私立中学教育。她对加拿大的公立及私立中学的优劣长短了如指掌。“公立中学的资源和综合素质肯定优于私立中学。不论是从教育经费、师资力量还是从教学硬件上来说,两者都有很大的差距。”朱婉瑶指出,首先在经费方面,各级政府每年在教育上投放的财政预算十分庞大,这是加拿大拥有高水平公立教育的基本保证。

“其次是公立学校的师资力量是私立中学无法比拟的。”朱婉瑶指,许多人误以为私校教师的人工普遍高于公校,但事实并非如此。“一个从师范毕业进入公校的教师,薪金绝对比到私校的同行高。即使是老牌私立名校的教师,工资也与普通公校的教师处于同一水平。”

朱婉瑶表示并非说私校就没有好教员,但一般有志于投身教师行业的,第一选择都会是公立学校。她说,高素质的好教师几乎都被公立学校招揽了,“由于教育资金的充足,公立学校能够提供优越的待遇聘用最好的教师。这是公校另一明显的优势。”“在课程上有优秀的教师,而在实践中,公立中学给学生们提供了最好的硬件设施。”朱婉瑶补充说。


公校成绩不佳之因

上月,菲沙研究所(Fraser Institute)公布2007/2008年度卑诗省中学成绩单。结果显示,排在前10名的中学均为私校。而打进头20名内的公校却只有一间。

对于公立学校成绩普遍差强人意,列治文学务委员区泽光认为主要是学生来源所造成的。加拿大实行的是平等的普及教育,公校有责任接收学区内的所有学生,这就令同一班学生中存在水平参差不齐的现象,为教学带来了不少的难度。而私校,由于有入学考试,同一班的学生在同一水平上,老师可以有针对地进行施教,学生成绩提高也会较快。公立学校也可以培养出优于私校的尖子学生,但由于公校学生的差距较大,所以,平均下来,在排名榜上往往逊于部分私校。

“学生的家庭环境也是公校排名不佳的原因之一,”朱婉瑶指出,有些公校学生父母收入低,经常是为生计而奔波,子女与父母见面的机会都不多,有些父母甚至连自己都照顾不了自己,而需要政府照顾。“公校在照顾这些学生方面花了不少精力。”


公校未满足学生要求

不论是从教育经费、师资力量还是从教学硬件上来说,公校都优胜于私校,那为什么近年来出现公校学生减少,私校学生反而增加的现象呢?

朱婉瑶认为,家长选择私立学校的原因有很多。有的是对公校系统不满意;有的是希望孩子在一种父母欣赏和认同的特殊教育方法中成长;有的是因为孩子有特别的天分和需要,是普通公校无法提供的。

“对于移民学生来说,情况更为严重。”朱婉瑶表示,目前的公校系统并不能满足移民学生的特殊要求。提到David的案例,朱婉瑶透露,这样的情况她碰到不少。“教育就是要帮学生,提供他们所需要的东西。”朱婉瑶指出,“显然,全加的公校都未能完全帮助移民学生,尤其是‘断层学生’。”

“原因是我们的教育系统对移民学生的背景不明白,也不愿意去了解,即使了解后也不愿意去帮他们。”朱婉瑶认为,这是某些人怕移民学生的成绩上来后,会抢了一些人的风头和资源。


私校补公校之不足

资深教育工作者、教育心理学博士卢群毅对记者说,其实不少人对本地的私立学校了解并不多,甚至存有误解。“加拿大的私立学校并不都是以赢利为目的的。”她介绍说,本地私校大概分为三类,一是传统的贵族学校,这些学校一般历史悠久,要求较高,不愁生源;二是教会或宗教组织学校,由于公校不允许宣扬宗教,所以不少宗教举办学校,弘扬教义,这类学校并不以赢利为目的;三是新兴的商业学校,这些学校大多以帮助学生升学为目的。

卢群毅认为私立学校具有其独特性,是公立学校的补充。“私校弥补了公校弹性不足的弱点,对学生进行针对性的个别辅导,有利于移民学生尽快完成ESL课程。


私校发展问题多多

不过,由于过去曾屡次发生事情,私立学校的管理一向是人们关心的问题。卢群毅认为,本地的私立学校大部分都是好的,教育质量也有保证,但也有一些私校以赢利为目的,学生成绩单的水分较多。以致有在这些学校高分毕业的学生,进入大学后才发觉自己的成绩原来并不是想像那样好,功课也十分吃力,有学生更因多门功课不合格而被勒令退学。

“当教育成为一种商品的时候,就成为严重的社会问题了。”卢群毅认为,在私立学校补充不足的同时,也不能忽视公校的改革,让公立教育体系适应学生的需要。朱婉瑶也认为,为移民学生提供所需教育服务是政府、教育厅、教育局和学校责无旁贷的职责。当局应该采取更有效和开放的措施帮助新移民学生,尤其是 “断层学生”。

对于是上公校还是私校好,卢群毅认为要视乎学生的具体情况而定。当然在这过程中,家长一直很配合,所以说,无论孩子上公校还是私校,都离不开家长的关心。 _(网文转载) (博讯 boxun.com)

点击这里对此新闻发表看法

pwjnd : 2009-07-24#723
回复: 加拿大教育情况有用素材收集帖子--梁溪香榭整理

一看前面的整理,太感动了,感谢守法领导和溪溪为坛子里做得贡献,先表谢意再慢慢仔细读。

vivian1989 : 2009-07-25#724
回复: 加拿大教育情况有用素材收集帖子--梁溪香榭整理

新移民,我是在坐大?生,想去?哥攘坐成人高中再坐本科,???哥攘的成人高中可以坐到多少?呢?著著

vivian1989 : 2009-07-25#725
回复: 加拿大教育情况有用素材收集帖子--梁溪香榭整理

???一下,在坐大?生在??要??什麽手理去那彦坐??

梅朵 : 2009-07-27#726
回复: 加拿大教育情况有用素材收集帖子--梁溪香榭整理

超有价值的帖子,守法和溪溪辛苦了!

laicanada : 2009-07-29#727
回复: 加拿大教育情况有用素材收集帖子--梁溪香榭整理

看了一下午还没看完。先谢谢守法和梁溪!

梁溪香榭 : 2009-07-30#728
回复: 加拿大教育情况有用素材收集帖子--梁溪香榭整理

新移民,我是在坐大?生,想去?哥攘坐成人高中再坐本科,???哥攘的成人高中可以坐到多少?呢?著著

应该是可以读到老

梁溪香榭 : 2009-07-30#729
回复: 加拿大教育情况有用素材收集帖子--梁溪香榭整理

???一下,在坐大?生在??要??什麽手理去那彦坐??

这个俺不专业。不敢瞎说。抱歉。

梁溪香榭 : 2009-07-30#730
回复: 加拿大教育情况有用素材收集帖子--梁溪香榭整理

非常有用的帖子

有用就好 :wdb6:

移民学生多“断层” 上公校还是私校难倒华人家长(图)
(博讯北京时间2009年7月23日 转载)

来源:《环球华报》

据报道,Gap Student,是加拿大教育界人士对16岁左右抵加移民学生的称谓。中文可以译作“断层学生”。这些学生来加后便要进入开始选科的10年级,由于加拿大与原居地的教育体系,在授课语言、学科选择、课程设置、教学方法及考核制度等方面,都存在不少的差异。所以,移民学生在两地教育系统接轨时都会出现断层现象,从而令他们成为移民家庭中最为负累的一群。而加拿大的公立学校往往不能完全照顾这些移民学生,于是,针对这一群体的私立学校便应运而生了。到底是上公校好?还是选私校好呢?许多移民家长仍是十分彷徨。

谢谢惴惴!

一看前面的整理,太感动了,感谢守法领导和溪溪为坛子里做得贡献,先表谢意再慢慢仔细读。

超有价值的帖子,守法和溪溪辛苦了!

看了一下午还没看完。先谢谢守法和梁溪!

灰常感动ING。。。。。。

惴惴不安 : 2009-07-30#731
回复: 加拿大教育情况有用素材收集帖子--梁溪香榭整理

好久没碰上梁溪妹妹了!

梁溪香榭 : 2009-07-30#732
回复: 加拿大教育情况有用素材收集帖子--梁溪香榭整理

好久没碰上梁溪妹妹了!

呵呵, 不好意思。
要被领导批评鸟 :wdb8::wdb8:

Luck Li : 2009-07-31#733
回复: 加拿大教育情况有用素材收集帖子--梁溪香榭整理

呵呵, 不好意思。
要被领导批评鸟 :wdb8::wdb8:
能经常回来看看我们就好,呵呵!:wdb20::wdb20:

生命的狂想 : 2009-07-31#734
回复: 加拿大教育情况有用素材收集帖子--梁溪香榭整理

好贴

vivian1989 : 2009-08-02#735
回复: 加拿大教育情况有用素材收集帖子--梁溪香榭整理

应该是可以读到老
谢谢姐姐

Luckychen : 2009-08-02#736
回复: 加拿大教育情况有用素材收集帖子--梁溪香榭整理

:wdb10::wdb10::wdb10:很好的贴!

jjjxsj0501 : 2009-08-04#737
回复: 加拿大教育情况有用素材收集帖子--梁溪香榭整理

我09.06月刚体检,正在等待打款信,我儿子今年高考被中国传媒录取,现在好纠结,不知该咋办...望明白的人给点建议好吗?眼看要开学了,签证又不来,孩子是该上大学还是等候出国,如果是出国,又该怎么上学呢...

:wdb2::wdb2::wdb2:

Luck Li : 2009-08-05#738
回复: 加拿大教育情况有用素材收集帖子--梁溪香榭整理

我09.06月刚体检,正在等待打款信,我儿子今年高考被中国传媒录取,现在好纠结,不知该咋办...望明白的人给点建议好吗?眼看要开学了,签证又不来,孩子是该上大学还是等候出国,如果是出国,又该怎么上学呢...

:wdb2::wdb2::wdb2:

在国内上大学用不了多少银子,先上学别让孩子闲着。该学车学车,呵呵!等签证下来了,再走也不迟,你说对吧。一磨蹭就半年,一个学期过去了。:wdb9::wdb9::wdb6::wdb6:

Luck Li : 2009-08-05#739
回复: 加拿大教育情况有用素材收集帖子--梁溪香榭整理

反正孩子过去了,也要从新开始学语言过关,不可能马上就上上大学。

王晶 : 2009-08-05#740
回复: 加拿大教育情况有用素材收集帖子--梁溪香榭整理

我09.06月刚体检,正在等待打款信,我儿子今年高考被中国传媒录取,现在好纠结,不知该咋办...望明白的人给点建议好吗?眼看要开学了,签证又不来,孩子是该上大学还是等候出国,如果是出国,又该怎么上学呢...

:wdb2::wdb2::wdb2:
当然在国内读大学,等签证下来再去加拿大也没关系。

jjjxsj0501 : 2009-08-06#741
回复: 加拿大教育情况有用素材收集帖子--梁溪香榭整理

谢谢各位给的建议,不是说国内的学历不认吗?不能转学分吗?那该怎么办呢?

故乡的云 : 2009-08-11#742
回复: 加拿大教育情况有用素材收集帖子--梁溪香榭整理

不错的

shilin6008221520 : 2009-09-03#743
回复: 子女教育信息素材收集帖

嘎嘎

这个帖子也算历经了劫难了,丢了又重新找回来了。

主要整理的是关于教育的一些素材

从147楼开始,主要就是梁溪香榭搜集的一些的,推荐推荐!

推荐本书--从普通女孩到银行家,http://forum.iask.ca/showthread.php?t=233637

目录

tony 3213, PEI地区教育情况 Page 1, 5楼
tiantianxingchen 高中课程简介 Page 1, 6-9楼
守法公民 初三学生多伦多求学记 Page1, 18-22楼
守法公民 安省新移民高中指南 Page 2, 24-30楼
守法公民 安省新移民小学指南 Page 2, 34-36楼
守法公民 高中家长如是说 Page 3, 44-49楼
雨中冷百合 大多伦多地区高中排名 Page 3, 57楼, 59楼
守法公民 温哥华十佳公立中学 Page 4, 65楼
一缕阳光 菲沙关于BC省中学排名及相关简介 Page 4, 69楼
守法公民 部分大学对是否需要雅思或托福成绩的年限规定 Page 6, 101楼
守法公民 教育篇 Page 6, 113-121楼
梁溪香榭 高中留学须谨慎 Page 8, 149-152楼
梁溪香榭 加拿大高中教育感悟 Page 8, 153-155楼
梁溪香榭 根据成绩选择加拿大大学 Page 8, 159楼
梁溪香榭 如何选择最适合的加拿大大学 Page 9, 161楼
梁溪香榭 读高二的孩子去加拿大需要准备什么 Page 9, 164-165楼
梁溪香榭 高中英语课程介绍 Page 9, 166-169楼
梁溪香榭 高中英语课程介绍 Page 10, 188-191楼
梁溪香榭 多伦多著名私立学校收费一览 Page 10, 192楼
梁溪香榭 孩子的英语过关了吗? Page10, 194楼
梁溪香榭 关于A-LEVEL课程 Page 11, 217-228楼
shieley6811 关于A-LEVEL课程及学制 Page 13, 256-257楼
梁溪香榭 2007年安省高中拍名 Page 17, 322楼
朱三雀 安省, BC省, AB省高中拍名, Page 17, 329楼
响铃当当 加拿大教育优势 Page18, 347楼
梁溪香榭 A-LEVEL与IB课程的比较 Page 18, 358楼
朱三雀 中西学生的差异 Page 19, 364楼
梁溪香榭 大多伦多地区IB高中简介 Page 19, 372-375楼
梁溪香榭 IB课程与其他特殊教育课程的比较 Page 19, 376楼
梁溪香榭 IB与AP的比较, Page 19, 377楼
梁溪香榭 美国相关研究机构关于IB与AP的选择比较 Page 20, 387-391楼
梁溪香榭 群星灿烂不如一轮皓月当空-大学录取篇 Page 21, 409-413楼
梁溪香榭 在教育子女的过程中教育自己 Page 22, 422-424楼
梁溪香榭 关于美国大学 Page 22, 429-432楼
梁溪香榭 多伦多高中阶段特殊教育程序 Page 23, 445-446楼
梁溪香榭 加拿大高中生学科竞赛 Page 23, 447-448楼
梁溪香榭 美国大学 Page 24, 466楼
梁溪香榭 如何选择适合自己孩子的中小学 Page 25, 487-489楼
梁溪香榭 论孩子成长的内环境 Page 25, 490楼
梁溪香榭 犹太人的教育风格:朴实无华 讲求实际 Page 25, 497楼
梁溪香榭 培养子女的参政意识 Page 25, 498楼
梁溪香榭 美国大学点评 Page 25, 499-529楼
梁溪香榭 加拿大大学点评(商科) Page 27, 533-535楼
梁溪香榭 加拿大大学点评(理工科) Page 27, 547-549楼

shilin6008221520 : 2009-09-03#744
回复: 加拿大教育情况有用素材收集帖子--梁溪香榭整理

谢谢溪溪提供了这么多有用的资料,先谢谢,再仔细看。

梁溪香榭 : 2009-09-04#745
回复: 加拿大教育情况有用素材收集帖子--梁溪香榭整理

加拿大中学教育

加拿大的中学教育制度比较复杂,各个不同省份之间又存在很大的差异。一般情况下,中学一般由八年级到十二年级,大概在十年级时就可以分出就读大学还是社区学院的兴趣来,这主要通过选择不同的课程实现

在加拿大,中学开始沿用选课方式,年级越高可选课程越多,通常学校有辅导老师专门协助学生选课或改选课程。一般中学可供选择的课程非常多,课程总体上重视学术、智能、人际关系、社会发展、技能及职业培训。一般英文、社会研究、数学和科学是必修课。不准备上大学的可选择绘图、木工、计算机操作等,相当于中国的职业高中,希望上大学的可选择学术课程。在中学阶段,学生必须完成一定学分才能中学毕业,相当于中国的普通高中,另外学校可能要求学生有作义工和社会打工的学时或经历。

中学的课程由不同的教师讲授,学生可以根据选择的课程到不同的教室上课。中学的作业也逐渐增加。公立学校还通常提供奖学金给成绩优秀的学生,如果学生在某些方面有专长和天份,还可以与辅导员商量,看看选择哪些课程能够特别适合其个性的成长和能力的发挥。

一般进入加拿大大学要综合考虑中学平时成绩和省试成绩,一些研究机构会把各个中学的省试成绩排队,说明各个学校的办学水平,学生家长往往根据这些排名为子女选择学校。另外大学录取除了学习成绩外,各学生的运动能力、课外活动、特长、社区服务、个性和志向都会影响申请成功与否,多数学校都希望招收综合表质高的学生。申请入读大学的条件各省的要求不同,如在卑诗省,学生要读完11、12年级的13个学分且通过省考,才可取得省高中毕业文凭,有了此文凭才可申请大学。在安大略省,学生必须修完三十个学分才能获得安省中学毕业文凭,学生如想升读大学,修读的三十个学分中,必须有六个安省大学预科科目。

鉴于加拿大中学教育制度的复杂性和各个不同省份教育制度的差异性,以下着重介绍安大略省的中学教育制度。

Luck Li : 2009-09-04#746
回复: 加拿大教育情况有用素材收集帖子--梁溪香榭整理

加拿大中学教育

加拿大的中学教育制度比较复杂,各个不同省份之间又存在很大的差异。一般情况下,中学一般由八年级到十二年级,大概在十年级时就可以分出就读大学还是社区学院的兴趣来,这主要通过选择不同的课程实现

在加拿大,中学开始沿用选课方式,年级越高可选课程越多,通常学校有辅导老师专门协助学生选课或改选课程。一般中学可供选择的课程非常多,课程总体上重视学术、智能、人际关系、社会发展、技能及职业培训。一般英文、社会研究、数学和科学是必修课。不准备上大学的可选择绘图、木工、计算机操作等,相当于中国的职业高中,希望上大学的可选择学术课程。在中学阶段,学生必须完成一定学分才能中学毕业,相当于中国的普通高中,另外学校可能要求学生有作义工和社会打工的学时或经历。

中学的课程由不同的教师讲授,学生可以根据选择的课程到不同的教室上课。中学的作业也逐渐增加。公立学校还通常提供奖学金给成绩优秀的学生,如果学生在某些方面有专长和天份,还可以与辅导员商量,看看选择哪些课程能够特别适合其个性的成长和能力的发挥。

一般进入加拿大大学要综合考虑中学平时成绩和省试成绩,一些研究机构会把各个中学的省试成绩排队,说明各个学校的办学水平,学生家长往往根据这些排名为子女选择学校。另外大学录取除了学习成绩外,各学生的运动能力、课外活动、特长、社区服务、个性和志向都会影响申请成功与否,多数学校都希望招收综合表质高的学生。申请入读大学的条件各省的要求不同,如在卑诗省,学生要读完11、12年级的13个学分且通过省考,才可取得省高中毕业文凭,有了此文凭才可申请大学。在安大略省,学生必须修完三十个学分才能获得安省中学毕业文凭,学生如想升读大学,修读的三十个学分中,必须有六个安省大学预科科目。

鉴于加拿大中学教育制度的复杂性和各个不同省份教育制度的差异性,以下着重介绍安大略省的中学教育制度。

进来学习一下,新内容。:wdb17:

梁溪香榭 : 2009-09-04#747
回复: 加拿大教育情况有用素材收集帖子--梁溪香榭整理

安大略省中学教育制度

从1999年起,安大略省政府对全省的中学的学制进行了改革,由原来的5年制缩短为4年制。新的中学学制要求学生从9年级开始,需要逐渐明确自己将来的发展方向:是继续大学、大专还是直接工作。换句话说,也就是将学生分为两类:一类是想继续读大学、大专的学生,另一类则是打算直接工作的学生。从中学9年级开始,所有的基础科目都按照两套教育方案进行授课。

一套为学术型(Academic)。学术型课程强调知识的理论性和抽象性,如果学生中学毕业后想继续读大学或大专的话,应选择学术型的课程。
另一套为应用型(Applied)。应用型课程强调知识的具体化,即与现实生活密切联系,包括大量的社会实践机会,以提高学生知识的实际应用能力。如果学生想中学毕业后直接参加工作的话,则需选读应用型课程。

选课
进入中学以后,课程种类繁多,选择合适的课程非常关键。
首先,中学8年级的时候,所有学生都会得到一本各类课程介绍的手册。每年学生和家长都有机会参加一系列由学校举办的课程介绍讲座。学生的家长应和学校保持联系,及早了解这类讲座的时间安排。选课时要重点考虑学生的特长、兴趣和未来职业的目标,从而选择不同类型和内容的课程。不同的大学、大专、学徒计划和各种培训计划都有不同的要求,学生应该根据这些要求来决定选择什么课程,学校会提供这方面的信息。
具体地说,9年级的学生要在数学、英语、科学、地理和法语课程上选读学术型课程或应用型课程,历史课要在10年级才开始分类。在9年级和10年级还有一类课程是开放型课程(Open Courses),即所有的学生都可以选择,如艺术、健康、体育、商业研究等就属于这一类。9年级时,一个学生可以某些课程选择学术型的,某些课程选择应用型的,以便发现自己到底适合什么样的发展道路,10年级依然可以进行这样的探索。
如果说9年级和10年级,学生重在学习一些基本的知识和技能,那么在中学的11年级和12年级,学生的选课就应该和自己的未来发展方向直接相关。10年级选课的时候还应该注意的是,有些11年级和12年级的课程需要一些预备课程的学分。所以学生在10年级时就考虑11年级时自己会选修哪些课程,这些课程是否需要阶段预备学分,这样在10年级时就应该先修完预备课程,以便到时候能选上自己想选的课程,学校备有所有课程所需的预备学分的详细说明。
如果学生在9年级时已经确定了自己的方向,并选择了相应的课程,但后来的志向发生了改变,即便如此,学生也可以在10年级时比较容易地在两种类型课程之间转换。到了11年级以后,要转变课程类型,就必须再选择一种转换课程(Transfer course),以便把自己原来修的学分转换为另一方向所要求的学分,这类课程通常会在暑假进行。
在安大略省,中学还有一种划分方式为学期制学校和非学期制学校。学期制学校每年有两个学期,获得每个学分需要半学年的学习,非学期制学校获得每个学分需要修一整学年的课程。
在每年的开学时,学校都会下发校历,告诉学生学校会提供哪些课程,具体考试时间等,以便让学生及早安排。

成绩报告单
在每学年开始时,每个学生都会收到一份课程大纲和考试指南。学生的成绩是根据平时的实验室的工作、口头演讲报告、课堂参与、小测验及作业完成情况等综合因素进行评估的。通常情况下,学校主要从学生对知识的理解和掌握、思考能力与提问能力、交流沟通能力、应用能力四个方面进行考察。
按照以上的考核标准,最终的学分成绩70%为平时的表现,30%为期末考试成绩。非学期制学校一年有3次成绩报告单,学期制学校一年有4次成绩报告单,具体的时间学生的家长应主动向学校询问。

课外活动
中学有很多丰富多彩的课外活动,重在培养和锻炼学生的社会实践能力和社会交际能力。如体育队、音乐组、学生剧团和模拟学生政府等活动。对于新移民的学生来说,多参加这些活动,有助于结识更多的朋友,尽快适应新的生活。在一些学校,还有专门适合ESL学生参加的活动。同时中学生在4年的学习期间,还要完成40个小时的社会义务服务,在申请上大学时,这些课外活动及社区参与活动都是很重要的考评成分。

中学毕业要求
在安大略省,中学学习采用学分制。每修完一门课程,记一个学分。要想取得高中毕业文凭,学生必须达到以下的几项条件:
修满30个学分,其中包括18个必修学分,12个为选修学分。
18个必修课学分分配如下:
英语4分,法语1分,数学3分,科学2分,加拿大历史1分,加拿大地理1分,艺术1分,体育健康1分,职业教育1分,英语或第三世界外语或社会学或加拿大与世界1分,体育或商业或艺术1分,科学或高科技教育1分。
12个选修课学分可以从学校提供的多种课程选择,可按自己将来的发展方向自行选修。
完成40个小时的社区义务服务工作。此项工作是为了鼓励学生参与社会,利用业余时间为社区义务服务。学校会帮助学生与各类社区机构联系,让学生在实践中逐步了解并认识社会。
通过全省的英语统考。
全省英语统考是在学生10年级时进行,考查学生英语的读写能力。如果第一次考试没有通过,可以进行补考,直至统考通过方可领取中学毕业证书。

梁溪香榭 : 2009-09-04#748
回复: 加拿大教育情况有用素材收集帖子--梁溪香榭整理

加拿大高中选课制

在加拿大初中与高中的课程编排有很大的差异. 高中起进入选课学分制度. 学生在四年内要修足30个学分, 毕业时才能得到高中毕业证书. 没有修足学分的学生, 仍是一起参加快乐的毕业典礼, 甚至会得一些奖项, 如社区服务, 运动项目, 音乐美术, 特殊技艺等等, 受到相同的鼓励. 只是没有一份毕业证书, 当然就不能直接选择继续申读专科或大学.

学生正常每学年可修8个学分, 每学期4个学分. 4个学分也就是四门课. 一天只上四门课, 上下午各二堂, 每堂课 90分钟. 科目选择是学生自行按学校课表挑选, 再由学校统一规划分配. 如一Grade 9, 9年级学生, 上学期选了9年级的英文, 数学, 体育, 美术四堂课, 则他每天都是这四门课. 要上整个学期. 下学期时他的课可能变成上9年级的科学, 地理, 法文, 现代工艺. 而他的其它同学可能跟他完全颠倒, 或也可能部分一样. 学生每学期上完且学期成绩超过 50者, 即是及格并可取得1学分. 所以, 一年可修得8个学分credit, 四年合计应可轻松修足32学分. 若有二科不及格, 只要不是必修科目, 仍足够30学分, 可获得毕业证书. 对于不及格的科目, 如果又是必修的, 学生则需利用暑假重修, 或待新学年时重修该门课.

对于30个学分的内容, 安大略省教育部的规定是, 必修18个学分, 选修12个学分. 例如有4个英文学分, 也就是学生四年, 9 – 12年级中, 每年上一次. 科学有2 credits, 分别排在 Grade 9 and 10. 数学有3 credits, 除了在9, 10年级要上以外, 要学生在11, 12年级时还需再上一次较高年级的数学. 除此要求之外, 这些科目还有连贯性, 也就是9年级时若学生科学成绩不及格, 是不可以去上10年级的科学.

现将18个必修学分明细列出
4 英文, 3 数学, 2 科学, 1法文, 1地理, 1历史, 1艺术, 艺术有四种, 美术, 音乐, 舞蹈, 戏剧, 任选一种即可
1体育, 这里的体育课内容, 包括静态的健康教育
0.5 公民课, 0.5 生涯规划, 这二门课在10年级中的一学期要上,
剩下的三个学分课目, 设计的很科学, 也很有弹性, 学生需在, 强化英文, 第三国语言, 社会人文科学, 高阶史地科目中自选
学生需额外在体育, 艺术, 商业科目中自选一门
学生需在额外的现代工艺课, 或10年级后的生物, 化学, 物理选一门
由以上可看出学生在9, 10年级时, 上的大都是必修科目
商科类 - 财务会计, 行销, 商用计算机, 国际贸易, 企业组织
史地类 - 物质地理, 观光旅游, 环境与资源, 世界地理, 近代史, 中世纪史, 世界史, 西方史, 加拿大法律研究
高阶数学; 个人财管, 方程式, 数学与生活, 几何代数, 微积分, 数据管理,
科学; 11, 12年级生物, 化学, 物理
法文; 10, 11, 12 年级法文
艺术; 视觉艺术, 绘画, 爵士乐, 乐器, 合唱声乐, 舞台戏剧, 摄影
体育; 正常体育, 大型活动管理, 领导统御, 运动研究
社会人文科学; 父母学, 营养学, 多样社会
现代工艺; 10, 11, 12 工艺设计, 媒体设计, 动画设计, 工厂技能,
计算机; 11, 12 计算机工程, 信息管理, 程序设计
实习学徒课程; 美容美发, 汽车修护, 园艺, 建筑, 工厂技术

加拿大高中的选课设计, 目的是让学生有多样性的选择, 让学生在各种专业上均衡发展, 学校视各各学生都是一样的, 教育学生是他们的目的, 学生的天生资质不同, 各有所长. 他们不忽略任一个学生的教育机会, 以及学习照顾自己的技能.

梁溪香榭 : 2009-09-04#749
回复: 加拿大教育情况有用素材收集帖子--梁溪香榭整理

高中毕业生该如何选择加拿大大学

加拿大大学主要分两大类型:研究型和教学型。研究型大学又分医学法学博士博士后类(简称医学博士类)和综合研究类(简称综合类)两部分。除医、法两大学科的发达程度以及博士和博士后布点的全面性外,两类大学均为研究型。研究型大学的本科教育几乎全部由硕士生导师、博士生导师承担。教学型大学又叫基础类大学,普遍采取小班授课,注重培养学生理解和应用知识的能力。不管是研究类大学还是教学类大学,其学力均得到中国教育部的认可。

  中国学生想直接进入研究型大学读本,必须具有相当的读书能力和研究能力。学生除具备优秀的高中成绩、高考成绩和语言能力成绩(指托福或雅思)等硬件条件外,必须具备很强的探索和创造能力、有良好的自学和研究习惯以及合理的学术挑战技巧等软件的条件。由于师资力量主要为研究生而设,研究型大学本科教学普遍采用大班制。

  对交流能力不强、不适应在大庭广众面前提问答疑、不习惯课堂外自学的中国学生来讲,这种教学环境根本上打乱他们在中国养成的读书和接受知识的习惯和方法,从而影响他们的学习质量,动摇他们的学习信心。研究型大学的本科大一、大二淘汰率高较高。如果说,有学生能被一所中国若干所顶尖大学录取,并不等于说他能一步到位地被加拿大研究型大学中的前十名录取,即便被录取,也不一定能在最初两年里幸存下来。其关键原因是缺乏或严重缺乏软件能力。

  换句话说,如果您学学习和研究能力足够强,那么研究类大学应该是不错的选择。因为无论是科研设备,还是科研人员,教学型大学都不能与科研型大学相抗衡。如果您对自己的学习能力不够自信,那么最好选择教学类大学。其实,排名靠前的大学并不是最适合自己的大学,研究类大学过于注重科研的教学氛围,并不一定适合中国学生的职业发展。

  在中国,入读名牌大学是广大家长和学子的梦想,然而即使进入了名牌大学又有什么用处呢?难道名牌大学的学生都是企业的高管吗?难道名牌大学的学生就一定收到世界五百强企业的青睐吗?以上种种问题值得广大家长和莘莘学子三思而后行!

梁溪香榭 : 2009-09-04#750
回复: 加拿大教育情况有用素材收集帖子--梁溪香榭整理

高中生如何入读加拿大大学?

路线1:对于高二在读或者高中毕业生来说,先读加拿大预科学院是很好的选择。这些加拿大预科学院一般都不要求提供托福成绩,大学预科(含6个学科)需要8个月-12个月时间。被加拿大大学录取不需要考大学入学考试,录取准则以大学预科最好六门科目的总平均分数为主。

路线2:优秀的高中毕业生可以直接报读加拿大大学,但必须具备以下条件:1)托福550以上;2)优秀的高中毕业生,三年平均分80%以上;3)高考成绩上国内本科线。另外,还要在大学规定的申请截止时间内递出申请材料。

路线3:学生可以选择大学转学分课程(University Transfer Program),1或2年后转换大学学分到大学完成学士学位。众所周知加拿大大学学位紧张,不管对本国学生还是国际学生都是非常困难的。因此加拿大大学会授权质量保证的学院提供本大学的学分课程,按照大学统一的课程大纲和要求授课,学生修得的学分完全被大学认可。

梁溪香榭 : 2009-09-04#751
回复: 加拿大教育情况有用素材收集帖子--梁溪香榭整理

加拿大大学怎样录取新生

1. 中国学生申请本科需要注意些什么?
大量的中国学生申请工商管理专业,这使得申请的标准变的非常高,录取起来变得非常不容易。而其他一些专业,例如经济学,商业数学等专业则并不是那么热,标准就比较低,相对好录取。选择专业,一定要以适合自己为前提,不能盲目跟风,有的学生擅长计算,偏要申请工商管理,一方面录取很难,另一方面在上课中有很多论文撰写和案例分析,最后因为写作不好而导致成绩很差,不得不再去转到其他专业里面去。这样不仅浪费了时间和金钱,也使自己的成绩很差,对申请研究生甚至找工作都带来了很多麻烦。总的来说,管理类的课程对写作和记忆的要求很高,对英文水平的要求更高,适合那些英文相对出色,写作出众的学生;而经济学科类的课程主要是处理和构建一些经济模型,更适合那些数学好,逻辑思维强的同学。加拿大乃至北美的大学的各专业录取标准不同,但是一旦进入大学,转专业是很容易的。故此我建议我们中国同学在申请的时候以自己认为最强、最擅长的专业作为申请对象,不要一味的申请录取标准高的专业。进入大学后,可以在第一年学习结束后再按照自己的兴趣并结合自己的学习实力,并通过自己对不同课程的反应来确定最适合自己的专业。

2. 申请的时间方面学生需要注意什么?
尽快申请是太重要了,特别是工商管理和工程等热门学科。每年报名的人都以万计,而学校采取的政策是材料先到先审的政策。更重要的是,这类热门专业的标准会随人数的增多有一定调整,申请时间越往后,录取位置就越少,标准就越高,奖学金的可能性就越小。而且越往后,给结果的时间就拖的越长,因为人数太多,录取委员会要不断比较才能确定录取人选。比如一个学生9月份申请,可能11月就能获得结果了,但是如果他12月申请,可能要到4月才能获得结果,时间的差异可见一斑。所以我建议大家申请不要拖,申请季节一开始就马上处理,尽早获得申请结果,安排其他事情就会更加轻松。

3. 申请过程中资料很繁杂,无从入手?
第一,材料的全面性。学生务必要按照学校的要求准备齐全每一项材料,最好再附上一个清单,这样整理和检查申请材料的工作人员能清晰快捷地将材料纪录到申请人的个人数据库中,加快申请的进程。第二,申请材料一定要用公证件或者是原件,不要交复印件。因为复印件无法确定材料的真实性,故此学校不予接受。第三,自己的申请材料一定要自己留备份。原因在于录取办公室每天都接到大量的邮包,在处理过程中难免会有遗漏的情况出现。如果发生材料遗失,录取办公室只能让学生再去补。如果学生手上没有任何备份,那么只会耽误很多时间,严重的甚至会导致错过申请截止日期。最后,也是最重要的一点,虽然有些学校可能不需要,但是学生最好要主动提交一个个人陈述以及你对将要申请专业的理解,这样在一定程度上能弥补申请者在成绩方面的不足。对于个人陈述和专业理解,学生要经过反复的推敲尽自己最大的能力将文章写好,这样录取人员看了这样的材料后,会加深对学生的印象,也会对你的能力和想法有了一定的了解,这对申请的帮助是非常积极的。

梁溪香榭 : 2009-09-04#752
回复: 加拿大教育情况有用素材收集帖子--梁溪香榭整理

4. 提交材料后,学生还需要做什么事情吗?
很多中国同学在交完材料后就认为万事大吉了,但是当接到学校通知说自己缺材料缺文件为时已晚。大学申请跟做其他事情一样,需要自己去积极的参与。北美大学基本上还是通过邮寄平信的方式跟申请的学生交流,处理的速度相对较慢,如果信件再有丢失就非常耽误学生的申请。所以学生如果有机会,应该直接去学校的录取办公室面对面地和录取人员交流,这样可以及时从数据库中发现缺乏的材料,也能得到学校快速的信息反馈,口语好的同学还能给录取人员留下很好的印象,这样对自己语言成绩的不足会有一定的帮助。即使没法去录取办公室,学生也要勤打电话,问问自己申请的状态,及时了解申请的进程。

5. 奖学金
当然,学校发放奖学金的主要对象是优秀的学生。申请人如果希望获得奖学金,必须在以前的学期和学术竞赛中取得不错的成绩,这样能够为学校学术做出贡献,学校才有可能给这些学生发放一部分入学奖学金。第二,也是大家都不很知道的,在大学中有些专业比较难读或者比较冷门,例如数学、矿业工程等,申请人数比较少,学校为了鼓励学生报读,一方面标准会松些,一方面就以发放一部分奖学金的形式来吸引学生。所以这类奖学金其实和学习水平并不特别相关,只是学校的一个优惠政策而已。我的建议是如果一个学生家境并不富裕而且成绩不够获得第一类奖学金的标准,不妨可以从读冷门专业开始,用奖学金来支持自己的学业,等到合适的时候再转到其他专业上去,这样能够省很多家庭的支出。

6. 中国学生如何能够获得一份校内工作?
需要学生具有这样几项特点,一是很强的沟通能力,需要非常专业,同时要诙谐幽默. 二是需要很好的组织能力,材料的安排处理要井井有条,和同事之间的配合也要非常默契,这样才能保证申请过程的顺利。第三是需要有很强的职业精神和道德感,绝对不能够违反工作流程。必须要英语水平达到比较高的标准,

梁溪香榭 : 2009-09-04#753
回复: 加拿大教育情况有用素材收集帖子--梁溪香榭整理

加拿大大学对A-LEVEL的录取要求

University of Toronto多伦多大学

建立于1827年,连续多年排在加拿大大学的榜首,被公认为是加拿大综合实力数一数二的优秀大学。世界排名头二十位。

大学设本科学士学位300个,硕士学位148个,博士研究生学位95个。其中以统计学,经济贸易,商科,生物,财会,计算机科学,建筑学,工程学,心理学,法学,教育学,医学等尤为杰出。

文学,商科 ,管理 ,应用科学,工程学,音乐,物理,健康学等学科

l 至少5门Olevel成绩包括4门AS成绩(平均成绩B以上)或3门Alevel成绩(不包括General Paper)

l 学校更看重Alevel成绩,但是有些学科也接受优秀的Olevel和AS成绩。申请应用科学和工程学必须提供化学,数学和物理Alevel 成绩且均应为A,不接受AS成绩

l 音乐学要两门Alevel成绩

l GCSE/IGCSE/GCE O Level English, English Language, 或English as a Second Language 至少为B

l 学生有至少5门A/AS成绩,包括三门Alevel成绩,申请文学,科学和商业课程可获得最高30免修学分


Alevel成绩中必须包括以下传统科目中的两门 、

会计,英语,数学,生物,地理,哲学,化学,地质,物理,政治,宗教,古典文学,计算机,历史,统计,经济,语言

各学科要求

商科,计算机,物理,数学
A/AS数学,GCSE/O数学

生命科学
A/AS数学,GCSE/O数学;推荐AS 生物 化学

或 A/AS(推荐),GCSE/O 数学,生物,化学,物理

工程学
Alevel数学 化学 物理 均A

体育教育,健康学
A/AS 生物,数学,GCSE/O数学,推荐 化学 物理

商科,计算机科学,数学和统计
Alevel 数学

化学和物理科学
A/AS数学,A/AS 或GCSE/O 化学 或 物理

心理学
A/AS,GCSE/O数学 生物 化学


语言成绩:最低要求托福PBT600分、写作5.0 ;IBT100分、写作不低于22分;CBT250分,写作5.0; 或雅思6.5分单项不低于6.0


申请截止日

文理学院 ,商科/管理

St. George Campus
March 15
University of Toronto Mississauga
Theatre and Drama - April 1
Art & Art History - April 30
Commerce and Management - April 30
All other programs - June 12
University of Toronto Scarborough
Biological Sciences, Computer Science, Co-op Computer Science, Humanities, Neuroscience, Psychology, Sciences (including Chemistry, Biochemistry, Environmental Science, Mathematics, Physics and Statistics), Social Science - July 20

Professional Faculties

Applied Science and Engineering
April 1

Medical Radiation Sciences
March 6

Music
February 1

Nursing
February 15

Physical Education and Health/Kinesiology
April 1

梁溪香榭 : 2009-09-04#754
回复: 加拿大教育情况有用素材收集帖子--梁溪香榭整理

加拿大大学对A-LEVEL的录取要求

McGill University麦吉尔大学

是一所实力雄厚的私立大学。此校约180年历史,设有农业、艺术教育、工程、管理、音乐及科学七个院系。以文科和理科专业著名,并为学生提供有竞争能力的电子工程和计算机工程专业,大学不断发展微电子和软件工程专业。


申请截止日

申请
January 15

提交材料


奖学金申请
January 19

其他申请
March 15



商科 May 1
提交申请材料 June 1

建筑学 March 15

社会工作 April 15

音乐 January 15

宗教学 May 1

牙医,法律,医学 不同


入学要求
一般来讲需要本国大学入取通知书。高中阶段各学科成绩(包括不及格和重复学科)均被考虑,尤其是与申请专业相关的学科成绩.

Alevel/As基本要求:三门Alevel成绩 BBC,或两门Alevel成绩两门AS成绩 BBCC或BCBB。但是,许多有竞争力的学科需要更高的成绩。根据A/AS成绩最高可给予30免修学分。

各学科要求

农业,环境科学,饮食学,人类营养学
A-Level 数学以及生物,化学,物理其中一门
A-Level 没有科学成绩,则GCSE level 中需有科学成绩
建筑
A Level 数学,物理,化学其中一门
GCSE level 需有数学,物理,化学三门成绩
文学
无具体要求
文理学科
A Level 数学以及生物,化学,物理中至少一门
GCSE level 需有生物,数学,物理,化学四门成绩
教育
无特殊要求
工程学
A Level 数学以及化学,物理中至少一门
GCSE level 需有数学,物理,化学三门成绩
生物不作为申请必须学科
人体运动学
A Level 数学以及生物,化学,物理中至少一门
GCSE level 需有数学,物理,化学三门成绩
管理
A Level 数学须为B以上
音乐
无特殊要求
护理
A Level 数学以及生物,化学,物理中至少一门
A-Level 没有科学成绩,则GCSE level 中需有科学成绩
物理疗法
A Level 数学,生物,化学,物理BBCC(无顺序)以上
至少已在大学实验室学习一学期有机化学
宗教学
无特殊要求,根据成绩单录取
科学
A Level 数学以及生物,化学,物理中至少一门
A-Level 没有科学成绩,则GCSE level 中需有科学成绩
社会工作
至少一年大学学习

申请材料
如果已完成ALevel, AS, OLevel, GCSE学习, 需由学校提供certificate和/或 Candidate Statement of Results.

如果已参加或将要参加A Level, AS, 或 CAPE考试, 需指明考试委员会名称,学科编号和课程。 (如 Edexcel GCE Advanced Level Mathematics 9371 with modules CM1, 2, 3, 4, M1, 2).

其他文件:高中毕业证


语言成绩

雅思6.5以上或托福

本科学科 (CBT) (PBT) (iBT) *

教育,管理 250 600 100

其他 233 577 90 每门 21分以上

音乐 213 550 79-80


其他申请材料

高中成绩单:需直接从学校或考试委员会寄出,附英语翻译

如果是Alevel学生,成绩需直接由考试委员会寄出。

推荐信:只做参考,录取以考试成绩为主

以下专业需要推荐信:
建筑
教育
音乐
物理治疗
宗教
社会工作
申请建筑学需提交10份高质量原创作品

申请英语教学需参加额外的语言测试

梁溪香榭 : 2009-09-04#755
回复: 加拿大教育情况有用素材收集帖子--梁溪香榭整理

加拿大大学对A-LEVEL的录取要求

Queen’s University皇后大学

在众多加拿大本地人的眼中,皇后大学其实是加拿大最适合本科生就读的大学。虽然在整体排行上,皇后大学一直还无法超越多伦多大学,但如果以大一新生的平均入学分数来说的话,皇后大学的平均录取分数,一直高居全国第一位,是全加拿大最难进的大学。皇后大学在学术上一直保持着很高的水准, 与多伦多大学(University of Toronto)和麦吉尔大学(McGill University)一起并称加拿大的长青藤盟校。

皇后大学规模不大,拥有加拿大一流的商学院和医学院,理工科中的工程物理专业为加拿大第一,该专业在北美也仅次于Princeton和Cornell而排名第三。人文艺术、教育学、法律等都是非常有名。


入学成绩:至少2门Alevel 或4门AS成绩(不包括General Paper,以及5门Olevel成绩(至少为C)。学生应完成7门功课,其中至少2门Alevel(不包括General paper).一般来讲Alevel成绩不低于B,Olevel成绩不低于C。2门AS成绩相当于1门Alevel成绩.

文理学院

文科,文科(师范)
Olevel英语

理科,理科(师范),X射线技术
Alevel数学以及生物,化学,物理其中一门;Olevel英语以及生物,化学,物理其中两门

计算机,计算机(师范)
Alevel数学;Olevel或Alevel英语

艺术
Oleval英语

运动机能学和健康研究
Alevel生物,Olevel英语

人体运动学
Alevel生物以及数学,化学,物理其中一门;Olevel英语

商学院

商业
Alevel数学A,Olevel英语B

应用科学学院

工程学
Alevel数学以及化学,物理其中一门;Olevel英语(至少为B),化学,物理三门

护理学院

护理
Alevel生物,化学;Olevel英语(至少为B),生物,化学,物理四门


申请截止日期

主要的入学奖学金截止 Dec.1

网上申请截止 Mid Oct-February 15
提交PSE截止(必须提交,作为申请资料) March 1
提交所有证明文件 Apr.30

王晶 : 2009-09-04#756
回复: 加拿大教育情况有用素材收集帖子--梁溪香榭整理

谢谢。

pwjnd : 2009-09-04#757
回复: 加拿大教育情况有用素材收集帖子--梁溪香榭整理

溪溪:行动好迅速,太太感谢你了.

梁溪香榭 : 2009-09-04#758
回复: 加拿大教育情况有用素材收集帖子--梁溪香榭整理

谢谢。

溪溪:行动好迅速,太太感谢你了.

咱们都是同路人,一起努力! :wdb23::wdb23::wdb9::wdb9:

梁溪香榭 : 2009-09-04#759
回复: 加拿大教育情况有用素材收集帖子--梁溪香榭整理

进来学习一下,新内容。:wdb17:

儿子现在上高中?

梁溪香榭 : 2009-09-04#760
回复: 加拿大教育情况有用素材收集帖子--梁溪香榭整理

安省大学申请
http://www.ouac.on.ca/105/index.html

currently reside in Canada (Canadian citizens, permanent residents or those currently studying in Canada on a study permit or other visa) OR
are a Canadian citizen living elsewhere (not in Canada) AND
are not currently attending an Ontario secondary school in a day program

附件


梁溪香榭 : 2009-09-04#761
回复: 加拿大教育情况有用素材收集帖子--梁溪香榭整理

多伦多大学录取条件:(摘自多伦多大学官方网站)

1. 接收中国高中教育,
高中毕业证+中国高考成绩+语言成绩
China (PRC) Senior High School Graduation Examination + Chinese National University Entrance Examinations

2. 接受英式高中教育 A-LEVEL + 语言成绩
British-Patterned Education: Admission requirements for Arts, Science, Commerce/Management, Applied Science and Engineering, Music, Physical Education and Health/Kinesiology.

At least five different Ordinary level subjects, AND
Four different Advanced Subsidiary subjects OR three different Advanced level subjects (excluding General Paper)
Prerequisites should be presented at Advanced level, but excellent Ordinary Level and Advanced Subsidiary level results may be accepted for some programs. Applicants to the Faculty of Applied Science and Engineering MUST present Chemistry, Mathematics and Physics at Advanced level.
Possession of the minimum requirements does not guarantee admission.

NOTES: Students may be considered in the year in which they are writing their Advanced examinations.

At least two traditional academic subjects from the list below should be presented at Advanced level.

Accounting English Mathematics
Biology Geography Philosophy
Chemistry Geology Physics
Classics Government and Politics Religious Studies
Computer Science History Statistics
Economics Languages

语言成绩要求:
雅思: The minimum requirement is an overall band of 6.5, with no band below 6.0.
托福: Internet-based Test:

Minimum Requirement
total score of 100 + 22 on Writing section
Discretionary Range
total score 89-99 +19 - 21 on Writing section

Paper-based Test:

Minimum Requirement
total score of 600 + 5.0 on TWE
Discretionary Range
total score 573-597 + 4.5 on TWE

Those who present the Paper-based TOEFL must ensure that the Test of Written English (TWE) is also available on their test date. If you score in the discretionary range indicated above, and are otherwise well qualified for admission, we will automatically consider other academic evidence of English proficiency (for example, results in English courses) and we will advise you whether our English facility requirements have been satisfied. It is not necessary to request this special consideration. Letters of reference will NOT be used for this purpose. The TOEFL Institution Code for undergraduate studies at the University of Toronto is 0982-00. You must indicate this code correctly on the TOEFL forms, or we will not receive your results. We receive your results electronically from TOEFL. We do not accept paper results.

梁溪香榭 : 2009-09-05#762
回复: 加拿大教育情况有用素材收集帖子--梁溪香榭整理

西安大略大学录取条件:(摘自西安大略大学官方网站)

1. 接收中国高中教育,
高中毕业证+中国高考成绩+语言成绩
China (PRC) Senior High School Graduation Examination + Chinese National University Entrance Examinations

2. A-LEVEL
Applicants must offer passes in five General Certificate of Education (GCE)/General Certificate of Secondary Education (GCSE or IGCSE), of which two must be Advanced or Principal level, or in four General Certificate of Education (GCE)/General Certificate of Secondary Education (GCSE or IGCSE), of which three must be at the Advanced or Principal level.

2. 语言:
a) TOEFL Internet based (iBT), Paper-based (PBT) and the TWE
The Test of Spoken English is also required for students applying to a Nursing program at Western.The Test of English as a Foreign Language (TOEFL) and the Test of Written English (TWE) is offered by the Educational Testing Service (ETS) on several dates at a number of locations throughout the world.
The minimum score required on the TOEFL is 580 on the paperbased with a 5 on the TWE, and 88 on the internet-based tests with a score of 22 in speaking,
reading and writing and 20 in listening.

b) IELTS International
The International English Language Testing Service
(IELTS) is jointly managed by the University of Cambridge ESOL Examinations, British Council, and IDP Education Australia.

A minimum overall band score of 7.0 is required with no part less than 6.5.

cindyycm : 2009-09-05#763
回复: 加拿大教育情况有用素材收集帖子--梁溪香榭整理

学习

梁溪香榭 : 2009-09-05#764
回复: 加拿大教育情况有用素材收集帖子--梁溪香榭整理

麦克马斯特大学录取条件:(摘自麦克马斯特大学官方网站)

1. 接收中国高中教育,
高中毕业证+中国高考成绩+语言成绩
China (PRC) Senior High School Graduation Examination + Chinese National University Entrance Examinations

2. A-LEVEL
Students presenting qualifications based on the British system of education are required to present passes in at least five different subjects including two at Advanced Level. (Two Advanced Supplementary Level examinations are considered to be equivalent to one Advanced Level for purposes of admission.) Subjects listed as prerequisites for admission should be presented at Advanced Subsidiary or Advanced Level. For example, students should present AS or A Level Mathematics for programs requiring Calculus. To be considered on the basis of predicted grades, students must have achieved at least “C” in each of five subjects. Some programs will require higher grades. Applicants to the Faculty of Science and the Faculty of Engineering should present grades of “A” or "B" in order to be competitive for admission. In addition, Mathematics and one of Physics or Chemistry must be presented at the Advanced Level. Transfer Credit may be granted for Advanced Level qualifications. No credit is given for Advanced Subsidiary exams. Possession of the minimum grades does not guarantee admission.

3. 语言
Each student granted admission to McMaster must be proficient in the use of the English language. Students will be expected to speak and write clearly and correctly in English.

Applicants whose first language is not English must satisfy one of the following requirements for admission:

Attended a Secondary School (High School) or Post-Secondary College in full-time academic studies (excluding ESL courses), in an English speaking country, for at least 3 years immediately prior to your anticipated start date at McMaster, or

Attended an accredited English medium Secondary School (High School) or Post-Secondary College (excluding ESL courses) in full-time academic studies, for at least 3 years immediately prior to your anticipated start date at McMaster, or
Attended an accredited English medium University in full-time academic studies for at least 1 year immediately prior to your anticipated start date at McMaster, or

Resided in an English speaking country for at least 4 years immediately prior to your anticipated start date at McMaster, or
Achieved an acceptable score on an English Language Proficiency Test as outlined on the chart below. Official test results must be submitted directly from the testing centre. Photocopies of test results will not be considered.

TOEFL: IBT: 86 overall with a minimum score of 20 on each of the four components (Reading, Writing, Speaking, Listening)
PBT: 580 overall,
Valid for 2 years
IELTS: 6.5 overall,
Valid for 2 years

梁溪香榭 : 2009-09-05#765
回复: 加拿大教育情况有用素材收集帖子--梁溪香榭整理

皇后大学录取条件:(摘自皇后大学官方网站)

1. 接收中国高中教育,
高中毕业证+中国高考成绩80%+语言成绩
Upper Middle School Graduation Certificate. Students currently in secondary school will be required to submit the Senior Middle School Graduation Examination results. For students who have graduated from secondary school, a Chinese National University Entrance Examination is required. Grades require for admission will vary but, based on the Chinese grading scale, the minimum average needed to fall within the competitive range is approximately 80%.

2. A-LEVEL
Candidates who complete seven subjects with at least two at the Advanced Level or equivalent (excluding the General Paper) will be eligible for admission consideration. Competitive grades at the Advanced Level must be "B" or higher. Grades at the Ordinary level must be C or higher. Two Advanced Subsidiary (A/S) levels are considered equivalent to one Advanced Level subject, however, prerequisite courses must be at the Advanced Level.

梁溪香榭 : 2009-09-05#766
回复: 加拿大教育情况有用素材收集帖子--梁溪香榭整理

UBC录取条件:(摘自UBC官方网站)

1. 中国高中教育:
Upper Middle School Graduation Certificate
Certificates must be official. Photocopies are acceptable if certified by school principal, head, or counsellor. Notarized copies are not acceptable.
Grades required for admission will vary by program, but based on the China grading scale, the minimum average needed to fall within the competitive range is approximately 80%.

2. A-LEVEL
As an applicant, you must achieve standing in at least five subject areas, including English, with at least three subjects at the A Level (or six at the advanced subsidiary level). A subject may not be counted at both the A Level and the GCSE level (O Level).
Admission average is calculated on at least two GCSEs (O Levels) and three GCE A Levels (or six Advanced Subsidiary levels)

3. 语言
Complete three or more consecutive years of full-time education in English within Canada immediately prior to attending UBC.
Complete four or more consecutive years of full-time education in English in a country other than Canada where English is the principal language. These four years must be immediately prior to attending UBC.
Achieve a grade of 70% or better on the provincial examination portion of BC English 12 or English Literature 12 or the equivalent.
Achieve a final grade of 4 or better on Advanced Placement (AP) English Language & Composition or AP Literature & Composition; or achieve a final grade of 5 or better on International Baccalaureate English A1 or A2 (higher-level or standard-level).
Achieve the competence standard indicated by one of the tests of English language proficiency that evaluates skills in listening, reading, speaking, and writing.
Successfully complete six credits of post-secondary first-year English studies that are eligible for transfer credit to UBC. To be eligible, the course must be taken at a recognized university in an English-speaking country. A transcript showing completion of these courses is required by June 30. For students at North American post-secondary institutions, this usually requires you to complete the courses by the end of the January to April term.
Graduate from a recognized degree program at an accredited university at which English is the primary language of instruction in a country where English is the principal language.
Attend four or more years at an eligible international secondary school that uses English as the language of instruction but operates in a country where the primary language is not English. These four years must be immediately prior to attending UBC. If you wish to meet the English Language Admission Standard using this option, please submit a transcript as soon as possible so that the Admissions office can confirm that the requirement has been met.
If you are fully proficient in English but do not meet any of the eight options listed above, you may request a waiver of the English language requirement.

IELTS: 6.5 with no part less than 6.0
TOEFL:
Internet-based test: Overall score: 90; Reading: 22; Listening: 22; Writing: 21; Speaking: 21
Paper-based test with the TWE (Test of Written English): 55 each for Reading, Listening, and Writing (4.0 on TWE)

狗狗凡 : 2009-09-05#767
回复: 加拿大教育情况有用素材收集帖子--梁溪香榭整理

学习

ever : 2009-09-05#768
回复: 加拿大教育情况有用素材收集帖子--梁溪香榭整理

安省大学申请
http://www.ouac.on.ca/105/index.html

currently reside in Canada (Canadian citizens, permanent residents or those currently studying in Canada on a study permit or other visa) OR
are a Canadian citizen living elsewhere (not in Canada) AND
are not currently attending an Ontario secondary school in a day program

多谢梁溪这么好的帖子。是不是说申请安省的大学都要通过这个OUAC而不是到各个大学去申请?新移民的孩子算是are a Canadian citizen living elsewhere (not in Canada)? 谢谢!

梁溪香榭 : 2009-09-05#769
回复: 加拿大教育情况有用素材收集帖子--梁溪香榭整理

多谢梁溪这么好的帖子。是不是说申请安省的大学都要通过这个OUAC而不是到各个大学去申请?新移民的孩子算是are a Canadian citizen living elsewhere (not in Canada)? 谢谢!

如果已经拿到枫叶卡而不是在加拿大高中毕业的,应该算是are a Canadian citizen living elsewhere (not in Canada)。
我看到各个大学都有申请的链接,不一定要到OUAC吧。

ever : 2009-09-05#770
回复: 加拿大教育情况有用素材收集帖子--梁溪香榭整理

谢谢梁溪的热心解答!

王晶 : 2009-09-06#771
回复: 加拿大教育情况有用素材收集帖子--梁溪香榭整理

非常有帮助。
除了英语还是英语,中国孩子就是英语考不好,其他科目考得比较出色。

sunnytan : 2009-09-06#772
回复: 加拿大教育情况有用素材收集帖子--梁溪香榭整理

新内容!非常有用!:wdb17::wdb17: 辛苦梁溪了,可以称得上专业讲座!!

梁溪香榭 : 2009-09-06#773
回复: 加拿大教育情况有用素材收集帖子--梁溪香榭整理

新内容!非常有用!:wdb17::wdb17: 辛苦梁溪了,可以称得上专业讲座!!

祝旅途顺利愉快
加拿大新生活平安喜乐
God Bless You!

梁溪香榭 : 2009-09-06#774
回复: 加拿大教育情况有用素材收集帖子--梁溪香榭整理

以上收录的各大学录取标准只是一个最基本的杠杠。
各个专业各有小杠杠,选课要求也不同。

Luck Li : 2009-09-07#775
回复: 加拿大教育情况有用素材收集帖子--梁溪香榭整理

儿子现在上高中?

已经过了十八岁生日,昨天大学第一学期开学了。急人呀,过去加语言不过关,还要先读高中呢。:wdb7::wdb7:

梁溪香榭 : 2009-09-07#776
回复: 加拿大教育情况有用素材收集帖子--梁溪香榭整理

已经过了十八岁生日,昨天大学第一学期开学了。急人呀,过去加语言不过关,还要先读高中呢。:wdb7::wdb7:

嗯。那的确是很急人了。:wdb23:

王晶 : 2009-09-08#777
回复: 加拿大教育情况有用素材收集帖子--梁溪香榭整理

请问到加拿大读书应该考托福还是雅思?

梁溪香榭 : 2009-09-08#778
回复: 加拿大教育情况有用素材收集帖子--梁溪香榭整理

请问到加拿大读书应该考托福还是雅思?

看那些大学网站,应该是都可以呀。
通常来说,北美的就托福,英联邦的就雅思。
加拿大即是北美也是英联邦。 网上似乎推荐托福的多一些。

很多人说雅思更容易得高分,
因为IELTS技术含量高一些 也就是说,你可以使用一些应试技巧 :wdb23:

没考过,不知道。

梁溪香榭 : 2009-09-08#779
回复: 加拿大教育情况有用素材收集帖子--梁溪香榭整理

ZT 去美国和加拿大留学到底考托福还是考雅思好?

托福和雅思一直是申请出国留学的两个主力考试。而对于美国和加拿大留学来说,托福考试也是大多数人所必经的考试。但是最近几年托福考试非常不好报名,而雅思考试报名灵活,加上雅思考试的宣传攻势一浪高过一浪,使很多学生自觉或者被迫的从考托福转到了考雅思,并以雅思成绩来申请学校。在这样的一个背景下,我经常会被问及,到底申请北美的学校考托福还是考雅思,哪个更好点?

从考试本质来说,其实两个考试都是同等效力的,只要达到学校所规定的分数,考哪个考试都无所谓。但是,从事实来看,美国接受托福成绩的学校的数量还是远远多于接受雅思的学校,从这个角度来看,考托福还是要合算的多。任何考生都不希望考了一个考试只有一些学校接受,而另外一些自己心仪的学校却不接受,这必经不是一个好事。很多学生在考了雅思之后就有些后悔,因为不少学校没法申报,这虽然有了一个成绩,但是并不能发挥最大的效力。故此,雅思对于申请北美学校来说,并不是一个最好的选择。 

从分数的角度来说,托福是120分制,学生的分数变化很多,可以是90,也可以是85,还可以是79,这些分数在录取委员会来看是有一定灵活性的,例如一个考了96的学生,在托福要求线为100的学校录取过程中,可能就被视为托福成绩通过了,因为差了很少几分,录取人员基本上就可以算申请人英语达标了。而雅思的9分制度,总分是按照0..5来进位,使分数在录取人员眼中的灵活性差了许多,故此一个考了6.5的申请人,绝难在雅思规定分数为7分的学校那边被视为语言通过。故此,雅思的分数设计还是比较严格的,0.5 分的进制我个人认为不利于语言介于学校要求边缘的学生申请。从这个方面来看,我还是鼓励希望进入北美学校的学生考托福的。

从我个人经验来看,北美学校对托福的认同程度还是非常高的,这也表现在他们对托福和雅思分数的规定上面,这尤以加拿大的学校为甚。例如约克大学(York University), 雅思要求是7分,而托福要求是88分以上,从我们语言工作者的角度来看,雅思7分,就相当于托福考试100分了,这种明显的不同考试分数不平等体现了学校对两类考试的对待程度。在加拿大,如果读者有机会去翻翻大学的雅思要求,发现如果雅思没有6.5分,实际上是没有大学本科可以录取的。而托福80分以上,就有学校可以接受了。这之间的差异实在是太大了。从学生的实际水平来看,到到托福80分是相对比较容易的,而雅思考到6.5确实是不太简单的。雅思看上去容易,但是实际上拿高分相当困难。那些希望去加拿大留学的学生,在考托福还是考雅思的选择上可以根据我文章中所说的情况斟酌一下。

从雅思和托福的题型内容来看,我更偏向于托福。托福的词汇量要比雅思大的多,难度也大。从一方面来说,这些词汇的记忆对于学生提高英文水平是比较有帮助的。从另外一个方面,如果两个考试被认知的程度一致,那么难度理论上是应当平等的。在阅读部分,托福的词汇量大,那么理解要求就会稍微低些。而雅思的词汇量不大,那么理解难度就要高了。故此,我如果是学生,宁可选择死记硬背的单词,也不喜欢理解绕来绕去。在这点上,托福的阅读容易得高分。另外写作这块托福也比雅思简单些,文章毕竟不需要特别多得构思。而雅思的文章思路过于重要,对于仅有高中水平的学生来说,很难写好。当然,雅思的口语比托福的口语稍微容易一些。不过从整体来看,雅思因为词汇量比较低,特别强调灵活应用,那么是适合专门学英语的这类人的。而托福强调识记,对于水平偏低的人来说,相对提高容易些。 

雅思的不方便性还体现在邮寄成绩方面。现在托福的邮寄成绩完全都可以通过网上进行了,费用为136人民币,邮寄的方式为ETS电子寄送,和学校的数据库连接,可以说是比原来的笔考邮寄成绩提高了一大步,这基本上确保了学生的TOEFL成绩不会丢失。而雅思的邮寄成绩还相当原始。60块人民币一个平邮,拿到了学校就得至少一个月了。而如果要快递的话,一份EMS又是一百出头,整个下来要比ETS邮寄托福成绩贵。这还不说,因为是邮寄,所以在学校那边处理的时候会经常出现忘记纪录的情况,也就是成绩到了学校,学校也没把成绩放在学生的档案里面。特别是有些学生在考完雅思后直接就把成绩邮寄斤学校了,而申请可能几个月之后才进行,这种情况,出现雅思成绩到学校,但是学校不反映的情况是大量的,因为学校接到成绩没有档案也就把成绩放在一边了,所以斤学生添了很多麻烦,还得花双份的钱,非常讨厌。可以说,雅思这个邮寄成绩不改革,斤学生后续申请带来的麻烦学生会不断抱怨的。我在这个方面,双手倾向于托福,不喜欢雅思。

考托福还是考雅思之争,我本身也无法说出一个答案,对我来说,托福考个115,雅思考个8分都是很轻松的事情,所以影响不大。但是对于广大学生来说,关心的是如何迅速通过考试,能以自己的水平在某个考试发挥最高水平,那么我个人觉得,托福会更好些,虽然表面看上去难,但是学习进去之后上手相对还是容易的。而且一系列的后续服务和学校认知程度都会好些,是一个比较好的选择。当然,我也完全没有抹杀雅思的意思,那些擅长英语灵活应用的学生,也完全可以省却背单词的苦恼,在雅思考试发挥一个高水平,这也是一个很不错的考试策略。 

梁溪香榭 : 2009-09-08#780
回复: 加拿大教育情况有用素材收集帖子--梁溪香榭整理

ZT 不考托福雅思也能去加拿大留学

对于无托福无雅思或者低托福低雅思的学生,有以下三种方案可以入读加国名校。

  第一,大学方案:对于成绩优秀的高中毕业生和大学在读生如无合格的托福雅思成绩,可以通过申请“语言+专业”双录取直接入读部分加拿大名校。

  推荐学校有西安大略大学、滑铁卢大学、西蒙菲沙大学,约克大学,维多利亚大学,阿尔伯塔大学、萨省大学、圭尔夫大学、纽芬兰纪念大学、康卡迪亚大学、曼尼托巴大学、温莎大学等。这些学校审核确定学生申请条件后,会给学生同时发出两份录取通知:一份是该校或者该校指定的语言中心录取通知书,另一份是该校某专业录取通知书。学生入学之后,先进入语言中心学习英语,一旦通过了该语言中心的本身的测试,可以直接进入大学学习专业课,而无需考托福或雅思。

  专家提醒学生,目前在留学市场上,部分小公司号称可以帮助学生申请加拿大名校,但实际上它们拿到的只有一个语言学校或私立预科学校的录取信,而学生仅凭这样的录取信,不仅将来入读大学无法保证,签证通过的几率也不高。

  第二,学院方案:对于成绩一般的高中、中专、职高及大专生,可以考虑先申请加拿大公立学院。由于名校直接申请要求高,而且大班授课,淘汰率高,而学院课程设置灵活实用,小班授课,并与众多名校签订了学分互转协议,学生在完成至少一年学习后,如果成绩达到大学的相关要求,则可以申请进入大学相关专业继续就读,毕业后可获得该大学学士学位,推荐学校:兰加拉学院、百年理工学院、亚岗坤学院、汉堡学院、BCIT理工学院、奥提根学院、北阿尔伯塔理工学院、范沙学院等。

  第三,预科方案:对于高二、高三在读生、高中应届毕业生,如希望入读加拿大最优秀的大学但不想参加高考或高考成绩不理想,可以考虑申请到加拿大读中学或者预科。因为加拿大部分名校除了很高的英语成绩外,还要求学生提供优秀的高考成绩,而高中生课业繁重,没有充足时间和充分把握同时拿到英语及高考的高分。

  因此到加拿大完成高中并获得加拿大高中毕业文凭后再申请大学本科不失为一个很好的方法,这样不仅无需提供中国的高考成绩,大学的英语入学要求也会相应降低。推荐学校:加拿大公立中学,西蒙菲莎大学预科、哥伦比亚国际学院、哥伦比亚学院、博域学院、宝迪学院等。

  由于加拿大没有高考,学生凭借高中毕业成绩申请大学,而加拿大高中毕业成绩评估至少有70%会参考平时的作业、测验甚至出勤率,而非一次考试定乾坤,因此只要是认真努力学习的学生,有很多都获得了高分,进入了多伦多、皇后、滑铁卢、麦克马斯特、英属哥伦比亚等世界一流大学就读。

八桂桂花 : 2009-09-08#781
回复: 加拿大教育情况有用素材收集帖子--梁溪香榭整理

谢谢梁溪!辛苦了:wdb10::wdb19:!祝面试顺利!:wdb9:

梁溪香榭 : 2009-09-09#782
回复: 加拿大教育情况有用素材收集帖子--梁溪香榭整理

谢谢梁溪!辛苦了:wdb10::wdb19:!祝面试顺利!:wdb9:

谢谢! :wdb19::wdb6:

王晶 : 2009-09-09#783
回复: 加拿大教育情况有用素材收集帖子--梁溪香榭整理

ZT 去美国和加拿大留学到底考托福还是考雅思好?

托福和雅思一直是申请出国留学的两个主力考试。而对于美国和加拿大留学来说,托福考试也是大多数人所必经的考试。但是最近几年托福考试非常不好报名,而雅思考试报名灵活,加上雅思考试的宣传攻势一浪高过一浪,使很多学生自觉或者被迫的从考托福转到了考雅思,并以雅思成绩来申请学校。在这样的一个背景下,我经常会被问及,到底申请北美的学校考托福还是考雅思,哪个更好点?

从考试本质来说,其实两个考试都是同等效力的,只要达到学校所规定的分数,考哪个考试都无所谓。但是,从事实来看,美国接受托福成绩的学校的数量还是远远多于接受雅思的学校,从这个角度来看,考托福还是要合算的多。任何考生都不希望考了一个考试只有一些学校接受,而另外一些自己心仪的学校却不接受,这必经不是一个好事。很多学生在考了雅思之后就有些后悔,因为不少学校没法申报,这虽然有了一个成绩,但是并不能发挥最大的效力。故此,雅思对于申请北美学校来说,并不是一个最好的选择。 

从分数的角度来说,托福是120分制,学生的分数变化很多,可以是90,也可以是85,还可以是79,这些分数在录取委员会来看是有一定灵活性的,例如一个考了96的学生,在托福要求线为100的学校录取过程中,可能就被视为托福成绩通过了,因为差了很少几分,录取人员基本上就可以算申请人英语达标了。而雅思的9分制度,总分是按照0..5来进位,使分数在录取人员眼中的灵活性差了许多,故此一个考了6.5的申请人,绝难在雅思规定分数为7分的学校那边被视为语言通过。故此,雅思的分数设计还是比较严格的,0.5 分的进制我个人认为不利于语言介于学校要求边缘的学生申请。从这个方面来看,我还是鼓励希望进入北美学校的学生考托福的。

从我个人经验来看,北美学校对托福的认同程度还是非常高的,这也表现在他们对托福和雅思分数的规定上面,这尤以加拿大的学校为甚。例如约克大学(York University), 雅思要求是7分,而托福要求是88分以上,从我们语言工作者的角度来看,雅思7分,就相当于托福考试100分了,这种明显的不同考试分数不平等体现了学校对两类考试的对待程度。在加拿大,如果读者有机会去翻翻大学的雅思要求,发现如果雅思没有6.5分,实际上是没有大学本科可以录取的。而托福80分以上,就有学校可以接受了。这之间的差异实在是太大了。从学生的实际水平来看,到到托福80分是相对比较容易的,而雅思考到6.5确实是不太简单的。雅思看上去容易,但是实际上拿高分相当困难。那些希望去加拿大留学的学生,在考托福还是考雅思的选择上可以根据我文章中所说的情况斟酌一下。

从雅思和托福的题型内容来看,我更偏向于托福。托福的词汇量要比雅思大的多,难度也大。从一方面来说,这些词汇的记忆对于学生提高英文水平是比较有帮助的。从另外一个方面,如果两个考试被认知的程度一致,那么难度理论上是应当平等的。在阅读部分,托福的词汇量大,那么理解要求就会稍微低些。而雅思的词汇量不大,那么理解难度就要高了。故此,我如果是学生,宁可选择死记硬背的单词,也不喜欢理解绕来绕去。在这点上,托福的阅读容易得高分。另外写作这块托福也比雅思简单些,文章毕竟不需要特别多得构思。而雅思的文章思路过于重要,对于仅有高中水平的学生来说,很难写好。当然,雅思的口语比托福的口语稍微容易一些。不过从整体来看,雅思因为词汇量比较低,特别强调灵活应用,那么是适合专门学英语的这类人的。而托福强调识记,对于水平偏低的人来说,相对提高容易些。 

雅思的不方便性还体现在邮寄成绩方面。现在托福的邮寄成绩完全都可以通过网上进行了,费用为136人民币,邮寄的方式为ETS电子寄送,和学校的数据库连接,可以说是比原来的笔考邮寄成绩提高了一大步,这基本上确保了学生的TOEFL成绩不会丢失。而雅思的邮寄成绩还相当原始。60块人民币一个平邮,拿到了学校就得至少一个月了。而如果要快递的话,一份EMS又是一百出头,整个下来要比ETS邮寄托福成绩贵。这还不说,因为是邮寄,所以在学校那边处理的时候会经常出现忘记纪录的情况,也就是成绩到了学校,学校也没把成绩放在学生的档案里面。特别是有些学生在考完雅思后直接就把成绩邮寄斤学校了,而申请可能几个月之后才进行,这种情况,出现雅思成绩到学校,但是学校不反映的情况是大量的,因为学校接到成绩没有档案也就把成绩放在一边了,所以斤学生添了很多麻烦,还得花双份的钱,非常讨厌。可以说,雅思这个邮寄成绩不改革,斤学生后续申请带来的麻烦学生会不断抱怨的。我在这个方面,双手倾向于托福,不喜欢雅思。

考托福还是考雅思之争,我本身也无法说出一个答案,对我来说,托福考个115,雅思考个8分都是很轻松的事情,所以影响不大。但是对于广大学生来说,关心的是如何迅速通过考试,能以自己的水平在某个考试发挥最高水平,那么我个人觉得,托福会更好些,虽然表面看上去难,但是学习进去之后上手相对还是容易的。而且一系列的后续服务和学校认知程度都会好些,是一个比较好的选择。当然,我也完全没有抹杀雅思的意思,那些擅长英语灵活应用的学生,也完全可以省却背单词的苦恼,在雅思考试发挥一个高水平,这也是一个很不错的考试策略。 
非常有用。:wdb19::wdb17::wdb6::wdb9::wdb11:
我孩子问了从加拿大回来的老师,他认为都可以,但是他认为还是考托福好。

Luck Li : 2009-09-09#784
回复: 加拿大教育情况有用素材收集帖子--梁溪香榭整理

非常有用。:wdb19::wdb17::wdb6::wdb9::wdb11:
我孩子问了从加拿大回来的老师,他认为都可以,但是他认为还是考托福好。

我也这么认为。:wdb10:

余则成 Liftmaster : 2009-09-10#785
回复: 子女教育信息素材收集帖

嘎嘎

这个帖子也算历经了劫难了,丢了又重新找回来了。

主要整理的是关于教育的一些素材

从147楼开始,主要就是梁溪香榭搜集的一些的,推荐推荐!

推荐本书--从普通女孩到银行家,http://forum.iask.ca/showthread.php?t=233637

目录

tony 3213, PEI地区教育情况 Page 1, 5楼
tiantianxingchen 高中课程简介 Page 1, 6-9楼
守法公民 初三学生多伦多求学记 Page1, 18-22楼
守法公民 安省新移民高中指南 Page 2, 24-30楼
守法公民 安省新移民小学指南 Page 2, 34-36楼
守法公民 高中家长如是说 Page 3, 44-49楼
雨中冷百合 大多伦多地区高中排名 Page 3, 57楼, 59楼
守法公民 温哥华十佳公立中学 Page 4, 65楼
一缕阳光 菲沙关于BC省中学排名及相关简介 Page 4, 69楼
守法公民 部分大学对是否需要雅思或托福成绩的年限规定 Page 6, 101楼
守法公民 教育篇 Page 6, 113-121楼
梁溪香榭 高中留学须谨慎 Page 8, 149-152楼
梁溪香榭 加拿大高中教育感悟 Page 8, 153-155楼
梁溪香榭 根据成绩选择加拿大大学 Page 8, 159楼
梁溪香榭 如何选择最适合的加拿大大学 Page 9, 161楼
梁溪香榭 读高二的孩子去加拿大需要准备什么 Page 9, 164-165楼
梁溪香榭 高中英语课程介绍 Page 9, 166-169楼
梁溪香榭 高中英语课程介绍 Page 10, 188-191楼
梁溪香榭 多伦多著名私立学校收费一览 Page 10, 192楼
梁溪香榭 孩子的英语过关了吗? Page10, 194楼
梁溪香榭 关于A-LEVEL课程 Page 11, 217-228楼
shieley6811 关于A-LEVEL课程及学制 Page 13, 256-257楼
梁溪香榭 2007年安省高中拍名 Page 17, 322楼
朱三雀 安省, BC省, AB省高中拍名, Page 17, 329楼
响铃当当 加拿大教育优势 Page18, 347楼
梁溪香榭 A-LEVEL与IB课程的比较 Page 18, 358楼
朱三雀 中西学生的差异 Page 19, 364楼
梁溪香榭 大多伦多地区IB高中简介 Page 19, 372-375楼
梁溪香榭 IB课程与其他特殊教育课程的比较 Page 19, 376楼
梁溪香榭 IB与AP的比较, Page 19, 377楼
梁溪香榭 美国相关研究机构关于IB与AP的选择比较 Page 20, 387-391楼
梁溪香榭 群星灿烂不如一轮皓月当空-大学录取篇 Page 21, 409-413楼
梁溪香榭 在教育子女的过程中教育自己 Page 22, 422-424楼
梁溪香榭 关于美国大学 Page 22, 429-432楼
梁溪香榭 多伦多高中阶段特殊教育程序 Page 23, 445-446楼
梁溪香榭 加拿大高中生学科竞赛 Page 23, 447-448楼
梁溪香榭 美国大学 Page 24, 466楼
梁溪香榭 如何选择适合自己孩子的中小学 Page 25, 487-489楼
梁溪香榭 论孩子成长的内环境 Page 25, 490楼
梁溪香榭 犹太人的教育风格:朴实无华 讲求实际 Page 25, 497楼
梁溪香榭 培养子女的参政意识 Page 25, 498楼
梁溪香榭 美国大学点评 Page 25, 499-529楼
梁溪香榭 加拿大大学点评(商科) Page 27, 533-535楼
梁溪香榭 加拿大大学点评(理工科) Page 27, 547-549楼
梁溪香榭 安省高中教育咨询问答 Page 30, 584楼
梁溪香榭 毕业之际谈我的大学:滑铁卢大学 Page 30, 586-592楼
梁溪香榭 我在麦克马斯特 Page 30, 597楼
梁溪香榭 2005年度全球十大商学院 Page 30, 598楼
梁溪香榭 世界大学金融专业的排名 Page 30, 599楼
梁溪香榭 在加拿大IVEY商学院读MBA的日子 Page 31, 608楼
梁溪香榭 加拿大几个名牌大学比较之我见 Page 31, 616楼
梁溪香榭 加拿大的商学院MBA排名分析 Page 31, 618楼
梁溪香榭 加拿大本科申请指南 page 31-32, 620-624楼
yhyxll申请UBC请问需要什么条件,难度大否 Page 32, 633-634楼
willam131 关于A-LEVEL选课 page 34, 680楼
梁溪香榭 公立学校vs.私立学校
page 36, 713-716楼
梁溪香榭 路在自己脚下 --- 一位小留学生的自述 page 36, 717楼
惴惴不安 移民学生多“断层” 上公校还是私校难倒华人家长 page 36, 722楼
梁溪香榭 加拿大中学教育 page 38, 745楼
梁溪香榭 安大略省中学教育制度 page 38, 747楼
梁溪香榭 加拿大高中选课制 page 38, 748楼
梁溪香榭 高中毕业生该如何选择加拿大大学 page 38, 749楼
梁溪香榭 高中生如何入读加拿大大学?page 38, 750楼
梁溪香榭 加拿大大学怎样录取新生page 38, 751楼
梁溪香榭 加拿大大学对A-LEVEL的录取要求 page 38, 753-755楼
:wdb11:OMG! 绝了!:wdb6:

小丁丁 : 2009-09-10#786
回复: 子女教育信息素材收集帖

:wdb11:OMG! 绝了!:wdb6:
:wdb10::wdb10:

sandyli833 : 2009-09-10#787
回复: 加拿大教育情况有用素材收集帖子--梁溪香榭整理

谢谢梁溪香榭!:wdb10::wdb10::wdb10:

jessie_f : 2009-09-10#788
回复: 加拿大教育情况有用素材收集帖子--梁溪香榭整理

加拿大大学对A-LEVEL的录取要求

Queen’s University皇后大学

在众多加拿大本地人的眼中,皇后大学其实是加拿大最适合本科生就读的大学。虽然在整体排行上,皇后大学一直还无法超越多伦多大学,但如果以大一新生的平均入学分数来说的话,皇后大学的平均录取分数,一直高居全国第一位,是全加拿大最难进的大学。皇后大学在学术上一直保持着很高的水准, 与多伦多大学(University of Toronto)和麦吉尔大学(McGill University)一起并称加拿大的长青藤盟校。

皇后大学规模不大,拥有加拿大一流的商学院和医学院,理工科中的工程物理专业为加拿大第一,该专业在北美也仅次于Princeton和Cornell而排名第三。人文艺术、教育学、法律等都是非常有名。


入学成绩:至少2门Alevel 或4门AS成绩(不包括General Paper,以及5门Olevel成绩(至少为C)。学生应完成7门功课,其中至少2门Alevel(不包括General paper).一般来讲Alevel成绩不低于B,Olevel成绩不低于C。2门AS成绩相当于1门Alevel成绩.

文理学院

文科,文科(师范)
Olevel英语

理科,理科(师范),X射线技术
Alevel数学以及生物,化学,物理其中一门;Olevel英语以及生物,化学,物理其中两门

计算机,计算机(师范)
Alevel数学;Olevel或Alevel英语

艺术
Oleval英语

运动机能学和健康研究
Alevel生物,Olevel英语

人体运动学
Alevel生物以及数学,化学,物理其中一门;Olevel英语

商学院

商业
Alevel数学A,Olevel英语B

应用科学学院

工程学
Alevel数学以及化学,物理其中一门;Olevel英语(至少为B),化学,物理三门

护理学院

护理
Alevel生物,化学;Olevel英语(至少为B),生物,化学,物理四门


申请截止日期

主要的入学奖学金截止 Dec.1

网上申请截止 Mid Oct-February 15
提交PSE截止(必须提交,作为申请资料) March 1
提交所有证明文件 Apr.30



太好啦,很有用很及时,我正考虑让女儿朝这里努力着,:wdb19::wdb19::wdb19::wdb19: 非常非常地感谢中。。。,同时预祝溪溪面试顺利,马到成功:wdb29:

梁溪香榭 : 2009-09-10#789
回复: 加拿大教育情况有用素材收集帖子--梁溪香榭整理

加拿大所有大学的网址,

Bishop's University 主教大学 http://www.ubishops.ca
Brandon University 布兰顿大学 http://www.brandonu.ca
Brock University 布鲁克大学 http://www.brocku.ca

C

Carleton University 卡尔顿大学 http://www.carleton.ca
College of the North Atlantic 北大西洋学院 http://www.northatlantic.nf.ca
Concordia University 康卡迪亚大学 http://www.concordia.ca
CETAI (Centre for International Business Studies) 国际商务研究中心
http://cetai.hec.ca
Concordia University College of Alberta 康卡迪亚大学阿尔伯塔分院
http://www.concordia.ab.ca

D

Dalhousie University 戴尔豪斯大学 http://www.dal.ca

E

Ecole des Hautes Etudes Commerciales 蒙特利尔商业学院 http://www.hec.ca/en

F

FIMES(International Forum on Management, Ethics and Spirituality) 管理学、伦
理学思想教育学之国际论坛中心 http://www.hec.ca/fimes

G

Generale University 捷勒瑞尔大学 http://www.generale-canada.com


H

Homeopathic College of Canada 加拿大同种疗法学院 http://www.homepathy.edu

K

King's University College 皇家大学学院 http://www.kingsu.ab.ca

M

McGill University 麦吉尔大学 http://www.mcgill.ca
McGill University WeatherPage 麦吉尔气象学院
http://zephyr.meteo.mcgill.ca/gary/weatherpage.html
McMaster University 麦玛斯特大学 http://www.mcmaster.ca/home.html
Memorial University of Newfoundland http://www.mun.ca
Mount Allison University 曼特艾里森大学 http://www.mta.ca/home.html
Mount Saint Vincent University 曼特圣温森特大学 http://www.msvu.ca
Moncton Flight College 曼克顿佛莱特学院 http://www.mfc.nb.ca
Memorial University of Newfoundland 纽芬兰纪念大学 http://www.mun.ca
Mount Allison University http://www.unbc.ca

N

Nipissing University 尼彼星大学 http://www.canadorec.on.ca
Nova Scotia Agricultural College 新斯科舍省农业大学 http://www.nsac.ns.ca
Nova Scotia College of Art and Design 新斯科舍省艺术设计学院
http://www.nscad.ns.ca

O

Okanagan University College 阿卡纳甘大学 http://www.ouc.bc.ca

P

Providence College & Seminary 普罗维登斯神学院
http://www.providence.mb.ca

Q

Queen's University 皇后大学 http://www.queensu.ca

R

Redeemer College 耶稣基督学院 http://www.redeemer.on.ca
Royal Military College of Canada 加拿大皇家陆军学院
http://www.royalroads.ca
Ryerson Polytechnic University 瑞尔森科技大学 http://www.ryerson.ca

S

Saskatchewan Indian Federated College 萨斯卡彻温印度联盟学院
http://www.sifc.edu
Simon Fraser University 西蒙佛蕾泽大学 http://www.sfu.ca
St. Francis Xavier University 圣佛朗西萨维尔大学
http://www3.ns.sympatico.ca/adnet/Elens
St. Thomas University 圣托玛斯大学 http://www.stthomasu.ca
St. Mary's University 圣玛丽亚大学 http://www.stmarys.ca
Sainte-Anne http://www.ustanne.ednet.ns.ca

T

Technical University of British Columbia (TechBC) 哥伦比亚科技大学
http://www.techbc.ca
Technical University of Nova Scotia 新斯科舍省科技大学 http://www.dal.ca
Trent University 川特大学 http://www.trentu.ca
Trinity Western University 西垂尼提大学 http://www.twu.ca

U

Universite de Sherbrooke 西尔布鲁克大学 http://www.usherb.ca
University College of Cape Breton 凯波布兰顿大学
http://www.cgc.ns.ca/cgc/main_en.html
University of Sudbury 萨德伯里大学 http://www.usudbury.com
University College of the Cariboo (UCC) 卡里布大学学院
http://www.cariboo.bc.ca
University College of the Fraser Valley 富瑞泽河谷大学
http://www.mbseminary.com
University of Alberta 阿尔伯塔大学 http://www.ualberta.ca
University of British Columbia 哥伦比亚大学 http://www.ubc.ca
University of Calgary 卡加里大学 http://www.ucalgary.ca
University of Guelph 盖尔佛大学 http://www.uoguelph.ca
University of King's College 国王学院大学 http://www.ukings.ns.ca
University of Lethbridge 莱斯布瑞泽大学 http://home.uleth.ca/reg-
rcr/students
University of Manitoba 曼尼托巴大学 http://www.umanitoba.ca
University of New Brunswick 新布朗斯维克大学 http://www.unb.ca
University of Northern British Columbia 北哥伦比亚大学
http://www.mcmaster.ca/home.html
University of Ottawa 渥太华大学 http://www.uottawa.ca/splash-3.html
University of Prince Edward Island 爱德华王子岛大学 http://www.upei.ca
University of Regina 里贾纳大学 http://www.uregina.ca

梁溪香榭 : 2009-09-10#790
回复: 加拿大教育情况有用素材收集帖子--梁溪香榭整理

:wdb11:OMG! 绝了!:wdb6:


谢谢梁溪香榭!:wdb10::wdb10::wdb10:

太好啦,很有用很及时,我正考虑让女儿朝这里努力着,:wdb19::wdb19::wdb19::wdb19: 非常非常地感谢中。。。,同时预祝溪溪面试顺利,马到成功:wdb29:

谢谢支持! 分分送上! :wdb6::wdb6::wdb6:

梁溪香榭 : 2009-09-10#791
回复: 加拿大教育情况有用素材收集帖子--梁溪香榭整理

加拿大双录取大学

双录取,即有条件录取,申请人其他条件都符合该学校要求,但没有语言成绩或者语言成绩不够标准的情况下,学校发语言中心和专业课程两份通知书。“有条件” 的条件包括除语言成绩以外的高考成绩、会考成绩、高中毕业证、高中平均分、在读证明等等。根据各学校要求不同,需要的“条件”也不同。

有些要求比较高的学校对双录取也有一些语言要求,比如雅思、托福成绩要达到一定分数才能够进入该校语言中心继续提高语言水平,读到语言中心最高级或者学校要求的级别后即可入读专业课。下面列出加拿大大学设置双录取的学校清单,供参考。

阿尔伯塔大学 University of Alberta
卡尔加里大学 University of Calgary
戴尔豪西大学 Dalhousie University
萨省大学 University of Saskatchewan
西安大略大学 University of Western Ontario
曼尼托巴大学 University of Manitoba
西蒙菲莎大学 Simon Fraser University
维多利亚大学 University of Victoria
滑铁卢大学 University of Waterloo
圭?夫大学 University of Guelph
纽芬兰纪念大学 Memorial University of Newfoundland
纽布伦斯威克大学 University of New Brunswick
卡尔顿大学 Carleton University
温莎大学 University of Windsor
里贾纳大学 University of Regina
约克大学 York University
肯考迪亚大学 Concordia University
北英属哥伦比亚大学 University of Northern British Columbia
阿卡迪亚大学 Acadia University
圣弗朗西斯大学 St.Francis Xavier University
特伦特大学 Trent University
莱斯布里奇大学 University of Lethbridge
爱德华王子岛大学 University of Prince Edward Island
温尼伯大学 University of Winnipeg
圣玛丽大学 St.Mary’s University
布兰登大学 Brandon University
湖首大学 Lakehead University
布鲁克大学 Brock University
圣文森特山大学 Mount Saint Vincent University
主教大学 Bishop’s University
劳伦森大学 Laurentian University
圣托马斯大学 St.Thomas University
不列颠海角大学 Cape Breton University

sunxuan01 : 2009-09-10#792
回复: 加拿大教育情况有用素材收集帖子--梁溪香榭整理

加拿大所有大学的网址,

Bishop's University 主教大学 http://www.ubishops.ca
Brandon University 布兰顿大学 http://www.brandonu.ca
Brock University 布鲁克大学 http://www.brocku.ca

C

Carleton University 卡尔顿大学 http://www.carleton.ca
College of the North Atlantic 北大西洋学院 http://www.northatlantic.nf.ca
Concordia University 康卡迪亚大学 http://www.concordia.ca
CETAI (Centre for International Business Studies) 国际商务研究中心
http://cetai.hec.ca
Concordia University College of Alberta 康卡迪亚大学阿尔伯塔分院
http://www.concordia.ab.ca

D

Dalhousie University 戴尔豪斯大学 http://www.dal.ca

E

Ecole des Hautes Etudes Commerciales 蒙特利尔商业学院 http://www.hec.ca/en

F

FIMES(International Forum on Management, Ethics and Spirituality) 管理学、伦
理学思想教育学之国际论坛中心 http://www.hec.ca/fimes

G

Generale University 捷勒瑞尔大学 http://www.generale-canada.com


H

Homeopathic College of Canada 加拿大同种疗法学院 http://www.homepathy.edu

K

King's University College 皇家大学学院 http://www.kingsu.ab.ca

M

McGill University 麦吉尔大学 http://www.mcgill.ca
McGill University WeatherPage 麦吉尔气象学院
http://zephyr.meteo.mcgill.ca/gary/weatherpage.html
McMaster University 麦玛斯特大学 http://www.mcmaster.ca/home.html
Memorial University of Newfoundland http://www.mun.ca
Mount Allison University 曼特艾里森大学 http://www.mta.ca/home.html
Mount Saint Vincent University 曼特圣温森特大学 http://www.msvu.ca
Moncton Flight College 曼克顿佛莱特学院 http://www.mfc.nb.ca
Memorial University of Newfoundland 纽芬兰纪念大学 http://www.mun.ca
Mount Allison University http://www.unbc.ca

N

Nipissing University 尼彼星大学 http://www.canadorec.on.ca
Nova Scotia Agricultural College 新斯科舍省农业大学 http://www.nsac.ns.ca
Nova Scotia College of Art and Design 新斯科舍省艺术设计学院
http://www.nscad.ns.ca

O

Okanagan University College 阿卡纳甘大学 http://www.ouc.bc.ca

P

Providence College & Seminary 普罗维登斯神学院
http://www.providence.mb.ca

Q

Queen's University 皇后大学 http://www.queensu.ca

R

Redeemer College 耶稣基督学院 http://www.redeemer.on.ca
Royal Military College of Canada 加拿大皇家陆军学院
http://www.royalroads.ca
Ryerson Polytechnic University 瑞尔森科技大学 http://www.ryerson.ca

S

Saskatchewan Indian Federated College 萨斯卡彻温印度联盟学院
http://www.sifc.edu
Simon Fraser University 西蒙佛蕾泽大学 http://www.sfu.ca
St. Francis Xavier University 圣佛朗西萨维尔大学
http://www3.ns.sympatico.ca/adnet/Elens
St. Thomas University 圣托玛斯大学 http://www.stthomasu.ca
St. Mary's University 圣玛丽亚大学 http://www.stmarys.ca
Sainte-Anne http://www.ustanne.ednet.ns.ca

T

Technical University of British Columbia (TechBC) 哥伦比亚科技大学
http://www.techbc.ca
Technical University of Nova Scotia 新斯科舍省科技大学 http://www.dal.ca
Trent University 川特大学 http://www.trentu.ca
Trinity Western University 西垂尼提大学 http://www.twu.ca

U

Universite de Sherbrooke 西尔布鲁克大学 http://www.usherb.ca
University College of Cape Breton 凯波布兰顿大学
http://www.cgc.ns.ca/cgc/main_en.html
University of Sudbury 萨德伯里大学 http://www.usudbury.com
University College of the Cariboo (UCC) 卡里布大学学院
http://www.cariboo.bc.ca
University College of the Fraser Valley 富瑞泽河谷大学
http://www.mbseminary.com
University of Alberta 阿尔伯塔大学 http://www.ualberta.ca
University of British Columbia 哥伦比亚大学 http://www.ubc.ca
University of Calgary 卡加里大学 http://www.ucalgary.ca
University of Guelph 盖尔佛大学 http://www.uoguelph.ca
University of King's College 国王学院大学 http://www.ukings.ns.ca
University of Lethbridge 莱斯布瑞泽大学 http://home.uleth.ca/reg-
rcr/students
University of Manitoba 曼尼托巴大学 http://www.umanitoba.ca
University of New Brunswick 新布朗斯维克大学 http://www.unb.ca
University of Northern British Columbia 北哥伦比亚大学
http://www.mcmaster.ca/home.html
University of Ottawa 渥太华大学 http://www.uottawa.ca/splash-3.html
University of Prince Edward Island 爱德华王子岛大学 http://www.upei.ca
University of Regina 里贾纳大学 http://www.uregina.ca


梁溪辛苦哦!祝面试顺利!:wdb17::wdb17::wdb9::wdb9:

梁溪香榭 : 2009-09-10#793
回复: 加拿大教育情况有用素材收集帖子--梁溪香榭整理

梁溪辛苦哦!祝面试顺利!:wdb17::wdb17::wdb9::wdb9:

谢谢。也祝嫩很快FN,ME, DM, PL, VISA, LANDING :wdb17::wdb20:

Luck Li : 2009-09-10#794
回复: 加拿大教育情况有用素材收集帖子--梁溪香榭整理

祝:大家都顺顺利利!

sunxuan01 : 2009-09-10#795
回复: 加拿大教育情况有用素材收集帖子--梁溪香榭整理

谢谢。也祝嫩很快FN,ME, DM, PL, VISA, LANDING :wdb17::wdb20:


谢谢!你也一样哦!:wdb6::wdb6::wdb6::wdb6::wdb6:

sandyli833 : 2009-09-10#796
回复: 加拿大教育情况有用素材收集帖子--梁溪香榭整理

:wdb11:感梁溪香榭!
University of New Brunswick 新布朗斯维克大学是NB省?校可以吧?
如果小孩子完高二去加拿大科,在多多有那些好的公立校?

王晶 : 2009-09-10#797
回复: 加拿大教育情况有用素材收集帖子--梁溪香榭整理

:wdb11:感梁溪香榭!
University of New Brunswick 新布朗斯维克大学是NB省?校可以吧?
如果小孩子完高二去加拿大科,在多多有那些好的公立校?
继续读高三还是直接读大学?:wdb2::wdb2::wdb2:

sandyli833 : 2009-09-12#798
回复: 加拿大教育情况有用素材收集帖子--梁溪香榭整理

继续读高三还是直接读大学?:wdb2::wdb2::wdb2:

谢谢王晶!
读完高二去的话,就继续读高三,请问多伦多有那些好的公立高中?
读完高三去的话,先学语言,然后直入本科(想入UNB大学)。

梁溪香榭 : 2009-09-13#799
回复: 加拿大教育情况有用素材收集帖子--梁溪香榭整理

谢谢王晶!
读完高二去的话,就继续读高三,请问多伦多有那些好的公立高中?
读完高三去的话,先学语言,然后直入本科(想入UNB大学)。

只读高三恐怕时间太紧。因为要在一年内即要语言过12年级,托福雅思,且要修满高中三年学分,还有累计满社会活动时间。

sunxuan01 : 2009-09-13#800
回复: 加拿大教育情况有用素材收集帖子--梁溪香榭整理

只读高三恐怕时间太紧。因为要在一年内即要语言过12年级,托福雅思,且要修满高中三年学分,还有累计满社会活动时间。


梁溪说的有道理,看来高二去更合适!:wdb10::wdb10:

故乡的云 : 2009-09-13#801
回复: 加拿大教育情况有用素材收集帖子--梁溪香榭整理

翻页收藏

sunnytan : 2009-09-13#802
回复: 加拿大教育情况有用素材收集帖子--梁溪香榭整理

梁溪妹妹,谢谢你提供的学校信息!:wdb17:
关于我儿子的学习看样子要根据你前面提供的双录取或college方案了,朋友推荐登陆后不上高中而是上college 的语言课程,然后拿国内的高中毕业证书读COLLEGE 再转学分到大学,据说他们的孩子就是这样进入UBC的。我的孩子不是很优秀,可也有名校梦啊!

sandyli833 : 2009-09-13#803
回复: 加拿大教育情况有用素材收集帖子--梁溪香榭整理

只读高三恐怕时间太紧。因为要在一年内即要语言过12年级,托福雅思,且要修满高中三年学分,还有累计满社会活动时间。

梁溪说的有道理,看来高二去更合适!:wdb10::wdb10:

:wdb11:明白,感谢两位!
但按现时进度,我怕小孩读完高二还未收到DM,无奈啊!:wdb14:
真不知如何是好?

蓉子 : 2009-09-13#804
回复: 加拿大教育情况有用素材收集帖子--梁溪香榭整理

请问:多大,西安略大等录取条件之一中国教育:高中毕业证+高考成绩+语言成绩。请问《高考成绩》是否是必备条件:wdb2:

梁溪香榭 : 2009-09-13#805
回复: 加拿大教育情况有用素材收集帖子--梁溪香榭整理

梁溪妹妹,谢谢你提供的学校信息!:wdb17:
关于我儿子的学习看样子要根据你前面提供的双录取或college方案了,朋友推荐登陆后不上高中而是上college 的语言课程,然后拿国内的高中毕业证书读COLLEGE 再转学分到大学,据说他们的孩子就是这样进入UBC的。我的孩子不是很优秀,可也有名校梦啊!

很不错的选择啊!谢谢你提供的信息。:wdb19::wdb17::wdb9:

梁溪香榭 : 2009-09-13#806
回复: 加拿大教育情况有用素材收集帖子--梁溪香榭整理

请问:多大,西安略大等录取条件之一中国教育:高中毕业证+高考成绩+语言成绩。请问《高考成绩》是否是必备条件:wdb2:

关于那些大学的录取条件,完全是摘录自那些大学的官方网站。
高考成绩是网站上明确指出的,也没有看到有任何附加说明指出高考成绩不是必备条件。有些大学甚至明确指出高考成绩要求(比如80%等等)。
建议你可以直接发MAIL去心仪的大学问问。

梁溪香榭 : 2009-09-13#807
回复: 加拿大教育情况有用素材收集帖子--梁溪香榭整理

只读高三恐怕时间太紧。因为要在一年内即要语言过12年级,托福雅思,且要修满高中三年学分,还有累计满社会活动时间。



:wdb11:明白,感谢两位!
但按现时进度,我怕小孩读完高二还未收到DM,无奈啊!:wdb14:
真不知如何是好?

嗯。是挺为难的。建议还是要根据孩子的具体情况具体对待了。
如果孩子学习成绩好,不如就拿个高考成绩吧。
如果学习成绩一般般,我的认为是不如拿到VISA就过去。
多读一年高中,或读COLLEGE转学分,比如SUNNYTAN的选择就不错啊。

梁溪香榭 : 2009-09-13#808
回复: 加拿大教育情况有用素材收集帖子--梁溪香榭整理

翻页收藏

谢谢支持 :wdb19::wdb6:

梁溪香榭 : 2009-09-14#809
回复: 加拿大教育情况有用素材收集帖子--梁溪香榭整理

溪溪:行动好迅速,太太感谢你了.

刚看到嫩在小白的帖子里转的好帖
俺在俺自留地里也收藏哈。
谢谢!

梁溪香榭 : 2009-09-14#810
回复: 加拿大教育情况有用素材收集帖子--梁溪香榭整理

感谢PWJND提供滴资料。:wdb17::wdb17:

“转贴1:
住在加国之申请魁北克省助学金及贷款 (2007-08-27 10:58:30)
标签:校园生活 加拿大 魁北克 蒙特利尔 教育 学费 大专大学 奖学金 助学金 贷款
分类:魁北克省的生活指南系列


经过快乐漫长的暑假后,又有不少学生踏入大专院校。在加拿大,众所周知魁北克省是加拿大教育系统管理最为严格和完善,资助条件最好的省份,这也吸引总舵移民在魁省展开他们的求学生涯。每年得益于魁省贷款及助学金(Loan&Bursary)计划的学生遍及个中等职业培训学校或大专院校、大学及其他魁省教育部认可的教育机构。本地某媒体整理了这个资料,我就将这个非常重要的方式与符合条件的移民们一同分享吧。
贷款及助学金(Loan&Bursary)计划是“学生财政援助计划”(Aide financie reaux etudes,法语写,简称AFE)的主要资金来源。主要是用于那些希望全职学习又暂无足够资金的学生。
申请人资格要求
1)申请人是加拿大公民或永久居民(持有枫叶卡,还未入籍者)、难民或在移民和难民法的规定中属于受保护人;
2)申请人是魁省居民或在申请期间即将成为魁省居民;
3)申请人被魁省教育部(Ministere de l' education,du Loisir et du Sport)人口的教育机构接受或者即将被接受;
4)申请人在被认可的学习项目中,将进行全职学习;
5)申请人没有超过财政助学金所提供的时限;
6)根据申请人的教育水平、学位或学习项目的类型,没有达到债务限额;
7)申请人没有足够财力完成学业;
特别说明:
*服刑期间的人不能享受贷款和助学金计划。
*如果申请人及配偶均为学生,那么只有其中之一可以在指定年份,作为全职学生申请此计划。
*居住在魁省的界定:
a)申请人必须在魁省出生或被居住在魁省的人所收养,且收养时,收养人必须居住在魁省。
b)申请人父母之一居住在魁省。
c)申请人服务已去世,但父母其中之一在死亡时,居住在魁省;
d)申请人应该居住在魁省,尽管其父母现居住在其他省份;
e)在过去连续12个月内,魁省是申请人最后居住的地方,且在此期间,申请人没有进行全职学生的学习;
f)申请人持有根据魁省有关移民的法规颁发的甄选证书;
g)申请人在魁省居住至少满三个月,同时在加拿大其他省份居住的时间不超过三个月;
h)申请人在过去五年里,根据上述规定,连续在魁省居住了三年的时间;
i)申请人或者配偶符合上述规定之一,被认定或曾经被认定为魁省居民;
j)申请人在魁省之外学习,但居住在魁省,每天往返于住所和教育机构之间,且符合上述规定之一;
*另外,如果申请人目前在魁省之外学习,那么如果在离开魁省之日,符合上述条件之一,即被认定为魁省居民,且符合如下标准:
a)申请人父母居住在魁省;
b)申请人不居住在魁省,当迁居到其他省份时间不超过五年;
c)申请人在离开魁省后,在连续12个月内没有中断全职学习;
d)申请人在离开魁省后,其配偶没有在魁省工作超过12个月。
注释:
@ 出生在魁省的学生及没有父母陪伴而回到魁省的学生,在没有进行全职学习时,则不需工作12个月;
@ 申请人被要求在没有进行全职学习的情况下,居住在魁省达12个月,此规定是为防止出现学生同时享受魁省及其他省份贷款和助学金计划的情况;
@ 如果学生的父母不再居住在一起,且其中之一现居住在魁省之外,那么AFE将把在学生开始学业前,与学生居住的父母之一的居住地考虑在内。如果此父母之一居住在魁省,那么学生被认为魁省居民,如果此父母之一没有居住在魁省,那么学生即不被认定为魁省居民。
* 在魁北克省以外地区学习的学生,那些符合贷款/助学金资助条件?
a)所有接受政府补贴的加拿大学院和大学学习项目都被认可;
b)在魁省之外学习的学生(包括那些在加拿大其他省份和法国学习的学生),如注册学习魁省政府或者MELS在魁省有所限制的项目――医学或治安维持技术(警察技术),则不符合申请贷款和助学金计划的条件;另外远程教育、第二语言学习及飞行器领航项目不被贷款和助学金计划承认。
c)居住在魁省但在渥太华大学学习医学的学生根据贷款和助学金计划可以符合申请条件。
魁省助学金/贷款资助的发放、开始计息及其他相关规定:
1)MELS每月一次与教育机构合适学生的身份。
2)一旦通过每月的核实,学生的身份被确定,贷款和助学金将按照月份或在相关月份之前定时直接被存入学生银行账户。

3)教育机构在学生学业开始之间,给学生提供保证证明(Guarantee Certificate),证书学生的注册信息,同时意味着第一期学生贷款可以被发放。

4)如果学生在相关月份16号之前即开始被认可项目的学习,那么第一个月的学习将被承认。

5)在享受贷款和助学金计划当年,最后一个被承认的月份中,学生至少进行过一天的学习。

6)中断学习的学生一旦在接下来的学习期间回复学习,将被认定为全职学生。如果接下来开始学习的月份是在5月或8月,那么预期恢复学习的时间可以被推迟至9月。没有在预期日期恢复学习的学生家不再被认为是学生,并需要支付该月的学生贷款利息。在接下来的学习阶段不准备恢复学习的学生将从中断学习的月份开始,不在被认定为学生,因此需要开始自支付学生贷款的利息。

7)学生身份的改变将在变更之日后第二月开始被考虑。

8)接近完成可以获得DCS文凭的学生(如那些在最后学习阶段只剩三门课程需要完成的学生)符合贷款和助学金计划条件。

9)作为审计师或注册语言课程(英、法、或其他语言)注册学习的学生不符合贷款和助学金计划申请条件。但在被认可学习项目进行学习的全职学生即使课程内有语言学习内容,也符合贷款和助学金计划申请条件。

10)作为独立学生在大学进行学习的学生按照贷款和贷款和助学金计划规定,在学士学位、硕士学位以及博士学位每个级别最多可以申请8个月的贷款和助学金。在第一个月的学习中被认定的独立学生身份将在整个年度都被认可。为了管理方便,项目代码为09999.作为独立学生所收到的任何一个学习月份的贷款和助学金都被计入享受贷款和助学金的最大年限中。

11)实习课程如果属于学习项目中的必修课,则被认为等同为全职学习,同年公司如果实习课程被教育机构认定为全职实习,则也被承认。

12)进行全职实习的学生如果作为合作项目(或被称作工作-研究项目)则不符合贷款和助学金计划申请条件。合作项目除了课程教学外,还包括带薪全职实习(在工厂进行的实习培训)。在此期间,学生无需开始偿还学生贷款或贷款所产生的利息。

13)研究生在进行作为研究项目的一部分的研究时,不符合实习条件。
转贴2:
OSAP(Ontario Student Assistance Program) 安省学生助学项目是安大略省为了支持经济有困难的学生完成学业。

一般OSAP分为两部分,及grant (助学金,不需偿还)+ loan (贷款,需偿还且计息)。根据学生的经济状况,单身

学生最高可以获得350元/周的OSAP,而结婚有家庭的学生最高可以获得545 元/周的OSAP。由于grant 不需偿还,

且通常占OSAP总额的50%以上,因此学生毕业后实际只需偿还不超过OSAP总额的50%。

曾经有一个华人移民申请到了25,000,毕业后只需偿还10,500。 OSAP的总额取决于几个因素:学费、学生的经济

状况。为了能够有资格申请且申请到较高额度的OSAP,新移民不宜带太多现金来加拿大,更不能抵加后购买高档汽

车。曾经有一位新移民登陆时申报了50,000 加元现金,在一个学院申请入学时谎称自己只有5000元。他所申请的

是一个postgraduate program, 学时一年,学费20,000。结果学院协助其申请OSAP后,发现他只能申请到2000元

的OSAP。请注意,政府有权核实居民的经济状况,并调取申请人的银行账户信息。

转贴完。“


小白,魁省投移是否只能申请魁省助学金?以*居住在魁省的界定:
f)申请人持有根据魁省有关移民的法规颁发的甄选证书;

另外必须在安省住满12个月才有资格申请OSAP

sandyli833 : 2009-09-14#811
回复: 加拿大教育情况有用素材收集帖子--梁溪香榭整理

嗯。是挺为难的。建议还是要根据孩子的具体情况具体对待了。
如果孩子学习成绩好,不如就拿个高考成绩吧。
如果学习成绩一般般,我的认为是不如拿到VISA就过去。
多读一年高中,或读COLLEGE转学分,比如SUNNYTAN的选择就不错啊。
:wdb11:
感谢梁溪!
您的贴子,是我们的:wdb19::wdb10::wdb10::wdb10:

蓉子 : 2009-09-14#812
回复: 加拿大教育情况有用素材收集帖子--梁溪香榭整理

关于那些大学的录取条件,完全是摘录自那些大学的官方网站。
高考成绩是网站上明确指出的,也没有看到有任何附加说明指出高考成绩不是必备条件。有些大学甚至明确指出高考成绩要求(比如80%等等)。
建议你可以直接发MAIL去心仪的大学问问。
谢谢梁溪!

生命的狂想 : 2009-11-03#813
回复: 加拿大教育情况有用素材收集帖子--梁溪香榭整理

学习!

紫钻石妈妈 : 2009-11-03#814
回复: 加拿大教育情况有用素材收集帖子--梁溪香榭整理

感谢梁溪:wdb19:
好人呐

year : 2009-11-04#815
回复: 加拿大教育情况有用素材收集帖子--梁溪香榭整理

看了一下午,准确的说学习了一个下午,收获很大啊,谢谢各位提供信息的朋友,特别是梁溪香榭和守法啊,我没有分可加,只能感谢了.怎么献花啊?辛苦啦.

梁溪香榭 : 2009-11-04#816
回复: 加拿大教育情况有用素材收集帖子--梁溪香榭整理

看了一下午,准确的说学习了一个下午,收获很大啊,谢谢各位提供信息的朋友,特别是梁溪香榭和守法啊,我没有分可加,只能感谢了.怎么献花啊?辛苦啦.

谢谢
心意领了

梁溪香榭 : 2009-11-04#817
回复: 加拿大教育情况有用素材收集帖子--梁溪香榭整理

感谢梁溪:wdb19:
好人呐

呵呵。最近比较忙。
等有空了,俺再去搜集点。:wdb20:

梁溪香榭 : 2009-11-04#818
回复: 加拿大教育情况有用素材收集帖子--梁溪香榭整理

:wdb11:
感谢梁溪!
您的贴子,是我们的:wdb19::wdb10::wdb10::wdb10:

关于那些大学的录取条件,完全是摘录自那些大学的官方网站。
高考成绩是网站上明确指出的,也没有看到有任何附加说明指出高考成绩不是必备条件。有些大学甚至明确指出高考成绩要求(比如80%等等)。
建议你可以直接发MAIL去心仪的大学问问。
谢谢梁溪!

学习!

谢谢支持。
资料能对大家有用就好。

枫叶家 : 2009-11-06#819
回复: 加拿大教育情况有用素材收集帖子--梁溪香榭整理

好贴,谢谢分享!

xxwe : 2009-11-08#820
回复: 加拿大教育情况有用素材收集帖子--梁溪香榭整理

梁溪你好,我们12月登陆多伦多,孩子今年6月参加了高考,并被国内大学录取。因为马上登陆了,所以孩子已在大学办了休学。
孩子去那边申请明年9月的大学本科;目前孩子已有了:
1、雅思成绩,
2、高中三年的成绩单,
3,会考成绩单,
4、高考成绩单。
还需要带的材料有那些?麻烦你给我一个清单。
另外,以上成绩单是带翻译件还是带公证件?
在此谢谢了。

tianyan : 2009-11-14#821
回复: 加拿大教育情况有用素材收集帖子--梁溪香榭整理

支持一下

熊的瓶子 : 2009-11-18#822
回复: 加拿大教育情况有用素材收集帖子--梁溪香榭整理

支持一下,好东西,赶快奔去收藏

wliu88 : 2009-12-09#823
回复: 加拿大教育情况有用素材收集帖子--梁溪香榭整理

这么好的帖子,谢谢

yamaha : 2009-12-16#824
回复: 加拿大教育情况有用素材收集帖子--梁溪香榭整理

保存下先

潜伏 : 2009-12-24#825
回复: 子女教育信息素材收集帖

嘎嘎

这个帖子也算历经了劫难了,丢了又重新找回来了。

主要整理的是关于教育的一些素材

从147楼开始,主要就是梁溪香榭搜集的一些的,推荐推荐!

推荐本书--从普通女孩到银行家,http://forum.iask.ca/showthread.php?t=233637

目录

tony 3213, PEI地区教育情况 Page 1, 5楼
tiantianxingchen 高中课程简介 Page 1, 6-9楼
守法公民 初三学生多伦多求学记 Page1, 18-22楼
守法公民 安省新移民高中指南 Page 2, 24-30楼
守法公民 安省新移民小学指南 Page 2, 34-36楼
守法公民 高中家长如是说 Page 3, 44-49楼
雨中冷百合 大多伦多地区高中排名 Page 3, 57楼, 59楼
守法公民 温哥华十佳公立中学 Page 4, 65楼
一缕阳光 菲沙关于BC省中学排名及相关简介 Page 4, 69楼
守法公民 部分大学对是否需要雅思或托福成绩的年限规定 Page 6, 101楼
守法公民 教育篇 Page 6, 113-121楼
梁溪香榭 高中留学须谨慎 Page 8, 149-152楼
梁溪香榭 加拿大高中教育感悟 Page 8, 153-155楼
梁溪香榭 根据成绩选择加拿大大学 Page 8, 159楼
梁溪香榭 如何选择最适合的加拿大大学 Page 9, 161楼
梁溪香榭 读高二的孩子去加拿大需要准备什么 Page 9, 164-165楼
梁溪香榭 高中英语课程介绍 Page 9, 166-169楼
梁溪香榭 高中英语课程介绍 Page 10, 188-191楼
梁溪香榭 多伦多著名私立学校收费一览 Page 10, 192楼
梁溪香榭 孩子的英语过关了吗? Page10, 194楼
梁溪香榭 关于A-LEVEL课程 Page 11, 217-228楼
shieley6811 关于A-LEVEL课程及学制 Page 13, 256-257楼
梁溪香榭 2007年安省高中拍名 Page 17, 322楼
朱三雀 安省, BC省, AB省高中拍名, Page 17, 329楼
响铃当当 加拿大教育优势 Page18, 347楼
梁溪香榭 A-LEVEL与IB课程的比较 Page 18, 358楼
朱三雀 中西学生的差异 Page 19, 364楼
梁溪香榭 大多伦多地区IB高中简介 Page 19, 372-375楼
梁溪香榭 IB课程与其他特殊教育课程的比较 Page 19, 376楼
梁溪香榭 IB与AP的比较, Page 19, 377楼
梁溪香榭 美国相关研究机构关于IB与AP的选择比较 Page 20, 387-391楼
梁溪香榭 群星灿烂不如一轮皓月当空-大学录取篇 Page 21, 409-413楼
梁溪香榭 在教育子女的过程中教育自己 Page 22, 422-424楼
梁溪香榭 关于美国大学 Page 22, 429-432楼
梁溪香榭 多伦多高中阶段特殊教育程序 Page 23, 445-446楼
梁溪香榭 加拿大高中生学科竞赛 Page 23, 447-448楼
梁溪香榭 美国大学 Page 24, 466楼
梁溪香榭 如何选择适合自己孩子的中小学 Page 25, 487-489楼
梁溪香榭 论孩子成长的内环境 Page 25, 490楼
梁溪香榭 犹太人的教育风格:朴实无华 讲求实际 Page 25, 497楼
梁溪香榭 培养子女的参政意识 Page 25, 498楼
梁溪香榭 美国大学点评 Page 25, 499-529楼
梁溪香榭 加拿大大学点评(商科) Page 27, 533-535楼
梁溪香榭 加拿大大学点评(理工科) Page 27, 547-549楼
梁溪香榭 安省高中教育咨询问答 Page 30, 584楼
梁溪香榭 毕业之际谈我的大学:滑铁卢大学 Page 30, 586-592楼
梁溪香榭 我在麦克马斯特 Page 30, 597楼
梁溪香榭 2005年度全球十大商学院 Page 30, 598楼
梁溪香榭 世界大学金融专业的排名 Page 30, 599楼
梁溪香榭 在加拿大IVEY商学院读MBA的日子 Page 31, 608楼
梁溪香榭 加拿大几个名牌大学比较之我见 Page 31, 616楼
梁溪香榭 加拿大的商学院MBA排名分析 Page 31, 618楼
梁溪香榭 加拿大本科申请指南 page 31-32, 620-624楼
yhyxll申请UBC请问需要什么条件,难度大否 Page 32, 633-634楼
willam131 关于A-LEVEL选课 page 34, 680楼
梁溪香榭 公立学校vs.私立学校
page 36, 713-716楼
梁溪香榭 路在自己脚下 --- 一位小留学生的自述 page 36, 717楼
惴惴不安 移民学生多“断层” 上公校还是私校难倒华人家长 page 36, 722楼
梁溪香榭 加拿大中学教育 page 38, 745楼
梁溪香榭 安大略省中学教育制度 page 38, 747楼
梁溪香榭 加拿大高中选课制 page 38, 748楼
梁溪香榭 高中毕业生该如何选择加拿大大学 page 38, 749楼
梁溪香榭 高中生如何入读加拿大大学?page 38, 750楼
梁溪香榭 加拿大大学怎样录取新生page 38, 751楼
梁溪香榭 加拿大大学对A-LEVEL的录取要求 page 38, 753-755楼

mark

linzjijie : 2010-01-08#826
回复: jenniferlee的精彩发言

在陈述下面的理由之前,俺得先把钢盔戴上,以防遭拍时被砸破脑袋:wdb13:


昔日孟母三迁,是为了给孩子创造一个“谈笑有鸿儒,往来无白丁”的生活和学习环境。今天的张王李赵母,为了类似的,或亚类似的理由,也在不断地迁徙,甚至不远万里地择地而居。


不是矫情,但俺的确觉得,现在的孩子由于物质生活条件太过优越,因此而失去了太多奋斗的动力。这使得俺非常担心儿子的未来。


所谓富不过三代,是非常有道理的。第一代的创业者胼手胝足地打下了江山,自己不舍得吃不舍得花的。到了第二代,能够将祖业发扬光大者又有几人?多的是纨绔子弟,坐吃山空的败家子。


虽然俺们并非什么富人,但相对而言,孩子从小没有吃过一点苦。不穷则不思变,多年充裕丰富的物质享受,已然使得他的进取精神廖剩无几,前途堪忧。


因此,俺想改变一下这种状况,带着孩子去加拿大受点苦。这也是俺移民的理由之一。

工作多年,多少有些见识.


俺发现无论哪个领域的成功者,一般而言都是具有高度责任心的人.他们对社会,对事业,对工作,对家庭,对自己,对他人,都秉持着高度负责任的态度.正因为如此,他们才能踏踏实实,认认真真地做好每一件事,即便那是件在世人眼中,无足轻重的琐事一桩.也正因为如此,他们的成长伴随着一次又一次的成功,才得以不断地获得领导的首肯,群众的认同,才能够一路青云,扶摇直上.


无论是在哪一个范畴,无论是在社会,单位,还是在家庭,很难想象一个欠缺责任心的人,能够长期地在该范畴内获得稳定的成就.


俺们的孩子多是独生子女,生活条件优越,学业负担沉重.因此,他们无论在心理上,还是在实际生活中,很少为自己的家庭考虑或做过些什么,更遽论为社会,为他人做些什么了.在国内的这种惟有分数高的氛围中,成绩决定了一切,成绩主宰了一切,成绩绑架了一切.


因为心疼他们的辛苦,俺们自觉不自觉地忽视了他们承担家庭责任的意愿;因为欠缺社会环境的支持,他们也很少有机会得以学习承担社会责任.因为长期施以援手之故,俺们的孩子对自己都不是很负责任,对家长的依赖心理严重.


鉴于此,俺希望能够带着孩子去到学业负担轻一些的国度,让他有限的时间和精力,多放在培养自己对社会,家庭,工作,个人和他人的责任感之上.这也构成俺选择移民的理由.
超级认同,至少不用担心儿子找父母要钱买房娶媳妇了。:wdb17:

linzjijie : 2010-01-09#827
回复: 偶搜集到的一些关于加拿大高中教育的资料 与大家分享

握手握手 :wdb6::wdb6::wdb6:
俺也准备让儿子上A-LEVEL.
他现在初三, 高一就准备读A-LEVEL啦.
谢谢!A-LEVEL是什么意思呀?上哪学呀?抱歉,问题比较小儿科:wdb7:

满庭芳09行 : 2010-01-09#828
回复: 加拿大教育情况有用素材收集帖子--梁溪香榭整理

谢梁溪香榭,谢守法:

孩子今年初三,希望能早日去加读高中。

十二少 : 2010-01-09#829
回复: 加拿大教育情况有用素材收集帖子--梁溪香榭整理

好贴,记号一下

梁溪香榭 : 2010-01-09#830
回复: 加拿大教育情况有用素材收集帖子--梁溪香榭整理

谢谢!A-LEVEL是什么意思呀?上哪学呀?抱歉,问题比较小儿科:wdb7:

关于A-LEVEL, 这个帖子里有比较详细的介绍。
嫩也可以谷歌一下,网上太多了。
俺儿子现在高一,正在读A-LEVEL,很不错。推荐推荐。

谢梁溪香榭,谢守法:

孩子今年初三,希望能早日去加读高中。

不谢不谢
祝嫩顺利成行。

好贴,记号一下

谢谢鼓励

linzjijie : 2010-01-10#831
回复: 偶搜集到的一些关于加拿大高中教育的资料 与大家分享

推荐一个学习英语的网站.
www.usalearns.org
很不错哦.
谢谢梁妹妹,每看一段都有收获,真是辛苦了,加声望:wdb19::wdb17:

tudouw62 : 2010-01-10#832
回复: 加拿大教育情况有用素材收集帖子--梁溪香榭整理

请问:国内取得的音乐考级证书,在加拿大对孩子有帮助吗?

梁溪香榭 : 2010-01-10#833
回复: 加拿大教育情况有用素材收集帖子--梁溪香榭整理

请问:国内取得的音乐考级证书,在加拿大对孩子有帮助吗?

这个不知道呢
抱歉

rwx1378 : 2010-01-10#834
回复: 加拿大教育情况有用素材收集帖子--梁溪香榭整理

请问:国内取得的音乐考级证书,在加拿大对孩子有帮助吗?
可以考英皇考级,加拿大认可的。

rwx1378 : 2010-01-10#835
回复: 加拿大教育情况有用素材收集帖子--梁溪香榭整理

溪,想问问读医科如何,本科阶段如何申请,谢。

梁溪香榭 : 2010-01-10#836
回复: 加拿大教育情况有用素材收集帖子--梁溪香榭整理

溪,想问问读医科如何,本科阶段如何申请,谢。

不是很了解
据俺所知,很难很难。
词汇量是个大门槛。即使录取了也很辛苦
A-LEVEL中化学生物应该是必选科目。

rwx1378 : 2010-01-10#837
回复: 加拿大教育情况有用素材收集帖子--梁溪香榭整理

不是很了解
据俺所知,很难很难。
词汇量是个大门槛。即使录取了也很辛苦
A-LEVEL中化学生物应该是必选科目。
谢谢,在考虑读啥专业呢。

梁溪香榭 : 2010-01-10#838
回复: 加拿大教育情况有用素材收集帖子--梁溪香榭整理

谢谢,在考虑读啥专业呢。

看看这个帖子对你的思路是否有帮助
http://forum.iask.ca/showthread.php?t=254062

rwx1378 : 2010-01-10#839
回复: 加拿大教育情况有用素材收集帖子--梁溪香榭整理

国家排名 大学名称 录取通知总数 全球排名

1 英国牛津大学(Oxford) 7 5
2 英国剑桥大学(Cambridge) 5 2

3 英国帝国理工学院 (Imperial) 23 5
4 英国圣安德鲁斯大学(St Andrews) 15 87

5 英国伦敦大学学院(UCL) 27 4

6 英国华威大学( Warwick) 41 58

7 英国伦敦政经学院(LSE) 2 67

8 英国杜伦大学(Durham) 12 103

9 英国艾克斯特大学(Exeter) 8/

10 英国布里斯托大学(Bristol) 18 34

11 英国约克大学(York) 9 70

13 英国巴斯大学(Bath) 25 144

14 英国爱丁堡大学(Edinburgh) 10 20

15 英国南安普顿大学(Southampton) 7 95

15 英国雷切斯特大学(Leicester) 3 196

18 英国谢菲尔德大学(Sheffield) 5 82

20 英国诺丁汉大学(Nottingham) 2 97

22 英国伯明翰大学(Birmingham) 1 66

24 英国曼切斯特大学(Manchester) 3 26

27 英国利兹大学(Leeds) 3 99

28 英国东安吉利亚大学(East Anglia) 2/

40 赫瑞瓦特大学(Heriot-Watt ) 1/

49 英国城市大学 (City) 1 /

美国
18 范德堡大学(Vanderbilt University) 1 140

22 卡内基梅隆大学(Carnegie Mellon University) 1 27

33 纽约大学(New York University) 1 52

39 伊利诺伊大学-香槟分校(U of Illinois) 2 63

LAC 18 史密斯学院(Smith College)

rwx1378 : 2010-01-10#840
回复: 加拿大教育情况有用素材收集帖子--梁溪香榭整理

以上是1月8日前的录取情况

Luck Li : 2010-01-11#841
回复: 加拿大教育情况有用素材收集帖子--梁溪香榭整理

溪溪,看我的新头像,呵呵!

梁溪香榭 : 2010-01-11#842
回复: 加拿大教育情况有用素材收集帖子--梁溪香榭整理

溪溪,看我的新头像,呵呵!

哈哈
不敢跟嫩住对门哦!:wdb6:

linzjijie : 2010-01-12#843
回复: 加拿大教育情况有用素材收集帖子--梁溪香榭整理

哈哈. 有领导艺术. :wdb17:
那行吧, 就这儿一亩三分地儿啦.
请领导和同学一起着力耕耘哈
:wdb20::wdb6:
又来学习了,先报个到!
加声望会了,怎么送花呀?

梁溪香榭 : 2010-01-12#844
回复: 加拿大教育情况有用素材收集帖子--梁溪香榭整理

又来学习了,先报个到!
加声望会了,怎么送花呀?

在首贴下方有个Thanks按钮,点击就可以给楼主送花。
也只能给楼主送花。

Luck Li : 2010-01-12#845
回复: 加拿大教育情况有用素材收集帖子--梁溪香榭整理

哈哈
不敢跟嫩住对门哦!:wdb6:

这是为什么呢?:wdb2::wdb6:

梁溪香榭 : 2010-01-12#846
回复: 加拿大教育情况有用素材收集帖子--梁溪香榭整理

这是为什么呢?:wdb2::wdb6:

呵呵,嫩那耳朵真巨可爱。
原本怕怕,现在爱爱 :wdb6:

linzjijie : 2010-01-17#847
回复: 加拿大教育情况有用素材收集帖子--梁溪香榭整理

最近每天都有学习,真是受益匪浅!:wdb17::wdb6:

linzjijie : 2010-01-17#848
回复: 加拿大教育情况有用素材收集帖子--梁溪香榭整理

梁溪你好!
已经会送花了,给楼主。
可是为什么不能给你加声望了呢?电脑说在再次给你加声望前需要给周围的人加声望,我也给另一个人加了,可是还是给你加不了,好奇怪呀?

梁溪香榭 : 2010-01-17#849
回复: 加拿大教育情况有用素材收集帖子--梁溪香榭整理

梁溪你好!
已经会送花了,给楼主。
可是为什么不能给你加声望了呢?电脑说在再次给你加声望前需要给周围的人加声望,我也给另一个人加了,可是还是给你加不了,好奇怪呀?

呵呵, 谢谢啦。
要另外加满10个人才可以再给俺加呢。

linzjijie : 2010-01-17#850
回复: 加拿大教育情况有用素材收集帖子--梁溪香榭整理

呵呵, 谢谢啦。
要另外加满10个人才可以再给俺加呢。
梁溪,你好!我儿子是94年1月份的,现在也上高一,好像和你儿子一般大啊?我们在天津,好像没有A-LEVEL学校。你们在北京吗?上的A-LEVEL是全日制的吗?

Luck Li : 2010-01-17#851
回复: 加拿大教育情况有用素材收集帖子--梁溪香榭整理

我在北京,朋友孩子上A-LEVEL是全日制并且高中同步住校。具体的她看见你的帖子一定会回答吧,呵呵!

梁溪香榭 : 2010-01-17#852
回复: 加拿大教育情况有用素材收集帖子--梁溪香榭整理

梁溪,你好!我儿子是94年1月份的,现在也上高一,好像和你儿子一般大啊?我们在天津,好像没有A-LEVEL学校。你们在北京吗?上的A-LEVEL是全日制的吗?

这么巧啊。俺儿子也是94年1月的。 呵呵
A-LEVEL是英国高中教育体系,当然是全日制的。就像英国孩子上高中一样的课程,课本。老师也是经剑桥认可的。
我不在北京。儿子的学校是本市最好的高中。A-LEVEL是他们学校为适应形势开办的。目前看前2届的成绩相当炫。
可以住校也可以不住。俺儿子没有住校。

梁溪香榭 : 2010-01-17#853
回复: 加拿大教育情况有用素材收集帖子--梁溪香榭整理

我在北京,朋友孩子上A-LEVEL是全日制并且高中同步住校。具体的她看见你的帖子一定会回答吧,呵呵!

嗯!抱抱Luck!

Luck Li : 2010-01-18#854
回复: 加拿大教育情况有用素材收集帖子--梁溪香榭整理

嗯!抱抱Luck!

一定回抱溪溪!:wdb6:

linzjijie : 2010-01-19#855
回复: 加拿大教育情况有用素材收集帖子--梁溪香榭整理

这么巧啊。俺儿子也是94年1月的。 呵呵
A-LEVEL是英国高中教育体系,当然是全日制的。就像英国孩子上高中一样的课程,课本。老师也是经剑桥认可的。
我不在北京。儿子的学校是本市最好的高中。A-LEVEL是他们学校为适应形势开办的。目前看前2届的成绩相当炫。
可以住校也可以不住。俺儿子没有住校。
梁溪,你好!
真难得碰上一边大的孩子,:wdb6:
可惜天津没有A-LEVEL:wdb14:不知怎么办好了:wdb7:
就算撺掇天津最好的中学开设A-LEVEL课也来不及了:wdb7:
有什么建议吗?

梁溪香榭 : 2010-01-21#856
回复: 加拿大教育情况有用素材收集帖子--梁溪香榭整理

其实A-LEVEL也不是说就适合所有准备去加拿大的孩子嘛。
我儿子班里就有不少孩子很不适应呢。
当然也可以去北京住校读,但我一直认为这么大的孩子还是带在身边的好。身体正处于发育期,也是人生观价值观发育期。放出去万一出问题,这个问题会很大哦。

linzjijie : 2010-01-22#857
回复: 加拿大教育情况有用素材收集帖子--梁溪香榭整理

其实A-LEVEL也不是说就适合所有准备去加拿大的孩子嘛。
我儿子班里就有不少孩子很不适应呢。
当然也可以去北京住校读,但我一直认为这么大的孩子还是带在身边的好。身体正处于发育期,也是人生观价值观发育期。放出去万一出问题,这个问题会很大哦。
你说的有道理,我争取明年1月或者9月去,那时就高二中或高二毕业了,宁可让他再读一年11年级了。

linzjijie : 2010-01-22#858
回复: 加拿大教育情况有用素材收集帖子--梁溪香榭整理

梁溪,你们大约什么时候能去?

梁溪香榭 : 2010-01-22#859
回复: 加拿大教育情况有用素材收集帖子--梁溪香榭整理

梁溪,你们大约什么时候能去?

唉。。。俺也想知道啊。 :wdb23::wdb23::wdb23:

Luck Li : 2010-01-24#860
回复: jenniferlee的精彩发言

超级认同,至少不用担心儿子找父母要钱买房娶媳妇了。:wdb17:
同意,哈哈!

cindy988 : 2010-01-25#861
回复: 加拿大教育情况有用素材收集帖子--梁溪香榭整理

守法领导/梁溪香榭/各位同学
您们好!
能否提供NB省SAINT JOHN公立高中学校资料参考,谢谢!

jessie_f : 2010-01-26#862
回复: 加拿大教育情况有用素材收集帖子--梁溪香榭整理

这么巧啊。俺儿子也是94年1月的。 呵呵
A-LEVEL是英国高中教育体系,当然是全日制的。就像英国孩子上高中一样的课程,课本。老师也是经剑桥认可的。
我不在北京。儿子的学校是本市最好的高中。A-LEVEL是他们学校为适应形势开办的。目前看前2届的成绩相当炫。
可以住校也可以不住。俺儿子没有住校。


哈哈,梁溪,我们有相似啊,不过我女儿是93年1月,所以是高二了,当时中考就考进苏州最好的中学的,后来读的他们办的A-LEVEL,高一一结束就考了雅思了,本来他们老师说让他们先试水一下体验体验的,结果就考了7.5总分,阅读8.5,听力7.5,口语7,最低单科是写作6.5,其实她是没考好的,出来自己郁闷得很。说这些不是自夸什么,而是想说我们的选择是很正确滴!:wdb20::wdb6:不过小丫头本来也很喜欢英语的,等应付完5月的全球统考和大学申请后,打算明年高三找点时间摸摸法语,培养些感觉吧!

jessie_f : 2010-01-26#863
回复: 加拿大教育情况有用素材收集帖子--梁溪香榭整理

其实最好的我认为是这种学习方式对孩子适应国外教育更好,再说咱家闺女本来就是好动贪玩兴趣广泛,所以一早就给她定下这样的求学规划,免得在国内高中把人给读“死”了!

梁溪香榭 : 2010-01-26#864
回复: 加拿大教育情况有用素材收集帖子--梁溪香榭整理

哈哈,梁溪,我们有相似啊,不过我女儿是93年1月,所以是高二了,当时中考就考进苏州最好的中学的,后来读的他们办的A-LEVEL,高一一结束就考了雅思了,本来他们老师说让他们先试水一下体验体验的,结果就考了7.5总分,阅读8.5,听力7.5,口语7,最低单科是写作6.5,其实她是没考好的,出来自己郁闷得很。说这些不是自夸什么,而是想说我们的选择是很正确滴!:wdb20::wdb6:不过小丫头本来也很喜欢英语的,等应付完5月的全球统考和大学申请后,打算明年高三找点时间摸摸法语,培养些感觉吧!

呵呵。:wdb6::wdb6::wdb6:
俺儿子准备高一暑假考雅思。男孩子整天就知道瞎玩,一点不知道着急。:wdb23::wdb23:
老师说他考个7.0应该没问题。俺也希望他能冲个7.5.
嫩闺女着实厉害啊。阅读8.5. :wdb17::wdb17:
俺儿子就是不高兴背单词。不过这次期末考试成绩不错,仍然保持第一。:wdb6::wdb20:
同意你的意见,我们的选择真的是很对很正确。

Luck Li : 2010-01-26#865
回复: 加拿大教育情况有用素材收集帖子--梁溪香榭整理

都是有出息的好孩子,呵呵!

梁溪香榭 : 2010-01-26#866
回复: 加拿大教育情况有用素材收集帖子--梁溪香榭整理

其实最好的我认为是这种学习方式对孩子适应国外教育更好,再说咱家闺女本来就是好动贪玩兴趣广泛,所以一早就给她定下这样的求学规划,免得在国内高中把人给读“死”了!

是的是的。
儿子读了一个学期的A-LEVEL,俺对国外的教育体系也有了更深的理解。的确科学得多。

梁溪香榭 : 2010-01-26#867
回复: 加拿大教育情况有用素材收集帖子--梁溪香榭整理

都是有出息的好孩子,呵呵!

看到LUCK 就 :wdb6::wdb6::wdb6:

jessie_f : 2010-01-26#868
回复: 加拿大教育情况有用素材收集帖子--梁溪香榭整理

呵呵。:wdb6::wdb6::wdb6:
俺儿子准备高一暑假考雅思。男孩子整天就知道瞎玩,一点不知道着急。:wdb23::wdb23:
老师说他考个7.0应该没问题。俺也希望他能冲个7.5.
嫩闺女着实厉害啊。阅读8.5. :wdb17::wdb17:
俺儿子就是不高兴背单词。不过这次期末考试成绩不错,仍然保持第一。:wdb6::wdb20:
同意你的意见,我们的选择真的是很对很正确。



不背单词没关系的,呵呵,我们那丫头也是的,曾经一本正经试过一次,还没看两页单词书,就扔一边去了!不过她喜欢看原版小说倒是真的,一般还不查词典,连蒙带猜的看,说这样更好,其实是懒,我也不戳穿她,嘿嘿,确实看的多了词汇也是会累积起来的,还是:wdb10::wdb10: 高一过了暑假考是最佳安排,你家儿子看来自有分寸,一定也:wdb9::wdb9::wdb9:

jessie_f : 2010-01-26#869
回复: 加拿大教育情况有用素材收集帖子--梁溪香榭整理

都是有出息的好孩子,呵呵!


谢谢,哈哈,我们在这里忙活半天不就是希望给他们更好的平台吗,但也得他们一起使劲,孩子将来才会更有出息吧,:wdb6::wdb6::wdb19::wdb19:那俺们也就放心了啊,现在还都需要更努力才行:wdb23::wdb23:

紫钻石妈妈 : 2010-01-29#870
回复: 加拿大教育情况有用素材收集帖子--梁溪香榭整理

梁溪,您好
麻烦您再给一下您推荐的那个学英语的网站好吗
我找不到了
辛苦一下,谢谢

梁溪香榭 : 2010-01-29#871
回复: 加拿大教育情况有用素材收集帖子--梁溪香榭整理

梁溪,您好
麻烦您再给一下您推荐的那个学英语的网站好吗
我找不到了
辛苦一下,谢谢

是不是这个?
http://www.speak2me.cn

紫钻石妈妈 : 2010-01-31#872
回复: 加拿大教育情况有用素材收集帖子--梁溪香榭整理


不是好像是usa什么的真是添麻烦了再次感谢

Luck Li : 2010-01-31#873
回复: 加拿大教育情况有用素材收集帖子--梁溪香榭整理

不是好像是usa什么的真是添麻烦了再次感谢

www.usalearns.org

梁溪香榭 : 2010-01-31#874
回复: 加拿大教育情况有用素材收集帖子--梁溪香榭整理


嗯。 谢谢LUCK!:wdb19::wdb19:

Luck Li : 2010-01-31#875
回复: 加拿大教育情况有用素材收集帖子--梁溪香榭整理

嗯。 谢谢LUCK!:wdb19::wdb19:

不客气,说实话我也是更喜欢嫩过去推荐俺的这个。:wdb19:

紫钻石妈妈 : 2010-01-31#876
回复: 加拿大教育情况有用素材收集帖子--梁溪香榭整理

再次感谢!luck和梁溪!:wdb19:

Luck Li : 2010-01-31#877
回复: 加拿大教育情况有用素材收集帖子--梁溪香榭整理

看到LUCK 就 :wdb6::wdb6::wdb6:

同俺在一个大厦里上班的同事和朋友,会经常在电梯或雇员餐厅里见到俺,也总是这么说,俺不是个很闹腾了人呀,平平静静的就能给大家带来快乐,真是很高兴,呵呵!:wdb6::wdb19::wdb6:

Luck Li : 2010-01-31#878
回复: 加拿大教育情况有用素材收集帖子--梁溪香榭整理

俺也谢谢溪溪,喜欢溪溪。:wdb19::wdb19:

Luck Li : 2010-01-31#879
回复: 加拿大教育情况有用素材收集帖子--梁溪香榭整理

再次感谢!luck和梁溪!:wdb19:

别客气,给新移友+SW,新春快乐!:wdb6:

诚心请教 : 2010-01-31#880
回复: 加拿大教育情况有用素材收集帖子--梁溪香榭整理

good job, moms

rwx1378 : 2010-02-01#881
回复: 加拿大教育情况有用素材收集帖子--梁溪香榭整理

哈哈,梁溪,我们有相似啊,不过我女儿是93年1月,所以是高二了,当时中考就考进苏州最好的中学的,后来读的他们办的A-LEVEL,高一一结束就考了雅思了,本来他们老师说让他们先试水一下体验体验的,结果就考了7.5总分,阅读8.5,听力7.5,口语7,最低单科是写作6.5,其实她是没考好的,出来自己郁闷得很。说这些不是自夸什么,而是想说我们的选择是很正确滴!:wdb20::wdb6:不过小丫头本来也很喜欢英语的,等应付完5月的全球统考和大学申请后,打算明年高三找点时间摸摸法语,培养些感觉吧!
高二是不是AS,今年5-6月份考CIE?如果是,那俺们的孩子气是同期的。不过俺家的不如你家的英语考得好,托福一考只有88分,很受打击的样子。

wayne_z : 2010-02-01#882
回复: 加拿大教育情况有用素材收集帖子--梁溪香榭整理

先收藏,慢慢消化。

九寨天堂 : 2010-02-01#883
回复: 加拿大教育情况有用素材收集帖子--梁溪香榭整理

好帖,从现在开始慢慢看。:wdb10:俺儿子93年的,今年去澳大利亚读高中了,现在正读语言。我们正在办加移民,估计三年后可办妥,有点绕哦!:wdb2:移民后计划让儿子在加读大学,读这个帖子可让自己更明白一些。我现在是稀里湖涂,只管把孩子送出去得了!:wdb5:给楼主送花花喽!:wdb6:

kuke : 2010-02-03#884
回复: 偶搜集到的一些关于加拿大高中教育的资料 与大家分享

读高二的孩子到加拿大需准备什么

孩子尤为重要的准备:英文,英文,还是英文!
从去年10月开始,在加拿大境内没有受满4年义务教育的移民孩子上大学都要考托,而且是改了形式的考托(以前重读写,去年10月开始重听说)这次改进的托福形式考试对中国大陆孩子很不利。250分为合格。
如果是来加拿大读高三,对孩子来说还是很具挑战性的。因为在加拿大孩子如果快到18岁了(或19岁?)还没有拿到所有高中毕业的学分的话,教育局会让孩子到成人高中继续修课,直到所有学分拿到,完成高中毕业的条件才可以申请大学,另外还要加试托福(或雅丝)。这样就有可能会比国内的同学晚几年上大学。
安省的高中可以读到21岁,直到修完相应学分和必修课为止,很多高中过来的孩子都是在高中多读一两年,为的是适应英文写作,为将来上大学打好基础,也免得上大学太辛苦。所以孩子过来进入高中后,可以根据孩子的情况决定何时申请大学。
:wdb5:计划儿子(15岁)6月中考完了8月长登温哥华上高一,看样子得改变计划了:wdb14:。请问:看见有类似情况的移友的建议是让孩子去安省中小城镇就读高一,合理吗?教学水平能有保障吗?

Holmes : 2010-02-15#885
回复: 加拿大教育情况有用素材收集帖子--梁溪香榭整理

顶!

yijianxiao : 2010-02-15#886
回复: 加拿大教育情况有用素材收集帖子--梁溪香榭整理

带孩子去加国读书,在中国要准备哪些证明?

may2626 : 2010-03-26#887
回复: 加拿大教育情况有用素材收集帖子--梁溪香榭整理

如此好帖,发现太迟了,顶

wabjtam.6789 : 2010-03-30#888
回复: 加拿大教育情况有用素材收集帖子--梁溪香榭整理

谢谢LZ ,辛苦您了,真是太太有用了,收藏了,慢慢看。向您致敬!:wdb17::wdb10:

Catflyfly : 2010-03-31#889
回复: 加拿大教育情况有用素材收集帖子--梁溪香榭整理

由于我们办理移民比较晚,如果顺利办下来的话,小孩也已经读完高二,搞不好可能已经进入高三了:wdb24:。这时候要去温哥华,该怎样申请学校呢?好迷茫呀!

梁溪香榭 : 2010-03-31#890
回复: 加拿大教育情况有用素材收集帖子--梁溪香榭整理

由于我们办理移民比较晚,如果顺利办下来的话,小孩也已经读完高二,搞不好可能已经进入高三了:wdb24:。这时候要去温哥华,该怎样申请学校呢?好迷茫呀!

38页, 750楼,有比较详细的解释。
另外,一楼有目录哦。

Catflyfly : 2010-03-31#891
回复: 加拿大教育情况有用素材收集帖子--梁溪香榭整理

38页, 750楼,有比较详细的解释。
另外,一楼有目录哦。



感谢LZ的指引:wdb10::wdb17::wdb19:

wfeng363 : 2010-04-05#892
回复: 加拿大教育情况有用素材收集帖子--梁溪香榭整理

多谢,太有用了

sure : 2010-04-07#893
回复: 子女教育信息素材收集帖

梁溪,你好!请教一下:如果孩子大学在加上,国内高中阶段学文和学理哪个好?不知学文后到国外是否知识对接不上?

梁溪香榭 : 2010-04-07#894
回复: 子女教育信息素材收集帖

梁溪,你好!请教一下:如果孩子大学在加上,国内高中阶段学文和学理哪个好?不知学文后到国外是否知识对接不上?

我认为完全取决于孩子的天分。如果理科强而非要学文,或者文科强非要去攻克理科,都可谓是杯具哦。

2010115 : 2010-04-10#895
回复: 加拿大教育情况有用素材收集帖子--梁溪香榭整理

渥太华念高中,法语是不是必修课?

sure : 2010-04-11#896
回复: 子女教育信息素材收集帖

我认为完全取决于孩子的天分。如果理科强而非要学文,或者文科强非要去攻克理科,都可谓是杯具哦。

谢谢梁溪,但我想问的问题是,国内高中学的文科知识如历史地理什么的,是否到国外后基本没多大用处,另外,在国外学文科是不是不好找工作?

梁溪香榭 : 2010-04-11#897
回复: 子女教育信息素材收集帖

谢谢梁溪,但我想问的问题是,国内高中学的文科知识如历史地理什么的,是否到国外后基本没多大用处,另外,在国外学文科是不是不好找工作?

理解嫩滴意思了。
这个俺也说不好。其实历史地理都是非常有用的科目。但国内的教育方式实在是扼杀孩子的思想。
至于国外学文科是否不好找工作,建议嫩去温哥华版块,多伦多版块问问吧。 我个人并不如此看。至少我知道国外有不少大的投资公司是很注重有哲学背景的人才的。

sure : 2010-04-18#898
回复: 子女教育信息素材收集帖

理解嫩滴意思了。
这个俺也说不好。其实历史地理都是非常有用的科目。但国内的教育方式实在是扼杀孩子的思想。
至于国外学文科是否不好找工作,建议嫩去温哥华版块,多伦多版块问问吧。 我个人并不如此看。至少我知道国外有不少大的投资公司是很注重有哲学背景的人才的。

谢谢梁溪!

期待远方 : 2010-04-20#899
回复: 加拿大教育情况有用素材收集帖子--梁溪香榭整理

:wdb19香榭!:wdb10邻居!

期待远方 : 2010-04-20#900
回复: 加拿大教育情况有用素材收集帖子--梁溪香榭整理

我的表情怎么不显示呢?

rong30099 : 2010-04-27#901
回复: 加拿大教育情况有用素材收集帖子--梁溪香榭整理

好贴!先收藏,再天天学,个人学完孩子学,孩子学完他爹学!

cnd : 2010-05-02#902
回复: 加拿大教育情况有用素材收集帖子--梁溪香榭整理

收藏!

cindy988 : 2010-06-19#903
回复: 加拿大教育情况有用素材收集帖子--梁溪香榭整理

好贴收藏:wdb19:,非常有用,花花分分同时送上致谢!:wdb17:

libaoming : 2010-07-14#904
回复: 子女教育信息素材收集帖

嘎嘎

这个帖子也算历经了劫难了,丢了又重新找回来了。

主要整理的是关于教育的一些素材

从147楼开始,主要就是梁溪香榭搜集的一些的,推荐推荐!

推荐本书--从普通女孩到银行家,http://forum.iask.ca/showthread.php?t=233637

目录

tony 3213, PEI地区教育情况 Page 1, 5楼
tiantianxingchen 高中课程简介 Page 1, 6-9楼
守法公民 初三学生多伦多求学记 Page1, 18-22楼
守法公民 安省新移民高中指南 Page 2, 24-30楼
守法公民 安省新移民小学指南 Page 2, 34-36楼
守法公民 高中家长如是说 Page 3, 44-49楼
雨中冷百合 大多伦多地区高中排名 Page 3, 57楼, 59楼
守法公民 温哥华十佳公立中学 Page 4, 65楼
一缕阳光 菲沙关于BC省中学排名及相关简介 Page 4, 69楼
守法公民 部分大学对是否需要雅思或托福成绩的年限规定 Page 6, 101楼
守法公民 教育篇 Page 6, 113-121楼
梁溪香榭 高中留学须谨慎 Page 8, 149-152楼
梁溪香榭 加拿大高中教育感悟 Page 8, 153-155楼
梁溪香榭 根据成绩选择加拿大大学 Page 8, 159楼
梁溪香榭 如何选择最适合的加拿大大学 Page 9, 161楼
梁溪香榭 读高二的孩子去加拿大需要准备什么 Page 9, 164-165楼
梁溪香榭 高中英语课程介绍 Page 9, 166-169楼
梁溪香榭 高中英语课程介绍 Page 10, 188-191楼
梁溪香榭 多伦多著名私立学校收费一览 Page 10, 192楼
梁溪香榭 孩子的英语过关了吗? Page10, 194楼
梁溪香榭 关于A-LEVEL课程 Page 11, 217-228楼
shieley6811 关于A-LEVEL课程及学制 Page 13, 256-257楼
梁溪香榭 2007年安省高中拍名 Page 17, 322楼
朱三雀 安省, BC省, AB省高中拍名, Page 17, 329楼
响铃当当 加拿大教育优势 Page18, 347楼
梁溪香榭 A-LEVEL与IB课程的比较 Page 18, 358楼
朱三雀 中西学生的差异 Page 19, 364楼
梁溪香榭 大多伦多地区IB高中简介 Page 19, 372-375楼
梁溪香榭 IB课程与其他特殊教育课程的比较 Page 19, 376楼
梁溪香榭 IB与AP的比较, Page 19, 377楼
梁溪香榭 美国相关研究机构关于IB与AP的选择比较 Page 20, 387-391楼
梁溪香榭 群星灿烂不如一轮皓月当空-大学录取篇 Page 21, 409-413楼
梁溪香榭 在教育子女的过程中教育自己 Page 22, 422-424楼
梁溪香榭 关于美国大学 Page 22, 429-432楼
梁溪香榭 多伦多高中阶段特殊教育程序 Page 23, 445-446楼
梁溪香榭 加拿大高中生学科竞赛 Page 23, 447-448楼
梁溪香榭 美国大学 Page 24, 466楼
梁溪香榭 如何选择适合自己孩子的中小学 Page 25, 487-489楼
梁溪香榭 论孩子成长的内环境 Page 25, 490楼
梁溪香榭 犹太人的教育风格:朴实无华 讲求实际 Page 25, 497楼
梁溪香榭 培养子女的参政意识 Page 25, 498楼
梁溪香榭 美国大学点评 Page 25, 499-529楼
梁溪香榭 加拿大大学点评(商科) Page 27, 533-535楼
梁溪香榭 加拿大大学点评(理工科) Page 27, 547-549楼
梁溪香榭 安省高中教育咨询问答 Page 30, 584楼
梁溪香榭 毕业之际谈我的大学:滑铁卢大学 Page 30, 586-592楼
梁溪香榭 我在麦克马斯特 Page 30, 597楼
梁溪香榭 2005年度全球十大商学院 Page 30, 598楼
梁溪香榭 世界大学金融专业的排名 Page 30, 599楼
梁溪香榭 在加拿大IVEY商学院读MBA的日子 Page 31, 608楼
梁溪香榭 加拿大几个名牌大学比较之我见 Page 31, 616楼
梁溪香榭 加拿大的商学院MBA排名分析 Page 31, 618楼
梁溪香榭 加拿大本科申请指南 page 31-32, 620-624楼
yhyxll申请UBC请问需要什么条件,难度大否 Page 32, 633-634楼
willam131 关于A-LEVEL选课 page 34, 680楼
梁溪香榭 公立学校vs.私立学校
page 36, 713-716楼
梁溪香榭 路在自己脚下 --- 一位小留学生的自述 page 36, 717楼
惴惴不安 移民学生多“断层” 上公校还是私校难倒华人家长 page 36, 722楼
梁溪香榭 加拿大中学教育 page 38, 745楼
梁溪香榭 安大略省中学教育制度 page 38, 747楼
梁溪香榭 加拿大高中选课制 page 38, 748楼
梁溪香榭 高中毕业生该如何选择加拿大大学 page 38, 749楼
梁溪香榭 高中生如何入读加拿大大学?page 38, 750楼
梁溪香榭 加拿大大学怎样录取新生page 38, 751楼
梁溪香榭 加拿大大学对A-LEVEL的录取要求 page 38, 753-755楼
哇塞,啥也不说了,好贴,谢谢了。

libaoming : 2010-07-14#905
回复: 子女教育信息素材收集帖

守法先生,
我的女儿18岁,高二,我们在等ME,预计09年7、8月才会PL。女儿高中会考结束,现在就可以拿到高中毕业证了。以后的高三学习全是为了明年的高考。
请问:
1、女儿还继续在校学习,迎接明年的高考,拿到高中毕业证和高考成绩,然后去加申请大学?
2、让女儿现在就去新东方学英语,放弃高三的在校学习,去过英语关,明年到加后继续读12年级(高三)?
以上那种方法更好点呢?
还有:
3、加国新移民入12年级,在年龄和国内的学习上有什么限制吗?比如,在国内高二已算是高中毕业了,加国还让继续上12年级吗?还有孩子明年就19岁了,年龄上还允许上高中吗?
有人说,孩子过了18岁,到啦那边只能上成人高中,成人高中收费额高吗?与普通高中相比,对孩子有什么不利吗?
谢谢
也欢迎有经验的其他同学帮忙出出主意。
和我的情况很类似,我估计我申请成功女儿也要上高二了,怎么办呢?

libaoming : 2010-07-14#906
回复: 子女教育信息素材收集帖

嘎嘎

这个帖子也算历经了劫难了,丢了又重新找回来了。

主要整理的是关于教育的一些素材

从147楼开始,主要就是梁溪香榭搜集的一些的,推荐推荐!

推荐本书--从普通女孩到银行家,http://forum.iask.ca/showthread.php?t=233637

目录

tony 3213, PEI地区教育情况 Page 1, 5楼
tiantianxingchen 高中课程简介 Page 1, 6-9楼
守法公民 初三学生多伦多求学记 Page1, 18-22楼
守法公民 安省新移民高中指南 Page 2, 24-30楼
守法公民 安省新移民小学指南 Page 2, 34-36楼
守法公民 高中家长如是说 Page 3, 44-49楼
雨中冷百合 大多伦多地区高中排名 Page 3, 57楼, 59楼
守法公民 温哥华十佳公立中学 Page 4, 65楼
一缕阳光 菲沙关于BC省中学排名及相关简介 Page 4, 69楼
守法公民 部分大学对是否需要雅思或托福成绩的年限规定 Page 6, 101楼
守法公民 教育篇 Page 6, 113-121楼
梁溪香榭 高中留学须谨慎 Page 8, 149-152楼
梁溪香榭 加拿大高中教育感悟 Page 8, 153-155楼
梁溪香榭 根据成绩选择加拿大大学 Page 8, 159楼
梁溪香榭 如何选择最适合的加拿大大学 Page 9, 161楼
梁溪香榭 读高二的孩子去加拿大需要准备什么 Page 9, 164-165楼
梁溪香榭 高中英语课程介绍 Page 9, 166-169楼
梁溪香榭 高中英语课程介绍 Page 10, 188-191楼
梁溪香榭 多伦多著名私立学校收费一览 Page 10, 192楼
梁溪香榭 孩子的英语过关了吗? Page10, 194楼
梁溪香榭 关于A-LEVEL课程 Page 11, 217-228楼
shieley6811 关于A-LEVEL课程及学制 Page 13, 256-257楼
梁溪香榭 2007年安省高中拍名 Page 17, 322楼
朱三雀 安省, BC省, AB省高中拍名, Page 17, 329楼
响铃当当 加拿大教育优势 Page18, 347楼
梁溪香榭 A-LEVEL与IB课程的比较 Page 18, 358楼
朱三雀 中西学生的差异 Page 19, 364楼
梁溪香榭 大多伦多地区IB高中简介 Page 19, 372-375楼
梁溪香榭 IB课程与其他特殊教育课程的比较 Page 19, 376楼
梁溪香榭 IB与AP的比较, Page 19, 377楼
梁溪香榭 美国相关研究机构关于IB与AP的选择比较 Page 20, 387-391楼
梁溪香榭 群星灿烂不如一轮皓月当空-大学录取篇 Page 21, 409-413楼
梁溪香榭 在教育子女的过程中教育自己 Page 22, 422-424楼
梁溪香榭 关于美国大学 Page 22, 429-432楼
梁溪香榭 多伦多高中阶段特殊教育程序 Page 23, 445-446楼
梁溪香榭 加拿大高中生学科竞赛 Page 23, 447-448楼
梁溪香榭 美国大学 Page 24, 466楼
梁溪香榭 如何选择适合自己孩子的中小学 Page 25, 487-489楼
梁溪香榭 论孩子成长的内环境 Page 25, 490楼
梁溪香榭 犹太人的教育风格:朴实无华 讲求实际 Page 25, 497楼
梁溪香榭 培养子女的参政意识 Page 25, 498楼
梁溪香榭 美国大学点评 Page 25, 499-529楼
梁溪香榭 加拿大大学点评(商科) Page 27, 533-535楼
梁溪香榭 加拿大大学点评(理工科) Page 27, 547-549楼
梁溪香榭 安省高中教育咨询问答 Page 30, 584楼
梁溪香榭 毕业之际谈我的大学:滑铁卢大学 Page 30, 586-592楼
梁溪香榭 我在麦克马斯特 Page 30, 597楼
梁溪香榭 2005年度全球十大商学院 Page 30, 598楼
梁溪香榭 世界大学金融专业的排名 Page 30, 599楼
梁溪香榭 在加拿大IVEY商学院读MBA的日子 Page 31, 608楼
梁溪香榭 加拿大几个名牌大学比较之我见 Page 31, 616楼
梁溪香榭 加拿大的商学院MBA排名分析 Page 31, 618楼
梁溪香榭 加拿大本科申请指南 page 31-32, 620-624楼
yhyxll申请UBC请问需要什么条件,难度大否 Page 32, 633-634楼
willam131 关于A-LEVEL选课 page 34, 680楼
梁溪香榭 公立学校vs.私立学校
page 36, 713-716楼
梁溪香榭 路在自己脚下 --- 一位小留学生的自述 page 36, 717楼
惴惴不安 移民学生多“断层” 上公校还是私校难倒华人家长 page 36, 722楼
梁溪香榭 加拿大中学教育 page 38, 745楼
梁溪香榭 安大略省中学教育制度 page 38, 747楼
梁溪香榭 加拿大高中选课制 page 38, 748楼
梁溪香榭 高中毕业生该如何选择加拿大大学 page 38, 749楼
梁溪香榭 高中生如何入读加拿大大学?page 38, 750楼
梁溪香榭 加拿大大学怎样录取新生page 38, 751楼
梁溪香榭 加拿大大学对A-LEVEL的录取要求 page 38, 753-755楼
很好的帖子,请问有没有整理到一起的一个文件,内容太多,不方便看,想打印出来好好的研究,谢谢了。

家园移民 : 2010-07-14#907
回复: 子女教育信息素材收集帖

很好的帖子,请问有没有整理到一起的一个文件,内容太多,不方便看,想打印出来好好的研究,谢谢了。
呵呵
暂时还没有啊

libaoming : 2010-07-14#908
回复: 子女教育信息素材收集帖

呵呵
暂时还没有啊
那我就自己整理,整理好了,有需要的可以找我。

梁溪香榭 : 2010-07-15#909
回复: 子女教育信息素材收集帖

那我就自己整理,整理好了,有需要的可以找我。

多谢多谢
请直接附件贴上来啊

西子湖畔 : 2010-07-15#910
回复: 加拿大教育情况有用素材收集帖子--梁溪香榭整理

好帖!:wdb17:,得好好学习研究一下!:wdb10:

西子湖畔 : 2010-07-15#911
回复: 子女教育信息素材收集帖

多谢多谢
请直接附件贴上来啊


:wdb10::wdb10::wdb10:

wangwususan2 : 2010-07-18#912
回复: 加拿大教育情况有用素材收集帖子--梁溪香榭整理

好贴!楼主辛苦了,谢谢!
如果顺利的话,我孩子过去可能也要到高二.现在她十四岁,准备让她初三上国际学校.

libaoming : 2010-07-23#913
回复: 加拿大教育情况有用素材收集帖子--梁溪香榭整理

好贴!楼主辛苦了,谢谢!
如果顺利的话,我孩子过去可能也要到高二.现在她十四岁,准备让她初三上国际学校.
你家孩子和我家的孩子一样,我们交流一下好吗?

kedeba : 2010-07-23#914
回复: 加拿大教育情况有用素材收集帖子--梁溪香榭整理

梁溪,我们这里新开办了ACT预科班,是高二升高三招生,说是也可以申请加拿大大学,请问你了解吗?是否值得一读?在加拿大是否被认可?谢谢啦。

梁溪香榭 : 2010-08-14#915
回复: 加拿大教育情况有用素材收集帖子--梁溪香榭整理

好久没来这一亩三分地了。

说说A-LEVEL吧。
儿子在A-LEVEL一年了,稍微有一点点汇报哈。

关于A-LEVEL的教学方式,最近和儿子聊天时,儿子说的,汇报如下:
老师会在开始一个新课程阶段前布置一个大作业,无论是数理化还是商科,让孩子们设法解决一个问题,这个问题是与我们的日常生活结合的,其解决方法是孩子们还没有学过的。涉及很多未知领域,甚至是几个科目结合的。

无论用何种手段,可以去网络上找,可以问家长,可以同学间探讨,随便用什么方法,反正就是要交作业。至于作业质量就全在于孩子自己掌握了,可以扎得很深,也可以草草了事。孩子们的作业可谓千差万别,各有千秋。

儿子的心得,
让他们对将要学习的课程有强烈的兴趣感。真的是凭兴趣去学。
这个教学方式很能开发他们的探求心,让孩子学会如何设法结合已知的去探求未知的。
学习有了针对性,知道为何而学。

李先生 : 2010-08-14#916
回复: 加拿大教育情况有用素材收集帖子--梁溪香榭整理

收藏

mayijj : 2010-08-15#917
回复: 加拿大教育情况有用素材收集帖子--梁溪香榭整理

mark

zr483503 : 2010-08-22#918
回复: 加拿大教育情况有用素材收集帖子--梁溪香榭整理

:wdb17::wdb17::wdb17::wdb17::wdb17:

孩子他爹 : 2010-08-24#919
回复: 加拿大教育情况有用素材收集帖子--梁溪香榭整理

孩子上学的好贴,感谢楼主的劳动

梦想的开始 : 2010-08-24#920
回复: 加拿大教育情况有用素材收集帖子--梁溪香榭整理

学习中!:wdb17::wdb17:

梦想的开始 : 2010-08-24#921
回复: 加拿大教育情况有用素材收集帖子--梁溪香榭整理

意外哦,我竟然可以加声望了!梁溪姐姐加油!

sun5169@163.com : 2010-08-24#922
回复: 加拿大教育情况有用素材收集帖子--梁溪香榭整理

很详细,谢谢!

梁溪香榭 : 2010-08-26#923
回复: 加拿大教育情况有用素材收集帖子--梁溪香榭整理

怎么回事啊?:wdb24:
可以加分无限量了?:wdb2:
俺加到手都软了耶 :wdb6::wdb6::wdb6:

yan8816 : 2010-08-30#924
回复: 加拿大教育情况有用素材收集帖子--梁溪香榭整理

感谢 梁溪香榭。这么辛苦,给大家提供了这么多有用的信息。:wdb10::wdb17:SW

tana : 2010-09-05#925
回复: 加拿大教育情况有用素材收集帖子--梁溪香榭整理

好贴收藏!谢谢!

一朵一果 : 2010-09-17#926
回复: 加拿大教育情况有用素材收集帖子--梁溪香榭整理

这个帖子太好了,就需要这方面的知识,守法和梁溪香榭辛苦了,多谢.送花花和声望啦.:wdb17::wdb19::wdb6:

一朵一果 : 2010-09-17#927
回复: 加拿大教育情况有用素材收集帖子--梁溪香榭整理

也给本页的梦想的开始 sun5169@163.com 梁溪香榭 yan8816 tana 同加声望啦:wdb6::wdb20:

梁溪香榭 : 2010-09-19#928
回复: 加拿大教育情况有用素材收集帖子--梁溪香榭整理

2010年加拿大大学重点学科排名

  生态/环境科学专业:

  NO.1 多伦多大学 University of Toronto
  NO.2 卡尔加里大学 University of Calgary
  NO.3 滑铁卢大学 University of Waterloo

  农业科学专业:

  NO.1 英属哥伦比亚大学 Uni. of British Columbia
  NO.2 阿尔伯塔大学 University of Alberta
  NO.3 圭尔夫大学 University of Guelph

  植物与动物学科:

  NO.1 女皇大学 Queen’s University
  NO.2 约克大学 York University
  NO.3 渥太华大学 University Ottawa

  临床医学专业:

  NO.1 麦克马斯特大学 McMaster University
  NO.2 麦吉尔大学 McGill University
  NO.3 英属哥伦比亚大学 Uni. of British Columbia

  神经学专业:

  NO.1 英属哥伦比亚大学 Uni. of British Columbia
  NO.2 戴尔豪西大学 Dalhousie University
  NO.3 拉瓦尔大学 University of Laval

梁溪香榭 : 2010-09-19#929
回复: 加拿大教育情况有用素材收集帖子--梁溪香榭整理

心理学专业:

  NO.1 麦克马斯特大学 McMaster University
  NO.2 滑铁卢大学 University of Waterloo
  NO.3 麦吉尔大学 McGill University

  药学专业:

  NO.1 谢布克大学 University of Sherbrooke
  NO.2 麦克马斯特大学 McMaster University
  NO.3 多伦多大学 University of Toronto

  数学专业:

  NO.1 多伦多大学 University of Toronto
  NO.2 约克大学 York University
  NO.3 滑铁卢大学 University of Waterloo

  物理学专业:

  NO.1 西蒙弗雷泽大学 Simon Fraser University
  NO.2 卡尔顿大学 Carleton University
  NO.3 英属哥伦比亚大学 Uni. of British Columbia

  化学专业:

  NO.1 约克大学 York University
  NO.2 多伦多大学 University of Toronto
  NO.3 阿尔伯塔大学 University of Alberta

  生物学专业:

  NO.1 多伦多大学 University of Toronto
  NO.2 阿尔伯塔大学 University of Alberta
  NO.3 英属哥伦比亚大学 Uni. of British Columbia

  地球科学专业:

  NO.1 约克大学 York University
  NO.2 多伦多大学 University of Toronto
  NO.3 阿尔伯塔大学 University of Alberta

梁溪香榭 : 2010-09-19#930
回复: 加拿大教育情况有用素材收集帖子--梁溪香榭整理

天文学专业:

  NO.1 维多利亚大学 University of Toronto
  NO.2 阿尔伯塔大学 University of Alberta
  NO.3 蒙特利尔大学 University of Montreal

  新闻专业:

  NO.1 麦吉尔大学 McGill University Canada
  NO.2 加拿大多伦多大学 University of Toronto Canada
  NO.3 英属哥伦比亚大学 British Columbia University

  加拿大MBA院校排名:

  NO.1 女王大学 Queen’s University
  NO.2 约克大学 York University
  NO.3 英属哥伦比亚大学 British Columbia University

新手上路165 : 2010-09-25#931
回复: 加拿大教育情况有用素材收集帖子--梁溪香榭整理

好贴

侯妮妮 : 2010-10-04#932
回复: 加拿大教育情况有用素材收集帖子--梁溪香榭整理

请教:孩子留学家长可以办陪读去加国吗?刚递交移民材料。。

轻风 : 2010-10-28#933
回复: 加拿大教育情况有用素材收集帖子--梁溪香榭整理

ding

云之梦 : 2010-11-09#934
回复: 加拿大教育情况有用素材收集帖子--梁溪香榭整理

面试10月当场过的,正式通知一直没来,天天等打款信,到是2个星期忙着帮上初三的儿子找学校,商量来也只有上中加合办的学校,提前准备语言。等!

家园移民 : 2010-11-09#935
回复: 加拿大教育情况有用素材收集帖子--梁溪香榭整理

面试10月当场过的,正式通知一直没来,天天等打款信,到是2个星期忙着帮上初三的儿子找学校,商量来也只有上中加合办的学校,提前准备语言。等!
小恭喜下啊

newxiaobao : 2010-11-16#936
回复: 加拿大教育情况有用素材收集帖子--梁溪香榭整理

这个贴太实用了,我要留着慢慢看,呵呵!

cindy988 : 2010-11-17#937
回复: 加拿大教育情况有用素材收集帖子--梁溪香榭整理

天文学专业:

  NO.1 维多利亚大学 University of Toronto
  NO.2 阿尔伯塔大学 University of Alberta
  NO.3 蒙特利尔大学 University of Montreal

  新闻专业:

  NO.1 麦吉尔大学 McGill University Canada
  NO.2 加拿大多伦多大学 University of Toronto Canada
  NO.3 英属哥伦比亚大学 British Columbia University

  加拿大MBA院校排名:

  NO.1 女王大学 Queen’s University
  NO.2 约克大学 York University
  NO.3 英属哥伦比亚大学 British Columbia University

:wdb19:收藏了,加声望致谢了!

云之梦 : 2010-12-26#938
回复: 加拿大教育情况有用素材收集帖子--梁溪香榭整理

这个帖子真好!

wls1wls : 2011-01-12#939
回复: 子女教育信息素材收集帖子-冲置顶了

家长指南第一章
(PARENT GUIDE ONE) 入学须知(Starting School)
目录
1 1. 中学的种类
2 2. 报名入学
3 3. 评估中心(接待中心)
4 4. 第一年选课
5 5. 承认以前的学分
6 6. 视力和听力健康
7 7. 使用计算机
8 8. 拿签证的国际学生
9 9. 各中学之间的区别
10 10.报读学区之外的学校
11 11.在加拿大之外的高中毕业的学生
12 12.为报道第一天做好准备
13 13.已满十八岁的学生

新移民学生抵达加拿大之后,大多就要立即入学。这份指南描述了孩子入学之初会碰到哪些事情、如何做好准备。
新移民到达加拿大的头几年可能非常艰难。家长忙于找工作、安顿新家。孩子则要结交新朋友,每个人都在适应新环境。如果子女就读高中(9-12 年级,secondary schools),家长还另有一项责任:每年都要协助孩子选择课程。课程种类繁多,选课并不容易,而高中怎么选课,会直接影响学生毕业后的出路。
指南的其它几章会告诉家长,如何协助子女对高中课程作出明智的选择。本指南建议家长和子女,在选科之前一起订出长期的学业计划(an education plan)。有了计划才可以目标明确(读大学、大专、学徒课程还是直接就业)。家长也可以请学校的老师或辅导员(the guidance counsellor)提供协助。
本指南专为新移民介绍安省高中制度,一共有六章,本章是其中之一。每一章都为家长如何辅导子女完成高中提供了资料和建议。这份免费资料有十八种语言,登在网上:www.settlement.org/edguide 。详情请参阅本章末尾。

Starting School Simplified Chinese
1. 中学的种类(Kinds of Secondary (High) Schools) 安省高中有四个体系:英语公立、英语天主教、法语公立、法语天主教。每个体系都由地区教育局(district school boards)管理。所有未满二十一岁的人都可以进高中读书。许多教育局还为成人开办高中课程。法语学校是为讲法
语的学生设立的。有关法语学校的详情,请向法语教育局查询。安省教育局的详情,请见:
http://www.edu.gov.ca/eng/general/elemsec/es_overview.html
2. 报名入学(Registering For School)
家长为子女报名入学的时候,请携带下列证件:
a) 学生年龄证明,二者有一即可:
�6�1出生证明(a birth certificate)或者
�6�1护照 (a passport)

b) 住址证明,文件上必须有姓名和地址,下列文件有一即可:银行月结单(a bank statement)、电话单(a telephone bill)、电费单(an electrical bill)、租约(apartment lease)。

c) 监护证明:只有未满十八岁、而且不和父母同住的学生才需要。
d) 学生移民身份证明,下列文件有一即可:
�6�1加拿大护照 (Canadian Passport)
�6�1移民入境纸(IMM1000)
�6�1永久居民证明(IMM5292)
�6�1枫叶卡(Permanent Resident Card)

e) 防疫注射记录(Immunization Record):证明学生已注射过防疫针
Starting School Simplified Chinese
3. 评估中心(接待中心)(Assessment Centres , or Reception)
难民身份的学生
有些教育局要求新移民学生到评估中心,让老师测试数学和英
(Students with Refugee Status)
语水平。测试的结果会送到学生就读的学校,让老师了解该生已经学过哪些内容。如果你有孩子以前的成绩单、教科书、已
不同教育局对难民学生、或及任何有助于评估的教学资料,都请拿给评估中心的老师看。
者没有《永久居民证明表》(Confirmation of Permanent Residence)的学生,有不同的报名方式。请向当地的教育局查询。
4. 第一年选课(Placing Students in Courses in the First Year)
在新移民学生入学的时候,辅导员会先与他们讨论以前的教育、兴趣、毕业后的目标,然后再协助他们选择适当的科目。
学校会考虑:
�6�1 把已经学过的东西与安省高中课程的要求进行比较
�6�1学生的能力和兴趣
�6�1体现以前学业的文件,例如各科成绩表
�6�1如果做过数学和英语评估的话,评估的成绩
�6�1 学生在学校里上了多少年
�6�1 学生的年龄

学校会暂时将学生分到某一年级,选读各种课目。然后,根据学生体现出来的知识水平,学校会进行必要的调整,将他们分配到更适当的班级。

Starting School Simplified Chinese
通常,在学年末、一般在二月,学生要选择下学年的课目。阅读成绩单(report cards)、与老师面谈 (parent teacher interview)、和子女讨论,都有助于你和子女选择课目。
与子女一起定下一个学业计划(an education plan)是非常重要的。由于课程种类繁多,如果事先没有计划好,选择起来会非常困难,并且现在的选择会影响到子女毕业后的出路.
有了一个计划,就可以一步一步来、保证家长和学生的目标得以实现(不论是读大学、大专院校、学徒课程还是直接就业)。学校的老师和辅导员也可以提供协助.
寻求帮助(Getting Help)
如果家长有疑问或担心,最好向有关老师查询。如果是一般性的问题,可以向“英语作为第二语言的特殊教师”(the ESL teacher)或辅导员查询。
英语为第二语言老师(The ESL Teacher) ESL 是English as Second Language 的简称,就是那些给英语为第二语言的学生开设的课程,教这种课程的老师就叫ESL 老师。ESL 老师,除了专门给移民学生教授英语,还可以帮助学生和家长理解他们的选择、毕业后的出路。这些老师在协助新移民学生方面有丰富的知识和经验,懂得怎样帮助他们顺利完成高中学业。
辅导员(The Guidance Counsellor) 辅导员可以提供信息和建议,帮助学生和家长理解如何选课、如何择业、毕业之后有哪些机会。
辅导员也可以帮助学生解决个人的疑难、学校里的各种问题。家长可以向辅导员咨询子女在校的表现。辅导员的丰富经验对学生和家长相当有帮助。
Starting School Simplified Chinese
5. 承认以前的学分(Credits for Previous School Learning)
安省学校采用学分制。学生完成每个课目之后会得到一个学分(a credit),学生必须拿够三十个学分才可以毕业。其中有一些学分是必修的,有些则可以选择。学校可能会承认新移民学生在原居地受过的教育,给一些学分。
学校通常会先让学生在学习中展示以前学到了多少,然后才决定原居住地的学业可以算多少个学分。请向辅导员查询详情。
�6�1 所有可以体现学生以前学业的资料,都带到学校去,例如成绩表、教科书等等。

建 议
�6�1帮助子女先做好准备,再与老师讨论以前的学业、毕业后的计划。
�6�1 如有任何疑问,请致电辅导员查询.

6. 视力和听力健康(Vision and Hearing Health)
视觉或听觉有问题,学习起来会很困难。很多学生甚至不知道自己视觉或听觉有问题。他们可能认为自己的视听和别人是一样的。视觉和听觉会随着年龄增长而变化,所以定期检查是很重要的。
�6�1 每二至三年让验光师(an Optometrists)或眼科医生(an Ophthalmologists)检查一次眼睛。安省医疗保险计划(OHIP)为十九岁以下的人支付每年一次的检查费用。
建 议

�6�1 必要的时候,鼓励子女配戴眼镜、隐形眼镜、助听器。如果学生不想戴,与辅导员或老师见面,一起劝说鼓励。
�6�1 跟医生谈一谈,看是否应检查学生的听觉

Starting School Simplified Chinese
7. 使用计算机(Access to Computers) 使用计算机是中学课程的重要部分。每所学校都有计算机供学生使用。
公立图书馆、一些社区中心也设有计算机,可免费使用。社区中心的计算机称为CAP 计算机(CAP 就是Community Access Program)。哪些社区中心设有计算机,名单在网上:http://cap.ic.gc.ca/english/4000newonurban.asp.
8. 签证学生(Visa Students)
需要付学费的外国学生被称为”签证学生”、或外国出生的付费学生。如果这些学生的父母不在加拿大居住,他们就必须有一名正式的监护人(an guardian)住在学区之内。监护人要负起当家长的责任。有关签证学生或国际学生的课程、学费,请浏览各个教育局的网址。

wls1wls : 2011-01-12#940
回复: Lucia的说法2

“作者: wangxiaoyu-62
LUCIA,你写的很详细,很有帮助.我也是90年的,我是四月的.我现在正读高三,马上准备登录,但我有些问题想请教一下:我原本打算参加完国内的高考,拿到成绩后赴加.我是想到加后能进入象多大这样的大学学习.但听说这些名牌大学对象我这样的新移民申请时需提供TOFEL或者IELTS成绩,而我没有.象我这种情况,朋友给了我几种方案1)不要参加国内的高考,马上赴加来读这里的高三.------这样的话语言上会有很大进步,但我觉得还是不能申请大学,因为我还是没有TOFEL成绩,我上高三的课程那有时间考啊,还有就象你说的,学分也不够啊.其实我已留了一级,很有可能留两级呢有点浪费时间(2)参加完高考再赴加,然后读一年的预科,这期间学校有TOFEL的课程并可以参加考试.然后可以申请大学-----但听说这类预科学校都是私立的,不可靠,会随时倒闭,这又让我很迷惑.所以我很拿不定主意该怎么办? ”



就你的情况,我和儿子讨论了下,他认为,还是尽快过来,因为大学要求很高,如果成绩不好,或者fail课了,就会被踢出来,所以,到了大学首先语言要过关,不然课听不懂肯定fail了。高中过来读的好处呢,就是有时间适应英语,而且学分不是问题, 国内的学分可以转,他一个同学今年圣诞节过来的,到多伦多,她在国内正读高2,上个月学分转了,得到了20个学分,一年半轻轻松松就可以把学分拿够,学分到不是问题;还有你说的TOFEL,不是所有大学都要TOFEL,有的大学有它专门的英语考试,过了就可以上。在高中还有许多大学的竞赛,waerloo的macmaster的都经常在儿子的学校搞数学竞赛、物理竞赛什么的,他还在数学竞赛中拿了个全校第一,题目很简单;这个竞赛中成绩好的,大学录取的时候就会优先考虑。
在预科呢,他也有认识的同学在读,但是就是像你说的不可靠,主要不是怕倒闭,主要是学习环境差,教学质量也差,而且很贵。
我这里有个方案,你现在已经高三,没几个月就高考了,考不考无所谓,因为你考了过来还可以上高中,只要你是21岁以下的都可以上高中,然后就像我前面说的,大学要6门专业课学分,你就想好要学哪个专业,然后到http://www.electronicinfo.ca/en/上面看...难事。然后还有就是弄好国内成绩的翻译和公正,拿过来转学分,你的情况应该可以转24个学分吧。
就看你怎么安排,读一年,辛苦点,早点毕业,读2年,轻松点,可以有更多选择,比如你可以多?藜该鸥咧械目危?ù笱У氖焙蚓筒恢劣谥荒鼙ㄒ桓鲎ㄒ怠?
我儿子是90年10月的,现在才在这里高中读11年级,算正常年龄。你年龄不算大,不用着急的,主要是要能上个好大学,而且要能坚持住,不被T出来。

LISALLA : 2011-01-12#941
回复: 加拿大教育情况有用素材收集帖子--梁溪香榭整理

谢谢守法和梁大姐!这贴真好!:wdb19:

wls1wls : 2011-01-12#942
回复: 先来点排名吧。这是05年统计的,也不知道是不是管用,

大多伦多地区高中排名High School Ranking in Greater Toronto Area
以下高中学校排名(High School Ranking)是由Fraser Institute研究发表,National Post(国家邮报)发布的。目前最新的排名报告发表于2001年。是根据1998、99年度的学校资料进行的排名。表格中显示“n/a”的地方,表示资料不足,难以进行名次排定。此外,表格中也并没有包括本地区所有高中学校。 ・ 高中12年级人数不足15人的学校没有包括,因为其学生资料不够很难得出公正可靠的分析结果。 ・ 没有包括成人教育和继续教育中心、主要面向外国学生的学校、以及只是侧重某些方面课程教育的学校。当然,没有包括这些学校,并不意味否定这些学校的教育。 值得注意的是,多伦多教育局(Toronto DSB)的一些在华人社区中名气很大的高中,在Fraser排名中不一定理想。如A Y Jackson Secondary School,被认为资料不足,难以确定名次;而在华人社区较少听说的 R H King Academy ,确排名第20,是多伦多教育局在大多伦多地区排名最前的高中。安省排名 District Name School Name 本地区排名
Branksome Hall
Havergal College
St Andrew's College
St Michael's College School
St Clement's School
University Of Toronto Schools
The Crescent School
The Academy for Gifted Children
Holy Trinity School
Ner Israel Yeshiva College
St Mildred's-Lightbourn School
Richmond Hill High School
R H King Academy(Toronto DSB)
The Woodlands Secondary School
Pickering College
&amp;Eacute;cole secondaire Cardinal Carter de langue fran&amp;ccedil;aise
Bayview Glen
Kingsway College
Ursula Franklin Academy(Toronto DSB)
Beth Jacob Private School
Langstaff Secondary School
Northview Heights Secondary School(Toronto DSB)
Holy Name of Mary Secondary School
Georges Vanier Secondary School(Toronto DSB)
Unionville High School
Woburn Collegiate Institute(Toronto DSB)
Heart Lake Secondary School
Albert Campbell Collegiate Institute!(Toronto DSB)************
Forest Hill Collegiate Institute(Toronto DSB)
Martingrove Collegiate Institute(Toronto DSB)
Richview Collegiate Institute(Toronto DSB)
Parkdale Collegiate Institute(Toronto DSB)
Peoples Christian Academy
Leaside High School
The Country Day School(Toronto DSB)
Lorne Park Secondary School
Agincourt Collegiate Institute(Toronto DSB)
Thornlea Secondary School
Malvern Collegiate Institute(Toronto DSB)
North Toronto Collegiate Institute
Mayfield Secondary School
Streetsville Secondary School
Midland Avenue Collegiate Institute
David &amp; Mary Thomson Collegiate Institute
Glenforest Secondary School
Woodbridge College
St Joseph Secondary School
Markville Secondary School
Lester B Pearson Collegiate Institute
Loyola Catholic Secondary School
Dr Norman Bethune Collegiate Institute
Marc Garneau Collegiate Institute
O'Neill Collegiate and Vocational Institute
Father Leo J Austin Catholic Secondary School
St Ignatius of Loyola Secondary School
Erindale Secondary School
Stephen Leacock Collegiate Institute
Bayview Secondary School
The Humberview School
Brampton Centennial Secondary School
Pine Ridge Secondary School
Sir John A Macdonald Collegiate Institute
Bramalea Secondary School
Madison Academy
Milliken Mills High School
Westmount Collegiate Institute
&amp;Eacute;cole secondaire E J Lajeunesse

wls1wls : 2011-01-12#943
回复: 偶搜集到的一些关于加拿大高中教育的资料 与大家分享

“一般进入加拿大大学要综合考虑中学平时成绩和省试成绩”
据我自己了解的对这句话做个解释,以温哥华为例,申请大学时,中学成绩占60%,省试成绩占40%,但很多时候申请大学时,省考还没有进行,所以申请大学的成绩往往就是中学的平时成绩了。而这个中学成绩是不分学校的好坏的,不管是在好的学校还是在差的学校的学生根据平时的成绩申请好大学的几率是平等的。这里就可以扭转一部分新移民的思维,部门新移民家长刚开始登陆的时候总是固执地要求把孩子送到排名靠前(在加拿大中小学的排名,除华人等少数族裔外,基本上是既不被鼓励也不被看重)的中学,而不考虑孩子的实际情况,往往原本成绩比较好的孩子在那里成绩反倒越来越不好,或者丝毫没有优势了,倒申请不到好的大学了;其实根据孩子的实际情况,让他在一般的学校里学习,让他能跟得上学习进度,平时成绩及校考得比较好的话,倒能申请到好的大学。所以根据孩子的实际情况入学是最好,不能盲目地用国内的观点去安排孩子的教育问题。


转载,供参考

wls1wls : 2011-01-12#944
回复: 子女教育信息素材收集帖

嘎嘎

这个帖子也算历经了劫难了,丢了又重新找回来了。

主要整理的是关于教育的一些素材

从147楼开始,主要就是梁溪香榭搜集的一些的,推荐推荐!

推荐本书--从普通女孩到银行家,http://forum.iask.ca/showthread.php?t=233637

目录. :wdb37:

tony 3213, PEI地区教育情况 Page 1, 5楼
tiantianxingchen 高中课程简介 Page 1, 6-9楼
守法公民 初三学生多伦多求学记 Page1, 18-22楼
守法公民 安省新移民高中指南 Page 2, 24-30楼
守法公民 安省新移民小学指南 Page 2, 34-36楼
守法公民 高中家长如是说 Page 3, 44-49楼
雨中冷百合 大多伦多地区高中排名 Page 3, 57楼, 59楼
守法公民 温哥华十佳公立中学 Page 4, 65楼
一缕阳光 菲沙关于BC省中学排名及相关简介 Page 4, 69楼
守法公民 部分大学对是否需要雅思或托福成绩的年限规定 Page 6, 101楼
守法公民 教育篇 Page 6, 113-121楼
梁溪香榭 高中留学须谨慎 Page 8, 149-152楼
梁溪香榭 加拿大高中教育感悟 Page 8, 153-155楼
梁溪香榭 根据成绩选择加拿大大学 Page 8, 159楼
梁溪香榭 如何选择最适合的加拿大大学 Page 9, 161楼
梁溪香榭 读高二的孩子去加拿大需要准备什么 Page 9, 164-165楼
梁溪香榭 高中英语课程介绍 Page 9, 166-169楼
梁溪香榭 高中英语课程介绍 Page 10, 188-191楼
梁溪香榭 多伦多著名私立学校收费一览 Page 10, 192楼
梁溪香榭 孩子的英语过关了吗? Page10, 194楼
梁溪香榭 关于A-LEVEL课程 Page 11, 217-228楼
shieley6811 关于A-LEVEL课程及学制 Page 13, 256-257楼
梁溪香榭 2007年安省高中拍名 Page 17, 322楼
朱三雀 安省, BC省, AB省高中拍名, Page 17, 329楼
响铃当当 加拿大教育优势 Page18, 347楼
梁溪香榭 A-LEVEL与IB课程的比较 Page 18, 358楼
朱三雀 中西学生的差异 Page 19, 364楼
梁溪香榭 大多伦多地区IB高中简介 Page 19, 372-375楼
梁溪香榭 IB课程与其他特殊教育课程的比较 Page 19, 376楼
梁溪香榭 IB与AP的比较, Page 19, 377楼
梁溪香榭 美国相关研究机构关于IB与AP的选择比较 Page 20, 387-391楼
梁溪香榭 群星灿烂不如一轮皓月当空-大学录取篇 Page 21, 409-413楼
梁溪香榭 在教育子女的过程中教育自己 Page 22, 422-424楼
梁溪香榭 关于美国大学 Page 22, 429-432楼
梁溪香榭 多伦多高中阶段特殊教育程序 Page 23, 445-446楼
梁溪香榭 加拿大高中生学科竞赛 Page 23, 447-448楼
梁溪香榭 美国大学 Page 24, 466楼
梁溪香榭 如何选择适合自己孩子的中小学 Page 25, 487-489楼
梁溪香榭 论孩子成长的内环境 Page 25, 490楼
梁溪香榭 犹太人的教育风格:朴实无华 讲求实际 Page 25, 497楼
梁溪香榭 培养子女的参政意识 Page 25, 498楼
梁溪香榭 美国大学点评 Page 25, 499-529楼
梁溪香榭 加拿大大学点评(商科) Page 27, 533-535楼
梁溪香榭 加拿大大学点评(理工科) Page 27, 547-549楼
梁溪香榭 安省高中教育咨询问答 Page 30, 584楼
梁溪香榭 毕业之际谈我的大学:滑铁卢大学 Page 30, 586-592楼
梁溪香榭 我在麦克马斯特 Page 30, 597楼
梁溪香榭 2005年度全球十大商学院 Page 30, 598楼
梁溪香榭 世界大学金融专业的排名 Page 30, 599楼
梁溪香榭 在加拿大IVEY商学院读MBA的日子 Page 31, 608楼
梁溪香榭 加拿大几个名牌大学比较之我见 Page 31, 616楼
梁溪香榭 加拿大的商学院MBA排名分析 Page 31, 618楼
梁溪香榭 加拿大本科申请指南 page 31-32, 620-624楼
yhyxll申请UBC请问需要什么条件,难度大否 Page 32, 633-634楼
willam131 关于A-LEVEL选课 page 34, 680楼
梁溪香榭 公立学校vs.私立学校
page 36, 713-716楼
梁溪香榭 路在自己脚下 --- 一位小留学生的自述 page 36, 717楼
惴惴不安 移民学生多“断层” 上公校还是私校难倒华人家长 page 36, 722楼
梁溪香榭 加拿大中学教育 page 38, 745楼
梁溪香榭 安大略省中学教育制度 page 38, 747楼
梁溪香榭 加拿大高中选课制 page 38, 748楼
梁溪香榭 高中毕业生该如何选择加拿大大学 page 38, 749楼
梁溪香榭 高中生如何入读加拿大大学?page 38, 750楼
梁溪香榭 加拿大大学怎样录取新生page 38, 751楼
梁溪香榭 加拿大大学对A-LEVEL的录取要求 page 38, 753-755楼

wls1wls : 2011-01-12#945
回复: 加拿大教育情况有用素材收集帖子--梁溪香榭整理

的确是好帖,绝对雪中送碳,感谢守法和香榭!

wls1wls : 2011-01-15#946
回复: Lucia的成绩单

[/ATTACH]

:wdb37:

meiwen : 2011-01-16#947
回复: 加拿大教育情况有用素材收集帖子--梁溪香榭整理

谢谢wls1wls的热心推荐,真是雪中送炭,正在学习中。

喜庆一生 : 2011-01-28#948
回复: 加拿大教育情况有用素材收集帖子--梁溪香榭整理

好贴啊,收藏了

chenwj : 2011-02-13#949
回复: 偶搜集到的一些关于加拿大高中教育的资料 与大家分享

我女儿也在这个学校读书,这个学校的管理很差,如果学生自我管理能力不够,要想学好英语也不容易。
我也准备把女儿送过去上学啊,这所学校真的很差吗?还是中国与加拿大的教育模式不一样的原故?

陌上花 : 2011-03-12#950
回复: 加拿大教育情况有用素材收集帖子--梁溪香榭整理

太棒了,还没看完,一口气才看到38楼。受益匪浅,谢谢您们这么细心的整理。收了慢慢的来看。新手还不知怎么加声望。

巧克力薄荷10 : 2011-03-22#951
回复: 加拿大教育情况有用素材收集帖子--梁溪香榭整理

非常有用,非常感谢!早看到这些就不用到处找人打听了。

期待远方 : 2011-03-29#952
回复: 加拿大教育情况有用素材收集帖子--梁溪香榭整理

之前也读了香榭姐姐的此帖,因为没有经历,没有感受;再读香榭姐姐帖子,受益匪浅,谢谢姐姐:wdb17::wdb19::wdb45:

gonglx : 2011-04-15#953
回复: 加拿大教育情况有用素材收集帖子--梁溪香榭整理

辛苦了,很有用,收藏起来慢慢读。

彩虹鱼 : 2011-04-22#954
回复: 加拿大教育情况有用素材收集帖子--梁溪香榭整理

一气呵成,看完了,只是囫囵吞枣。因为离登陆还很遥远,所以也没有好好去消化。现在纠结的是,现在要选择怎样的初中,本来算算正好去加拿大上七年级的,但是因为面试不顺利要耽误一年多。按现在的进度估计等拿到签证是要上八年级下了吧。请教大侠们,加国教育局能不能小孩降一级重读一年的,是不是每个省的政策不一样的。听说哈利法克斯只能在18岁内读完高中的,所以没有办法降级。听听各位的建议。

xuexuexue488 : 2011-05-02#955
回复: 偶搜集到的一些关于加拿大高中教育的资料 与大家分享

好帖:wdb10:

芳子2009 : 2011-05-04#956
回复: 子女教育信息素材收集帖

那我就自己整理,整理好了,有需要的可以找我。
我很需要,不知道有没有已经整理好?先谢谢了。

sunnyabc : 2011-05-10#957
回复: 加拿大教育情况有用素材收集帖子--梁溪香榭整理

好贴!学习了。谢谢守法,谢谢梁溪!

林荫密草 : 2012-01-09#958
回复: 加拿大教育情况有用素材收集帖子--梁溪香榭整理

谢谢溪溪姐,太有用了,:wdb17:给嫩加声望:wdb10:

天涯同路 : 2012-01-09#959
回复: 加拿大教育情况有用素材收集帖子--梁溪香榭整理

收!

ggs : 2012-01-09#960
回复: 加拿大教育情况有用素材收集帖子--梁溪香榭整理

:wdb37::wdb45:

梁溪香榭 : 2012-02-18#961
回复: 加拿大教育情况有用素材收集帖子--梁溪香榭整理

好久没来了,好不容易找到这个老帖子。谢谢上面各位同学的鼓励肯定表扬。
儿子昨天收到了多伦多大学的电子工程专业的OFFER,就是俗称的DOUBLE E. 这是儿子的第一志愿,算是称心如意了。2月份就拿到这个OFFER,也说明他在所有申请者中占据相当大的优势。
我们2010年7月拿到取签函,思来想去选择了留在国内完成A-LEVEL,使得高中教育比较连贯完整。儿子在这三年中不光取得了非常好的成绩,个性也更阳光,思维更宽广更平衡。看到他的一些英文写作,我很惊讶这是出自初中老师眼中的作文老大难的儿子之手。
总体而言,当初的选择是对的。A-LEVEL在国内,或者说在他们学校,可谓是取长避短。即取了国内高中教育的长,也避了国内教育模式的短。

八桂桂花 : 2012-02-18#962
回复: 加拿大教育情况有用素材收集帖子--梁溪香榭整理

恭贺梁溪儿子顺利考取多大!:wdb45::wdb17::wdb6:!有时间能否给我们分享一下儿子在国内上国际学校的体会,谢谢!

honeygirl : 2012-02-18#963
回复: 加拿大教育情况有用素材收集帖子--梁溪香榭整理

恭喜啊!我儿子今年也要中考了,希望他能进自己心仪的高中。前辈有好的建议能指教啊!

家园移民 : 2012-02-18#964
回复: 加拿大教育情况有用素材收集帖子--梁溪香榭整理

好久没来了,好不容易找到这个老帖子。谢谢上面各位同学的鼓励肯定表扬。
儿子昨天收到了多伦多大学的电子工程专业的OFFER,就是俗称的DOUBLE E. 这是儿子的第一志愿,算是称心如意了。2月份就拿到这个OFFER,也说明他在所有申请者中占据相当大的优势。
我们2010年7月拿到取签函,思来想去选择了留在国内完成A-LEVEL,使得高中教育比较连贯完整。儿子在这三年中不光取得了非常好的成绩,个性也更阳光,思维更宽广更平衡。看到他的一些英文写作,我很惊讶这是出自初中老师眼中的作文老大难的儿子之手。
总体而言,当初的选择是对的。A-LEVEL在国内,或者说在他们学校,可谓是取长避短。即取了国内高中教育的长,也避了国内教育模式的短。
值得高兴的

crisy : 2012-02-19#965
回复: 加拿大教育情况有用素材收集帖子--梁溪香榭整理

好久没来了,好不容易找到这个老帖子。谢谢上面各位同学的鼓励肯定表扬。
儿子昨天收到了多伦多大学的电子工程专业的OFFER,就是俗称的DOUBLE E. 这是儿子的第一志愿,算是称心如意了。2月份就拿到这个OFFER,也说明他在所有申请者中占据相当大的优势。
我们2010年7月拿到取签函,思来想去选择了留在国内完成A-LEVEL,使得高中教育比较连贯完整。儿子在这三年中不光取得了非常好的成绩,个性也更阳光,思维更宽广更平衡。看到他的一些英文写作,我很惊讶这是出自初中老师眼中的作文老大难的儿子之手。
总体而言,当初的选择是对的。A-LEVEL在国内,或者说在他们学校,可谓是取长避短。即取了国内高中教育的长,也避了国内教育模式的短。
梁溪,你这两年没在加拿大吗?一直看你的帖子,以为你已经定居了呢. 算来你拿到签证的时候,儿子应该是高一结束,快上高二了吧, 估计我也差不多在孩子高一结束的时候拿签证.对于有高中生的家庭,选择合适的登陆日期,的确是让人思来想去,难以定夺的事情,恭喜你了!

梁溪香榭 : 2012-02-19#966
回复: 加拿大教育情况有用素材收集帖子--梁溪香榭整理

恭贺梁溪儿子顺利考取多大!:wdb45::wdb17::wdb6:!有时间能否给我们分享一下儿子在国内上国际学校的体会,谢谢!

谢谢!
其实我的感受基本都在这个帖子里分享了的。
如果你有特别的问题,请随时问吧。

梁溪香榭 : 2012-02-19#967
回复: 加拿大教育情况有用素材收集帖子--梁溪香榭整理

恭喜啊!我儿子今年也要中考了,希望他能进自己心仪的高中。前辈有好的建议能指教啊!

指教不敢啦。关键的看清楚孩子的闪光点在哪里。
学校老师总是希望孩子均衡发展,没有瘸腿科目。造成的结果是孩子不得不将大量的精力放在自己不擅长的地方。可惜啊!
其实正相反,何不将有限的精力放在刀刃上,扬长避短才是聪明的选择。

梁溪香榭 : 2012-02-19#968
回复: 加拿大教育情况有用素材收集帖子--梁溪香榭整理

值得高兴的

谢谢领导。
我们的移民之路多得您的大力帮助,怎一个谢字了得。
自我感觉,这条路我们走得还算是稳的,不左也不右。一步一个脚印,每一步都不浪费。现在回头看看,真的是满心感恩。

梁溪香榭 : 2012-02-19#969
回复: 加拿大教育情况有用素材收集帖子--梁溪香榭整理

梁溪,你这两年没在加拿大吗?一直看你的帖子,以为你已经定居了呢. 算来你拿到签证的时候,儿子应该是高一结束,快上高二了吧, 估计我也差不多在孩子高一结束的时候拿签证.对于有高中生的家庭,选择合适的登陆日期,的确是让人思来想去,难以定夺的事情,恭喜你了!

我们去旅游了两趟。第一趟是2011年1月办理登陆手续,顺便看了看几所大学。第二趟是01年暑假,约了另外一家过去玩了两周。
你算得对,我们拿到签证是儿子高一升高二的那个暑假。
个人认为,高中阶段转去加拿大要慎重再慎重。时间真的太紧太紧,会很大程度影响孩子将来的发展方向。
当然,具体要看各个孩子的情况了。

honeygirl : 2012-02-19#970
回复: 加拿大教育情况有用素材收集帖子--梁溪香榭整理

指教不敢啦。关键的看清楚孩子的闪光点在哪里。
学校老师总是希望孩子均衡发展,没有瘸腿科目。造成的结果是孩子不得不将大量的精力放在自己不擅长的地方。可惜啊!
其实正相反,何不将有限的精力放在刀刃上,扬长避短才是聪明的选择。
言之有理,小儿数理化历史名列前茅,英语中上,语文中。请问前辈怎么合理的选高中课程:wdb17:

八桂桂花 : 2012-02-19#971
回复: 加拿大教育情况有用素材收集帖子--梁溪香榭整理

我们去旅游了两趟。第一趟是2011年1月办理登陆手续,顺便看了看几所大学。第二趟是01年暑假,约了另外一家过去玩了两周。
你算得对,我们拿到签证是儿子高一升高二的那个暑假。
个人认为,高中阶段转去加拿大要慎重再慎重。时间真的太紧太紧,会很大程度影响孩子将来的发展方向。
当然,具体要看各个孩子的情况了。
现在看来,梁溪当初让儿子在国际学校读完高中是个正确的选择:wdb17::wdb45:,虽然儿子的学费额外付出了一些(如果去加拿大就会省下这两年的学费吧)但争气的儿子考取了如愿的大学,也感到欣慰了吧:wdb6:!我也在申请移民过程中,08年底递交的,现在还在等ME阶段,我也选择了给儿子读国际学校,现在刚上完一个学期,感觉现在也挺适应,我把你儿子作为他学习的榜样,我再好好重温你的帖子,多多学习!:wdb37::wdb45:

梁溪香榭 : 2012-02-19#972
回复: 加拿大教育情况有用素材收集帖子--梁溪香榭整理

言之有理,小儿数理化历史名列前茅,英语中上,语文中。请问前辈怎么合理的选高中课程:wdb17:
虽然那时我们的移民才刚刚开始,还没有面试,我们却毅然选择上A-LEVEL,彻底放弃高考,相当一部原分因是因为儿子的语文成绩是在太那个啥了。:wdb5:

梁溪香榭 : 2012-02-19#973
回复: 加拿大教育情况有用素材收集帖子--梁溪香榭整理

现在看来,梁溪当初让儿子在国际学校读完高中是个正确的选择:wdb17::wdb45:,虽然儿子的学费额外付出了一些(如果去加拿大就会省下这两年的学费吧)但争气的儿子考取了如愿的大学,也感到欣慰了吧:wdb6:!我也在申请移民过程中,08年底递交的,现在还在等ME阶段,我也选择了给儿子读国际学校,现在刚上完一个学期,感觉现在也挺适应,我把你儿子作为他学习的榜样,我再好好重温你的帖子,多多学习!:wdb37::wdb45:
加油!:wdb9:

不过我们上的不是国际学校,而是市一中的A-LEVEL班。

李先生 : 2012-02-19#974
回复: 加拿大教育情况有用素材收集帖子--梁溪香榭整理

有用,收藏

八桂桂花 : 2012-02-19#975
回复: 加拿大教育情况有用素材收集帖子--梁溪香榭整理

加油!:wdb9:

不过我们上的不是国际学校,而是市一中的A-LEVEL班。
你儿子真优秀!:wdb19:你儿子一个班有多少人?像你儿子这样考取世界名校的多吗?我儿子现在读的是武汉枫叶国际学校,是跟加拿大BC省联合办学的,有BC省学籍,对,我们好好加油!

梁溪香榭 : 2012-02-19#976
回复: 加拿大教育情况有用素材收集帖子--梁溪香榭整理

你儿子真优秀!:wdb19:你儿子一个班有多少人?像你儿子这样考取世界名校的多吗?我儿子现在读的是武汉枫叶国际学校,是跟加拿大BC省联合办学的,有BC省学籍,对,我们好好加油!

现在时间还早,出结果的主要是英国方面。
儿子班里30个人,两个班共大约60个
剑桥2个,帝国理工6个,圣安德鲁斯3个,杜伦6个
美国方面听说已经有收到OFFER的,还不知道详情。通常要到3月有大面积OFFER出来。
加拿大方面通常大面积发OFFER是在4-5月份。我儿子这算是提前录取。所以,他们这一届的整体情况还要等一等。

Luck Li : 2012-02-19#977
回复: 加拿大教育情况有用素材收集帖子--梁溪香榭整理

祝贺香溪有这么有出息的宝贝。+++

梁溪香榭 : 2012-02-20#978
回复: 加拿大教育情况有用素材收集帖子--梁溪香榭整理

祝贺香溪有这么有出息的宝贝。+++

Luck, 来我们抱一个吧:wdb19:。看到你好开心。

amyliu1226 : 2012-02-20#979
回复: 加拿大教育情况有用素材收集帖子--梁溪香榭整理

这几天正愁怎么能找到梁溪呢,可巧姐姐就出现了,真是太高兴了!方便的话,能否共享一下您儿子申请大学的经历?我儿子现在在国际学校上IGCSE2,今年九月份上高三,马上就面临申请大学的问题。我们是香港联投,刚体检完,估计今年暑假前后差不多能拿到签证,这样的话,我们准备像您一样拿到签证后先短登,儿子在国内继续上完高三上半年,明年1月份就该申请加拿大的大学了,只是不知道我们这样应该怎么申请?是按国际生还是按加国学生算,两者的申请一样吗?能否分享一下你们的经验或谈谈您的看法?谢谢了!

天涯同路 : 2012-02-20#980
回复: 加拿大教育情况有用素材收集帖子--梁溪香榭整理

好人,好贴,安省的情况非常详细,魁省的能有这样一个贴字就好了,感谢楼主

梁溪香榭 : 2012-02-20#981
回复: 加拿大教育情况有用素材收集帖子--梁溪香榭整理

这几天正愁怎么能找到梁溪呢,可巧姐姐就出现了,真是太高兴了!方便的话,能否共享一下您儿子申请大学的经历?我儿子现在在国际学校上IGCSE2,今年九月份上高三,马上就面临申请大学的问题。我们是香港联投,刚体检完,估计今年暑假前后差不多能拿到签证,这样的话,我们准备像您一样拿到签证后先短登,儿子在国内继续上完高三上半年,明年1月份就该申请加拿大的大学了,只是不知道我们这样应该怎么申请?是按国际生还是按加国学生算,两者的申请一样吗?能否分享一下你们的经验或谈谈您的看法?谢谢了!

IGCSE2的全球统考就快来了,这个成绩很关键,预祝你儿子取得好成绩。另外就是托福成绩要过关。其他其实没什么。就加拿大大学申请来说,成绩是硬道理。慈善义工,各种社团活动啥的,其实不太有用。我估计人家也明白,在中国这些东西不太靠谱。

只要完成了最后的登陆手续,你儿子就应该以加国永久居民的身份来申请。与国际生的申请不同,二者是在不同的POOL里面的。以我的观察,学校对二者的成绩要求是相同的,但在相同的成绩下,永久居民比国际生更容易获取OFFER。SO,身份很重要。

梁溪香榭 : 2012-02-20#982
回复: 加拿大教育情况有用素材收集帖子--梁溪香榭整理

好人,好贴,安省的情况非常详细,魁省的能有这样一个贴字就好了,感谢楼主

谢谢。
其实我觉得不管在哪里,基本道理是相通的。
这个帖子应该到也不是针对安省,但的确没有涉及到法语的问题。这个问题我从来没有考虑过,对魁省的高中教育不了解啊。

关于你说到的,目前的学校教学基础知识扎实,对国际学校心里没底。
我的意见是,国际学校的确良莠不齐,不要盲目转去国际学校,除非你心里有了底。IP算不了什么的,对升学其实帮助并不大。
不过我觉得,如果孩子过去的时间已经到了高中阶段或者接近高中阶段,还是要多慎重,更何况是去法语学校。你说呢?

八桂桂花 : 2012-02-20#983
回复: 加拿大教育情况有用素材收集帖子--梁溪香榭整理

谢谢梁溪每帖详细回复:wdb37::wdb19:,请问你儿子考的是托福吗?加拿大是不是考雅思就可以了,我儿子现在是10年级,是不是也该为考雅思做准备了。还有个问题想请教,过几个月要选课了,他现在成绩还不错,平均分可以在90分以上,但看不出哪门特别突出,理科不像某些男孩那么突出,文科也没特别的,我鼓励他干脆向生物挑战,毕竟是一门新的东西,他似乎也同意,其实我也没底,不知道究竟以后的路有多难,谢谢赐教!

Luck Li : 2012-02-20#984
回复: 加拿大教育情况有用素材收集帖子--梁溪香榭整理

Luck, 来我们抱一个吧:wdb19:。看到你好开心。

一定要抱抱,如果这辈子我们能有幸在加拿大相逢,才将是真正的缘分。:wdb17::wdb19::wdb6::wdb26::wdb9::wdb23:

梁溪香榭 : 2012-02-20#985
回复: 加拿大教育情况有用素材收集帖子--梁溪香榭整理

谢谢梁溪每帖详细回复:wdb37::wdb19:,请问你儿子考的是托福吗?加拿大是不是考雅思就可以了,我儿子现在是10年级,是不是也该为考雅思做准备了。还有个问题想请教,过几个月要选课了,他现在成绩还不错,平均分可以在90分以上,但看不出哪门特别突出,理科不像某些男孩那么突出,文科也没特别的,我鼓励他干脆向生物挑战,毕竟是一门新的东西,他似乎也同意,其实我也没底,不知道究竟以后的路有多难,谢谢赐教!

是的,我儿子考的是托福。
是的,加拿大也接受雅思。
是的,10年纪是应该为雅思做准备了。

关于选课,我儿子都是自己做主,我完全不过问。这次的大学申请,什么大学什么专业,也完全是他自己的主意,还好对我履行告知义务。

我建议你不妨让孩子作把主,不要剥夺了孩子摔跤受伤康复成长的机会。我看到很多很多父母,说是让孩子做主,其实不自觉地以各种方式试图影响孩子的决定,且最终赢的多是父母,因为孩子已经习惯于依赖了。

生物蛮好,如果孩子喜欢就没问题。俗话说,兴趣是最好的老师。顺着孩子的兴趣走应该是比较聪明的选择。而之所以看不到孩子的兴趣方向,有时就是因为家长干涉太多造成的。不知你是否同意。呵呵

梁溪香榭 : 2012-02-20#986
回复: 加拿大教育情况有用素材收集帖子--梁溪香榭整理

一定要抱抱,如果这辈子我们能有幸在加拿大相逢,才将是真正的缘分。:wdb17::wdb19::wdb6::wdb26::wdb9::wdb23:

好,我们一定有这个缘分!:wdb9:

nancy69 : 2012-02-20#987
回复: 加拿大教育情况有用素材收集帖子--梁溪香榭整理

好贴,真的太感谢了。

Robin666666 : 2012-02-21#988
回复: 加拿大教育情况有用素材收集帖子--梁溪香榭整理

值得高兴的
好好好!

zaeuro : 2012-02-22#989
回复: 加拿大教育情况有用素材收集帖子--梁溪香榭整理

非常实用

八桂桂花 : 2012-02-22#990
回复: 加拿大教育情况有用素材收集帖子--梁溪香榭整理

是的,我儿子考的是托福。
是的,加拿大也接受雅思。
是的,10年纪是应该为雅思做准备了。

关于选课,我儿子都是自己做主,我完全不过问。这次的大学申请,什么大学什么专业,也完全是他自己的主意,还好对我履行告知义务。

我建议你不妨让孩子作把主,不要剥夺了孩子摔跤受伤康复成长的机会。我看到很多很多父母,说是让孩子做主,其实不自觉地以各种方式试图影响孩子的决定,且最终赢的多是父母,因为孩子已经习惯于依赖了。

生物蛮好,如果孩子喜欢就没问题。俗话说,兴趣是最好的老师。顺着孩子的兴趣走应该是比较聪明的选择。而之所以看不到孩子的兴趣方向,有时就是因为家长干涉太多造成的。不知你是否同意。呵呵
多谢梁溪的耐心解答!非常同意:兴趣是最好的老师!我再跟儿子好好探讨下!:wdb6::wdb37:

crisy : 2012-02-22#991
回复: 加拿大教育情况有用素材收集帖子--梁溪香榭整理

梁溪,我儿子目前在国内读IB,九年级, 如果明年我的签证能办下来,我就想带他长登了, 我知道高中的孩子选择长登有一定的风险, 儿子成绩一般,即便留在国内上完高中,我也不知道他是否一定能如你儿子一样考上像多伦多大学这样的名校, 所以短登后回国上高中意味着也会有风险--损失时间,思来想去,我还是倾向于拿到签证就走, 让他尽早适应加拿大的生活,然后再作定夺,我想问你的是: 学IB的孩子, 如果到了多伦多,找到教IB课程的高中, 插班难度大吗? 还是不一定非上Ib,选择普通的高中,他们之间的区别大吗?

梁溪香榭 : 2012-02-23#992
回复: 加拿大教育情况有用素材收集帖子--梁溪香榭整理

梁溪,我儿子目前在国内读IB,九年级, 如果明年我的签证能办下来,我就想带他长登了, 我知道高中的孩子选择长登有一定的风险, 儿子成绩一般,即便留在国内上完高中,我也不知道他是否一定能如你儿子一样考上像多伦多大学这样的名校, 所以短登后回国上高中意味着也会有风险--损失时间,思来想去,我还是倾向于拿到签证就走, 让他尽早适应加拿大的生活,然后再作定夺,我想问你的是: 学IB的孩子, 如果到了多伦多,找到教IB课程的高中, 插班难度大吗? 还是不一定非上Ib,选择普通的高中,他们之间的区别大吗?

抱歉,这个问题我回答不了。
据说加拿大IB高中不容易进,学习强度比普通高中大很多。

amyliu1226 : 2012-02-23#993
回复: 加拿大教育情况有用素材收集帖子--梁溪香榭整理

IGCSE2的全球统考就快来了,这个成绩很关键,预祝你儿子取得好成绩。另外就是托福成绩要过关。其他其实没什么。就加拿大大学申请来说,成绩是硬道理。慈善义工,各种社团活动啥的,其实不太有用。我估计人家也明白,在中国这些东西不太靠谱。

只要完成了最后的登陆手续,你儿子就应该以加国永久居民的身份来申请。与国际生的申请不同,二者是在不同的POOL里面的。以我的观察,学校对二者的成绩要求是相同的,但在相同的成绩下,永久居民比国际生更容易获取OFFER。SO,身份很重要。
非常感谢您的解答!还有一疑问请教:只拿到签证而未登陆,是否算加国的永久居民?如是不是的话,那是否意味着最好在孩子申请大学前()登陆?

crisy : 2012-02-23#994
回复: 加拿大教育情况有用素材收集帖子--梁溪香榭整理

抱歉,这个问题我回答不了。
据说加拿大IB高中不容易进,学习强度比普通高中大很多。

哦,看来这事还得再做打算, 谢谢梁溪!

瓜果妈妈 : 2012-02-23#995
回复: 加拿大教育情况有用素材收集帖子--梁溪香榭整理

谢谢梁溪的无私分享!A-LEVEL难考吗?初二毕业能考吗?初三大部分是为中考进行的题海训练了吧,不如放弃。。。

梁溪香榭 : 2012-02-23#996
回复: 加拿大教育情况有用素材收集帖子--梁溪香榭整理

非常感谢您的解答!还有一疑问请教:只拿到签证而未登陆,是否算加国的永久居民?如是不是的话,那是否意味着最好在孩子申请大学前()登陆?

只拿到移民纸和签证还没有真正完成移民。必须成功登陆才成为加拿大永久居民。
是的,最好在孩子申请大学前登陆,比如高二升高三的那个暑假。否则的话就是按照国际生的身份申请。

梁溪香榭 : 2012-02-23#997
回复: 加拿大教育情况有用素材收集帖子--梁溪香榭整理

谢谢梁溪的无私分享!A-LEVEL难考吗?初二毕业能考吗?初三大部分是为中考进行的题海训练了吧,不如放弃。。。

A-LEVEL目前有很多学校都在开办,但办的情况是良莠不齐的。
我们这边的A-LEVEL招生是根据中考成绩,且分数线高居全市所有高中录取分数线的第二名(第一名就是他们学校的本部高中)。儿子学校的A-LEVEL平均成绩是全国第二名(第一名在杭州)。
所以,如果想选择A-LEVEL,最好多做做功课,选择学校很重要。

梁溪香榭 : 2012-02-23#998
回复: 加拿大教育情况有用素材收集帖子--梁溪香榭整理

哦,看来这事还得再做打算, 谢谢梁溪!

嗯,再多做点功课,综合考量吧。不谢!

西子湖畔 : 2012-02-23#999
回复: 加拿大教育情况有用素材收集帖子--梁溪香榭整理

恭喜梁溪,儿子争气的:wdb17:!我们的移民之路还遥遥无期,已准备让儿子今年9月份先去留学了。无限纠结!

梁溪香榭 : 2012-02-24#1000
回复: 加拿大教育情况有用素材收集帖子--梁溪香榭整理

恭喜梁溪,儿子争气的:wdb17:!我们的移民之路还遥遥无期,已准备让儿子今年9月份先去留学了。无限纠结!

谢谢
这移民之路的确太磨人了。加油:wdb9:

Luck Li : 2012-02-24#1001
回复: 加拿大教育情况有用素材收集帖子--梁溪香榭整理

谢谢
这移民之路的确太磨人了。加油:wdb9:
一言难尽!:wdb1::wdb26::wdb9::wdb23:

都是浮云 : 2012-05-13#1002
回复: 加拿大教育情况有用素材收集帖子--梁溪香榭整理

感谢LZ和大家的奉献,顶一下!

泠波微风 : 2012-05-15#1003
回复: 加拿大教育情况有用素材收集帖子--梁溪香榭整理

好贴,收藏

就爱红木 : 2012-05-16#1004
回复: 加拿大教育情况有用素材收集帖子--梁溪香榭整理

我孩子今年上完初一我们就登陆,我还能再让他上一年七年级吗?关键想让他多补补外语。

pacsun : 2012-07-17#1005
回复: 加拿大教育情况有用素材收集帖子--梁溪香榭整理

请教有知道西安大略大学及皇后大学商科的录取条件吗?谢谢啦,孩子目前在加国上高中,明年夏天申请大学